You are on page 1of 215

Editorial Board

Pratiyogita Darpan

UPKAR PRAKASHAN, AGRA–2


© Publishers

Publishers
UPKAR PRAKASHAN
(An ISO 9001 : 2000 Company)

2/11A, Swadeshi Bima Nagar, AGRA–282 002


Phone : 4053333, 2530966, 2531101
Fax : (0562) 4053330, 4031570
E-mail : care@upkar.in, Website : www.upkar.in
Branch Offices :
4845, Ansari Road, Daryaganj, Pirmohani Chowk, 1-8-1/B, R.R. Complex (Near Sundaraiah
New Delhi—110 002 Kadamkuan, Park, Adjacent to Manasa Enclave Gate),
Phone : 011–23251844/66 Patna—800 003 Bagh Lingampally,
Phone : 0612–2673340 Hyderabad—500 044 (A.P.)
Phone : 040–66753330
28, Chowdhury Lane, Shyam B-33, Blunt Square, Kanpur
Bazar, Near Metro Station, Taxi Stand Lane, Mawaiya,
Gate No. 4 Lucknow—226 004 (U.P.)
Kolkata—700004 (W.B.) Phone : 0522–4109080
Phone : 033–25551510

● The publishers have taken all possible precautions in publishing this book, yet if
any mistake has crept in, the publishers shall not be responsible for the same.
● This book or any part thereof may not be reproduced in any form by
Photographic, Mechanical, or any other method, for any use, without written
permission from the Publishers.
● Only the courts at Agra shall have the jurisdiction for any legal dispute.

ISBN : 978-93-5013-418-4

Code No. 1819


Chemical Sciences
CSIR UGC-NET/JRF Exam.
Solved Paper
June 2012
Chemical Science
Useful Fundamental Constants Chromium Cr 24 52.00
Cobalt Co 27 58.93
m Mass of electron 9.11 × 10–31 Kg Copper Cu 29 63.54
h Planck’s constant 6.63 × 10–34 Jsec Curium Cm 96 (247)
e Charge of electron 1.6 × 10–19 C Dysprosium Dy 66 162.50
Einsteinium Es 99 (254)
k Boltzmann constant 1.38 × 10–23 J/k
Erbium Er 68 167.26
c Velocity of Light 3.0 × 108 m/sec Europium Eu 63 151.96
IcV 1.6 × 10–14 J Fermium Fm 100 (253)
amu 1.67 × 10–27 kg Fluorine F 9 19.00
Francium Fr 87 (223)
G 6.67 × 10–11 Nm2 kg–2
Gadolinium Gd 64 157.25
Ry Rydberg constant 1.097 × 107 m–1 Gallium Ga 31 69.72
NA Avogadro number 6.023 × 1023 mole–1 Germanium Ge 32 72.59
ε0 8.854 × 10–12 Fm–1 Gold Au 79 196.97
Hafnium Hf 72 178.49
μ0 4π × 10–7 Hm–2 Helium He 2 4.003
R Molar Gas constant 8.314 JK–1 mole–1 Holmium Ho 67 164.93
Hydrogen H 1 1.0080
List of the Atomic Weights of the Elements Indium In 49 114.82
Atomic Atomic Iodine I 53 126.90
Element Symbol Iridium Ir 77 192.2
Number Weight
Iron Fe 26 55.85
Actinium Ac 89 (227)
Krypton Kr 36 83.80
Aluminium Al 13 26.98
Lanthanum La 57 138.91
Americium Am 95 (243)
Lawrencium Lr 103 (257)
Antimony Sb 51 121.75
Lead Pb 82 207.19
Argon Ar 18 39.948
Lithium Li 3 6.939
Arsenic As 33 74.92
Lutetium Lu 71 174.97
Astatine At 85 (210)
Magnesium Mg 12 24.312
Barium Ba 56 137.34
Manganese Mn 25 54.94
Berkelium Bk 97 (249)
Mendelevium Md 101 (256)
Beryllium Be 4 9.012
Mercury Hg 80 200.59
Bismuth Bi 83 208.98
Molybdenum Mo 42 95.94
Boron B 5 10.81
Neodymium Nd 60 144.24
Bromine Br 35 79.909
Neon Ne 10 20.183
Cadmium Cd 48 112.40
Calcium Ca 20 40.08 Neptunium Np 93 (237)
Californium Cf 98 (251) Nickel Ni 28 58.71
Carbon C 6 12.011 Niobium Nb 41 92.91
Cerium Ce 58 140.12 Nitrogen N 7 14.007
Cesium Cs 55 132.91 Nobelium No 102 (253)
Chlorine Cl 17 35.453 Osmium Os 76 190.2
4J | Chemical Sciences 2012

Oxygen O 8 15.9994 (B) High altitude and low air temperature


Palladium Pd 46 106.4 (C) Low altitude and low air temperature
Phosphorus P 15 30.974
(D) High altitude and high air temperature
Platinum Pt 78 195.09
Plutonium Pu 94 (242) 2. How many squares are there in this figure ?
Polonium Po 84 (210)
Potassium K 19 39.102
Praseodymium Pr 59 140.91
Promethium Pm 61 (147)
Protactinium Pa 91 (231)
Radium Ra 88 (226)
Radon Rn 86 (222)
Rhenium Re 75 186.23
Rhodium Rh 45 102.91
Rubidium Rb 37 85.47 (A) 9 (B) 14
Ruthenium Ru 44 101.1 (C) 15 (D) 17
Samarium Sm 62 150.35
Scandium Sc 21 44.95 3. A mountain road has 3 sections of different
Selenium Se 34 78.96 slopes as shown. What is the average slope m
Silicon Si 14 28.09 of the entire climb ?
Silver Ag 47 107.870
Sodium Na 11 22.9898
Strontium Sr 38 37.62
Sulfur S 16 32.064
Tantalum Ta 73 180.95
Technetium Tc 43 (99)
Tellurium Te 52 127.60
Terbium Tb 65 158.92
Thallium
Thorium
Tl
Th
81
90
204.37
232.04 (A) 1 (B) (13) < m < (12)
Thulium Tm 69 168.93
Tin Sn 50 118.69 (C) 1 < m < ⎯
√3 ⎛ 1 ⎞< m < 1
(D) ⎜ ⎟
Titanium Tl 22 47.90 ⎝√
⎯ 3⎠
Tungsten W 74 183.85
Uranium U 92 238.03 4. Which of the following graphs shows the
Vanadium V 23 50.94 concentration of a sugar solution as a function
Xenon Xe 54 131.30 of the cumulative amount of sugar added in
Ytterbium Yb 70 173.04 the process of preparing a saturated solution
Yttrium Y 35 88.91 (the temperature remaining constant) ?
Zinc Zn 30 65.37
Zirconium Zr 40 91.22
* Based on mass of C12 at 12.00… . The ratio of these
weights of those on the order chemical scale (in which (A) (B)
oxygen of natural isotopic composition was assigned a
mass of 16.0000…) is 1.000050. (Values in parentheses
represent the most stable known isotopes).

Part A
1. In still air, fragrance of a burning incense
(C) (D)
stick will be smelt by an observer quickest
when the experiment is carried out at—
(A) Low altitude and high air temperature
Chemical Sciences 2012 | 5J

5. There are sand-piles which are geometrically (A) Chewing gum is a pain killer
similar but of different heights. The ratio of (B) Chewing equilibrates pressure on both
the masses of the sand comprising two sides of the ear drum
randomly chosen piles will be equal to the
(C) Chewing gum closes the ear drum
ratio of the—
(A) Pile heights (D) Chewing distracts the person
(B) Squares of the pile heights 10. The reason why a lunar eclipse does not occur
(C) Cubes of the pile heights at every full moon is—
(D) Cube-roots of the pile heights (A) The position of the sun is not favourable
at all full moons
6. There are two identical vessels of volume V
each, one empty, and the other containing a (B) The orbital planes of the moon and that
block of wood of weight w. The vessels are of the earth are inclined to each other by
then filled with water up to the brim. The two a small angle
arrangements are shown as A and B in the (C) The shape of the earth is not a perfect
figure. If the density of water is ρ and g is the sphere
acceleration due to gravity, then— (D) The moon reflects only from one hemi-
sphere
11. A boy throws a stone vertically upwards with
a certain initial velocity. Which of the
following graphs depicts the velocity as a
function of time, if the acceleration due to
gravity is assumed to be uniform and
constant ?
(A) A and B have equal weights
(B) A is heavier than B by an amount w (A)
(C) A is heavier than B by an amount
Vρg – w
(D) B is heavier than A by an amount
Vρg – w
7. If the father has blood group O and the (B)
mother has blood group AB, what are the
possible blood groups of their children ?
(A) O, AB, A (B) A, B
(C) A, O (D) B, AB
8. Nuclei of 32P and 32S, accelerated through the (C)
same potential difference enter a uniform,
transverse magnetic field (Z = 15 for P and Z
= 16 for S). As they emerge from the
magnetic field—
(A) Both nuclei emerge undeflected
(B) 32P is deflected less than 32S (D)
(C) 32P is deflected more than 32S
(D) Both are equally deflected
9. A person chewing a bubble gum did not 12. A rigid uniform bar of a certain mass has two
experience ear pain in a jet plane while bobs of the same size, but with different
landing whereas another person not chewing a densities ρ and 2ρ suspended identically from
gum had ear pain. The reason could be— its ends.
6J | Chemical Sciences 2012

16. The 14C dating method is not usually used for


dating organic substances older than–60,000
years, because—
(A) Such objects rarely contain carbon
(B) Such objects accumulated 14C after their
When the bar is level on a fulcrum as shown formation
in the figure, d and d′ are related by— (C) In those times there was no production of
14C
(A) 2d = d′ (B) d > 2d′
(D) Most of the 14 C in the sample would
(C) d = 2d′ (D) d < 2d′
have decayed
13. There are two points A and A′ on the equator
17. A seismograph receives a S-wave 60 s after it
at longitudes 0° and 90°E, and two other
receives the P-wave. If the velocities of P-and
points B and B′ on the same longitudes,
S-waves are 7 km/s and 6 km/s respectively,
respectively, but at latitude 60°S. The
then the distance of the seismic focus from
distances (along the latitudes) between the
the seismograph is—
points A, A′ and B, B′ are related by—
(A) 2520 km (B) 42 km
(A) AA′ = BB′ (C) 7070 km (D) 72 km
(B) AA′ = 2BB′ 18. The decay of a radioactive isotope P produces
(C) AA′ = (√⎯ 3) BB′ a stable daughter isotope D. The ratio of the
number of atoms of D to the number of atoms
(D) AA′ = (√⎯ 2) BB′ of P after 2 half lives would be—
1 3
(A) (B)
4 4
14. (C) 3 (D) 2
19. The scatter plots represents the values
measured by two similar instruments. Point A
Water is flowing through a tube as shown. in the figure represents the true value. Which
The cross-sectional areas at A and C are of the following is a correct description of the
equal, and greater than the cross-sectional are quality of these measurements ?
at B. If the flow is steady, then the pressure
on the walls at B is—
(A) Less than that at A and that at C
(B) More than that at A and that at C
(C) Same as that at A and that at C Fig. 1 Fig. 2
(D) More than that at A but less than that at C (A) Fig. 1 : good accuracy, good precision
Fig. 2 : good accuracy, good precision
15. Match the two lists— (B) Fig. 1 : poor accuracy, poor precision
Raw Material Product Fig. 2 : good accuracy, poor precision
(a) Limestone 1. Porcelain (C) Fig. 1 : poor accuracy, good precision
(b) Gypsum 2. Glass Fig. 2 : poor accuracy, poor precision
(c) Silica sand 3. Plaster of Paris (D) Fig. 1 : poor accuracy, poor precision
Fig. 2 : poor accuracy, good precision
(d) Clay 4. Cement
20. Even though the concentration of CO2 is the
(a) (b) (c) (d)
same at sea level and at high altitude, the
(A) 1 2 3 4
photosynthetic rate is higher in a plant grown
(B) 4 3 2 1 at sea level than in a plant (of the same
(C) 1 3 4 2 species) grown at high altitude. The reason
(D) 4 1 3 2 for this is—
Chemical Sciences 2012 | 7J

(A) Light intensity is more at sea level 24. The total number of lone pairs of electrons in
(B) Temperature is lower at higher altitude I–3 is—
(C) Atmospheric pressure is higher at sea (A) Zero (B) Three
level (C) Six (D) Nine
(D) Relative humidity is higher at sea level 25. If Mössbauer spectrum of Fe(CO)5 is recorded
in the presence of a magnetic field, the
Part B original spectrum with two lines changes into
21. In the reactions (A) and (B), the one with—
nH2O + Cl– → [Cl (H2 O) n ]– …(A) (A) Three lines (B) Four lines
6H2O + Mg2+ → [Mg (H2O) 6 ]2+ …(B) (C) Five lines (D) Six lines
water behaves as— 26. The spectrophotometric response for the
(A) An acid in both (A) and (B) titration of a mixture of Fe3+ and Cu2+ ions
against EDTA is given below :
(B) An acid in (A) and a base in (B)
(C) A base in (A) and an acid in (B)
(D) A base in both (A) and (B)
22. The size of the d orbitals in Si, P, S and Cl
follows the order—
(A) Cl > S > P > Si
(B) Cl > P > S > Si
(C) P > S > Si > Cl The correct statement is—
(D) Si > P > S > Cl (A) Volume ab ≡ [Fe3+ ] and volume c d ≡
23. The correct structure of basic beryllium [Cu2+]
nitrate is— (B) Volume ab ≡ [Cu2+ ] and volume cd ≡
[Fe3+]
(C) Volume ab ≡ [Fe3+ ] and volume c d ≡
excess EDTA
(A)
(D) Volume ab ≡ [Cu2+ ] and volume cd ≡
excess EDTA
27. In ‘carbon-dating’ application of radioiso-
topes, 14C emits—
(A) β-particle (B) α-particle
(B)
(C) γ-radiation (D) Positron
28. The actual base pairs present in the double
helical structure of DNA containing adenine
(A), thymine (T), cytosine (C) and guanine
(G), are—
(C) (A) AG and CT
(B) AC and GT
(C) AG and AC
(D) AT and GC
29. The oxidation state of iron in met-hemoglobin
(D) is—
(A) Three (B) Two
(C) Four (D) Zero
8J | Chemical Sciences 2012

30. The reactions of Ni(CO)4 with the ligand L [L 37. The least basic among the following is—
= PMe3 or P(OMe)3 ] yields Ni(CO) 3 L. The (A) Al(OH)3 (B) La(OH) 3
reaction is— (C) Ce(OH)3 (D) Lu(OH)3
(A) Associative (B) Dissociative
38. For any operator A and its adjoint A+ , the
(C) Interchange (Ia) (D) Interchange (Id) incorrect statement is—
31. As a ligand Cl– is— (A) AA+ is hermitian
(A) Only a σ-donor (B) AA+ + A+A is hermitian
(B) Only a π-donor (C) A + A+ is hermitian
(C) Both a σ-donor and a π-donor (D) A – A+ is hermitian
(D) a σ-donor and a π-acceptor 39. For hydrogen-like atom with a nuclear charge
Z, the energy of orbital with principal
32. The correct d-electron configuration showing
quantum number ‘n’ follows the relation—
spin-orbit coupling is—
Z2
6
(A) t2g eg 2 6
(B) t2g eg 0
(A) En ∝ n2Z2 (B) En ∝ –
n
(C) t2g4 eg0 (D) t2g3 eg2 Z Z2
(C) En ∝ – (D) En ∝ – 2
33. The correct statement for the aggregating n n
nature of alkyl lithium (RLi) reagent is— 40. The average value of the radius <r> in the 1s
(A) The carbanion nucleophilicity increases state of the hydrogen atom is (ao is Bohr
with aggregation radius)—
(B) The observed aggregation arises from its (A) a0 (B) 1.5 a0
electron deficient nature
(C) 0.75 a0 (D) 0.5 a0
(C) Carbanion nucleophilicity does not
depend on aggregation 41. Among the following, the correct statement
(D) The extent of aggregation is maximum in is—
polar dative solvents (A) The number of irreducible representa-
34. For the reaction, trans-[IrCl(CO) (PPh3 )2] + tions is equal to classes of symmetry
operations
Cl2 → trans-[IrCl3 (CO) (PPh3 )2], the correct
(B) The number of irreducible representa-
observation is—
tions is equal to the order of the
(A) VCO (product) > V CO (reactant) symmetry point group
(B) VCO (product) < V CO (reactant) (C) The irreducible representations contained
(C) VCO (product) = V CO (reactant) in any point group are always of one
dimension
(D) VCO (product) = V CO (free CO) (D) A symmetry point group may not contain
35. The nucleophilic attack on olefins under mild a totally symmetric irreducible represen-
conditions— tation
(A) Is always facile 42. For a diatomic molecule AB, the energy for
(B) Is more facile than electrophilic attack on the rotational transition from J = 0 to J = 1
olefins state is 3.9 cm–1. The energy for the rotational
(C) Is facile for electron-rich olefins transition from J = 3 to J = 4 state would be—
(A) 3.9 cm –1 (B) 7.8 cm–1
(D) Requires activation by coordination to –1
metal (C) 11.7 cm (D) 15.6 cm–1

36. Among the following, the strongest oxidizing 43. For the vibrational Raman spectrum of a
homonuclear diatomic molecule, the selection
agent is—
rule under harmonic approximation is—
(A) [WO4 ]2– (B) [CrO 4 ]2– (A) Δv = 0 only (B) Δv = ± 1 only
(C) [MoO4]2– (D) [ReO 4 ]–1 (C) Δv = ± 2 only (D) Δv = 0, ±1
Chemical Sciences 2012 | 9J

44. With increase in temperature, the Gibbs free 49. For a reaction, the rate constant k at 27°C was
energy for the adsorption of a gas on to a found to be :
solid surface— k = 5·4 × 1011 e– 50
(A) Becomes more positive from a positive The activation energy of the reaction is—
value
(A) 50 J mol–1
(B) Becomes more negative from a positive
value (B) 415 J mol–l
(C) Becomes more positive from a negative (C) 15,000 J mol–1
value (D) 125,000 J mol–1
(D) Becomes more negative from a negative 50. During the addition polymerisation, the reac-
value tion proceeds via—
45. The vapour of a pure substance, when cooled (A) Step-growth process
under a pressure less than its triple-point (B) Free-radical chain reaction
pressure—
(C) Cascade process
(A) Liquefies
(B) Liquefies first and then solidifies (D) Addition reaction
(C) Solidifies directly 51. How many atoms are there in an element
(D) Remains unchanged packed in a fcc structure ?
46. The quantities, which are held fixed in a (A) 1 (B) 2
canonical ensemble are— (C) 4 (D) 8
(A) N, T and P 52. The structure obtained when all the tetrahe-
(B) V, T and N dral holes are occupied in a fcc structure, is of
(C) N, V and E the type—
(D) μ, V and P (A) NaCl (B) CsCl
47. The correct value of E0 of a half cell in the (C) CaF 2 (D) ZnS
following graph of E vs log m (molality) is—
53. Dispersion of a solid in a liquid, a liquid in a
gas and a liquid in a liquid are respectively
known as—
(A) Aerosol, emulsion, sol
(B) Sol, aerosol, emulsion
(C) Emulsion, sol, aerosol
(D) aerosol, sol, emulsion
(A) CC′/AC′ (B) AB′ 54. The data obtained from two sets of experi-
(C) BB′ (D) CC′ ments A and B have the following charac-
teristics :
48. One of the assumptions made in the conven-
tional activated complex theory is— Experiment A B
(A) Equilibrium is maintained between the Mean 50 units 100 units
reactants and the activated complex Standard deviation 2 units 2 units
(B) Equilibrium is maintained between the It may be concluded that—
reactants and the products (A) A is more precise than B
(C) Equilibrium is maintained between the
(B) A is less precise than B
products and the activated complex
(D) Equilibrium is maintained between the (C) A and B are of the same precision
reactants, the activated complex and the (D) Relative precision of A and B cannot be
products assessed
10J | Chemical Sciences 2012

55. The IUPAC name of the compound given (A) Enantiomers


below is— (B) Diastercomers
(C) Constitutional isomers
(D) Homomers
60. The major product formed in the following
concerted reaction is—

(A) ethyl (R)-2-methyl-4-oxocyclohex-2-


enecarboxylate
(B) ethyl (S)-2-methyl-4-oxocyclohex-2-
enecarboxylate (A)
(C) (R)-4-ethoxycarbonyl-3-methylcyclohex-
2-enone
(D) (S)-4-ethoxycarbonyl-3-methylcyclohex-
2-enone (B)
56. The major product formed in the following
reaction is—

(C)

(A) (D)

(B) 61. The structure of meso-tricarboxylic acid that


is formed on potassium permanganate oxida-
tion of abietic acid is—
(C)

(A)
(D)

57. The number of signals that appear in the


broad-band decoupled 13C NMR spectrum of
phenanthrene and anthracene, respectively, (B)
are—
(A) ten and four (B) ten and ten
(C) seven and four (D) seven and seven
58. The co-enzyme that is involved in the reduc-
tion of a double bond in fatty acid
biosynthesis is— (C)
(A) NADH (B) Biotin
(C) Pyridoxal (D) FADH2
59. Epoxidation of (R)-cyclohex-2-enol with
peracetic acid yields a 95 : 5 mixture of (D)
compounds A and B. Compounds A and B
are—
Chemical Sciences 2012 | 11J

62. The major product formed in the following


reaction is—
(D)

66. Among the following, the amino acid which


is basic in nature is—
(A) (B)
(A) Tyrosine (B) Asparagine
(C) Leucine (D) Arginine

(C) (D) 67. “Phosphorescence” is represented as—


(A) T1 → S0 + hv (B) T1 → S0 + Δ
63. The major product formed in the following (C) S 1 → S0 + hv (D) S 1 → T1 + Δ
reaction is— 68. Among the following diacids, the one that
forms an anhydride fastest on heating with
acetic anhydride is—

(A)

(A) (B)
(B)

(C) (C)
(D)

64. Among the following, the synthetic equiva- (D)


lent for acyl anion is—
(A) Nitroethane and base 69. The major product formed in the following
(B) α-chloroacrylonitrile reaction sequence is—
(C) Ethylmagnesium bromide
(D) Acetyl chloride and triethylamine
65. Among the following, the compound that
undergoes deprotection easily on treatment
with hydrogen in the presence of 10% Pd/C to (A) (B)
generate RNH2 is—

(A) (C) (D)

70. In the 400 MHz 1 H NMR spectrum, an


(B) organic compound exhibited a doublet. The
two lines of the doublet are at δ 2.35 and 2.38
ppm. The coupling constant (J) value is—
(C) (A) 3 Hz (B) 6 Hz
(C) 9 Hz (D) 12 Hz
12J | Chemical Sciences 2012

Part C 77. The decreasing order of dipole moment of


molecules is—
71. The strength of pn – d2 bonding in E—O (E =
(A) NF3 > NH3 > H2 O
Si, P, S and Cl) follows the order—
(A) Si—O > P—O > S—O > Cl—O (B) NH3 > NF3 > H2 O
(B) P—O > Si—O > S—O > Cl—O (C) H2O > NH3 > NF3
(C) S—O > Cl—O > P—O > Si—O (D) H2O > NF 3 > NH3
(D) Cl—O > S—O > P—O > Si—O 78. The cluster having arachno type structure is—
72. In the following reactions carried out in liquid (A) [Os5(CO) 16] (B) [Os3(CO) 12]
NH3, (C) [Ir4 (CO) 12] (D) [Rh6 (CO) 16]
Zn(NH2 )2 + 2KNH2 → K2 [Zn(NH2 )4]
79. The carbonyl resonance in 13C NMR spec-
K2 [Zn(NH2 )4] + 2NH4NO3 → trum of (η 5 –C 5 H5) Rh(CO)] 3 (103 Rh, nuclear
Zn(NH2 )2 + 2KNO3 + 4NH3 spin, I = 1/2, 100%) shows a triplet at – 65 °C
KNH 2 and NH4 NO3 act respectively as— owing to the presence of—
(A) Terminal CO (B) μ2 – CO
(A) Solvo-acid and solvo-base
(B) Solvo-base and solvo-acid (C) μ3 – CO (D) η5 – C5 H5
(C) Conjugate acid and conjugate base 80. Low oxidation state complexes are often air-
(D) Conjugate base and conjugate acid sensitive, but are rarely water sensitive
because—
73. The pair of lanthanides with the highest third-
ionization energy is— (A) Air is reducing in nature while water is
inert
(A) Eu, Gd (B) Eu, Yb
(B) Both air and water are oxidizing in nature
(C) Dy, Yb (D) Lu, Yb
(C) Both air and water are not π-acceptors
74. The lanthanide (III) ion having the highest (D) Complexes with low oxidation states will
partition coefficient between tri-n-butylphos- easily lose electrons to O2 but will not
phate and concentrated HNO3 is—
bind to a π-donor molecule like H2 O
(A) La (III) (B) Eu (III)
(C) Nd (III) (D) Lu (III) 81. The metal complex that exhibits a triplet as
well as a doublet in its 31P NMR spectrum
75. The quantitative determination of N2 H4 with is—
KIO3 proceeds in a mixture of H2 O/CCl4 as (A) mer–[IrCl3(PPh3)3]
follows : (B) trans–[IrCl(CO)(PPh3)2]
N2H4 + KIO3 + 2HCl ⎯→
(C) fac–[IrCl 3 (PPh3)3]
N2 + KCl + ICl + 3H 2 O
(D) [Ir(PPh3)4]+
The end point for the titrimetric reaction is—
(A) Consumption of N2H4 82. The complex that does not obey 18 electron
rule is—
(B) ICl formation
(A) [(η5 –C 5 H5)RuCl(CO)(PPh3)]
(C) Disappearance of the yellow colour due
to Cl2 in CCl4 layer (B) [W(CO)3(SiMe3 )(Cl)(NCMe)2]
(D) Disappearance of the red colour due to I2 (C) [IrCl3(PPh3)2(AsPh2 )] –
in CCl4 layer (D) [O s (N)Br2 (PMe3)(NMe2)] –
76. Among the halides, NCl3 (A), PCl3 (B) and 83. The number of spin-allowed ligand field
AsCl 3 (C), those which produce two different transitions for octahedral Ni (II) complexes
acids upon hydrolysis are— with 3A2g ground state is—
(A) A and B (B) A and C (A) Two (B) Three
(C) B and C (D) A, B and C (C) One (D) Four
Chemical Sciences 2012 | 13J

84. The correct structure of P4 S 3 is— (A) 5,000 years (B) 4,000 years
(C) 877 years (D) 617 years
89. The reaction 3[Rh4 (CO) 12 ] → 2 [Rh6 (CO) 16]
(A) + 4CO [25 °C, 500 atm CO] is—
(A) Exothermic as more metal-metal bonds
are formed
(B) Endothermic as stronger metal carbonyl
bonds are cleaved while weaker metal-
(B) metal bonds are formed
(C) Is entropically favorable but enthalpi-
cally unfavorable such that ΔG = 0
(D) Thermodynamically unfavorable (ΔG > 0)
90. A column is packed with 0.5 g of a strongly
(C) acidic ion exchange resin in H + form. A 1.0
M NaCl solution is passed through the
column until the eluant coming out becomes
neutral. The collected eluant is completely
neutralized by 17 ml of 0.5 M NaOH. The ion
exchange capacity of the resin is—
(A) 1.00 meq/g (B) 1.25 meq/g
(D) (C) 1.50 meq/g (D) 1.75 meq/g
91. The molar extraction coefficient of B (MW =
180) is 4 × 103 lit mol–1 cm–1 . One litre
85. The final product of the reaction [Mn(CO)6 ]+ solution of C1 which contains 0.1358 g
+ MeLi → is— pharmaceutical preparation of B, shows an
(A) [Mn(CO)6 ]– Me absorbance of 0.441 in a 1 cm quartz cell. The
percentage (w/w) of B in the pharmaceutical
(B) [Mn(CO)5 Me] preparation is—
(C) [Mn(CO)6 ] (A) 10.20 (B) 14.60
(D) [(MeCO) Mn(CO)5 ] (C) 20.40 (D) 29.12
86. The reaction that yields Li [AlH 4 ] is— 92. The changes (from A–D given below) which
occur when O2 binds to hemerythrin are—
(A) HCl (excess) + AlCl3 + Li →
1. One iron atom is oxidized
(B) H2 + Al + Li →
2. Both the iron atoms are oxidized
(C) LiH (excess) + AlCl3 → 3. O2 binds to one iron atom and is also
(D) LiH (excess) + Al → hydrogen bonded
87. The number of microstates for d 5 electron 4. O2 binds to both the iron atoms and is
configuration is— also hydrogen bonded
(A) 21 × 63 (B) 14 × 63 (A) 2 and 3 (B) 2 and 4
(C) 7 × 6 2 (D) 28 × 63 (C) 1 and 4 (D) 1 and 3
88. The carbon-14 activity of an old wood sample 93. In photosynthetic systems the redox metallo-
is found to be 14.2 disintegrations min –1 g–1. proteins involved in electron transfer are
Calculate age of old wood sample, if for a cytochrome (cyt b), cytochrome bf complex
fresh wood sample carbon–14 activity is 15.3 (cyt bf) and plastocyanin (PC). The pathway
disintegrations min– 1 g– 1 (t1/2 carbon–14 is of electron flow is—
5730 years), is— (A) PC → cyt b → cyt bf
14J | Chemical Sciences 2012

(B) cyt bf → cyt b → PC 2. Sum of the zeroth order and first order
(C) cyt b → cyt bf → PC corrections to the ground state energy is
(D) PC → cyt bf → cyt b ALWAYS greater than the exact ground
state energy.
94. The total numbers of fine and hyperfine EPR
3. Sum of the zeroth order and first order
lines expected for octahedral high-spin Mn
corrections to the ground state energy is
(II) complexes are respectively (I = 5/2 for
Mn)— less than the exact ground state energy.
(A) 3 and 30 (B) 5 and 33 From the following which one is correct ?
(C) 5 and 30 (D) 4 and 24 (A) Only 1 is true
95. The Mossbauer spectra of two iron complexes (B) Both 1 and 2 are true
are shown below. They may arise from (i) (C) Only 3 is true
high-spin iron (III), (ii) high-spin iron (II) and (D) Both 2 and 3 are true
(iii) low-spin iron (III)
99. Using Huckel molecular orbital approxi-
mation, the two roots of secular equation of
ethene are—
(A) α+√ ⎯⎯2β, α – ⎯√⎯2β
(B) α + β, α
(C) α + β, α – β
(D) α + 2β, α – 2β
100. For H2 molecule in the excited state σ1g σ1 z,
The correct matches of spectra (A) and (B)
the spin part of the triplet state with ms = 0
with the iron complexes are—
is proportional to—
(A) A with (i) and B with (ii)
(A) α(1) β(2)
(B) A with (ii) and B with (i)
(B) [α(1) β(2) – β(1) α(2)]
(C) A with (iii) and B with (ii)
(D) A with (ii) and B with (iii) (C) α(1) α(2)
(D) [α(1) β(2) + β(1) α(2)]
96. The probability of finding the particle in a one
dimensional box of length L in the region 101. A square pyramidal, MX4, molecule belongs
L 3L to C4 V point group. The symmetry opera-
between and for quantum number n = 1
4 4 tions are : E, 2C 4 , C2 , 2σ Y , and 2σΔ. The
is— trace for the reducible representation, when
1 1 1
(A) (B) + symmetry operations of C4, applied to MX 4 ,
2 2 π
is—
1 1 2
(C) – (D) (A) 5 1 1 1 3 (B) 1 1 1 1 1
2 π 3
(C) 5 1 1 1 1 (D) 4 1 1 1 3
97. A particle in three dimensional cubic box of
14h2 102. Character table of C2V , point group is
length L has energy of . The degeneracy
8mL2 C2 E C2 σV σV
of the state is—
A1 1 1 1 1 z
(A) 2 (B) 3
(C) 6 (D) 9 A2 1 1 –1 –1 –
98. The following are the three statements about B1 1 –1 1 –1 x
perturbation theory— B2 1 –1 –1 1 y
1. Second order perturbation correction to
the ground state energy is ALWAYS If the initial and final states belong to A1 and
negative. B 1 irreducible representations respectively,
Chemical Sciences 2012 | 15J

the allowed electronic transition from A1 to 108. Indicate which one of the following relations
B 1 is— is not correct—
∂T ∂ρ
(A) z-polarized
(C) x-polarized
(B) y-polarized
(D) x, z polarized
(A) – ( ) ( )
∂V S
=
∂S V
∂T ∂V
(B) – ( ) = ( )
103. Using cuvette of 0.5 cm path length, a 10 –4
M solution of a chromophore shows 50% ∂ρ ∂S
S P
transmittance at certain wavelength. The
∂S ∂ρ
molar extinction coefficient of the chromo- (C) – ( ) = – ( )
phore at this wavelength is (log 2 = 0.301)— ∂V T ∂T V

(A) 1500 M–1 cm–1 (B) 3010 M–1 cm–1 ∂S ∂V


(D) – ( ) = ( )
(C) 5000 M–1 cm–1 (D) 6020 M–1 cm–1 ∂ρ ∂T
T P

104. The set of allowed electronic transitions 109. The energy levels of the harmonic oscillator
among the following is— (neglecting zero point energy) are εn = nhv
1. 4 ∑ → 2 Π 2. 3 ∑ → 3 Π for n = 0, 1, 2, …, ∞. Assuming hv = k aT,
3. 1 Δ→ Δ 1 4. 2 Π → 2 Π the partition function is—
5. 2 ∑→ Δ 3
1
(A) e (B)
(A) 1, 2, 5 (B) 1, 3, 5 e
(C) 2, 3, 4 (D) 3, 4, 5 1 1
(C) 1 – (D)
105. The following data were obtained from the e 1
1–
vibrational fine structure in the vibronic e
spectrum of a diatomic molecule : 110. The correct entropy for 6 identical particles
ωe = 512 cm–1, ωexe ≡ 8 cm–1 with their occupation number |0, 1, 2, 3| in
where ωe is the energy associated with the four states is—
natural frequency of vibration and x e is the (A) ka ln 6 (B) ka ln 12
anharmonicity constant. The dissociation (C) kb ln 60 (D) kb ln 720
energy (De) of the molecule is— 111. The correct Nernst equation for the
(A) 4096 cm–1 (B) 6144 cm–1 concentration cell :
(C) 8192 cm –1 (D) 16384 cm–1 68
106. An ideal gas was subjected to a reversible, Pt |H2 (P) | HCl (a ± )1 | AgCl(S) | Ag Ag |
adiabatic expansion and then its initial AgCl(S) | HCl (a ± )2 | II2 (P) | Pt without
volume was restored by a reversible, isother- liquid junction would be—
mal compression. If ‘q’ denotes the heat
2RT (a )
added to the system and ‘w’ the work done (A) E = Ln ± 1
by the system, then— F (a±)2
(A) w < 0, q < 0 (B) w > 0, q < 0 (a )
RT
(C) w < 0, q > 0 (D) w > 0, q > 0 (B) E = Ln ± 2
F (a±)1
107. The gas phase reaction 2NO2(g) → N2 O4(g)
2RT (a )
is an exothermic process. In an equilibrium (C) E = Ln ± 2
mixture of NO2 and N2 O4, the decomposi- F (a±)1
tion of N2 O4 can be induced by— (a )
RT
(A) Lowering the temperature (D) E = Ln ± 2
2F (a±)1
(B) Increasing the pressure
(C) Introducing an inert gas at constant 112. Main assumption(s) involved in the deriva-
volume tion of Debye-Huckel equation is(are) the
(D) Introducing an inert gas at constant validity of—
pressure (A) Only Poisson equation
16J | Chemical Sciences 2012

(B) Poisson equation and Boltzmann distri- [h = 6.626 × 10–34 Js, c = 3 × 108 ms–1]
bution (A) 1000 nm, UV (B) 1000 nm, IR
(C) Poisson equation, Boltzmann distribu- (C) 500 nm, visible (D) 500 nm, FAR IR
tion and |±Zeφ| >> kBT
118. The lattice parameter of an element stabi-
(D) Poisson equation, Boltzmann distribu- lized in a fcc structure is 4.04 Å. The atomic
tion and |± Zeφ| << kBT radius of the element is—
113. In the base (OH – ) hydrolysis of a transition (A) 2.86 Å (B) 1.43 Å
metal complex [ML6 ]Z+, the slope between (C) 4.29 Å (D) 5.72 Å
log(k/k0 ) and √
⎯ 1 is found to be –2.1. The —
119. The number-average molar mass (Mn ) and
charge on the complex is— —
weight-average molar mass (Mw ) of a poly-
(A) +1 (B) +2
mer are obtained respectively by—
(C) +3 (D) +4
(A) Osmometry and light scattering mea-
114. The rate law for one of the mechanisms of surements
the pyrolysis of CH3 CHO at 520 °C and 0.2 (B) Osmometry and viscosity measure-
⎛ k ⎞1/2 ments
bar is Rate = – k2 ⎜ 1 ⎟ [CH3 CHO]3/2
⎝ 2kd⎠ (C) Light scattering and sedimentation mea-
surements
The overall activation energy Ea in terms of
(D) Viscosity and light scattering measure-
the rate law is— ments
(A) Ea (2) + Ea(1) + 2Ea(4)
120. Two data sets involving the same variables
1 X and Y are given below :
(B) Ea (2) + E a (1) – Ea (4)
2 X 4.1 4.2 4.3 4.4 4.5 4.6
1 1
(C) Ea (2) + E a (1) – E a (4) Y (Set A) 10.2 10.6 10.9 11.5 11.8 12.2
2 2
Y (Set B) 10.2 10.6 11.1 11.3 11.8 12.2
1 1
(D) Ea (2) – E a (1) + E a (4) If the slopes and intercepts of the regression
2 2
lines for the two sets are denoted by (mA,
115. In the Michaelis-Menten mechanism for en- mB) and (c A, cB), respectively, then—
zyme kinetics, t he expression obtained is
(A) mA > mB, cA > cB
V 104 V
= 1.4 × 1012 – (B) mA < mB, cA > cB
[E]0 [S] [E]0
The values of k 3 (k cat , mol L–1s–1 ) and K (C) mA > mB, cA < cB
(Michaelis constant, mol L–1), respectively, (D) mA < mB, cA < cB
are—
(A) 1.4 × 1012, 104 (B) 1.4 × 108 , 104 121. Compounds A and B exhibit two singlets,
(C) 1.4 × 108 , 10–4 (D) 1.4 × 1012, 10–4 each in their 1 H NMR spectra. The expected
chemical shifts are at δ—
116. The most used acid catalyst in oil industry
and the relevant process are respectively—
(A) Aluminophosphate and reforming
(B) Aluminosilicate and cracking
(C) Aluminosilicate and reforming
(D) Aluminophosphate and cracking
(A) 6.9 and 2.1 for A; 7.7 and 3.9 for B
117. The wavelength and the spectral region for a
(B) 7.7 and 3.9 for A; 6.9 and 2.1 for B
single electron transfer across the band gap
in a semiconductor (Eg = 1·98 × 10–19 J) (C) 6.9 and 3.9 for A; 7·7 and 2.1 for B
are— (D) 7.7 and 2.1 for A; 6.9 and 3.9 for B
Chemical Sciences 2012 | 17J

122. In the following reaction sequence, the (A) Photochemical electrocyclic disrotatory
major products A and B are— ring opening; and thermal antarafacial
[1, 7]-H shift
(B) Photochemical electrocyclic conrota-
tory ring opening; and thermal antara-
facial [1, 7]-H shift
(C) Thermal electrocyclic conrotatory ring
opening; and photochemical suprafacial
[1, 7]-H shift
(A) (D) Thermal electrocyclic disrotatory ring
opening; and thermal suprafacial [1, 7]-
H shift
125. The intermediate A and the major product B
in the following reaction are—
(B)

(C)
(A)

(B)
(D)
123. The structure of the tricyclic compound
(C)
formed in the following two steps sequence
is—

(D)

126. For the following two reactions A and B, the


correct statement is—

(A) (B)

(C) (D)

124. The two steps conversion of 7-dehydro- (A)


cholesterol to vitamin D3 proceeds through—

(B)
18J | Chemical Sciences 2012

130. Structure of the starting material A in the


(C) following photochemical Norrish reaction,
is—

(D)

127. The major compound B formed in the


reaction sequence given below exhibited a
carbonyl absorption band at 1770 cm–1 in
the IR spectrum. The structures A and B (A) (B)
are—

(C) (D)

(A) 131. Considering the following reaction, among


a–c, the correct statements are—

(B)

(C)
1. The carbonyl group has enantiotopic
faces;
(D) 2. The hydride attack is re-facial;
3. It is a diastereoselective reduction
128. Consider the following reaction sequence (A) 1 and 2 only (B) 1 and 3 only
starting with monoterpene α-pinene. Identify
the correct statement— (C) 2 and 3 only (D) 1, 2 and 3
132. The major product formed in the following
reaction sequence is—
(A) A has a disubstituted double bond; B
and C are dicarboxylic acids
(B) A has a trisubstituted double bond; B is
a methyl ketone; and C is a dicar-
boxylic acid
(C) A has a disubstituted double bond; B is (A) (B)
a methyl ketone; and C is a dicaroxylic
acid
(D) A has an exocyclic double bond; B and
C are monocarboxylic acids (C) (D)
129. The major product formed when (3R, 4S)-3,
4-dimethylhexa-1, 5-diene is heated at 240 133. The major product formed in the following
°C is— reaction sequence is—
(A) (2Z,6Z)-octa-2,6-diene
(B) (2E,6E)-octa-2,6-diene
(C) (2E,6Z)-octa-2,6-diene
(D) (3Z,5E)-octa-3,5-diene
Chemical Sciences 2012 | 19J

(A) (C)

(B)

(D)
(C)

136. The major product formed in the following


reaction is—
(D)

134. Match the following—


Compound
(a) Acetic acid
(b) Acetonitrile (A) (B)
(c) Acetone
(d) Carbon tetrachloride
13C NMR chemical shift (δδ ppm)
1. 95 2. 115 (C) (D)
3. 175 4. 205
(a) (b) (c) (d) 137. The reagents A and B in the following
(A) 3 2 4 1 reactions are—
(B) 3 4 1 2
(C) 1 2 4 3
(D) 3 1 2 4
135. The major products A and B in the
following reaction sequence are— (A) A = CH2 I2,Zn–Cu; B = Me4S +I, NaH
(B) A = CH2 I2,Zn–Cu; B = Me3S +(O)I–,
NaH
(C) A = Me3 S +I–,NaH; B = Me3S +(O)I–,
NaH
(D) A = Me3 S +(O)I–,NaH; B = CH2 I2,
Zn–Cu
(A)
138. The major products A and B formed in the
following reaction sequence are—

(B) (A)
20J | Chemical Sciences 2012

141. The major product formed in the following


(B) reaction is—

(C)

(D)
(A)

139. The major products A and B formed in the


following reaction sequence are— (B)

(C)
(A)

(D)
(B)

142. The correct sequence of reagents for effec-


ting the following conversion is—

(C)

(D) (A) (a) (CH2 OH)2 , PTSA, Δ;

(b) CO2Ti Cl _ AlMe2 (Tebbe’s

140. The correct reagent combination/reaction reagent); (c) H3 O+; (d) KOH
sequence for effecting the following conver- (B) (a) (CH 2 OH)2 , PTSA, Δ; (b) Ph3 P–CH2;
sion is— (c) H3 O+; (d) KOH
(C) (a) CO2Ti Cl _ AlMe2 (Tebbe’s
reagent);
(A) (a) Me3 SiCH2OMe, BuLi; (b) H3 O+; (b) H3O+; (c) KOH
(c) NaBH4 , MeOH (D) (a) Ph3 P–CH2; (b) H 3 O+; (c) KOH
(B) (a) Ph 3 P +CH2 OMeCl– , BuLi; (b) H 3 O+; 143. The major products A and B formed in the
(c) NaBH4 , MeOH following reaction sequence are—
(C) (a) NH2 NHTs; (b) NaOEt; (c) ClCOOEt
(D) (a) NH2 NHTs; (b) 2eq. nBuLi;
(c) HCHO
Chemical Sciences 2012 | 21J

Answers with Explanations


(A) 1. (D)
2. (C) Total number of squares are 15.
3. (D) 4. (B) 5. (C) 6. (A) 7. (B)
(B) 8. (B)
9. (B) Chewing equilibrates the pressure on both
side of the ear drum.
(C) 10. (A) 11. (A) 12. (D) 13. (B) 14. (A)
15. (B) In the most general sense of the word, a
cement is a binder, a substance that sets and
hardens independently, and can bind other
(D) materials together. The word “cement” traces
to the Romans, who used the term opus
144. The reagent A required and the major caementicium to describe masonary resem-
product B formed in the following reaction bling modern concrete that was made from
sequence are— crushed rock with burnt lime as binder.
16. (D) Radiocarbon dating (usually referred to as
simply carbon dating) is a radiometric dating
method that uses the naturally occurring
radioisotope carbon-14 ( 14C) to estimate the
age of carbon-bearing materials up to about
(A) A = CH2 Br2 and KOtBu 58,000 to 60,000 years. Raw i.e., uncali-
brated, radiocarbon ages are usually reported
in radiocarbon years “Before Present” (BP),
with “present” defined as AD 1950. Such raw
(B) A = CH2 Br2 and KOtBu ages can be calibrated to give calendar dates.
One of the most frequent uses of radiocarbon
dating is to estimate the age of organic
remains from archaeological sites. When
plants fix atmospheric carbon dioxide (CO2 )
(C) A = CHBr3 and KOtBu into organic material during photosynthesis
they incorporate a quantity of 14 C that
approximately matches the level of this
isotope in the atmosphere (a small difference
(D) A = CHBr3 and KOtBu occurs because of isotope fractionation, but
this is corrected after laboratory analysis.
17. (A) 18. (C) 19. (C) 20. (C) 21. (B)
145. Among the choices, the correct statement for
A formed in the following reaction— 22. (D) 23. (A) 24. (D) 25. (D) 26. (C)
27. (A) Carbon 14, unlike carbon 12 and carbon
13, is radioactive; meaning that over time the
carbon 14 atoms will decay. When the isotope
carbon 14 decays it gives off a beta particle
and in doing so becomes nitrogen 14. The
(A) A is a single enantiomer amount of carbon 12 and carbon 13, however,
(B) A is a racemic mixture remains constant.
(C) A is a mixture of two diastereomers 28. (D) The base pairing-rules for DNA are that,
(D) A is a mixture of two epimers only the Nitrogen Bases of DNA which are;
22J | Chemical Sciences 2012

Adenine “A”-which only pairs with-Thymine triple point involving two different fluid
“T”, and Cytosine “C”-which only pairs with- phases. In general, for a system with p
Guanine “G” can only pair to one another possible phases, there are triple points.
within that sequence.
29. (A) 30. (B) () p 1
3 = 6 p (p – 1) (p – 2)
31. (C) In coordination chemistry, a ligand is an 46. (B) 47. (C)
ion or molecule that binds to a central metal 48. (A) The key assumption of Activated
atom to form a coordination complex. The Complex Theory (ACT), that the AC is in
bonding between metal and ligand generally thermodynamic equilibrium with the reac-
involves formal donation of one or more of tants, needs to be reconsidered. This is
the ligand’s electron deficient pairs. The because the formation of the AC is slower
nature of metal-ligand bonding can range than its collapse to product. However, this can
from covalent to ionic. Furthermore, the be remedied by assuming that the AC is
metal-ligand bond order can range from one formed in a rapid pre-equilibrium as a
to three. Ligands are viewed as Lewis bases, thermally activated species, which collapses
although rare cases are known involving to products in a slow step involving the
Lewis acidic “ligands”. diffusion of another AC molecule (or solvent
32. (C) 33. (B) 34. (A) 35. (D) in the case of a unimolecular reaction).
36. (B) Chromate salts contain the chromate 49. (D)
anion, CrO 4 2–. Dichromate salts contain the 50. (B) Additional Polymerization : A chemical
dichromate anion, Cr2 O72– . They are oxya- reaction in which simple molecules (mono-
nions of chromium in the oxidation state +6. mers) are added to each other to form long-
They are moderately strong oxidizing agents. chain molecules (polymers) without by
products. The molecules of the monomer join
37. (A) For hydrogen and other nuclei stripped to together to form a polymeric product in which
one electron, the energy depends only upon the molecular formula of the repeating unit is
the principal quantum number n. identical with that of the monomer. The
–13.6 Z2 eV molecular weight of the polymer so formed is
En =
n2 thus the total of the molecular weights of all
This fits the hydrogen spectrum unless you of the combined monomer units.
take a high resolution look at fine structure 51. (C) First, consider the atoms at the 8 corners.
where the electron spin and orbital quantum Each corner atom is occupied by 8 unit cells,
numbers are involved. At even higher resolu- which means 1/8 of the atom is actually in
tions, there is a tiny dependence upon the one unit cell. There are 8 of these corner
orbital quantum number in the Lamb shift.
38. (D) 39. (D) 40. (B) 41. (A) 42. (D)
atoms, so 8 () 1
8
= 1.

Next, consider the atoms at the 6 faces of the


43. (B) 44. (C) unit cell. Each face atom is occupied by 2 unit
45. (C) In thermodynamics, the triple point of a cells, which means 1/2 of the atom is in one
substance is the temperature and pressure at unit cell. There are 6 of these face atoms, so 6
which the three phases (gas, liquid and solid) (1/2) = 3.
of that substance coexist in thermodynamic Add up the two values… 1 + 3 = 4
equilibrium. For example, the triple point of
So, there are 4 atoms contained in a face-
mercury occurs at a temperature of
centered.
–38.8344°C and a pressure of 0.2 mPa .
52. (C) Liquids, solids and gases all may be
In addition to the triple point between solid, mixed to form colloidal dispersions.
liquid and gas, there can be triple points
involving more than one solid phase, for Aerosols : solid or liquid particles in a gas.
substances with multiple polymorphs. Examples : Smoke is a solid in a gas. Fog is
Helium-4 is a special case that presents a a liquid in a gas.
Chemical Sciences 2012 | 23J

Sols : solid particles in a liquid. Example : 58. (D)


Milk of Magnesia is a sol with solid magne- 59. (B) Due to generation of two chiral centers by
sium hydroxide in water. epoxidation it will generate diastereomers. A
Emulsions : liquid particles in liquid. diastereomer is simply any stereoisomer that
Example : Mayonnaise is oil in water. is not an enantiomer. Technically, cis-trans
isomers are diastereomers. However, typically
Gels : liquids in solid. Examples : gelatin is
the term is reserved for stereoisomers that
protein in water. Quicksand is sand in water.
differ at some but not all stereocenters.
53. (B) 54. (B)
55. (B) As ester is first priority group compared
to ketone and alkene, it will go in first priority
and after applying Cahn-Prelog Rule (A
molecule may contain any number of stereo
centers and any number of double bonds, and
each gives rise to two possible configurations)
option 2 is correct.
56. (D) Due to the generation of allelic and Diastereomers formed by inverting some but not all
stereocenters
benzylic carbonium that is resonance stabi-
lized so it will attack on the bromo allyl in 60. (A) 61. (B)
very facile condition that’s why product 4 is 62. (B) Due to the cis bromonium addition and
formed as major product. after that SN2 reaction of OMe - gives the
57. (C) In the phenathrene there are 7 types of product two due to oxygen lone pair.
magnetically equivalent while in anthracene 63. (D) 64. (A) 65. (B) 66. (D) 67. (A)
there are 4 types of magnetically equivalent
carbons. Phenanthrene is a polycyclic 68. (A)
aromatic hydrocarbon composed of three 69. (C) Due to Birch reduction first intermediate
fused benzene rings. The name phenanthrene is the one which prefers the formation of
is a composite of phenyl and anthracene. In product 3 as para directing strength of methyl
its pure form, it is found in cigarette smoke group is more than that of methoxy.
and is a known irritant, photo-sensitizing skin
to light. Phenanthrene appears as a white 70. (D) 71. (D) 72. (A) 73. (B) 74. (D)
powder having blue fluorescence. 75. (D) 76. (C) 77. (C) 78. (B) 79. (B)
80. (D) 81. (A) 82. (D) 83. (B) 84. (A)
85. (D) 86. (C) 87. (C) 88. (D) 89. (B)
90. (D) 91. (A) 92. (A)
93. (C)
24J | Chemical Sciences 2012

94. (C) 95. (B) 96. (B) 97. (C) 98. (B) 133. (A) In case of DIBAL reaction opening of
99. (C) 100. (D) 101. (A) 102. (C) 103. (D) lactam took place because here is only 1
104. (C) 105. (C) 106. (B) 107. (D) 108. (B) equation DIBAL is used so further reduction
109. (D) 110. (C) of aldehyde is not possible second reaction
111. (B) The two (ultimately equivalent) equa- is Grignard on aldehyde group which give
tions for these two cases (half-cell, full cell) secondary allelic alcohol which on oxidation
are as follows : with PCC gives ketone.
RT a 134. (A) 135. (A) 136. (B)
Ered = Eθ red – ln Red (half-cell reduction
zF aOx 137. (B) The alpha beta unsaturated ketones on
potential) reaction with Ch2I2 Zn–Cl give cyclopropyl
RT on alpha beta unsaturated bond on the other
Ecell = Eθ cell – ln Q (total cell potential) hand alpha beta unsat alcohols on reaction
zF
with tetramethyl sulphonium iodide in pre-
112. (D) 113. (B) 114. (C) 115. (C) 116. (B)
sence of hydride gives cyclopropane at alpha
117. (B) 118. (B) 119. (A) 120. (C) 121. (A) beta unsat positions.
122. (B) 123. (D) 124. (B) 125. (D) 126. (B)
138. (C) Reaction with LAH gives cis ally
127. (B) alcohol which on sharpless epoxidation
128. (B) A has a trisubstituted double bond which gives up epoxide.
is endocyclic and after KMnO4 oxidation it
139. (B) The hydroboration of alkynes gives
gives B a methyl ketone and after the trans hydroborene due to steric hindrance of
reaction with NaOH and Br2 gives bulkier cyclopropyl and hydroborating group
dicarboxylic acid C. which on further substitution gives same
129. (C) side conjugated alkene.
130. (C) This is the Norrish type reaction in 140. (D) A simple stark examine reaction after
which abstraction of gama hydrogen takes that anionic attack on formaldehyde gives
place by the formation of double bond that the alcohol.
gives product 3.
141. (D) 142. (A)
131. (C) The hydride ion attacks from the lower
face of the plane that is called re facial and it 143. (D) Abstraction of H from indolic N attacks
reduces the ketone that results in another on oxirane giving product 4A which after
chiral centre so it will be diastereoselective addition with benzaldehyde gives product
reduction. 4B.
132. (D) 144. (C) 145. (B)
Chemical Sciences
CSIR UGC-NET/JRF Exam.
Solved Paper
December 2012
Chemical Science
Time : 3 Hours] [Maximum Marks : 200

Direction List of the Atomic Weights of the


1. This Test Booklet contains one hundred and Elements
forty five (20 Part ‘A’ + 50 Part ‘B’ + 75 Part Atomic Atomic
Element Symbol
‘C’) Multiple Choice Questions (MCQs). You Number Weight
are required to answer a maximum of 15, 35 Actinium Ac 89 (227)
and 25 questions from part ‘A’, ‘B’ and ‘C’ Aluminium Al 13 26.98
respectively. If more than required number of Americium Am 95 (243)
questions are answered, only first 15, 35 and Antimony Sb 51 121.75
25 questions in Parts ‘A’, ‘B’ and ‘C’ Argon Ar 18 39.948
respectively, will be taken up for evaluation. Arsenic As 33 74.92
Astatine At 85 (210)
2. Each question in Part ‘A’ and ‘B’ carries 2 Barium Ba 56 137.34
marks and Part ‘C’ questions carry 4 marks Berkelium Bk 97 (249)
each respectively. There will be negative Beryllium Be 4 9.012
marking @ 25% for each wrong answer. Bismuth Bi 83 208.98
3. Below each question in Part ‘A’, ‘B’ and ‘C’ Boron B 5 10.81
four alternatives or responses are given. Only Bromine Br 35 79.909
Cadmium Cd 48 112.40
one of these alternatives is the ‘correct’ option
Calcium Ca 20 40.08
to the question. You have to find, for each
Californium Cf 98 (251)
question, the correct or the best answer. Carbon C 6 12.011
Useful Fundamental Constants Cerium Ce 58 140.12
m Mass of electron 9.11 × 10–31 kg Cesium Cs 55 132.91
Chlorine Cl 17 35.453
h Planck’s constant 6.63 × 10–34 J-sec Chromium Cr 24 52.00
e Charge of electron 1.6 × 10–19 C Cobalt Co 27 58.93
Copper Cu 29 63.54
K Boltzmann constant 1.38 × 10–23 J/K Curium Cm 96 (247)
c Velocity of Light 3.0 × 108 m/sec Dysprosium Dy 66 162.50
Einsteinium Es 99 (254)
IeV 1.6 × 10–19 J
Erbium Er 68 167.26
amu 1.67 × 10–27 kg Europium Eu 63 151.96
G 6.67 × 10–11 Nm2 kg–2 Fermium Fm 100 (253)
Fluorine F 9 19.00
Ry Rydberg constant 1.097 × 107 m–1 Francium Fr 87 (223)
NA Avogadro number 6.023 × 1023 mole–1 Gadolinium Gd 64 157.25
ε0 8.854 × 10–12 Fm–1 Gallium Ga 31 69.72
Germanium Ge 32 72.59
μ0 4π × 10–7 Hm–1 Gold Au 79 196.97
R Molar Gas constant 8.314 JK–1 mole–1 Hafnium Hf 72 178.49
4 | CSIR–Chemical Sciences (Dec.–12)

Helium He 2 4.003 Tellurium Te 52 127.60


Holmium Ho 67 164.93 Terbium Tb 65 158.92
Hydrogen H 1 1.0080 Thallium Tl 81 204.37
Indium In 49 114.82 Thorium Th 90 232.04
Iodine I 53 126.90 Thulium Tm 69 168.93
Iridium Ir 77 192.2 Tin Sn 50 118.69
Iron Fe 26 55.85 Titanium Ti 22 47.90
Krypton Kr 36 83.80 Tungsten W 74 183.85
Lanthanum La 57 138.91 Uranium U 92 238.03
Lawrencium Lr 103 (257) Vanadium V 23 50.94
Lead Pb 82 207.19 Xenon Xe 54 131.30
Lithium Li 3 6.939 Ytterbium Yb 70 173.04
Lutetium Lu 71 174.97 Yttrium Y 39 88.91
Magnesium Mg 12 24.312 Zinc Zn 30 65.37
Manganese Mn 25 54.94 Zirconium Zr 40 91.22
Mendelevium Md 101 (256) 12
* Based on mass of C at 12.00… . The ratio of these
Mercury Hg 80 200.59 weights of those on the order chemical scale (in which
Molybdenum Mo 42 95.94 oxygen of natural isotopic composition was assigned a
Neodymium Nd 60 144.24 mass of 16.0000…) is 1.000050. (Values in parentheses
Neon Ne 10 20.183 represent the most stable known isotopes).
Neptunium Np 93 (237) Part–A
Nickel Ni 28 58.71
1. Which of the following numbers is the
Niobium Nb 41 92.91
largest ?
Nitrogen N 7 14.007 4 3 4 2 3 2
23 , 24 ,3 2 , 34 , 42 , 43 .
Nobelium No 102 (253) 4 2
(A) 23 (B) 34
Osmium Os 76 190.2 (C) 4 3 2
(D) 42 3
Oxygen O 8 15.9994 2. The cube ABCDEFGH in the figure has each
Palladium Pd 46 106.4 edge equal to a. The area of the triangle with
Phosphorus P 15 30.974 vertices at A, C and F is—
Platinum Pt 78 195.09
Plutonium Pu 94 (242)
Polonium Po 84 (210)
Potassium K 19 39.102
Praseodymium Pr 59 140.91
Promethium Pm 61 (147)
Protactinium Pa 91 (231)
Radium Ra 88 (226) ⎯ 3 a2
√ ⎯ 3 a2

Radon Rn 86 (222) (A) (B)
4 2
Rhenium Re 75 186.23
Rhodium Rh 45 102.91 (C) √⎯3 a2 (D) 2√
⎯ 3 a2
Rubidium Rb 37 85.47 3. What is the number of distinct arrangements
Ruthenium Ru 44 101.1 of the letters of the word UGC-CSIR so that
Samarium Sm 62 150.35 U and I cannot come together ?
Scandium Sc 21 44.95 (A) 2520 (B) 720
Selenium Se 34 78.96
(C) 1520 (D) 1800
Silicon Si 14 28.09
Silver Ag 47 107.870 4. Suppose the sum of the seven positive
Sodium Na 11 22.9898 numbers is 21. What is the minimum possible
Strontium Sr 38 87.62 value of the average of the squares of these
Sulphur S 16 32.064 numbers ?
Tantalum Ta 73 180.95 (A) 63 (B) 21
Technetium Tc 43 (99) (C) 9 (D) 7
CSIR–Chemical Sciences (Dec.–12) | 5

113 + 2 13 + 3 13 + ··· + 10013 10. Amar, Akbar and Anthony are three friends,
5. Let A = ,
100 one of whom is a doctor, another is an
113 + 313 + 5 13 + ··· + 9913 engineer and the third is a professor. Amar is
B= , not an engineer. Akbar is the shortest. The
50
tallest person is a doctor. The engineer’s
213 + 413 + 6 13 + ··· + 10013 height is the geometric mean of the heights of
C= ,
50 the other two. Then which of the following is
Which of the following is true ? true ?
(A) B < C < A (B) A < B < C (A) Amar is a doctor and he is the tallest
(C) B < A < C (D) C < A < B (B) Akbar is a professor and he is tallest
(C) Anthony is an engineer and he is shortest
6. A circle of radius 5 units in the XY plane has
its centre in the first quadrant, touches the (D) Anthony is a doctor and he is the tallest
x-axis and has a chord of length 6 units on the 11. If 100 cats catch 100 mice in 100 minutes,
y-axis. The coordinate of its centre are— then how long will it take for 7 cats to catch
(A) (4, 6) (B) (3, 5) 7 mice ?
(C) (5, 4) (D) (4, 5) (A) 100/7 minutes (B) 100 minutes
(C) 49/100 minutes (D) 7 minutes
7. A wire of length 6 metres is used to make a
tetrahedron of each edge 1 metre, using only 12. What does this digram demonstrate ?
one strand of wire for each edge. The
minimun number of times the wire has to be
cut is—

n · (n + 1)
(A) 1 + 2 + 3 + ··· + n =
2
(A) 2 (B) 3 n· (n + 1)·(2n + 1)
(C) 1 (D) 0 (B) 12 + 22 + 32 + ··· + n2 =
2
8. If the sum of the next two terms of the series (C) 1 + 3 + ··· + (2n + = –1) = n2
below is x, what is the value of log2x ? 2·n(n + 1)·(2n + 1)
(D) 22 + 4 2 + ··· + (2n)2 =
2, – 4, 8, – 16, 32, – 64, 128, ………… 3
(A) 128 (B) 10 13. Suppose there are socks of N different colours
(C) 256 (D) 8 in a box. If you take out one sock at a time,
what is the maximum number of socks that
9. you have to take out before a matching pair is
found ? Assume that N is an even number.
(A) N (B) N + 1
(C) N – 1 (D) N/2
14. At what time after 4 O’ clock, the hour and
the minute hands will lie opposite to each
A conical vessel with semi-vertical angle 30° other ?
and height 10·5 cm has a thin lid. A sphere (A) 4 – 50′ – 31′′ (B) 4 – 52′ – 51′′
kept inside it touches the lid. The radius of (C) 4 – 53′ – 23′′ (D) 4 – 54′ – 33′′
the sphere (in cm) is— 15. Which of the following curves just touches
(A) 3·5 (B) 5 the x-axis ?
(C) 6·5 (D) 7 (A) y = x 2 – x + 1
6 | CSIR–Chemical Sciences (Dec.–12)

(B) y = x 2 – 2x + 2 Part–B
(C) y = x 2 – 10x + 25 21. For an odd nucleon in ‘g’ nuclear orbital and
(D) y = x 2 – 7x + 12 parallel to I, spin and parity are—
16. (A) 9/2 and (+) (B) 7/2 and (+)
(C) 9/2 and (–) (D) 7/2 and (–)
22. For the deposition of Pb by electroplating, the
best suited compound among the following
is—
(A) PbCl2 (B) PbSO4
(C) Pb(Et)4 (D) Pb(BF4 )2
If AB is paralleled to CD and AO = 2OD,
then the area of triangle OAB is bigger than 23. Appropriate reasons for the deviation form
the area of triangle OCD by a factor of— the Beer’s law among the following are—
(A) 2 (B) 3 1. Monochromaticity of light
(C) 4 (D) 8 2. Very high concentration of analyte
3. Association of analyte
17.
4. Dissociation of analyte
(A) 1, 2 and 4 (B) 2, 3 and 4
(C) 1, 3 and 4 (D) 1, 2 and 3
24. Which one of the following shows the highest
solubility in hot concentrated aqueous NaOH?
A semi-circular arch of radius R has a vertical (A) La(OH) 3 (B) Nd(OH)3
pole put on the ground together with one of
(C) Sm(OH)3 (D) Lu(OH)3
its legs. An ant on the top of the arch finds the
angular height of the tip of the pole to be 45°. 25. In the vibrational spectrum of CO2 , the
The height of the pole is— number of fundamental vibrational modes
(A) √ 2R
⎯ (B) √ 3R
⎯ common in both infrared and Raman are—
(A) three (B) two
(C) √
⎯ 4R (D) √
⎯ 5R (C) one (D) zero
18. Suppose we make N identical smaller spheres 26. The light pink colour of [Co(H2O) 6 ]2+ and the
from a big sphere. The total surface area of
deep blue colour of [CoCl4]2– are due to—
the smaller spheres is X times the total surface
area of the big sphere, where X is— (A) MLCT transition in the first and d-d
transition in the second
(A) √
⎯N (B) 1 (B) LMCT transitions in both
(C) N1/3 (D) N3 (C) d-d transitions in both
19. What is the next number in the sequence 24, (D) d-d transition in the first and MLCT
30, 33, 39, 51, ········ ? transition in the second
(A) 57 (B) 69
27. In Mo2 (S2 )6]2– cluster the number of bridging
(C) 54 (D) 81 S 2 2– and coordination number of Mo
20. Four lines are drawn on a plane with no two respectively, are—
parallel and no three concurrent. Lines are (A) 2 and 8 (B) 2 and 6
drawn joining the points of intersection of the (C) 1 and 8 (D) 1 and 6
previous four lines. The number of new lines
obtained this way is— 28. 1 H NMR spectrum of HD would show—
(A) 3 (B) 5 (A) a singlet
(C) 12 (D) 2 (B) a doublet
CSIR–Chemical Sciences (Dec.–12) | 7

(C) a triplet with intensity ratio : 1 : 2 :1 37. Degradation of penicillin G—


(D) a triplet with intensity ratio 1 : 1 : 1
29. The number of possible isomers of
[Ru(PPh3)2(acac)2] (acac = acetylacetonate)
is—
(A) 2 (B) 3 gives penicillamine that can utilize nitrogen,
(C) 4 (D) 5 oxygen or sulfur atoms as donors to bind with
lead(II), mercury(II) or copper(II). The
30. The total number of Cu – O bonds present in structure of penicillamine is—
the crystalline copper (II) acetate monohy-
drate is—
(A) 10 (B) 6 (A)
(C) 8 (D) 4
31. The electronegativity difference is the highest
for the pair— (B)
(A) Li, Cl (B) K, F
(C) Na, Cl (D) Li, F (C)
2– ,
32. Which ones among CO3 SO3 , XeO3 and
NO3– have planar structure ?
(A) CO3 2–, SO3 and XeO3 (D)
(B) SO3 , XeO3 and NO3 –
(C) CO3 2–, XeO3 and NO3 – 38. The molecule that has an S6 symmetry
(D) CO3 2–, SO3 and NO3 – element is—
33. The substitution of η5 – Cp group with nitric (A) B 2 H6 (B) CH4
oxide is the easiest for— (C) PH5 (D) SF6
(A) η5 – Cp 2 Fe (B) η5 – Cp 2 CoCl
39. The electric dipole allowed transition in a d2
(C) η5 – Cp 2 Ni (D) η5 – Cp 2 Co atomic system is—
34. The molecule (A) 3 F → 1D (B) 3 F → 1P
(C) F → D
3 3 (D) 3 F → 3P
40. When a hydrogen atom is placed in an electric
field along the y-axis, the orbital that mixes
obeys 18 e rule. The two ‘M’ satisfying the
most with the ground state 1s orbital is—
condition are—
(A) Cr, Re+ (B) Mo, V (A) 2s (B) 2px
(C) V, Re+ (D) Cr, V (C) 2py (D) 2pz
35. The correct set of the biologically essential 41. For water, ΔH vap ~
~ 41 kJ mol–1. The molar
elements is— entropy of vaporization at 1 atm pressure is
(A) Fe, Mo, Cu, Zn (B) Fe, Cu, Co, Ru approximately—
(C) Cu, Mn, Zn, Ag (D) Fe, Ru, Zn, Mg (A) 410 J K–1 mol–1 (B) 110 J K–1 mol–1
(C) 41 J K–1 mol–1 (D) 11 J K–1 mol–1
36. The number of lines exhibited by a high
resolution EPR spectrum of the species, 42. If A and B are non-commuting hermitian
[Cu(ethylenediamine2]2+ is [Nuclear spin (I) operators, all eigenvalues of the operator given
of Cu = 3/2 and that of N = 1]— by the commutator [A, B] are—
(A) 12 (B) 15 (A) complex (B) real
(C) 20 (D) 36 (C) imaginary (D) zero
8 | CSIR–Chemical Sciences (Dec.–12)

43. The value of the commutator [x , px2 ] is given (C) hydrochloric acid in water
by— (D) benzoic acid in benzene
(A) 2 i (B) 2 i h– 50. A dilute silver nitrate solution is added to a

(C) 2 i h x (D) 2 i –h p x slight excess of sodium iodide solution. A
solution of Agl is formed whose surface
44. The correlation coeffcient between two adsorbs—
arbitrary variable x and y is zero, if— (A) I

(B) NO3–
(A) <xy> = <yx > (B) <x2> = <x>2 (C) Na + (D) Ag+
2
(C) <y > = <y > 2 (D) <xy > = <yx > <y>
51. The absorption spectrum of O2 shows a vibra-
45. A Carnot engine takes up 90 J of heat from tional structure that becomes continuum at
the source kept at 300 K. The correct 56875 cm–1. At the continuum, it dissociates
statement among the following is— into one ground state atom (Og ) and one
(A) it transfers 60 J of heat to the sink at excited state atom (Oe). The energy difference
200 K between O e and Og is 15125 cm–1 . The
(B) it transfers 50 J of heat to the sink at dissociation energy (in cm–1) of ground state
200 K of O2 is—
(C) it transfers 60 J of heat to the sink at 56875 15125
250 K (A) (B)
15125 56875
(D) it transfers 50 J of heat to the sink at (C) 72000 (D) 41750
250 K 52. The angle between the two planes represented
46. The relative population in two states with by the Miller indices (110) and (111) in a
energies E1 and E2 satisfying Boltzman simple cubic lattice is—
distribution is given by n1 /n2 = (3/2) exp. (A) 30° (B) 45°
[–(E1 – E2 )/KBT]. The relative degeneracy (C) 60° (D) 90°
g2 /g1 is— 53. The correct representation of the variation of
(A) 2 (B) 2/3 molar conductivity (y-axis) with surfactant
concentration (x-axis) is [CMC = Critical
(C) 3/2 (D) 3 Micelle Concentration]—
47. The Daniell cell is—
(A) Pt1 (s)⎜Zn(s) ⎜Zn2+ (aq)⎜⎜Cu2+ (aq)⎜Cu(s)⎜
Pt11 (s)
(B) Pt1 (s)⎜Zn(s) ⎜Zn2+ (aq) ⎜⎜Ag+ (aq)⎜Ag(s)⎜ (A) (B)
Pt11 (s)
(C) Pt1 (s)⎜Fe(s)⎜Fe2+ (aq)⎜⎜Cu 2+ (aq)⎜Cu(s)⎜
Pt11 (s)
(D) Pt1 (s)⎜H2 (g)⎜H2 SO4 (aq)⎜⎜Cu2+ (aq)
⎜Cu(s)⎜Pt11 (s)
48. If the concept of half-life is generalized to (C) (D)
quarter-life of a first order chemical reaction,
it will be equal to—
(A) ln 2 / k (B) ln 4 / k
(C) 4 / k (D) 1 / 4k 54. The major product formed in the following
reaction is—
49. Kohlrausch’s law is applicable to a dilute
solution of—
(A) potassium chloride in hexane
(B) acetic acid in water hν
CSIR–Chemical Sciences (Dec.–12) | 9

59. The correct statement(s) A-D are given for


(A) the following reaction. The correct one(s) is
(are)—

(B)

1. aromatic ipso substitution reaction


(C) 2. aromatic nucleophilic substitution
3. aromatic electrophilic substitution
4. aromatic free radical substitution
(D) (A) 1 and 2 only
(B) 1 and 3 only
55. If the pK a value for p-methoxybenzoic acid is (C) 3 and 4 only
4·46 and that of benzoic acid is 4·19, the σpara (D) 3 only
for methoxy group is— 60. The following photochemical transformation
(A) 8·65 (B) 4·32 proceeds through
(C) 0·27 (D) –0·27
56. The biosynthetic precursor of cadinene is—

(A) Norrish type I reaction


(B) Norrish type II reaction
(C) Barton reaction
(D) Paterno-Buchi reaction
Cadinene
61. A tripeptide gives the following products on
(A) shikimic acid (B) mevalonic acid Edman degradation.
(C) arachidonic acid (D) prephenic acid
57. The correct order of acidity of the compounds
A-C is—

The tripeptide is—


(A) Phe-Ala-Gly (B) Phe-Gly-Ala
(C) Ala-Gly-Phe (D) Gly-Ala-Phe
A B C 62. In the 1 H NMR spectrum recorded at 293 K,
(A) A > B > C (B) B > C > A an organic compound (C 3 H7NO), exhibited
(C) C > A > B (D) B > A > C signals at δ 7·8 (1 H, s), 2·8 (3 H, s) and 2·6
(3 H, s). The compound is—
58. The mechanism involved in the following
conversion is— (A) (B)

(C) (D)

63. In the IR spectrum of p-nitrophenyl acetate,


the carbonyl absorption band appears at—
(A) E2 -elimination (B) E1 -elimination (A) 1670 cm–1 (B) 1700 cm–1
(C) syn-elimination (D) E1cb-elimination (C) 1730 cm –1 (D) 1760 cm–1
10 | CSIR–Chemical Sciences (Dec.–12)

64. The absolute configuration at the two chiral 69. The most appropriate reagent to effect the
centres of (–)-camphor is— following chemoselective conversion is—

(A) HCl, EtOH, reflux


(A) 1R, 4R (B) 1R, 4s (B) Bu 4 NF
(C) 1s, 4R (D) 1s, 4s (C) K2CO3 MeOH
65. The major product formed in the following (D) CF3COOH, EtOH, rt
reaction is—
70. Among the following, an example of a ‘Green
Synthesis’ is—
(A) synthesis of malachite green
(B) Friedel-Craft’s acylation of anisole with
Ac 2 O/anhydrous AlCl 3
(A) (B)
(C) Jones’ oxidation of benzyl alcohol to
benzoic acid
(D) Diels-Alder reaction of furan and maleic
(C) (D) acid in water

66. The first person to separate a racemic mixture Part–C


into individual enantiomers is— 71. The recoil energy of a Mössbauer nuclide of
(A) J. H. Van’t Hoff (B) L. Pasteur mass 139 amu is 2·5 Mev. The energy emitted
(C) H. E. Fischer (D) F. Wöhler by the nucleus in keV is—
67. Consider the following statements for [18]- (A) 12·5 (B) 15·0
annulene : (C) 20·5 (D) 25·0
1. it is aromatic 72. Complexes of general formula, fac-
2. the inner protons resonate at δ 9·28 in its [Mo(CO)3 (phosphine)3 ] have the C-O
1 H NMR spectrum
stretching bands as given below :
3. there are six protons in the shielded zone.
Phosphines : PF3 (A); PCl3 (B); P(Cl)Ph 2 (C);
The correct statements are—
PMe3 (D)
(A) 1, 2, 3 (B) 1 and 2 only
(C) 2 and 3 only (D) 1 and 3 only v(CO), cm–1 : 2090 (i); 2040 (ii); 1977 (iii);
1945 (iv)
68. In the compound given below, the relation
The correct combination of the phosphine and
between HA, HB, and between Br1 , Br2 is—
the streching frequency is—
(A) (A – i) (B – ii) (C – iii) (D – iv)
(B) (A – ii) (B – i) (C – iv) (D – iii)
(C) (A – iv) (B – iii) (C – ii) (D – i)
(D) (A – iii) (B – iv) (C – i) (D – ii)
(A) H A , HB are enantiotropic; and Br1 , Br2
are diastereotopic 73. On subjecting 9·5 ml solution of Pb2+ of X M
to polorographic measurements, Id was found
(B) H A , HB are diastereotopic; and Br1, Br2
are enantiotropic to be 1 μA. When 0·5 ml of 0·04 M Pb2+ was
added before the measurement, the Id was
(C) H A , HB are diastereotopic; and Br1, Br2
are homotopic found to be 1·25 μA. The molarity X is—
(D) H A , HB are enantiotropic; and Br1 , Br2 (A) 0·0035 (B) 0·0400
are homotopic (C) 0·0067 (D) 0·0080
CSIR–Chemical Sciences (Dec.–12) | 11

74. Match each item from the List-I (compound Consider the following statements about its
in solvent) with that from the List-II (its be- room temperature spectral data :
haviour) and select the correct combination 1H
1. NMR has singles at 5·48 and 3·18
using the codes given below— ppm.
List-I 2. 1 H NMR has multiplet at 5·48 and singlet

(a) CH3 COOH in pyridine at 3·18 ppm.


(b) CH3 COOH in H2SO4 3. IR has CO stretching bands at 1950 and
1860 cm–1.
(c) HClO4 in H2 SO4
4. IR has only one CO stretching band at
(d) SbF5 in HF 1900 cm–1.
List-II The correct pair of statements is—
1. Strong acid 2. Weak acid (A) 1 and 3 (B) 2 and 3
3. Strong base 4. Weak base (C) 1 and 4 (D) 2 and 4
Codes : 79. In the cluster [Co3 (CH)(CO)9 ] obeying 18e
(a) (b) (c) (d) rule, the number of metal-metal bonds and the
(A) 1 2 3 4 bridging ligands respectively, are—
(B) 2 1 3 4 (A) 3 and 1 CH (B) 0 and 3 CO
(C) 3 4 2 1 (C) 3 and 1 CO (D) 6 and 1 CH
(D) 4 2 3 1 80. Consider the ions Eu(III), Gd(III), Sm(III)
75. Structure of a carborane with formula, and Lu(III). The observed and calculated
C 2 B 4 H8 is formally derived from— magnetic moment values are closest for the
pair—
(A) Closo-borane (B) Nido-borane (A) Gd (III), Lu (III)
(C) Arachno-borane (D) Conjuncto-borane (B) Eu (III), Lu (III)
76. Boric acid is a weak acid in aqueous solution. (C) Sm (III), Gd (III)
But its acidity increases significantly in the (D) Sm (III), Eu (III)
presence of ethylene glycol, because—
81. Silicates with continuous 3D framework
(A) ethylene glycol releases additional H+ are—
(B) B(OH)4 – is consumed in forming a (A) Neso-silicates (B) Soro-silicates
compound with ethylene glycol (C) Phyllo-silicates (D) Tecto-silicates
(C) ethylene glycol neutralizes H+ released
82. The correct spinel structure of Co3 O4 is—
by boric acid
(A) (Co2+)t(2Co3+)oO4
(D) Boric acid dissociates better in the
mixed-solvent (B) (Co3+)t(Co2+Co3+)oO4
(C) (Co2+Co3+)t(Co3+)oO4
77. Coordination number of ‘C’ in Be2 C 3 whose
structure is correlated with that of CaF2, is— (D) (2Co3+)t(Co2+)oO4
(A) 2 (B) 4 83. In the solid state, the CuCl5 3– ion has two
(C) 6 (D) 8 types of bonds. There are—
(A) three long and two short
78. For the molecule given below—
(B) two long and three short
(C) one long and four short
(D) four long and one short
84. In metalloenzymes, the metal centers are
covalently linked through the side chains of
the amino acid residues. The correct set of
12 | CSIR–Chemical Sciences (Dec.–12)

amino-acids which are involved in the 88. Successive addition of NaCl, H 3 PO4 , KSCN
primary coordination spheres of metalloen- and NaF to a solution of Fe(NO3 )3·9H2 O
zymes is— gives yellow, colourless, red and again
(A) Ala, Leu, His (B) Glu, His, Cys colourless solutions due to the respective
(C) Leu, Glu. Cys (D) Ala, His, Glu formation of—
85. Consider the catalyst in Column-I and reaction (A) [Fe(H2 O) 5 Cl]2 + , [Fe(H2 O) 5 (PO4 )],
in Column-II : [Fe(H2 O) 5 (SCN)]2+, [Fe(H2 O) 5 F]2+
Column-I (B) [Fe(H2 O) 4 Cl(OH)]1+, [Fe(H2O) 5 (PO4 )],
[Fe(H2 O) 5 (SCN)]2+, [Fe(H2 O) 5 F]2+
(a) [(R)-BINAP]Ru2–
(b) [Rh(CO) 2 I2]– (C) [Fe(H2 O) 5 Cl]2 + , [Fe(H 2 O) 6 ]3+,
[Fe(H2 O) 5 (SCN)]2+, [Fe(H2 O) 5 F]2+
(c) Pd(PPh3 )4
(D) [Fe(H2 O) 5 Cl]2 + , [Fe(H2 O) 5 (PO4 )],
[Fe(H2 O) 5 (SCN)]2+, Fe(H2 O) 4 (SCN)F]1+

(d) 89. Which one of following will not undergo


oxidative addition by methyl iodide ?
(A) [Rh(CO) 2 I2]–
(B) [Ir(PPh3)2(CO)Cl]
Column-II
(C) [η5 –CpRh(CO)2]
1. Hydroformylation
(D) [η5 –Cp2 Ti(Me)(Cl)]
2. Asymmetric hydrogenation
3. Asymmetric hydrogen transfer 90. In hydroformylation reaction using
[Rh(PPh3)3(CO)(H)] as the catalyst, addition
4. Heck coupling
of excess PPh 3 would—
The best match of a catalyst of Column-I with (A) increase the rate of reaction
the reaction under Column-II is—
(B) decrease the rate of reaction
Codes : (C) not influence the rate of reaction
(a) (b) (c) (d) (D) stop the reaction
(A) 2 1 4 3 91. Find out the number of lines in the 31P NMR
(B) 1 2 3 4 signal for—
(C) 3 1 4 2
(D) 4 3 2 1
86. A solution of 2·0 g of brass was analysed for
Cu electrogravimetrically using Pt-gauze as
electrode. The weight of Pt-gauze changed
from 14·5 g to 16·0 g. The percentage weight (A) 3 (B) 6
of Cu in brass is— (C) 18 (D) 90
(A) 50 (B) 55
(C) 60 (D) 75 92. The rate of exchange of OH2 present in the
coordination sphere by 18OH of,
2
87. The platinum complex of NH3 and Cl – ligands (i) [Cu(OH2 )6] ;2+ (ii) [Mn(OH2 )6]2+;
is an anti-tumour agent. The correct isomeric (iii) [Fe(OH 2 )6]2+; (iv) [Ni(OH2 )6]2+, follows
formula of the complex and its precursor are— an order—
(A) cis–Pt(NH3)2Cl2 and PtCl42– (A) (i) > (ii) > (iii) > (iv)
(B) trans–Pt(NH3 )2Cl2 and PtCl42– (B) (i) > (iv) > (iii) > (ii)
(C) cis–Pt(NH3)2Cl2 and Pt(NH 3 )42+ (C) (ii) > (iii) > (iv) > (i)
(D) trans–Pt(NH3 )2Cl2 and Pt(NH 3 )42+ (D) (iii) > (i) > (iv) > (ii)
CSIR–Chemical Sciences (Dec.–12) | 13

93. Based on the behaviour of the metalloen- 96. A metal crystallizes in f.c.c. structure with a
zymes, consider the following statements : unit cell side of 500 pm. If the density of the
1. In the enzymes, the zinc activates O 2 to crystal is 1·33 g/cc, the molar mass of the
form peroxide species. metal is close to—
2. In the enzymes, the zinc activates H2 O (A) 23 (B) 24
and provides a zinc bound hydroxide. (C) 25 (D) 26
3. In the oxidases, the iron activates O2 to 97. The activation energy for the bimolecular
break the bonding between the two reaction A + BC → AB + C is E0 in the gas
oxygens. phase. If the reaction is carried out in a
4. Zinc ion acts as a nucleophile and attacks confined volume of λ3, the activation energy
at the peptide carbonyl. is expected to—
The set of correct statements is—
(A) remain unchanged
(A) 1 and 2 (B) 2 and 3
(C) 3 and 4 (D) 1 and 4 (B) increase with decreasing λ
(C) decrease with decreasing λ
94. Fe2+ –porphyrins fail to exhibit reversible
oxygen transport and cannot differentiate CO (D) oscillate with decreasing λ
from O2 . However, the haemoglobin is free 98. In a many-electron atom, the total orbital
both these pitfalls. Among the following : angular momentum (L) and spin (S) are good
1. Fe2+ –porphyrins undergo μ–oxodimer quantum numbers instead of the individual
formation and the same is prevented in electron orbital (l1 , l2 ) and spin (s1 , s2 ) angular
case of the haemoglobin. momenta in the presence of—
2. Fe–CO bond strength is much low in (A) inter-electron repulsion
case of haemoglobin when compared to (B) spin-orbit interaction
the Fe2+–porphyrins.
(C) hyperfine couping
3. While Fe–CO is linear, Fe-O2 is bent and
is recognized by haemoglobin. (D) external magnetic field
4. The interlinked four monomeric units in 99. The packing fraction of a simple cubic lattice
the haemoglobin are responsible to is close to—
overcome the pitfalls. (A) 0·94 (B) 0·76
The correct set of statements is— (C) 0·52 (D) 0·45
(A) 1 and 2 (B) 1 and 3
100. The number of IR active vibrational modes of
(C) 3 and 4 (D) 2 and 4 pyridine is—
95. Reactions A and B are, termed as respecti-
vely— C2v E2 C2 σv σ′ v
1. A 1 1 1 1 1 z
A2 1 1 –1 –1 Rz
B1 1 –1 1 –1 x, Ry
B2 1 –1 –1 1 y, Rx
2. (A) 12 (B) 20
(C) 24 (D) 33
101. One of the excited states of Ti has the
electronic configuration [Ar]4s2 3d1 4p1 . The
(A) Insertion, Metathesis number of microstates with zero total spin (S)
(B) Metathesis, Insertion for this configuration is—
(C) Oxidative addition, Metathesis (A) 9 (B) 15
(D) Oxidative addition, Insertion (C) 27 (D) 60
14 | CSIR–Chemical Sciences (Dec.–12)

102. For the reaction A2 2A in a closed (A) A has fcc lattice while B has bcc lattice
container, the relation between the degree of (B) A has bcc lattice while B has fcc lattice
dissociation (α) and the equilibrium constant (C) A and B both have fcc lattice
Kp at a fixed temperature is given by— (D) A and B both have bcc lattice
(A) α = [Kp / (Kp + 4p)] 108. The decomposition of NH3 on Mo surface
follows Langmuir-Hinshelwood mechanism.
(B) α = [Kp / (Kp + 4p)]1/2
The decomposition was carried out at low
(C) α = [Kp + 4p) / Kp] pressures. The initial pressure of NH3 was
(D) α = [(Kp + 4p) / Kp]1/2 10–2 torr. The pressure of NH3 was reduced to
103. The fugacity of a gas depends on pressure and 10–4 torr in 10 minutes. The rate constant of
— decomposition of NH 3 is—
the compressibility factor Z (= pV /RT)
— (A) 9·9 × 10–4 torr min–1
through the relation [ V is the molar
(B) 0·4606 min –1
volume]—
(C) 9·9 × 10–3 torr min–1
[∫
ƒ = p . exp o
pZ–1
p
dp] (D) 0·693 min –1
109. A polymer sample has the following com-
For most gases at temperature T and up to position.
moderate pressure, this equation shows that—
(A) ƒ < p, if T → 0 (B) ƒ < p, if T → ∞ Number of molecules Molecular weight
(C) ƒ > p, if T → 0 (D) ƒ = p, if T → 0 10 1000
104. The internal pressure (∂U/∂V)T of a real gas is 50 2000
— 40 4000
related to be compressibility factor Z = pV/RT

by [V is the molar volume]— The polydispersity Index (P.D.I.) of the
(A) (∂U/∂V)T = RT (∂Z/∂V)T polymer is—
— 85000 85
(B) (∂U/∂V)T = RT / (V Z) (A) (B)
— 27 81
(C) (∂U/∂V)T = (RT2 / V) (∂Z/∂T)V 850 729
— (C) (D)
(D) (∂U/∂V)T = (V /RT2) (∂Z/∂T)V 729 850
105. Suppose, the ground stationary state of a 110. The equilibrium constant for an electro-
harmonic oscillator with force constant k is chemical reaction 2Fe3+ + Sn2+ 2Fe2+ +
given by ψo = exp [–Ax ]2
Sn 4+ is [E0 (Fe 3+ / Fe2+) = 0·75V, E 0 (Sn4+ /
Then, A should depend on k as— Sn 2+) = 0·15V, (2·303RT / F) = 0·06V]—
(A) A ∝ k –1/2 (B) A ∝ k (A) 1010 (B) 1020
(C) A ∝ k1/2 (D) A ∝ k1/3 (C) 1030 (D) 1040
106. Combining two real wave functions φ 1 and φ2, 111. A bacterial colony grows most commonly by
the following functions are constructed : cell division. The change in the population
A = φ1 + φ 2 , B = φ1 + iφ2, C = φ 1 – iφ2, D = due to cell division in an actively growing
i(φ1 + φ2). The correct statement will then colony is d N = λgN d t. The population of
be— bacterial colony at time t is [N0 = N(t = 0)]—
(A) A and B represent the same state (A) N0 λgt (B) N0 exp[–λgt]
(B) A and C represent the same state (C) N 0 exp[λg t] (D) N0 (λgt)2
(C) A and D represent the same state 112. The Arrhenius parameters for the thermal
(D) B and D represent the same state decomposition of NOCl, 2NOCl(g) → 2NO(g)
107. Crystal A diffracts from (111) and (200) + Cl2 (g), are A = 1013 M–1 s–1, E a = = 105 kJ
planes but not from (110) plane, while the mol –1 and RT = 2·5 kJ mol–1. The enthalpy (in
Crystal B diffracts from (110) and (200) kJ mol–1) of the activated complex will be—
planes but not from the (111) plane. From the (A) 110 (B) 105
above, we may conclude that— (C) 102·5 (D) 100
CSIR–Chemical Sciences (Dec.–12) | 15

113. The rotational partition function of H2 is— 119. The trial wave function of a system is
(A) ∑ (2J + 1)e –βhcBJ(J + 1) expanded as ψt = c1 φ1 + c2φ2. The matrix
J = 0‚1‚2‚ … elements of the Hamiltonian are <φ1⏐H⏐φ1〉
= 0; (φ1 ⏐H⏐φ2〉 = 2·0 = <φ2|H|φ1〉 and
(B) ∑ (2J + 1)e –βhcBJ(J + 1)
J = 1‚3‚5‚ … <φ2⏐H⏐φ2〉 = 3·0. The approximate ground-
state energy of the system from the linear
(C) ∑ (2J + 1)e –βhcBJ(J + 1)
J = 0‚2‚4‚ … variational principle is—
1 (A) –1·0 (B) –2·0
(D)
4
[ ∑ (2J + 1)e –βhcBJ(J + 1) (C) +4·0 (D) +5·0
J = 0‚2‚4‚ …
120. One molecular orbital of a polar molecule AB
+3 ∑ (2J + 1)e–βhcBJ(J + 1)] has the form CAψ A + CB ψB , where ψA and
J = 1‚3‚5‚ … ψB are normalized atomic orbitals of centred
114. The potential in Debye-Hückel theory is on A and B, respectively. The electron in this
proportional to— orbital is found on atom B with a probability
(A) 1/kr (B) exp[–kr] of 90%. Neglecting the overlap between ψA
(C) exp[–kr]/r (D) kr and ψB , a possible set of CA and CB is—
115. The vibrational frequency and anharmonicity (A) C A = 0·95, CB = 0·32
constant of an alkali halide are 300 cm–1 and (B) C A = 0·10, CB = 0·90
0·0025, respectively. The positions (in cm–1) (C) C A = –0·95, CB = 0·32
of its fundamental mode and first overtone are
respectively— (D) C A = 0·32, CB = 0·95
(A) 300, 600 (B) 298·5, 595·5 121. 4-Hydroxybenzoic acid exhibited signals at
(C) 301·5, 604·5 (D) 290, 580 δ 171, 162, 133, 122 and 116 ppm in its
116. The adsorption of a gas is described by the broadband decoupled 13C NMR spectrum. The
Langmuir isotherm with the equilibrium cons- correct assignment of the signals is—
tant K = 0·9 kPa–1 at 25°C. The pressure (in (A) δ 171 (C–4), 162 (COOH), 133 (C–3
kPa) at which the fractional surface coverage & 5), 122 (C-1) and 116 (C-2 & 6)
is 0·95, is— (B) δ 171 (COOH), 162 (C-4), 133 (C–2
(A) 1/11·1 (B) 21·1 & 6), 122 (C-1) and 116 (C-3 & 5)
(C) 11·1 (D) 42·2 (C) δ 171 (C–4), 162 (COOH), 133 (C–2
117. The energy of a harmonic oscillator in its & 6), 122 (C-1) and 116 (C-3 & 5)
1
ground state is hω. According to the virial (D) δ 171 (COOH), 162 (C–4), 133 (C–3
2 & 5), 122 (C-1) and 116 (C-2 & 6)
theorem, the average kinetic (T) and potential 122. An organic compound (C9 H10O3) exhibited
(V) energies of the above are— the following spectral data :
1 1
(A) T = hω; V = hω IR : 3400, 1680 cm–1;
4 4
1 H NMR : δ 7·8 (1 H, d, J = 8 Hz), 7·0 (1 H,
1 3
(B) T = hω; V = hω d, J = 8 Hz), 6·5 (1 H, s), 5·8 (1 H, s, D2 O
8 8
1 exchangeable), 3·9 (3 H, s), 2·3 (3 H, s).
(C) T = hω; V = – hω The compound is—
2
3 1
(D) T = hω; V = hω
8 8 (A) (B)
118. The energy of a hydrogen atom in a state is
–hcRH
(RH = Rydberg constant). The degen-
25
eracy of the state will be—
(C) (D)
(A) 5 (B) 10
(C) 25 (D) 50
16 | CSIR–Chemical Sciences (Dec.–12)

123. The [α ] D of a 90% optically pure


2-arylpropanoic acid solution is +135°. On (C)
treatment with a base at RT for one hour, [α]D
changed to +120°. The optical purity is
reduced to 40% after 3 hours. If so, the (D)
optical purity of the solution after 1 hour, and
its [α ] D after 3 hours, respectively, would 126. The major product formed in the following
be— reaction is—
(A) 80%; and 60° (B) 70%; and 40°
(C) 80%; and 90° (D) 70%; and 60°
124. In the following pericyclic reaction, the
structure of the allene formed and its configu-
ration are— (A)

(B)

(C)
(optically pure)
(D)
(A) R 127. The following conversion involves—

(B) S
(A) a 1, 3-dipolar species as reactive inter-
mediate and a cycloaddition.
(B) a carbenium ion as reactive intermediate,
(C) R
and a cycloaddition
(C) a 1, 3-dipolar species as reactive inter-
mediate, and an aza Wittig reaction
(D) S (D) a carbanion as reactive intermediate, and
an aza Cope rearrangement
128. The following transformation involves—
125. In the following sequence of pericyclic
reactions X and Y are—

(A) an iminium ion, [3, 3]-sigmatropic shift


and Mannich reaction
(B) a nitrenium ion, [3, 3]-sigmatropic shift
X Y and Michael reaction
(C) an iminium ion, [1, 3]-sigmatropic shift
(A) and Mannich reaction
(D) a nitrenium ion, [1, 3]-sigmatropic shift
and Michael reaction
(B) 129. With respect to the following biogenetic
conversion of chorismic acid (A) to 4-
CSIR–Chemical Sciences (Dec.–12) | 17

hydroxyphenylpyruvic acid (C), the correct


statement is— (D)

132. In the following reaction A and B are—

(A) X is Claisen rearrangement; Y is oxida-


tive decarboxylation (A) (B)
(B) X is Fries rearrangement; Y is oxidative
decarboxylation (A)
(C) X is Fries rearrangement; Y is dehydra-
tion
(B)
(D) X is Claisen rearrangement; Y is dehy-
dration
130. Match the following— (C)
List-I
(a) β-amyrin (b) squalene (D)
(c) morphine (d) ephedrine
List-II 133. Match the following biochemical transforma-
tions with the coenzymes involved—
1. alkaloid, secondary alcohol
List-I
2. alkaloid, phenol
(a) α-ketoglutarate to glutamic acid
3. triterpene, secondary alcohol
(b) uridine to thymidine
4. acyclic triterpene, polyene (c) pyruvic acid to acetyl coenzyme A
Codes : List-II
(a) (b) (c) (d) 1. tetrahydrofolate
(A) 3 4 2 1 2. NADH
(B) 2 1 3 4 3. thiamine pyrophosphate
4. pyridoxamine
(C) 3 2 4 1
(D) 1 4 2 3 Codes :
(a) (b) (c)
131. In the following reaction, the structure of B,
and the mode of addition are— (A) 4 1 3
(B) 1 2 4
(C) 2 1 3
(D) 4 2 3

(A) 134. The structure of the major product B formed


in the following reaction sequence is—

(B)

(C)
18 | CSIR–Chemical Sciences (Dec.–12)

(A) (B) (A) (B)

(C) (D) (C) (D)

138. 12·0 g of acetophenone on reaction with


135. Given the energy of each gauche butane 76·2 g of iodine in the presence of aq. NaOH
interaction is 0·9 k-cal/mol, ΔG value of the gave solid A in 75% yield. Approximate
following reaction is— amount of A obtained in the reaction and its
structure are—
(A) 80 g, CI4 (B) 40 g, CI4
(C) 60 g, CHI3 (D) 30 g, CHI3
(A) 0·9 k-cal/mol (B) 1·8 k-cal/mol 139. Consider the following reaction mechanism.
(C) 2·7 k-cal/mol (D) 3·6 k-cal/mol
|
136. In the following reaction, the reagent I and l
the major product II are—
I

The steps A, B and C, respectively, are—


II (A) oxidative addition; transmetallation; re-
I II ductive elimination
(A) (B) oxidative addition; carbopalladation; β-
hydride elimination
(C) carbopalladation; transmetallation; re-
(B) ductive elimination
(D) metal halogen exchange; transmetallation;
metal extrusion
(C) 140. The major product formed in the following
reaction sequence is—

(D)
(A)

(B)
137. The major product formed in the following
reaction sequence is—
(C)

(D)
CSIR–Chemical Sciences (Dec.–12) | 19

141. The major product B formed in the following 144. The major product formed in the following
reaction sequence is— reaction is—

(A) (B)
(A)

(C) (D)
(B)
145. In the following enantioselective reaction, the
major product formed is—

(C)

(D)

142. The major product B formed in the following


reaction sequence is— (A) (B)

(C) (D)

Answers with Explanations


(A) (B)
Part–A
4 3 4 2
1. (A) 23 = 2 81, 24 = 2 64, 32 = 3 16, 34 = 3 16,
3 2
42 = 4 8 = 2 16, 43 = 4 9 = 2 18.
(C) (D) 4
Therefore, 23 is the largest.
2. (B) D a
C A 2 B
143. The osazone A could be obtained from— A B
H a 2 a 2
G
E F
F
Equilateral triangle is formed with diagonal.
⎯ 3 (Side)2

∴ Area ACF =
4
A
⎯ 3 (a√

(A) glucose and mannose = ⎯ 2)2
(B) mannose and galactose 4
(C) gulose and fructose ⎯ 3a2

=
(D) galactose and fructose 2
20 | CSIR–Chemical Sciences (Dec.–12)

3. (D) Total number of distinct arrangements OB2 = OC2 + BC2


7 52 = OC2 + 3 2
= = 2520 OC = ± 4
2 ⇒ OC = 4
Number of arrangements when U and I come –ve not taken in first quadrant
together So, centre is (4, 5).
7. (C) The minimum single cut is required
6
= × 2 = 720 because wire ABCDB = 5 m
2 A
Thus, the no. of arrangements when U and I
cannot come together
= 2520 – 720
= 1800 D
4. (C) Let numbers are a, b, c, d, e, f, g B C
Given, a + b + c + d + e + f + g = 21
and Wire AD = 1m
Q Arithematic Progression ≥ Geometrical
8. (D) 2, – 4, 8, –16, 32, – 64, 128, – 256, 512
Progression
– 256 + 512 = x
A.P. = G.P.
x = 256
(When all the numbers are equal) log2 x = log 256
a+b+c+d+e+f+g
Now, = 3 = log2 2 8
7
If a=b=c=d=e=f=g = 3 log2 x = 8 log2 2 = 1
a2 + b2 + c2 + d2 + e2 + f2 + g2 9. (A) Let r is radius, then
⇒ = 9
7 OR = 10·5 – r
5. (C) 50B = 113 + 3 13 + 5 13 + … 9913 …(1)
50C = 213 + 4 13 + 6 13 + … 10013 …(2) r
Each term of (2) is greater to term of (1) O
P
Then, 50C > 50B ⇒ C > B
Now, 100A = 113 + 2 13 + 3 13 + 4 13
30°

+ … 100 13
R
100A = 50B + 50C
In Δ OPR
2A = B + C
OP r 1
Therefore, 2A > 2B and 2A < 2C sin 30° = ⇒ =
OR 10·5 – r 2
Then, A > B and A < C
r
Therefore, B<A<C ⇒ 0·5 =
10·5 – r
6. (D) In right angled triangle BOC
10·5
y ⇒ 2 = –1
r
II A Ist quadrant 10·5
3 5 ⇒ 3 =
r
C O
3 35 ⇒ r = 3·5
B x 10. (A) Amar Doctor (tallest)
(Shortest) Akbar Engineer e2 = dp
Anthony Professor
Therefore, Amar is a doctor and he is the
III IV tallest.
CSIR–Chemical Sciences (Dec.–12) | 21

11. (B) If 100 cats catch 100 mice in 100 minutes, ⇒ R 3 = Nr3
then 7 cates will catch 7 mice also in
100 minutes. ⇒ ()r 3
R
=
1
N
12. (A) Each half diagram shows increase in one Also, 4πR2 ·X = N(4πr2 )
n(n + 1)
unit, thus 1 + 2 + 3 + … +n = . ⇒ R 2 X = Nr2
2
13. (B) Since, socks is in pair, so atleast N + 1
number of socks would be taken out.
⇒ X =
r 2
R ()
N

14. (D) 11 12 =
N ()
1 2/3
·N
10 ⇒ X = N1–2/3 = N1/3
9 3 19. (A) 24 30 33 39 51 57
4 2 4 6 3 0 3 3 3 63 9 12 5 1 6
5 Thus, the next number is 57.
6 20. (A) No parallel and No concurrent lines.
4 – 54′ – 33′′ 3
15. (C) When curve touches the x-axis, y = 0 for 4 2
real x. P Q R
1
∴ 0 = x 2 – 10x + 25
⇒ (x – 5)2 = 0 UT S
⇒ x = 5
No other option have real value of x. New lines are PU, QS and RT.
16. (C) AB // CD
C D Part–B
21. (A) Like atomic and molecular orbital, there
are also nuclear orbital with same rules. We
O know that
Unitary operator have two parity,
A B P = ±1
When even serial number (azimuthal quantum
Let OD = r
No.) is found for orbital, it is positive (P) and
Thus, ∠ COD = ∠ AOB vice-versa. We know that spin for orbitals are
and ∠ DCO = ∠ ABO Orbitals Serial No. Parity Spin
Therefore, Δ OCD and Δ OBA are symmetric (Azimuthal
triangle Quantum No.)
Area of triangle Δ OAB
Thus,
Area of triangle Δ OCD
= ( )
AO 2
OD
s 0 + 1
2
= ( )
2OD 2
OD
p 1 – 3
2
= 4 5
d 2 +
17. (*) 2
18. (C) Let r is the radius of small sphere and R is 7
f 3 –
the radius of big sphere. 2
4 4 9
Thus, πR3 = N πr3 g 4 +
3 3 2
4 11
Volume of sphere = π (radius)3 h 5 –
3 2
22 | CSIR–Chemical Sciences (Dec.–12)

22. (D) Electroplating is a process that uses Therefore, there are no fundamental vibra-
electrical current to reduce dissolved metal tional modes common in both IR and Raman
cations so that they form a coherent metal spectra.
coating on an electrode. 26. (C) [CO(H 2 O) 6 ]2+, d7 system
Also, PbCl2 and PbSO 4 are ionic and insolu- eg Transition from + g to eg
ble in cold water Pb(Et) 4 is toxic.
e e 2g
Cathode Anode Octahedral environment
[COCl 4 ]2–, d7 system
Pb 2 Transition from e to + 2

e
2
Pb BF4 So, both complexes show d-d transition.
27. (A) [MO2 (S2 )6]–2
Fig : Electroplating of a Metal with Lead
in a Pd(BF4)2 bath S 2 2– is 2e– donor
It is generally agreed that electro deposited S S
metals are crystalline and external appearance S S
S
depends mainly on the rate at which the S MO MO S
S
crystal grow and on the rate of the formation S S
of fresh nuclei. S S
–2
Briolging S2 ligand are 2 and coordination
23. (B) Beer’s law is,
number are 8.
() I
log 0 = ecl= A
I 28. (D) For D, I = 1
Since, absorbance is directly proportional to Multiplicity = 2nI + 1
concentration, very high concentration of n = No. of D atom
analyte deviates the Beer’s law. Further Asso- I = spin of D atom
ciation or dissociation of analyte increases or Therefore, M = 2×1×1+1
decreases the concentration of analyte, so it = 3
also cause deviation.
and Intensity ratio will be non-pascal.
24. (D) In Lanthanoids in (+3) oxidation state So, 1 H NMR spectrum of HD gives a triplet
shows decrease in size when we go down with intensity ratio 1 : 1 : 1.
from La to Lu.
29. (B) [Ru(PPh3 )2(acac)2] (acac = acetylaceto-
In hot concentrated aqueous NaOH, OH have
small size and strongly binds with Lu +3 and nate)
forms [Lu(OH)6 ]–3. That is the reason PPh3 acac
O
Lu(OH)3 shows the highest solubility in hot O O O PPh3
acac Ru acac Ru
concentrated aqueous NaOH. O O O PPh3
PPh3 acac O minor
25. (D) The structure of CO 2 is
Trans Cis-d
Symmetric
O C O acac
O
Since, symmetric stretching gives no change Ph3P O
Ru
in dipole moment so it is IR-inactive mode. Ph3P O
Since, CO2 is centrosymmetric molecule, O acac
therefore all the modes that is inactive in IR, Cis-l
are active in Raman. 3 possible isomer are shown above.
CSIR–Chemical Sciences (Dec.–12) | 23

30. (A) This complex exists or dimer and structure OCH3


is 34. (A) (OC)5M C
OH2 Ph
This is an example of metal carbene complex.
O O
Since, the molecule obeys 18e – rule and
Cu carbene is 2e– donor.
CH3C CCH3
O O Therefore, we need a metal with 6e–.
δ bond
O O This is only Cr and Re+ pair.
CH3C CCH3
Cu 35. (A) Fe is found in haemoglobin myoglobin
O O and cytrochromes. Mo is for nitrogen fixation.
OH2 Cu for heamocyanin and Zn for metallo-
There are 10 Cu-O bond presents in this enzymes.
complex. 36. (D) H2
N NH2 2
31. (B) Among the given elements K has least CH2 CH2 I of Cu = 3
electronegativity and F has most electro- Cu , 2
CH2 CH2 I of N = 1
negativity. Therefore, the electronegativity NH2
N
difference is greater in K and F. H2
32. (D) O O Therefore,

O
C
O O
S
O
(2n1I1 + 1) (2n2 I2 + 1) = (2 × 1 × 23 + 1)
(2 × 4 × 1 + 1)
sp2-hybridization sp2 -hybridization
Trigonal planar Trigonal planar = 36
O n1 = no. of Cu atom, I1 = I of Cu
n2 = no. of N atom, I 2 = I of N.
Xe N
37. (A) Degradation of penicillin G
O O O O O donor atom
sp 3 -hybridization sp2 -hybridization
Trigonal pyramidal Trigonal planar
Therefore, CO3 2–, SO 3 and NO3– have planar donor atom
structure.
donor atom
33. (C) Cl Penicillamine can utilize N, O, S atoms as
donors to bind with Pb(II), Hg(II) and Cu(II).
Fe CO CO 38. (D) B2 H6 → D2h point group and S2 axis
CH4 → Td point group and S4 axis
PH5 → D3 h point group and S3 axis
18°-e– system 18e– system 19e– system SF6 → Oh point group and S6 axis
stable stable unstable (symmetry element)
But Cp is not replaceable with NO in this case. 39. (C) For electronic transition selection rule is,
Therefore, ΔS = O, Δl = ± 1 and Δ J = 0, ± 1
For d2, 3 F → 3D is allowed transition.
Because ΔS = 0, Δl = –1.
Ni NO Ni 18 e system 40. (C) When a hydrogen atom is placed in an
stable electric field along the y-axis, electron density
N will be oriented along y-axis and therefore 2py
20 e system O orbital mixes most with the ground state 1s
unstable orbital.
24 | CSIR–Chemical Sciences (Dec.–12)

41. (B) For water, ΔHvap ≈ 41kJ mol–1 g1 3


∴ =
For vaporization, Tb = 273 + 100 g2 2
= 373 K g2 2
⇒ =
Therefore, molar entropy of vaporization, g1 3
ΔHvap 47. (A) In the Daniell cell, Copper and Zinc elec-
ΔS vap = trodes are immersed in a solution of Copper
Tb
41 × 1000J mol–1 (II) sulphate and Zinc sulphate respectively.
=
373K Oxidation → Zn(s) → Zn+2(aq) + 2e– at anode
≈ 109·9 J K –1 mol–1 Reduction → Cu+2(aq) + 2e– → Cu(s)
at cathode
42. (C) For commuting operators,
Overall cell rh is
[A, B] = 0 Zn(s) + Cu+2(aq) → Zn+2(aq) + Cu(s)
For non-commuting hermitian operators, there 48. (B) We know that for first order reaction,
is always a real value multiplie with i is found. A+ = A0e–k1 +
Therefore, it is always imaginary. According to question,
43. (D) [x, px2 ] = ih·2px A0
= A0e–K1 + 1/4
= 2ihpx 4
1 1
It is non-commuting operator. ⇒ = K
4 e 1 + 1/4
44. (D) The correlation coefficient between two ⇒ 4 = eK1 + 1/4
arbitrary variables x and y is given by ⇒ ln 4 = K1+ 1/4
n[Σxy – Σx Σy] ln 4
r = ⇒ t1/4 =
⎯⎯⎯⎯⎯⎯⎯⎯⎯⎯⎯⎯⎯⎯⎯⎯⎯⎯⎯
√ [nΣx – (Σx)2] [nΣy 2 – (Σy)2]
2 K1
= 0 49. (C) According to Kohlrausch’s law at infinite
⇒ Σxy = ΣxΣy dilution, when dissociation is complete, each
ion makes a definite contribution towards
or, <xy > = <x> <y >
molar conductance of the electrolyte irrespec-
45. (A) T2 = 300 K tive of the nature of the other ion with which
q2 = 90 J it is associated and the molar conductance at
(hot) infinite dilution for any electrolyte is given by
T2 the sum of the contribution of the two ions.
Source q2 In given option, only HCl in H2O completely
work done by dissociated and contributes equal ion.
Carnot engine
the engine 50. (A) A dilute silver nitrate solution is added to
q1 a slight excess of sodium iodide solution, a
Sink
T1 negatively charged solution of silver iodide is
(cold) formed.
q2 – q 1 T – T1 Na
We know = 2 I
q2 T2 Na I I Na

∴ q2 – q1 = 90 × (
300 – 200
300 ) Na I I Na

= 30 I AgI I
Na Na
q1 = 60 J I I
46. (B) Boltzman Distribution law is Na I I Na
ni g I
= i e–(Ei – EJ)/KBT Na
nJ gJ
Fig. : Electrical property of colloid based on
n1
and given that
n2
=
3
2()e–(E1 – E2)/KBT
concept of electrical double layer
AgNO 3 + NaI → AgI + NaNO3
CSIR–Chemical Sciences (Dec.–12) | 25

The negatively charged AgI attracts Na + and factant with part of the charge of the micelle
repels NO3 – . neutralized by associated counter ions.
log CMC = –a log ci + b
Therefore, Na+ try to form a rigid layer with
I– . 54. (A)

51. (D) We know that, Me Me


Ag2O Walf
Ph MeOH or Rearrange-
O C Ph
Dissociation energy, hν N2 ment
Me
De = Emax – E0 MeOH
Me OMe Me Me
E0 is ground state energy Proton
Ph OH Ph C OH TransferPh C O
It is clear from question that OMe HOMe
Emax = 56875 cm–1 Me O
Ph OMe
and E0 = 15125
55. (D) We know that,
∴ De = 56875 – 15125
KP-OMe
= 41750 cm–1 log = σ
KCOOH
52. (*) The angle between two planes having σ = log KP-OMe – log KCOOH
miller indices (h1 k1l1 ) and (h2 k2l2 ) is given as = –log KCOOH – (–log KP-OMe )
h1 h2 + k1k2 + l1l2 = P Ka(COOH) – PKa(P-OMe)
cos θ =
√⎯⎯⎯⎯⎯⎯⎯⎯
⎯ h1 2 + k12 + l12 √
⎯⎯⎯⎯⎯⎯⎯⎯⎯
h2 2 + k22 + l22 = 4·19 – 4·46
σ = – 0·27
| X | + | X | + OX
⇒ cos θ = Electron releasing group have –ve σ value.
⎯⎯⎯⎯⎯⎯⎯⎯
√ + 12 + 02 √
12 ⎯⎯⎯⎯⎯⎯⎯⎯
12 + 1 2 + 1 2
56. (B) Biosynthetic precursor of Cadinene
2 √2
⎯ H HO O
= =
⎯ 2·⎯√ 3 √
√ ⎯3 is mevalonic acid, HO OH .
θ = cos–1 (0·816)
⇒ θ = 35·31 O O O
57. (C) H ,
All options are incorrect.
53. (B) The molar conductance of an ionic sur- O O O
factant of the type Na+R – in water is plotted A B
against the square root of the normality of the more acidic aromatic ring
solution. The curve obtained, instead of being with reactive makes proton less
the smoothly decreasing curve characteristic methylene gp acidic due to e–
of ionic electrolytes of this type, has a sharp withdrawing nature
break in it at low concentrations. This sharp HO O O O

break in the curve accompanied by reduction H


in the conductance of the solution, indicating
a sharp increase in the mass per unit charge of HO O O O

the material in soluton, is interpreted as C


evidence for the formation of miceller at that acidic like H2SO 4 aromatic in nature
point from the unassociated molecular of sur- Therefore, C>A>B
26 | CSIR–Chemical Sciences (Dec.–12)

58. (D) δ 7·8(1H, s)


HN
Ph
Piperidine δ 2·8(3H, s)
H O COOEt δ 2·6(3H, s)
O N
H From the above observation the structure of
compound is
O O CH3(2.8) Chemically
Ph
HN H NMe2 N non-equivalent
O COOEt H CH3(2.6)
O (7.8)
Carbanion (Stable) O
↓ E1CB 63. (D) O2N O C CH3
A

Ph
Normally, ester groups show absorption at
H 2N CO2
COOEt
1750 – 1770 in phenyl acetate, is shower
absorption at 1765 but in compound (A) due
59. (B) to conjugation with ring and O-atom absor-
F O ption decreases and it appears at 1761 cm–1.
DMSO
K2CO3
MeO2S N O 64. (D) (–)-Camphor
MeO2S N 1
H O
This reaction is aromatic ipso substitution and 4
aromatic nucleophilic substitution. 2
60. (B) 2 4
O Ph O CH3 1
hν 3
O 3 1
Ph N Me Ph N Ph
view
Norrish type-II reaction 4
OH 1s
γ-H abstraction OH CH2 Ph
by O 2
Ph N Ph N
Ph 2
O 3 1
1, 4-biradical 4
3 H 1
4 view
61. (A) Edman’s reagent is N C S 4

Phenyisothio cyanate 4s
O Absolute configuration is 1s, 4s.
HN Ph
O H2N CH C OH 65. (A) This reaction is Vielsmeir-Haac Reaction.
S N
CH2 Ph Cl
Ph Me Cl
Cl H C N Cl
Phe(Phenyl alamine) Me Me Cl
O P Cl O P O
O O O N
H Cl Me P O
N OH + H2N C
H 2N OH H2N OH H Cl
O O
Ala(alanine) Gly (Glycine)
H
Therefore, the sequence is Phe-Ala-Gly, Since Cl
H C
Edman degradation occur from N-T-A H C NMe2 NMe2
H C NMe2
From N-terminal to Acid terminal. N Cl N
62. (A) C3 H7NO Cl H H
reactive aromatization
No. of C-atoms + 2 – No. of H-atoms species
+ No. of N-atoms H
DBE = CHO C NMe2
2 H3O
6+2–7+1 N N
= =1
2 H H
CSIR–Chemical Sciences (Dec.–12) | 27

66. (B) The first person to separate a racemic 69. (C)


mixture into individual enantiomers is L. Boc
Boc N
Pasteur. N OAc
H OH
H OTBDMS OTBDMS
Pasteur separated the left and right crystal
shapes from each other to form two piles of The hydrolysis of ester is faster in alkaline
crystals; in solution one form rotated light to medium therefore the correct one is K2 CO3 ,
the left, the other to the right, while an equal MeOH.
mixture of the two forms cancelled each 70. (D) Diels-Alder reaction of furan and maleic
other’s effect and does not rotate the polarized acid in water is an example of ‘Green
light. Synthesis’.
O O
OH H2O
O
OH COOH
Furan
O COOH
Maleic acid Endo product

Mirror
image Part–C
67. (D) 71. (D) Recoil energy,
H H E2
Er =
H H 2Mc2
536E2
or, Er = eV
H H M
H H H
H H H
2·5 × 10 6 × 139 × 931·5
E =
H H 536
= 24·57 keV
H H
72. (A) CO is π-acid ligend and have tendency to
H H back-bonding with metal.
[18]-Annulene CO
* aromatic, planar
H 3P CO
* shows aromatic substitution
* 12 H outer, deshielded zone δ = 9·28 Mo
* 6 H inner, shielded zone δ = –3·0 PH3 CO
* 37 k-cal/mol resonance energy
PH3
* diatropic
Electron donating phosphine group increase
68. (B) HA HB e– density on metal and back bonding with M
and CO becomes strong. As a result of this C
Me Me
= O bond becomes weak.
Therefore, stretching frequency, υC = 0
Br1 Br2
decreases.
Molecule has a plane of symmetry bisecting Phosphine V(CO) (cm–1)
HA and HB reflecting Br1/Br2 and CH3/CH 3 .
PF3 (A) 2090 (i)
(a) Br1 and Br2 are reflected by plane, so they
PCl3 (B) 2040 (ii)
are enantiotropic to each other.
(b) HA and H B are bisecting by plane. Hence, P(Cl)Ph2 (C) 1977 (iii)
they are diastereotopic to each other. PMe3 (D) 1945 (iv)
28 | CSIR–Chemical Sciences (Dec.–12)

73. (C) Id = K(C*—C) Thus, Be +2 have 4 coordination number and


where (C*—C) is concentration gradient and C 3 –2 have 8 coordination number.
K is constant Id α concentration (mol ml–1)
⎧⎪ Be 2
X
C1 = C3 2

⎨ 9·5
⎪⎩
C2 =
0·04
0·5
Id1 C1 78. (A) H
= H H
Id2 C2
⇒ X = 0·0067 H
Mo
H
74. (C) (A) CH3 COOH in pyridine CH3
Singlets N CO
at 3.18 ppm CO
Strong base CH3

→ CH3COO– + C5H5NH solvocation All 5-protons r are equal and resonate at 5·48
ppm.
(B) CH3 COOH in H2SO4
Strong base Also, both CO are is different environment,
⊕ therefore gives two signal at 1950 and 1860
→ CH3COOH2 + HSO 4 – solvo anion cm–1.
(C) HClO4 in H2 SO4 79. (A) —CH is 3e– donor ligand.
→ ClO4 – + H3 S +O4 (weak acid) No. of M—M bond
(D) SbF5 in HF → SbF 6 + H2 F strong acid
– + 18 × 3 – (9 × 3 + 9 × 2 + 3)
=
2 mole 2
75. (B) C 2 B 4 H8 = (BH) × 4 + (CH) × 2 + 2 54 – 48
= =3
= 2×4+3×2+2 2
= 16e– or 8e– pair Therefore, structure of [CO3 (CH)(CO)9 ]
or (m + 2)e – pair (Nido) CO CO CO
—BH contributes 2e– s to framework.
—CH contributes 3e– s to framework. Co
OC CO
76. (B) Boric acid is a weak acid in aq. solution.
OC Co Co CO
H OC C CO
O
B(OH)4 OH B H
with HO—CH2 —
O O Thus, No. of M—M bond = 3
H
H and Bridging ligand = 1
CH2 —OH 80. (A) Since, Gd +3 and Lu+3 have 4 f 7 and 4f 14
B(OH)4 – consumed in forming a compound electronic configuration. Therefore, no orbital
with ethylene glycol. contribution observed and magnetic moment
HO CH2 value is closest w.r.t. observed and calculated.
HO OH HO CH HO OCH2 81. (D) Neso-silicates = ortho silicates
B 2
B
HO OH H2O HO OCH2 SiO4 –2 unit
HO CH2 Soro-silicates = Si2O7–2 unit
HO CH2
CH 2 O OCH 2 Tecto-silicates = SiO 2 unit and 3D
B
H2O CH2O OCH2 framework
82. (A) The structure of Co3O4 is normal spinel.
77. (D) Be2C 3 structure is correlated with CaF 2 . In normal spinel the Co+2 ions occupy tetra-
Therefore, Be +2 in tetrahedral voids. hedral voids and Co+3 ions occupy octahedral
Since, molecules have FCC arrangement. voider.
CSIR–Chemical Sciences (Dec.–12) | 29

Therefore structure is, 88. (A) In solution Fe(NO3)3·9H2 O exists as


(Co+2)t (2Co+3)O O4 [Fe(H2 O) 6 ]+3·(NO3– )3·3H2 O
For Inverse spinel structures it is [Fe(H2O)6] 3
+3 +2
(Co )t (Co Co )O O4+3
NaCl H3PO4 KSCN NaF
3– [Fe(H2O)5Cl] 2 [Fe(H2O)5(PO4)] [Fe(H2O)5(SCN)] 2 [Fe(H2O)5F]2
83. (A) In the solid state, the CuCl5 ion have Yellow Colourless Red Colourless
trigonal bipyramidal geometry and d-orbitals
are 89. (D) For oxidative addition reaction, metal
1 2 2
dz > dxy = dx – y > dxz = dyz
2 2 2 should have e – rich and have tendency to
charge its oxidation state.
The two ligands on the z-axis and three are in
between xy plane and directly in path of Therefore, [η5 – Cp2 Ti(Me)Cl] do not have
orbitals lobe and feels. There are four e – s in tendency to show oxidative addition reaction.
dxz2 and dyz2 and repel each other, due to this 90. (B)
Ph3P CO
bond length becomes long. Thus, three long
and two short bond is found.
Rh PPh3
84. (B) In metalloenzymes, the metal centers are
covalently linked through the side chains of Ph3P H
the amino acid residues. Therefore Glu, His
In hydroformylation reaction, 16e– species is
Cys are involved in the primary coordination
reactive catalysts. Due to excess PPh3 , it
spheres of metalloenzymes. –s species and lost it catalytic
becomes 20 e
85. (A) [R-BINAP]Ru2– is used for Asymmetric reactivity.
hydrogenation.
So, the rate of reaction decreases in the pre-
[Rh(CO) 2 I2]– for Hydroformylation sence of excess PPh3.
[Pd(PPh3)4] for Heck coupling.
91. (D) Species I

1H
1
Ru 2 a
Cl for Asymmetric hydrogen transfer H 15 Ha
Ts N NH2 1 N
15N
2 b F
H P
1 F
86. (D) The weight of pt-gauze as electrode is 19F
N15
changed as 2 Ha Ha
= 16·0 – 14·5 31P
1
= 1·5g 2
Thus, weight of Cu is 2·0 g brass is 1·5 g. No. of lines
Therefore, percentage weight of Cu in brass = (2n1I1 + 1) (2n2I2 + 1) (2n3I3 + 1) (2n4I4 + 1)
1·5 2N 2F 4H 1H
= × 100 1 1 1
2·0 = (2 × 2 × + 1) (2 × 2 × + 1) (2 × 4 × + 1)
= 75% 2 2 2
1
(2 × 1 × + 1)
87. (A) Cl Cl 2 Cl NH3 2
NH3
Pt Pt =3×3×5×2
Cl Cl Cl Cl = 90
Cl– have trans effect > NH 3 92. (A) Since in [Cu(H2 O) 6 ]+2, John-Teller distor-
Cl NH3 tion observed, so rate of exchange is fast.
NH3
Pt Rest of others are 3d-series dipositive metal
Cl NH3 cations. Thus, rate of exchange with 18H2O
Cis-platin decreases with decrease in size and increase
(anti-tumour agent) in atomic number.
30 | CSIR–Chemical Sciences (Dec.–12)

93. (B) Since, Zn is a lewis acid, so 97. (C) Bimolecular reaction


H H
A + BC → AB + C
Zn 2 OH2 Zn O Zn(OH) There will be greater possibilities of collisions
H
Activates H2O Zinc bound
hydroxide between molecule when volume is smaller
and therefore activation energy decreases
In oxidases an enzyme that catalyse the with decreasing λ.
reduction of O 2 to H2 O or H2 O2, the iron
activates O2 to break the bonding between the 98. (A) There is a strong electron-electron i.e.,
two oxygen's. inter-electron repulsion in many-electron
atom. Due to this the orbital angular momen-
94. (B) Fe+2-porphyrins fail to exhibit reversible
tum (l1 , l 2 ) and sign angular momentum (s1 ,
oxygen transport and cannot differentiate CO
s2 ) are couple together to for total orbital
from O2 . However, the haemoglobin is free
angular momentum (L) and (S).
from both these pitfalls.
O
99. (C) Packing fraction
Fe 3 Fe μ-oxodimer
3 (No. of particles per unit cell)
(hematin) × (Volume of 1 particle)
Unable to bind O2 =
Total volume of unit cell
* Fe+ 2 -porphyrins undergo μ-oxodimer For simple cubic lattice
formation and the same is prevented in case
4
of haemoglobin. 1 × πr3
3
*1 Fe-CO is linear, Fe—O2 is bent recognized = 3
a
by haemoglobin.
4 22 r3
95. (A) SnCl2 behaves as a carbene and it insert = × ×
into the CO—CO bond. 3 7 (2r) 3
Insertion : = 0·52
SnCl2 100. (C) H
(OC)4CO CO(CO)4 Insertion H H
Cl
H N H
(OC)4CO Sn CO(CO)4
Order of group = 4 (C2v = 2 × 2 = 4)
Cl C2v E C2 σV(xz) σV′(yz) √
Metathesis : A1 1 1 1 1 z
Re(CO)5 A2 1 1 –1 –1 Rz
Me
Me Cl B1 1 –1 1 –1 x, Ry
2NaRe(CO)5
Sn (OC)5Re Sn Re(CO)5 B2 1 –1 –1 1 y, Rx
Me Cl
Me No. of unshifted 11 3 3 11
Re(CO)5 atoms
96. (C) We know that, Character per 3 –1 1 1
nM atoms
Density p =
Na3 Reducible 33 –3 3 11
where n is no. of particles per unit cell Representation
M = Molar Mass 1
No. of A1 = [33 × 1 × 1 + (–3) × 1 × 1 + 3
N = Avogadro’s number 4
a = edge length of cube × 1 × 1 + 11 × 1 × 1] = 11
pNa3 1
∴ M = No. of A2 = [33 × 1 × 1 + (–3) × 1 × 1 + 3
n 4
(1·33g cm –3) (6·023 × 1023 mole–1) × (–1) × 1 + 11 × (–1) × +1] = 4
(500 × 100–10 cm)3 1
= No. of B1 = [33 × 1 × 1 + (–3) × (–1) × 1
4 4
M = 25 + 3 × 1 × 1 + 11 × (–1) × 1] = 7
CSIR–Chemical Sciences (Dec.–12) | 31

1 pV
No. of B2 = [33 × 1 × 1 + (–3) (–1) × 1 + 3 103. (A) Z =
4 RT
× (–1) × 1 + 11 × 1 × 1] = 11 p
∫ Z P– 1
( ) dp
Therefore, f = p·e0
R.R., τred = 11A1 + 4A2 + 7B1 + 11B2 For Van der Waal gas
τtrans = B 1 + B2 + A1 f = pePb/RT
(Right side of box) ⇒ If T → ∞, f = p
τrot = B 2 + B1 + A2 If T → 0, then f < p.

∴ τvib = 10A1 + 3A2 + 5B1 + 9B2 pV
104. (C) Z =
Since, A2 is not IR-active. RT
ZRT
Therefore, total IR active vibration modes are ⇒ p = —
10 + 5 + 9 = 24 V
∂P
101. (B) 22Ti – [Ar] 4s2 3d1 4p1
Excited state
⇒ ( )
∂T V
R
= —
V
[(∂Z∂T) T + Z] V

4l + 2 ∂U
No. of microstates = i.p. = ( )
4l + 2 – q q ∂V T

∂P
where l is azimuthal quantum no. of valence = T( ) – p
shell and q is no. of e–s in valence shell ∂T V

∂Z
= T [( ) ·T + Z] – p
R
4×1+2
= — ∂T V
V
4+2–2 2
RT ∂Z 2

6×5× 4
=
V
— ( ∂T ) +
ZRT
V
–p
V —
=
4 ×2×1 ⎧ ZRT
⎨Q — = p
= 15 ⎩ V
102. (B)
at t = 0
A2
1
2A
0
(∂U
∂V) T
RT2 ∂Z
= —
V
( )
∂T V
at equilibrium 1 – α 2α 105. (C) Harmonic Oscillator wave function is
2α 2α given as :
Since, PA = XAP ⇒ XA = =
1 – α + 2α 1 + α 2
ψn (x) = NnHn e–αx /2
P A2 = XA2P and given wave function is
2
1–α 1–α ψ0 = e–Ax
⇒ XA2 = =
1 – α + 2α 1 + α α
Thus, A ∝
2
P
Thus, Kp = A =
2 ( )
2α 2 2
1+α
P
=
4α2
P Since,
⎛ kμ⎞1/2
α = ⎜ h2 ⎟
1–α 1 – α2
P A2
( )
1+α
P ⎝ ⎠
Therefore, A ∝ k1/2
(1 – α )K p = 4α2p
2
106. (C) A = φ1 + φ2
Kp – Kpα2 – 4α2p = 0 D = i(φ1 + φ2)
⎡⎢ Kp ⎤⎥1/2 Combing two real wave functions φ1 and φ2 ,
⇒ α = ⎢⎣ (K p + 4p)⎥⎦ constructed functions φ1 + φ 2 and i(φ1 + φ 2 )
represent the same state because if the two
Kp = α2(K p + 4p) function multiplied by any constant, no
32 | CSIR–Chemical Sciences (Dec.–12)

effect on the state and on the energy because 111. (C) dN = λgNdt
multiply only in eigen value, not in energy Integrating it with appropriate limits
and no effect on wave function.
∫ ∫
N dN t
107. (A) For BCC, h + k + l should be even FCC, = λg dt
N0 N 0
h, k, l all either even or odd
where 0 is even number. [ln N]NN = λgt
0
Thus, A has fCC lattic while B has bcc lattice. N = N0eλg t
108. (B) At low pressure, Langmuir-Hinshelwood 112. (D) 2NOCl(g) → 2NO(g) + Cl 2 (g)
mechanism follows first order kinetics.
ΔH = Ea + (Δng * – 1)RT
2·303 P
∴ K1 = log10 0 Δng * = –1 (Bimolecular reaction)
T Pf
∴ ΔH = 105 kJ mol–1
2·303 10–3
=
10
log10 –4
10 + (–1 – 1) × 2·5 kJ mol–1
2·303 = 100 kJ mol–1
= ×2
10 113. (D) Rotational partition function of H2
= 0·4606 min –1 1
qr = [ Σ (2J + 1) e –βhc BJ(J + 1)
— n M + n2 M 2 + n3 M 3 4 J = 0‚ 2‚ 4
109. (C) Mn = 1 1
n1 + n2 + n3 + 3 Σ (2J + 1) e –βhc BJ(J + 1)]
J = 1‚ 3‚ 5
10 × 1000 + 50 × 2000 + 40
× 4000 114. (C) The potential in Debye Hückel theory is
=
10 + 50 + 40 Z e e–kr
ψr = i 0
= 2700 q r
— n M 2 + n2 M 2 2 + n3 M 3 2 e–kr
Mw = 1 1 ψr ∝
n1 M 1 + n2 M 2 + n3 M 3 r
85000 115. (B) Fundamental vibration :
=
27 ~ ~
Therefore, ΔE0 → 1 = υe(1 – 2xe)
— = 300(1 – 2 × 0·0025)
M
P.D.I. = —w = 298·50 cm–1
Mn
First overtones
85000 1 850
= · = ~ ~
27 2700 729 ΔE0 → 2 = 2ve (1 – 3xe)
110. (B) 2 × (Fe+3 + e– → Fe+2) reduction at = 2 × 300(1 – 3 × 0·0025)
cathode = 595·50 cm–1
Sn +2 → Sn+4 + 2e– Oxidation 116. (B) According to Langmuir Isotherm, frac-
at anode tional surface coverage is given as :
2Fe+3 + Sn+2 → 2Fe+2 + Sn+4 1 1
= +1
θ KeqP
E0 Cell = E cathode – Eanode
1 1
= 0·75V – 0·15V ⇒ –1 =
θ Keq P
= 0·60V
2·303RT θ
Thus, E0 Cell = log Keq ⇒ P =
nF (1 – θ)Keq
0·60V × 2 0·95
log Keq = = 20 =
0·06V 0·05 × 0·9
Keq = 1020 = 21·1
CSIR–Chemical Sciences (Dec.–12) | 33

117. (A) According to virial theorem for simple 121. (B) δ 171 at COOH
harmonic oscillator, the kinetic energy and δ 162 at C-4
potential energy is equal and half of the total
energy. δ 133 at C-2 and C-6
E = K+U QU=K δ 122 at C-1
hw δ 116 at C-3 and C-5
= 2K OH
2
5
hw 5 4
K = =U 3 3 4-hydroxy benzoic acid
4 2
COOH
118. (C) Energy of a hydrogen atom in a state is 1

–hcRH –hCRH Z2
h 2 =
25 (
Q E = –13·6 · 2
n ) 5
OH
4
3
2
Degeneracy = n = 25 6 2
1
119. (A) ψt = c1φ1 + c2φ2 COOH

⎪⎪ H11 – ES11 H12 – ES12 ⎪⎪ 122. (D) C9 H10O3


⎪⎪ H – ES H – ES ⎪⎪ = 0 18 + 2 – 10
21 21 22 22 DBE =
2
Q S 11 = 1, S22 = 1, S12 = S21 = 0
= 5 double bond or ring
^ | φ > = 0, H = H = <φ | H
H11 = <φ1 | H ^ |
1 12 21 1 IR : 3400, 1680 cm–1
^ ^
φ2 = 2·0 = <φ 2 |H | φ1>, H22 = <φ2 | H | φ2> 1 H NMR :
= 3·0 δ 7·8 (1H‚ d‚ J = 8Hz)
⎪⎪ 0 – E 2 – 0 ⎪⎪ }
aromatic protons
7·0 (1H‚ d‚ J = 8 Hz) with m-position
∴ ⎪⎪ 2 – 0 3 – E ⎪⎪ = –3E + E2 – 4 = 0 6·5 (1H, S) aromatic proton with O-
position to —OH to —OCH3
E2 – 3E – 4 = 0
⇒ E2 – 4E + E – 4 = 0 5·8 (1H, S, D2O exchangeable) —OH
⇒ (E + 1) (E – 4) = 0 3·9 (3H, S) with electronegative atom like
oxygen.
Since, ground state energy is lowest.
2·3 (3H, S) adjacent to carbonyl group.
Thus, E = –1·0
120. (D) Polar molecule AB → A + B – has H H
O
molecular orbital CAψA + CBψB HO
CH3
2
C A + CB 2 = 1 (normalization) H OCH3
(6.5)
Since, C B 2 = 90%
123. (A) Since, specific rotation [α] D of a 90%
90 optically pure 2-arylpropanoic acid is +135°.
= = 0·9
100
So, 100% optically pure isomer will show
⇒ CB = 0·95 135
× 100 = 150° [a]D.
C A2 = 10% 90
10 After one hour [α]D reduced to 120°
= = 0·1
100 120
⇒ CA = 0·32 Thus, Optical purity = × 100
150
⇒ C A = 0·32, CB = 0·95 = 80%
34 | CSIR–Chemical Sciences (Dec.–12)

After 3 hours optical purity is reduced to 128. (A)


40%. O OH
150 × 40 C H OH Me CH N Me
So, Specific rotation = = 60°
100 MeHN
N N
124. (A) OH
Me
4 Me
AcO 1
View H
PhNEt2 H N Me C Me
C H C N
Ac2O 2 CH2 Δ 3
2
Claisen 3Ph [3, 3] 1
Allene N 3 1
rearrangement N OH OH
[3, 3] 2
3 Me Me
2
Mannich Intermediate Iminium ion
1 2 1 O
O 3
Me 2 3 4 CH N Me
H N
Ph R-configuration CH
N N Me
Thermal condition, so retention in configu- Me OH
ration w.r.t. Me and Ph.
COOH HOOC COOH
125. (C) 129. (A) 3
2 3 Claisen O
Δ 1 Rearrang-
Conrotation H O 2 COOH ment
Ph 8-membered ring
Ph 1 [3, 3]
COOH 8π-system formation COOH OH OH
H A Oxidation
Disrotation Δ Six membered ring
6π-system formation B
H
Ph COOH
O HOOC COOH
COOH
Decarboxylation O
H CO2
Y
126. (C) OH O
C
O H O Claisen rearrangement followed by oxidative
K H, 1
THF
decarboxylation.
2 1 O
H2 3 130. (A)
3 2
3
Δ [3, 3]-sigmatropic
Keto-enol H3O 2
O OH H
Tautomerism O 1 β-amyrin
HO
127. (A) Triterpene and Secondary alcohol.
NO2 NO2 Acyclic triterpene
Ph Et3N Ph
N N polyene
Cl Acid-base Cl
rh
H Squalene
COOEt Δ
NO2 NO2 Ph NO2 CH3
Ph N Base Ph N N OH
Cl Cl Cl NHCH3
(Et3N) H 1, 3-dipolar Alkaloid H
Cycloaddition species
Phenol O N H OH
Cl
H CH3
OH Ph Ephedine
NO2 NO2
Ph N H3O Ph N Morphine Alkaloid, Secondary
alcohol
CSIR–Chemical Sciences (Dec.–12) | 35

131. (C) 3 OLi


OH O H Cu(acac)2 H
H B 136. (A) N N C :C
Ph 2 Ph COOEt N2 COOEt
O Si face attack
Me 1 Carbene
Re face
COOEt
Re face of carbonyl and Si-face of enolate, : CHCOOEt
syn product will be formed. Chelotropic
MeO reaction MeO
N i-PrMgCl
N hydrogenolysis
132. (A) -i-PrH
MgCl
O O MeOH↓H2 , 10% Pd/C
A
Grignard reagent COOOEt

N
MeO
O Mg 2 Cl least hindered bond will be break
N O
OMe O
137. (C)
N
O Me
Me H
O
Me LiAlH4 Me OH
Pyridoxamine O
133. (A) α -Ketoglutarate ⎯⎯⎯⎯→ Glutamic Et2O
20 Ac2O, Py
acid
Tetrahydrofolate O
Uridine ⎯⎯⎯⎯⎯→ Thymidine
Me2CuLi Flipping Me O C CH3
Thiamine
Pyruvic acid ⎯⎯⎯⎯→ Acetyl coenzyme A Me Et2O
Pyrophosphate Me equatorial Me
1, 3-a, e Me attack O C CH3
(For structure see Lehninger text book)
O
O trans
CHO C O H Me
H OH H OH 138. (D) NaOH + I2 → NaOI + HI
Br2
134. (A) H OH H2O H OH PhCOCH3 + 3NaOI → PhCOCI3 + 3NaOH
H OH Mild H OH PhCOCI3 + NaOH → PhCOONa + CHI3
oxidising
agent
CH2OH CH2OH COCH3 M.w.t. = 120 M.w.t. = 394
H O2
CO2 Fe2 (SO 12g CHI3
2 4)3

CHO 120g acetophenone gives 394g iodoform.


12g acetophenone willl give
H OH 394
× 12 = 39·4g
H OH 120
Since, yield is 75%
CH2OH So, 12g acetophenone will give
39·4 × 75
H CH3 =
100
H
CH3 = 29·5 g CHI3
135. (B) H2,Pd 139. (A)
CH3 CH3
250 C PhI ligand transfer from
Pd(O)L2 Oxidative Ph PdL2
No. 1, 3 interaction Two, H/Me one metal to another
(A) addition
1, 3 interaction I SnBu3
Since, 1 gauche butane interaction energy = Oxidation state increased by two units.
0·9 k-cal/mol. (B) Transmetallation
So, ΔG for given reaction (C)
Reductive
Pd(O)L2 elimination Ph PdL2
= 2 × 0·9 k-cal/mol
Ph
= 1·8 k-cal/mol
36 | CSIR–Chemical Sciences (Dec.–12)

140. (B) This is Reimer-Tiemann Reaction NNHPh NNHPh


CHCl3 NNHPh O
OH NaOH OH
HO H HO H
CH H NH2NHPh H
OH OH
O H OH H OH
H2O2 NaOH O O H
CH2OH CH2OH
OH OH Ozazone
O O
O
OH C H O H O
OH
O HO H
OH OH H C O H HO H
H OH
O O O
C HCOO
H OH
OH H CH2OH
OH D-mannose
OH
144. (B) Li, liq.NH3 H /H3O
141. (A) O tBuOH
O
B D N N N
Ph C C D O H
Syn addition
P B O H
Ph D H , H2O H3O
H3O
OO H O N
P I Ph P B(OH)2 NH2 H
Ph Ph Pd(OAc)2 Ph MeOH KOH
D PPh3, Et3N
D
cis product
Suzuki H
coupling
O O O O
142. (A) O More acidic O O OH H
H H2C N2 MeOH KOH
Et2O CH3 N2
O O
O
OH O O
Me 145. (*) Doubtful question ?
MeMgCl N2
Et O CH3 N2
OCH3 2 OCH3 O O
1, 2-addition
H3O C OEt COOEt COOEt
n-BuLi
OH2 Me Me OMe
Me N
H2O H2O HN OMe
SiMe3
O N
O CH O
3 CH3 CH3OH OMe N
N
N OMe SiMe3
143. (A) H O NNHPh N N SiMe3
COOEt H3O COOEt
H OH H OH
HO H NH2NHPh
HO H
H H H2, Raney Ni
OH OH
H OH H OH NH2
CH2OH CH2OH COOEt
D-glucose
Chemical Sciences
CSIR UGC-NET/JRF Exam.
Solved Paper
June 2013
Chemical Sciences
them, 19.9% of the women and 8.8% of the
PART A men were willing to pay for the facilities—
1. What is the ratio of the men to women
1. During an evening party, when Ms. Black, customers who wanted child care
Ms. Brown and Ms. White met, Ms. Brown
facilities ?
remarked, “It is interesting that our dresses
are white, black or brown, but for each of us 2. If the survey had been conducted during
the name does not match the colour of the the weekend instead, how will the result
dress !”. Ms. White replied, “But your white change ?
dress does not suit you !”. Pick the correct With the above data—
answer. (A) Only 1 can be answered
(A) Ms. White’s dress was brown (B) Only 2 can be answered
(B) Ms. Black’s dress was white (C) Both 1 and 2 can be answered
(C) Ms. White’s dress was black (D) Neither 1 nor 2 can be answered
(D) Ms. Black’s dress was black 6. The map given below shows contour lines
2. Of all the triangles that can be inscribed in a which connect points of equal ground surface
semicircle of radius R with the diameter as elevation in a region. Inverted ‘V’ shaped
one side, the biggest one has the area— portions of contour lines represent a valley
along which a river flows. What is the
(A) R 2 (B) R 2⎯
√2 downstream direction of the river ?
(C) R 2⎯√3 (D) 2R 2 N
3. A square pyramid is to be made using a wire 200
such that only one strand of wire is used for 180
each edge. What is the minimum number of 160
times that the wire has to be cut in order to
make the pyramid ? Scale = 1 : 5000
(A) 3 (B) 7 (A) North (B) South
(C) 2 (D) 1 (C) East (D) West
4. Identify the next figure in the sequence— 7. During a summer vacation, of 20 friends from
a hostel, each wrote a letter to each of all
others. The total number of letters written
was—
(A) 20 (B) 400
(A) (B) (C) 200 (D) 380
8. D C
(C) (D)

5. In a customer survey conducted during North


Monday to Friday, of the customers who
asked for child care facilities in super markets, East
23% were men and the rest, women. Among A B
4 | CSIR-Chemical Sciences (June-2013)

A person has to cross a square field by going (A) 56 (B) 70


from A to C. The person is only allowed to (C) 86 (D) 100
move towards the east or towards the north or
use a combination of these movements. The 14. Define a ⊗ b = lcm (a, b) + gcd (a, b) and
total distance travelled by the person— a ⊕ b = ab + ba . What is the value of (1 ⊕ 2)
⊗ (3 ⊕ 4) ? Here lcm = least common
(A) depends on the length of each step
multiple and gcd = greatest common
(B) depends on the total number of steps divisor—
(C) is different for different paths
(A) 145 (B) 286
(D) is the same for all paths
(C) 436 (D) 572
9. A crow is flying along a horizontal circle of
radius R at a height R above the horizontal 15. There is an equilateral triangle in the XY
ground. Each of a number of men on the plane with its centre at the origin. The
ground found that the angular height of the distance of its sides from the origin is 3.5 cm.
crow was a fixed angle θ (< 45°) when it was The area of its circumcircle in cm2 is—
closest to him. Then all these men must be on (A) 38.5 (B) 49
a circle on the ground with a radius— (C) 63.65 (D) 154
(A) R + R sin θ (B) R + R cos θ 1 1 1
16. What is the value of + + +…
(C) R + R tan θ (D) R + R cot θ 1×2 2×3 3×4
10. How many pairs of positive integers have gcd to ∞ ?
20 and lcm 600 ? 2
(A) (B) 1
(gcd = greatest common divisor; lcm = least 3
common multiple) (C) 2 (D) ∞
(A) 4 (B) 0 17. A sphere of iron of radius R/2 fixed to one
(C) 1 (D) 7 end of a string was lowered into water in a
11. Two integers are picked at random from the cylindrical container of base radius R to keep
first 15 positive integers without replacement. exactly half the sphere dipped. The rise in the
What is the probability that the sum of the level of water in the container will be—
two numbers is 20 ?
3 1
(A) (B)
4 21
1 1
(C) (D)
105 20
12. A daily sheet calendar of the year 2013
contains sheets of 10 × 10 cm size. All the
sheets of the calendar are spread over the (A) R /3 (B) R /4
floor of a room of 5 m × 7.3 m size. What (C) R /8 (D) R /12
percentage of the floor will be covered by 18. Choose the largest number—
these sheets ? (A) 2500 (B) 3400
(A) 0.1 (B) 1 (C) 4 300 (D) 5200
(C) 10 (D) 100
19. A crystal grows by stacking of unit cells of
13. How many rectangles (which are not squares) 10 × 20 × 5 nm size as shown in the diagram
are there in the following figure ? given below. How many unit cells will make
a crystal of 1 cm 3 volume ?

5 nm nm
20
10 nm
Unit cell (not to scale)
CSIR-Chemical Sciences (June-2013) | 5

24. The oxidation state of molybdenum in [(η7 –


tropylium) Mo (CO3 ]+ is—
(A) +2 (B) +1
(C) 0 (D) –1
nm
5 nm 20 25. The reaction of [PtCl4 ]2– with two equivalents
10 nm of NH3 produces—
Crystal (not to scale) (A) cis-[Pt(NH3 )2Cl2]
(A) 106 (B) 109 (B) trans-[Pt(NH3)2Cl2]
(C) 1012 (D) 1018 (C) Both cis-[Pt(NH3 )2Cl2] and
20. A solid cylinder of basal area A was held trans-[Pt(NH3)2Cl2]
dipped in water in a cylindrical vessel of basal (D) cis-[Pt(NH3 )2Cl4]2–
area 2A vertically such that a length h of the
cylinder is immersed. The lower tip of the 26. The electronic transition responsible for the
cylinder is at a height h from the base of the colour of the transition metal ions is—
vessel. What will be the height of water in the (A) dπ → dσ (B) dπ → dσ*
vessel when the cylinder is taken out ? (C) dπ → dπ* (D) dσ → dπ*
27. The number of metal-metal bonds in
[W2 (OPh)6] is—
h (A) 1 (B) 2
h (C) 3 (D) 4
28. The Mulliken symbols for the spectroscopic
3 states arising from the free-ion term F are—
(A) 2h (B) h
2 (A) T2g + Eg (B) T1g + T2g + T1u
4 5
(C) h (D) h (C) T1g + T2g + A2g (D) A1g + T2g + T1g
3 4
29. Which of the following is used as propellant
for whipping creams ?
PART B
(A) N2O (B) NO
21. Which of the following pairs has the highest (C) N2O3 (D) N2O5
difference in their first ionization energy ? 30. Flame proof fabrics contain—
(A) Xe, Cs (B) Kr, Rb (A) H2NC(O)NH2·Na2 SO4
(C) Ar, K (D) Ne, Na (B) H2NC(S)NH 2 ·Na2 SO4
22. The ligand in uranocene is— (C) H2NC(O)NH2·H3 PO4
(A) C 8 H82– (B) C 5 H52– (D) H2NC(S)NH 2 ·H3 PO4
(C) C 6 H6 (D) C 4 C 4 2–
31. Among the compounds A–D, those which
23. In metal–olefin interaction, the extent of hydrolyse easily are—
increase in metal → olefin π-back-donation (A) NCl 3 (B) NF3
would— (C) BiCl3 (D) PCl3
(A) Lead to a decrease in C = C bond length
(B) Change the formal oxidation state of the 32. The coordination geometry of copper(II) in
metal the type I copper protein plastocyanin is—
(C) Change the hybridisation of the olefin (A) Square planar
carbon from sp2 to sp3 (B) Tetrahedral
(D) Increase with the presence of electron (C) Octahedral
donating substituents on the olefin (D) Distorted tetrahedral
6 | CSIR-Chemical Sciences (June-2013)

33. The metal ions present in the active site of respectively. What is the probability that the
nitrogenase enzyme co-factor are— system energy will be observed to be E1 ?
(A) Fe, Mo (B) Fe, W 3 3
(C) Fe, Cu (D) Fe, Ni (A) (B)
16 4
34. The reaction [(CO)5 Mn(Me)] + CO →
1 1
[(CO)5 Mn{(C(O)Me}] is an example for— (C) (D)
4 4
(A) Oxidation addition
(B) Electrophilic substitution 40. What is the atomic term symbol for helium
atom with electronic configuration 1s2 ?
(C) Nucleophilic substitution
(D) Migratory insertion (A) 2 S 1/2 (B) 1 P 0
(C) 1 S 0 (D) 1 S 1
35. The number of EPR signals observed for
octahedral Ni (II) complexes is— 41. A molecule contains the following symmetry
(A) One (B) Two operations : E, 2C 6 , 2C3 , C2 , 3σd , 3σ v. The
(C) Three (D) Zero number of classes and order of the symmetry
point group is—
36. For neutron activation analysis of an element, (A) 3, 12 (B) 5, 12
the favourable characteristics of both the
target and the product are from the (C) 6, 12 (D) 6, 6
following— 42. A triatomic molecule of the type AB 2 shows
1. High neutron cross-section area of target two IR absorption lines and one IR-Raman
2. Long half-life of the product line. The structure of the molecule is—
3. Low neutron cross-section area of target (A) B–B–A (B) B–A–B
4. Low half-life time of the product B A
The correct characteristics from the above
are— (C) (D)
(A) 1 and 2 (B) 2 and 3 B A B B
(C) 3 and 4 (D) 1 and 4
37. The concentrations of a species A undergoing 43. In NMR spectroscopy, the product of the
the reaction A → P is 1.0, 0.5, 0.33, 0.25 mol nuclear ‘g’ factor (gN), the nuclear magneton
dm–3 at t = 0, 1, 2 and 3 seconds, respectively. (βN ) and the magnetic field strength (B0 )
The order of the reaction is— gives the—
(A) Two (B) One (A) Energy of transition from α to β state
(C) Zero (D) There (B) Chemical shift
(C) Spin-spin coupling constant
38. The difference in energy levels of n = 2 and
(D) Magnetogyric ratio
n = 1 of a particle in a one dimensional box is
6 units of energy . In the same units, what is 44. An aqueous mixed solution of NaCl and HCl
the difference in energy levels of n = 3 and is exactly neutralized by an aqueous NaOH
n = 2 for the above system ? solution. The number of components in the
(A) 4 (B) 5 final mixture is—
(C) 9 (D) 10 (A) 1 (B) 2
(C) 3 (D) 4
39. The wave function ψ of a certain system is
the linear combination 45. The lowest pressure at which the liquid phase
of a pure substance can exist is known as—
1 3 (A) Critical point pressure
ψ= ψ + ψ
4 1 4 2 (B) Super-incumbent pressure
where ψ1 and ψ 2 are energy eigen functions (C) Triple-point pressure
with eigen values (non-degenerate) E1 and E 2 , (D) Saturation vapour pressure
CSIR-Chemical Sciences (June-2013) | 7

46. A chemical reaction involving non-linear 54. The correlation coefficient of two parameters
molecule + non-linear molecule non-linear is found to be –0.99. It may be concluded that
activated complex. The number of vibrational the two parameters are—
degrees of freedom in the activated complex, (A) Strongly correlated
containing N atoms, is—
(B) Almost uncorrelated
(A) 3N – 5 (B) 3N – 6
(C) Connected by a cause-effect relationship
(C) 3N – 7 (D) 3N – 8
(D) Not connected by a cause-effect relation-
47. Calculate the total number of microstates for ship
6 identical particle with their occupation
numbers {1, 2, 3} in three states is— 55. The IUPAC name for the compound given
(A) 6 (B) 12 below is—
(C) 60 (D) 720 OH
PH
48. If the concentration (c) is increased to 4 times
its original value (c), the change in molar (A) (2R, 3Z)-7-phenylhept-3-en-2-ol
conductivity for strong electrolytes is (where
b is Kohlrausch constant)— (B) (2S,3Z)-7-phenylhept-3-en-2-ol
(C) (2R,3E)-7-phenylhept-3-en-2-ol
(A) 0 (B) b√
⎯c
(D) (2S,3E)-7-phenylhept-3-en-2-ol
(C) 2b√
⎯c (D) 4b√
⎯c
56. Among the following esters, the one that
49. In atom recombination reactions— undergoes acid hydrolysis fastest is—
(A) Ea = 0, ΔS# = +ve, ΔH# = +ve
OCOCH3
(B) Ea = 0, ΔS# = –ve, ΔH# = –ve OCOCH3
(B)
(C) Ea = +ve, ΔS# = –ve, ΔH# = –ve (A) OH
(D) Ea = +ve, ΔS# = +ve, ΔH# = +ve
OCOCH3
50. In the Lindemann mechanism of unimolecular OCOCH3
(C)
reactions, the observed order at low concen- (D)
tration is— OH CH3
(A) 0.5 (B) 1
(C) 1.5 (D) 2 57. Reaction of cyclohexyl benzyl ether with
hydrogen in the presence of 10% Pd/C
51. The aggregation of surfactant molecules is
yields—
known as—
(A) cyclohexanol and toluene
(A) Micelles (B) Clusters
(B) cyclohexanol and benzyl alcohol
(C) Gel (D) Colloid
(C) cyclohexane and benzyl alcohol
52. The coordinates for the atoms in a body-
(D) cyclohexane and toluene
centred cubic unit cell are—
(A) (0, 0, 0) and (1/2, 0, 0) 58. Among the following dibromocyclohexanes,
(B) (0, 0, 0) and (1/2, 1/2, 1/2) the one that reacts fastest with sodium iodide
to give cyclohexene is—
(C) (0, 0, 0) and (0, 1/2, 0)
(D) (0, 0, 0) and (0, 0, 1/2) Br
Br
(B)
° for (100) plane (A) Br
53. The interplanar distance (A) Br
in a cubic structure with the lattice parameter
° is— Br Br
of 4 A
(A) 1 (B) 2 (C) (D)
(C) 4 (D) 8 Br Br
8 | CSIR-Chemical Sciences (June-2013)

59. Match the following drugs with their 63. In the compound given below, the hydrogens
medicinal activity— marked A and B are—
(a) 5-fluorouracil 1. Anti-bacterial O
(b) Amoxicillin 2. Cholesterol
lowering 3. Anticancer
Ph N Ph
4. Anti-inflammatory HA
(a) (b) HB
COOH
(A) 1 2 (A) Homotopic (B) Isotopic
(B) 4 3 (C) Enantiotopic (D) Diastereotopic
(C) 3 4
64. In the IR spectrum, the absorption band due
(D) 3 1 to carbonyl group in phenyl acetate appears
60. The major product formed in the following at—
reaction sequence is— (A) 1800 cm–1 (B) 1760 cm–1
Me 1. CH3CO3H (C) 1710 cm–1 (D) 1660 cm–1
O 2. LiAIH4
65. The reactive intermediate involved in the
following reaction is—
Me Me OH
OH NaNH2

(A) OH (B) OH Br N-Me N


H Me

Me (A) A carbocation (B) A carbanion


Me
OH OH (C) A free radical (D) An aryne
(C) (D) 66. Number of isoprene units present in lupeol
OH OH is—

61. The biosynthetic precursor for the steroids


H
is—
(A) Secologanin H
(B) Shikimic acid H
(C) Mevalonic acid HO H
Lupeol
(D) α-ketoglutaric acid
(A) Two (B) Four
62. The major product formed in the following
(C) Six (D) Eight
reaction sequence is—
67. The heterocyclic ring present in the amino
Br
n Bu acid histidine is—
1. 3 4 SnCl‚ NaBH ‚ AlBN
⎯⎯⎯⎯⎯⎯⎯⎯⎯→ (A) Pyridine (B) Tetrahydropyrrole
O OEt 2. H2SO 4 ; CrO3
(C) Indole (D) Imidazole
68. The gauche conformation (ϕ = 60°) of
(A) (B) n-butane posseses—
O O O O
(A) Plane of symmetry; and is achiral
(B) C 2 -axis of symmetry; and is chiral
(C) (D) (C) Centre of symmetry; and is achiral
O O O O (D) Plane of symmetry; and is chiral
CSIR-Chemical Sciences (June-2013) | 9

69. The following photochemical conversion 74. Among the following, those can act as
proceeds through— Mossbauer nuclei are—
O Me O 1. 129 I 2. 57Co
H
3. 57Fe 4. 121 Sb

(A) 1, 2, 3 and 4
H (B) 2, 3 and 4 only
(C) 1, 2 and 4 only
(A) Barton reaction
(D) 1, 3 and 4 only
(B) Paterno-Buchi reaction
(C) Norrish type I reaction 75. Which of the pairs will generally result in
tetrahedral coordination complexes, when
(D) Norrish type II reaction ligands are Cl – or OH– —
70. Among the following dienes, the one that 1. Be(II), Ba(II) 2. Ba(II), Co(II)
undergoes a degenerate Cope rearrangement 3. Co(II), Zn(II) 4. Be(II), Zn(II)
is—
(A) 1 and 2 (B) 2 and 3
H
(C) 3 and 4 (D) 1 and 4
(A) (B) 76. Silica gel contains [CoCl4 ]2– as an indicator.
H When activated, silica gel becomes dark blue
while upon absorption of moisture, its colour
(C) (D) changes to pale pink. This is because—
(A) Co(II) changes its coordination from
tetrahedral to octahedral
PART C (B) Co(II) changes its oxidation state to
Co(III)
(C) Tetrahedral crystal field splitting is NOT
71. A radioisotope 41Ar initially decays at the rate
equal to octahedral crystal field splitting
of 34,500 disintegrations/minute, but decay
rate falls to 21,500 disintegrations/minute (D) Co(II) forms kinetically labile while
after 75 minutes. The t1/2 for 41Ar is— Co(III) forms kinetically inert complexes
(A) 90 minutes (B) 110 minutes 77. For the metalloprotein hemerythrin, the
statement that is not true is—
(C) 180 minutes (D) 220 minutes
(A) There are two iron centers per active site
72. The orders of reactivity of ligands, NMe 3 , (B) Both iron centres are hexacoordinated in
PMe3 and CO with complexes MeTiCl3 and the active state
(CO) 5 MO (thf) are— (C) One iron is hexacoordinated while the
(A) CO > PMe3 > NMe3 and CO > NMe3 > other is pentacoordinated in the active
PMe3 state
(B) PMe3 > CO > NMe3 and NMe3 > CO > (D) It is found in marine invertebrates
PMe3 78. For a tetragonally distorted Cr(III) complex,
(C) NMe 3 > PMe3 > CO and CO > PMe3 > zero-field splitting results in the following
NMe 3 number of Kramers doublets—
(A) 1 (B) 2
(D) NMe 3 > CO > PMe3 and PMe3 > NMe3 > (C) 3 (D) 4
CO
79. Intense band at 15000 cm–1 in the UV-visible
73. The number of lone-pairs are identical in the spectrum of [Bu4 N] 2 Re 2 Cl8 is due to the
pairs— transition—
(A) XeF4, ClF3 (B) XeO4 , ICl4 – (A) π → π* (B) δ → δ*

(C) XeO2 F 2 , ICl4 (D) XeO4 , ClF3 (C) δ → π * (D) π → δ*
10 | CSIR-Chemical Sciences (June-2013)

80. Electron change in reduction of Ce(SO4 )2, 86. The species with highest magnetic moment
KMnO4, HNO 2 and I 2 with hydrazine in (spin only value) is—
acidic medium, respectively is— (A) VCl 6 4 –
(A) 1e, 1e, 2e and 4e (B) (η5–C 5 H5)2Cr
(B) 1e, 3e, 2e and 4e (C) [Co(NO2 )6]3–
(C) 2e, 3e, 1e and 4e
(D) [Ni(EDTA)]2–
(D) 2e, 4e, 1e and 3e
81. The compound that will behave as an acid in 87. The number of metal-metal bonds in
Ir 4 (CO) 12 is—
H2SO4 is—
(A) CH3 COOH (B) HNO 3 (A) 4 (B) 6
(C) HClO (D) H O (C) 10 (D) 12
4 2

82. Among the oxides of nitrogen, N2 O3, N2 O4 88. Three bands in the electronic spectrum of
[Cr(NH3 )6] 3 + are due to the following
and N2 O5, the compound(s) having N–N bond
transitions :
is/are—
(A) N2O4 and N 2 O5 1. 4 A2g → 4 T1g 2. 4 A2g → 4 T2g
(B) N2O3 and N 2 O5 3. 4 A2g → Eg
2

(C) N2O3 and N 2 O4 Identify the correct statement about them—


(D) N2O5 only (A) Intensity of 1 is lowest
(B) Intensity of 3 is lowest
83. The treatment of PhBr with n-BuLi yields—
(A) 2n-BuPh + Br2 + Li 2 (C) Intensities of 1, 2 and 3 are similar
(B) PhPh + octane + 2LiBr (D) Intensities of 2 and 3 are similar
(C) n-BuPh + LiBr 89. Identify the pairs in which the covalent radii
(D) PhLi + n-BuBr of elements are almost similar—
1. Nb, Ta 2. Mo, W
84. Though cyclobutadiene (C4 H4) is highly
unstable and readily polymerizes in its free 3. La, Lu 4. Sc, Y
state, its transition metal complexes could be (A) 1 and 2 only
isolated because— (B) 1 and 3 only
(A) It engages in long-range interaction with (C) 2 and 3 only
transition metals
(D) 1, 2 and 3 only
(B) It gains stability due to formation of
C 4 H42– on binding to transition metals 90. Consider the following lanthanide(III) ions :
(C) Its polymerization ability reduces in 1. Nd(III) 2. Gd(III)
presence of transition metal 3. Dy(III)
(D) It becomes stable in presence of The magnetic moment closest to the spin only
transition metals due to formation of value is(are) for—
C 4 H42+ (A) 2 only (B) 1 and 2 only
85. Identify the order representing increasing (C) 1 and 3 only (D) 2 and 3 only
π-acidity of the following ligands— 91. The Δt of the following complexes—
C 2 F 4 , NEt3 , CO and C2 H4 1. [CoCl4]2–
(A) CO < C2F 4 < C2H4 < NEt3 2. [CoBr4 ]2– and
(B) C 2 F 4 < C2H4 < NEt3 < CO 3. [Co(NCS)4 ]2– follows the order
(C) C 2 H4 < NEt3 < CO < C2 F 4 (A) 3 > 1 > 2 (B) 1 > 2 > 3
(D) NEt3 < C2H4 < C2F 4 < CO (C) 2 > 1 > 3 (D) 3 > 2 > 1
CSIR-Chemical Sciences (June-2013) | 11

92. In complexometric titration (B) AB + AB A2 B 2 (fast)


S(substrate) + T(titrant) → P(product) slow
The end point is estimated spectrophoto- A2B 2 + B2 ⎯→ 2AB2
metrically. If S and P have ε = 0, the shape of slow
(C) AB + B2 ⎯→ AB3
the titration curve would look like—
fast
AB3 + AB ⎯→ 2AB2
(D) AB + B2 AB3 (fast)
(A) A (B) A
slow
AB3 + AB ⎯→ 2AB2
T T 97. Observe the following statements—
1. In the H2 –O2 reaction, explosion occurs
when the rate of chain branching exceeds
(C) A (D) A that of chain termination.
2. The order of the reaction, nA → products,
T T is 2.5. For this reaction,
93. Identify the chiral complexes from the t1/2 ∝ [A]0 –3/2
following— 3. Unimolecular gas phase reactions are
1. [Cr(EDTA)]– 2. [Ru(bipy)3]3+ second order at low pressure but become
3. [PtCl(diene)] + first order at high pressure.
(A) 1 only (B) 1 and 2 only Which of the following is correct ?
(C) 1 and 3 only (D) 2 and 3 only (A) 1, 2 and 3 are correct
94. Distribution ratio of ‘A’ between CHCl3 and (B) Only 2 is correct
water is 9.0. It is extracted with several, 5 mL (C) Only 3 is correct
aliquot of CHCl3 . The number of aliquots (D) 1 and 2 are correct
needed to extract 99.9% of ‘A’ from its 5 mL
aqueous solution are— 98. For the particle-in-a-box problem in (0, L), an
approximate wave function is given
(A) 2 (B) 3 – as x (L/2
– x) (L – x). The average energy E for such a
(C) 4 (D) 5
state will obey—
95. The correct equilibrium order for the inter- – h2 –
h2 h2
conversion of different forms of SiO2 is— (A) 2 <E 2 (B) E >
8mL 2mL 2mL2
(A) Tridymite quartz cristobalite
h2 – h2 – h2
liquid SiO2 (C) <E< (D) 0 < E <
4mL2 2mL2 8mL2
(B) Quartz tridymite cristobalite
99. For two variables x and y, the following data
liquid SiO2 set is given :
(C) Quartz cristobalite tridymite x y
liquid SiO2 –1 1
(D) Cristobalite tridymite quartz 0 2
liquid SiO2 1 3
96. The rate equation for the reaction, 2AB + B2 The correct statement for the covariance A
→ 2 AB2 , is given by and correlation coefficient B of x and y is—
rate = k [AB] [B2 ] (A) A = 2/3, B=1
A possible mechanism consistent with this (B) A = –2/3, B =1
rate law is— (C) A = –2/3, B = –1
slow
(A) 2AB + B2 ⎯→ 2AB2 (D) A = 0, B=0
12 | CSIR-Chemical Sciences (June-2013)

100. The hydrogenic orbital with the form of the 107. Identify the Hückel determinant for cyclo-
radial function butadiene—
r2 (α1 – r) (α2 – r) exp [–βr], where α1α2 ⎪
α–E β 0 0 ⎪
and β are constants, may be identified as a— ⎪β α–E β 0 ⎪
(A) 3d orbital (B) 4f orbital (A)
⎪0 β α–E β ⎪
(C) 5d orbital (D) 5f orbital ⎪0 0 β α–E ⎪
⎪ ⎪
2
101. The operator [x, [x, p ]] is identical with—
α–E β 0 β
(A) [px, [x, p]] (B) [xp, [x, p]]
⎪β α–E β 0 ⎪
(C) –[p, [x2, p]] (D) [x, [x2 , p]] (B)
⎪0 β α–E β ⎪
102. For the particle-in-a-box problem in (0, L), ⎪ β β 0 α–E ⎪
the value of (x3) in the n → ∞ limit would
be— ⎪ α–E β 0 β ⎪
⎪ β α–E β 0 ⎪
⎪ 0 β ⎪
(A) L3 /6 (B) L3 /3 (C)
β α–E
⎪ β α–E ⎪
(C) L3 /4 (D) L4 /4
0 β
103. Identity the Mulliken notation for the
following irreducible representation ⎪ α–E β 0 β ⎪
E En nC i σh ⎪ β α–E β 0 ⎪
⎪ 0 β ⎪
2
(D)
β α–E
⎪ 0 α–E⎪
1 1 –1 –1 –1
(A) A'1u (B) A"2u 0 β
(C) B'2u (D) A'2u 108. On mixing 120 mL of 0.05 M CH 3 COOH
104. Identify the point group symmetry of the and 40 mL of 0.05 M of NaOH, the pH of
following molecule (all C–C bond lengths the solution is—
are equal)— ( pK a = –log Ka )
X (A) pKa + 0.69 (B) pKa + 0.301
(C) pKa (D) pKa – 0.69
109. A system consists of gaseous H2 , O2 , H2 O
X and CO2 where the amount of CO2 is speci-
X
fied and the equilibrium constant for the
reaction 2H2 (g) + O2 (g) 2H2 O(g) is
known. The number of degrees of freedom
X of the system is—
(A) C 2v (B) S 4 (A) 2 (B) 3
(C) D2d (D) D4d (C) 4 (D) 5
110. “Colloids are thermodynamically unstable
105. The ground state term symbol for Nb with reference to bulk but kinetically
(atomic number 41) is 6 D. The electronic stable”. Identify the correct pair—
configuration corresponding to this term
symbol is— Statements Reasons
(A) [Kr] 4d3 5s2 (B) [Kr] 4d4 5s1 (a) Thermodynamically (c) Interfacial surface
tension
(C) [Kr] 4d5 5s0 (D) [Kr] 4d3 5s15p1 (b) Kinetically stable (d) Electrical double
106. In the presence of an external magnetic layer
field (normal Zeeman effect), the transition (A) (a) ↔ (d) and (b) ↔ (c)
1 D → 1 P splits into—
2 1 (B) (a) ↔ (c) and (b) ↔ (d)
(A) 9 lines (B) 8 lines (C) (a) ↔ (c) and (b) ↔ (c)
(C) 7 lines (D) 6 lines (D) (a) ↔ (d) and (b) ↔ (d)
CSIR-Chemical Sciences (June-2013) | 13

111. An AX system gave 4 lines at 4.72, 4.6, 1.12 117. A plane of spacing d shows first order Bragg
and 1.0 ppm away from TMS using an nmr diffraction at angle θ. A plane of spacing
spectrometer operating at 100 MHz. What 2d—
are the values of JAX (in Hz) and δ AX (in
(A) Shows Bragg diffraction at 2θ
ppm), respectively ?
(A) 12 and 3.6 (B) 6 and 3.6 (B) Shows Bragg diffraction at θ/2
(C) 12 and 2.86 (D) 6 and 2.86 (C) Shows Bragg diffraction at sin–1
sin θ
112. The equilibrium population ratio (nj/ni) of a
doubly-degenerate energy level (Ej) lying at
( ) 2
energy 2 units higher than a lower (D) Shows Bragg diffraction at sin–1
nondegenerate energy level (Ei), assuming
kBT = 1 unit, will be— ( sin 2θ
2 )
(A) 2e–2 (B) 2e2
118. In the formation of H2 molecule from 2H
(C) e2 (D) e–2 atoms placed at positions A and B, and
113. Which of the following statements is true for separated by a distance rAB, a part of the
a cyclic process ? spatial wave function is—
(A) Oƒ dq = 0 ϕA(1) ϕA(2) + ϕB(1) ϕB(2)
(B) Oƒ dw = 0 (A) This is a covalent term and is important
(C) Heat can be completely converted into as rAB → ∞
work
(B) This is an ionic term and is important
(D) Work can be completely converted into as rAB → ∞
heat
(C) This is a covalent term and is important
114. Identify, from the following, the correct as rAB → 0
ionic strengths for (a) a 0.01 molal solution
of NaCl and (b) a 0.01 molal solution of (D) This is an ionic term and is important
Na 2 SO4 — as rAB → 0
(A) (a) 0.010 mol kg –1 (b) 0.010 mol kg–1 119. A 0.1 M solution of compound A shows
(B) (a) 0.010 mol kg –1 (b) 0.030 mol kg–1 50% transmittance when a cell of 1 cm
(C) (a) 0.010 mol kg –1 (b) 0.025 mol kg–1 width is used at λ 1 nm. Another 0.1 M
(D) (a) 0.010 mol kg –1 (b) 0.015 mol kg–1 solution of compound B gives the optical
115. A system has 100 degenerate energy levels density value of 0.1761 using 1 cm cell at λ1
and 100 bosons are kept in it. Find the nm. What will be the transmittance of a
entropy of the system at equilibrium— solution that is simultaneously 0.1 M in A
(A) 10–2 k B (B) 102 k B and 0.1 M in B using the same cell and at
(C) 460.6 kB (D) 4.606 kB the same wavelength ?

116. Which is correct Nernst equation for redox (log 20 = 1.301; log 30 = 1.4771; log 50 =
reaction O + ne R? 1.699)
RT [O] (A) 33.3% (B) 50%
(A) E = E0 – ln
nF [R] (C) 66.7% (D) 70%
nF
[O] (E – E 0 )
(B) = e RT 120. Using standard equation for intrinsic
[R] —a
[O] –
nF
(E – E 0 ) viscosity [η ] = KMv , for a solution of
(C) = e RT polymer and any information from the graph
[R]
RT

[O] (E – E 0 ) identify viscosity-average molar mass ( Mv)
(D) = e nF
[R] [given that a = 0.5, K = 5 × 10–5 Lg–1].
14 | CSIR-Chemical Sciences (June-2013)

0.055 (A) A is the major product and it is Cram


product
η − η0 1 (B) A is the major product and it is anti-
η0 c Cram product
(C) B is the major product and it is a Cram
product
0.050 (D) B is the major product and it is anti-
0 c 10
Cram product
(A) 103 g/mol (B) 104 g/mol
124. Which one of the following statements is
(C) 105 g/mol (D) 106 g/mol true for the following transformation ?
121. Among the following, the correct statement O O
for the following reaction is— O O O
+
O
1. MeMgBr, Et2O
+
2. H3PO4
A B A B
(A) A is the major product and it will have (A) Suitable reagent is m-CPBA and B is
five signals in the proton decoupled the major product
13C NMR spectrum (B) Suitable reagent is m-CPBA and A is
(B) A is the minor product and it will have the major product
eight signals in the proton decoupled (C) Suitable reagent is aq. H2 O2/NaOH and
13C NMR spectrum
B is the major product
(C) B is the major product and it will have (D) Suitable reagent is aq. H2 O2/NaOH and
five signals in the proton decoupled A is the major product
13C NMR spectrum
125. The compound formed in the following
(D) B is the minor product and it will have reaction sequence is—
five signals in the proton decoupled
13C NMR spectrum 1. Li, liq.NH3, t-BuOH
2. 10% aq. H2SO4
122. For the following three step conversion of A MeO
to B, the appropriate sequence of reactions
HO3S
is—
(A)
OH O O H2 N
CHO
OH HO3S
(B)
A B HO
(A) MnO2 ; (CH2OH)2 /p-TSA; PCC
(C)
(B) PCC; MnO2; (CH2OH)2 /p-TSA
(C) PCC; (CH2OH)2 /p-TSA; Jones’ reagent
(D) Jones’ reagent; (CH2 OH)2 /p-TSA; (D)
MnO2 O

123. Which one of the following statements is 126. Among the following compounds, the one
true for the following transformation ? which has highest dipole moment is—
OH OH
CHO MgBr, Et2O
(A) (B)
Ph Ph + Ph
Me Me Me
A B (C) (D)
CSIR-Chemical Sciences (June-2013) | 15

127. In the UV-Vis spectrum, a diterpenoid 130. An organic compound exhibited the follo-
exhibited a λ max at 275 nm. The compound, wing 1H NMR spectral data :
among the choices given below, is— δ 7.80 (2, H, d, J = 8 Hz), 6.80 (2 H, d, J = 8
Hz), 4.10 (2 H, q, J = 7.2 Hz); 2.4 (3 H, s),
1.25 (3 H, t, J = 7.2Hz). The compound,
among the choices given below, is—
(A) O
H
HOOC
(A)
EtO
O
(B)
H
HOOC (B)
MeO
O

(C) (C)
H
HOOC HO
O

(D) OMe
(D) Et
H
HOOC 131. α-Pinene on reaction with dilute alkaline
KMnO4 produces a diol, which on further
128. The major product formed in the following oxidation with chromium trioxide gives
reaction is— product A, which undergoes a positive
1. Δ haloform test. The compound A is—
+
NO2 2. TiCl3, HCl
O
COOH
O (A) (B)
COOH
(A) (B)
O O
O
NH2 (C) (D)
(C) (D) COOH
NH2
132. The major product formed in the reaction of
129. In the broadband decoupled 13 C NMR guanosine with one equivalent of methyl
spectrum, the number of signals appearing iodide is—
for the two pyrenediols A and B, O
Me
respectively, are— N I
HN
OH (A)
OH H2N N N
R
O
HO OH Me N
A B N
(A) Eight and eight (B) Eight and sixteen (B)
H 2N N N
(C) Five and ten (D) Five and eight R
16 | CSIR-Chemical Sciences (June-2013)

O (B) Phe-Leu + toluene + carbon dioxide


N (C) Phe-Leu + benzyl alcohol + carbon
HN
(C) dioxide
H2N N N I (D) Gly-Leu + benzyl alcohol + carbon
R dioxide
Me

O 135. Among the following, the most suitable


reagent for carrying out resolution of
N racemic 3-methylcyclohexanone is—
(D) HN
N HO
Me-HN N
R

133. The major product formed in the following (A)


reaction is— OH

O OH
CN COOMe
Δ (B) MeOOC
+
OH
O Me
O
OH
HH (C) Ph
O
(A)
HH
OH
O

O
(D)
HH
HO
(B)
HH 136. In the following reaction sequence, structures
O
of the major products X and Y are—
CN O
H NaOEt Zn/AcOH
+ (COOEt)2 ⎯⎯→ X ⎯⎯⎯→ Y
NO2
(C)
H O
O
(A) X is COOEt
HO NO2
NC
(D)
Y is COOEt
OH
N
134. Reaction of the dipeptide, given below, with H
hydrogen in the presence of 10% palladium O
over carbon, produces a mixture of—
(B) X is COOEt
NO2
H O
Ph O N OH OH
N
O H O Y is
Ph
N O
(A) Gly-Leu + toluene + carbon dioxide H
CSIR-Chemical Sciences (June-2013) | 17

O O amount of p-TSA furnished the major


O product A. The structure of A is—
(C) X is OEt
Y is O
N HO
NO2 (A) O O
H
Ph
O
O COOEt HO
(D) X is OEt Y is (B) O O
NO2 NH
Ph
137. Consider the following reaction sequence : OH
OH OH
(C) O
Sn/HCl
O
O2N (90%) H2N Ph
OH (70%)
HNO3
OH
OH O
Ac2O
(D) O
(90%) AcHN Ph
OH OH Ac2O OH
Sn/HCl 140. The major product formed in the following
(90%) (90%)
(25%)
NO2 NH2 NHAc reaction is—
The overall yield for the formation of p- OTs
H
hydroxyacetanilide and o-hydroxyaceta-
KH/THF
nilides from phenol, respectively, are appro- ⎯⎯⎯→
ximately—
(A) 57 and 20% (B) 57 and 68% OH
(C) 83 and 68% (D) 83 and 20% H
H
138. The most stable conformations of 1, 2–diflu- (A) O (B)
oroethane and dl-2,3-butanediol are—
OH
F OH
H F Me H
(A) and
H H H
(C) (D)
Me
H OH O O
F OH
H H Me OH 141. The major product formed in the following
(B) and reaction is—
H H H Me
F H OH
1. NaH‚ CS ‚ Mel
2
F OH ⎯⎯⎯⎯⎯⎯→
H H H OH 2. 200°C
(C) and
H H Me H
F Me

F Me (A) (B)
H F H OH
(D) and
H H H OH
H Me

139. Reaction of (S)-1,2,4-butanetriol with (C) (D)


benzaldehyde in the presence of a catalytic
18 | CSIR-Chemical Sciences (June-2013)

142. The major product formed in the following H


reaction is—
N2
O O
1. hv‚ aq. THF H
2. Δ
(A) Con-rotatory in photochemical; and
dis-rotatory in thermal conditions
O
N
(B) Con-rotatory in thermal; and dis-
O rotatory in photochemical conditions
(A) N (B)
O (C) Con-rotatory in thermal; and con-
rotatory in photochemical conditions
O
(D) Dis-rotatory in photochemical; and dis-
O rotatory in thermal conditions
(C) (D) OH
OH
Answers with Hints
143. The major product formed in the following
reaction is— Part–A
O 1. (C) Ms. Black Ms. Brown Ms. White
hv‚ Et2O ↓ ↓ ↓
brown white black
Each bears different colour dress according to
OH
their name.
(A) (B) O
2. (A)
3. (D) E
OH

(C) O (D) D C
square pyramid
H
A B
144. Predict the condition A and the structure of Only one cut is required as ABCEDA is
the major product B in the following formed without cut and then after one cut CD
sequence— is made.
4. (C) For every next figure, dark cut is rotating
A 1. O3; Me2S
B clockwise with one part adjacent to next and
2. Ph3P = CHCOOEt
(excess) small circles are increasing 2 and 1 adjacent
to dark area and next after adjacent to dark
COOEt area respectively.
(A) A is hv B is
COOEt

COOEt 5. (A)
(B) A is hv B is
COOEt 6. (B)
N
200
COOEt
180
(C) A is Δ B is
COOEt 160

Since inverted ‘V’ shaped portions of contour


(D) A is Δ B is COOEt
lines represent a valley along which a river
COOEt
flows and that direction is north according to
145. The most appropriate mode of cyclisation in given figure. Thus, the downstream direction
the following transformation is— of the river is south.
CSIR-Chemical Sciences (June-2013) | 19

7. (D) 24. (C) [(n7–tropylium) Mo(CO)3 ]+


8. (D) D C North

MO
East
A B OC CO CO
Since, there is square and allowed only to In zero oxidation state of Mo, complex have
North and East direction move the total 18 e– (stable)
distance travelled by the person is same for all 7 + 6 + 6 – 1 = 18
paths. tropylium is a 7e – donor ligand in neutral
9. (D) 10. (A) 11. (B) 12. (C) 13. (B) method.
25. (A)
14. (C) 15. (D) 16. (B) 17. (D) 18. (B) −2
Cl Cl Cl NH3 Cl NH3
19. (D) 20. (B) +NH3 +NH3
Pt Pt Pt
Part–B Cl Cl NH3
Cl Cl Cl
21. (D) As we go down the group in periodic table Cis-platin
for sodium family, the number of electrons (anticancer drug)
increases and their attraction towards nucleus Cl– have more trans directing ability than
increases and removal of e – become difficult. NH3.
So Na loses e – s very easily. 26. (A) 27. (C) 28. (C) 29. (A) 30. (C)
In the case of inert gas family, Ne has its 31. (C) Among given compounds NF3 is not
complete octet, strongly connected with hydrolyse easily because, the more electro-
nucleus e– attraction and do not give e–s to negative F atom strongly attracts the e–s
lose easily. present in N atom and they are no longer
available to get hydrolysed.
Thus Na, Ne pair have highest difference
in their first ionization energy in given option.
N
22. (A) The structure of uranocene is 10 F
2 .5
F °
U+4 (C8H8– 2)2. F
32. (D) 33. (A)
34. (D) O
U C
(CO)4Mn CH3 (CO)4Mn C CH3
O
Most stable actenocene. (Co insertion)

23. (C) In Metal-Olefin interaction, the extent of O O


increase in metal → Olefin π-back donation C C CH3
would strengthen the Metal-Olefin bond
H H (CO)4Mn CH3 (CO)4Mn
(methyl migration)
C = vacant site
M So overall migratory insertion.
C 35. (A)
H H
36. (A) Neutron activation analysis is a sensitive
multi-element analytical technique used for
which produces a increase in C = C bond both qualitative and quantitative analysis of
length and somewhat single bond character major, minor, trace and rare elements. It
which results a change in hybridisation from requires high neutron cross-section area of
sp2 to sp3 . target and long half-life of the product.
20 | CSIR-Chemical Sciences (June-2013)

37. (C) A → P 42. (B) In triatonic centrosymmetric molecule, the


d [A] vibrational mode which are active in IR, are
rate = – inactive in Raman and vice-versa. Thus AB2
dt
⇒ [A]t = [A]0 – n Kt molecule showing two IR absorption lines and
one IR–Raman line have following structure—
← →
B —A— B
Conc.

asymmetric stretching and bending → IR


active symmetric stretching → IR-Raman
active
Time 43. (A) In NMR spectroscopy energy of transition
From given observation, we find this graph from α to β state is given as :
and it indicates that order of reaction is zero. Vα → β = gNβNB0
aq. NaOH
38. (D) For a particle in a one dimensional 44. (B) NaCl + HCl ⎯⎯⎯→ 2 NaCl + H2O
box energy is given as (aq) Neutrali- 2 components
zation
n2 h2
En = There are no HCl and NaOH in final mixture.
8 ma2
45. (C)
where a is length of box, m is mass of
particle. Thus, liquid
solid
h2 P3
E1 =
8 ma2 vapour
4h2
and E2 = T3
8 ma2
3h2 Temperature
So, E2 – E1 =
8 ma2 The lowest pressure at which the liquid phase
= 6 units of energy of a pure substance can exist is known as
h2 ‘Triple-point pressure’.
Therefore, E3 – E2 = (9 – 4) 46. (C)
8 ma2
= 5×2 47. (C) Number of microstates
= 10 units of energy n
39. (C) =
n1 n2 n3 ……
40. (C) 2He : 1S 2 1S
where n is total identical particles and n1, n2 ,
Electronic term symbol = 2S + 1L
J
n3 are occupation number.
n 6
S = =0
2 Thus, =
⇒ 2S + 1 = 1 1 2 3
L = 0 (S) 6×5×4× 3
J = 0 =
1S 1×2×1× 3
Thus, term symbol for He = 0
= 60
41. (C) Symmetry operation : E, 2C 6 , 2 C 3 , C 2 , 3σd ,
48. (B) 49. (B)
3σv symmerty operations with different sym-
50. (D) Lindeman Mechanism :
metry element is equal to number of classes
K1
= 6 A+A A* + A
order of the symmetry point group K– 1
K2
= 1+2+2+1+3+3 A* ⎯→ D
= 12 Rate of formation of product = K2 [A* ]
CSIR-Chemical Sciences (June-2013) | 21

Applying SSA on [A* ] 56. (B)


d [A* ] 57. (A)
= K1[A]2 – K – 1 [A * ] [A] – K2[A]*
dt H2
O CH2
= 0 10% Pd/c
Cyclohexyl benzyl ether
K1 [A]2
[A * ] =
K – 1 [A] + K2 OH + H3C
K2K1 [A]2
Rate = Cyclohexanal Toluene
K – 1 [A] + K2
Br
If K2 >> K – 1 [A] at low concentration Br
Na+I−
Rate = K1 [A]2 58. (C) Br
Second order kinetics. Br
51. (A) no available position to form cyclohexene
52. (B) When coordinates are changed with one Br
Br
axis, the unit cell is simple cubic, when I−Na+
changes with two axis, it is face-centred cubic.
−IBr
Thus, when coordinates are changed with Br
three axis, the unit cell is body centred cubic. Trans-1,2, dibromo Br
cyclohexane
(0, 0, 0) to
1
2 (
‚ 0‚ 0 )
– simple cubic

(0, 0, 0) to
1 1
‚ ‚0
2 2 ( – f.c.c. ) Br
Cyclohexene

(0, 0, 0) to
1 1 1
‚ ‚
2 2 2 ( – b.c.c. ) 59. (D)
60. (B) Me Me
53. (C) For cubic system, interplanar distance is O O
O
given as CH3CO3H
a2 esterification
d2hkl = 2
h + k2 + l2 Me
42
Thus, d2100 = 2 = 16 LiAlH4 OH
1 + 02 + 02 CH2 OH
⇒ d100 = 4 Å
LiAlH4 reduces as
54. (A)
O OH
55. (D) OH H
Ph
2 4 6 Me OH
1 3 5 7 −CHO CHOH
For, 2-position O OH
1 4 CH2
OH H 4
3 2
OH OH
3 2
25 1 61. (C) Steroids are type of organic compound
view
that contains a characteristic arrangement of
For, 3–position four cycloalkane ring that are joined to each
2 other as—
H
1
3E
H C D
2

Thus, IUPAC Name is originated as A B


(2S, 3E)–7–Phenylhept-3-en-2-ol cholestane, a polytypical steroid skeleton
22 | CSIR-Chemical Sciences (June-2013)

Mevalonic Acid—
OH O N Me
H
5 3 1
HO 4 2 OH Six-membered ring
(3R)–3, 5–Dihydroxy-3-methyl pentanoic acid is stable
used as biosynthetic precursor for steroids.
62. (B)
nBu Sn N N
Br 3 Cl
AIBN NaBH4 H H Me
Me
O OEt C Cl O OEt
1
nBu SnBr
3
66. (C) H
2
CrO3
OEt Isoprene 5 H
O H2SO4 O O 3
4
6 H
hυ HO
C N N C C + N2 H
CN CN CN Lupeol
(AIBN) Six isoprene units in Lupeol is identified.
O O N Histidine (His)
63. (C) 67. (D)
+X N CH2 CH COOH
−HA
Ph N Ph Ph N Ph NH2
Imidazole ring
HA X (A)
HB COOH
+X
HB 68. (B) Gauche conformation of n-butane with
COOH
−HB 60° dihedral angle (φ)
O
CH3
H CH3

Ph N Ph φ = 60°
HA (B) H
X H
COOH
H
Structures (A) and (B) have non-superimpo-
sable mirror images, Staggered
Thus, HA and HB are enantiotopic C 2 –axis bisects molecule with one methyl at
each side (front & back) and is chiral, no
O
plane of symmetry, no centre of symmetry.
64. (B) O C CH3 69. (D)
Ester group have carbonyl stretching H
frequency at 1765 to 1770. But due to phenyl O H Me O CH2 OH CH2
ring, —O—atom involves in conjugation and hυ
– CO frequency decreases to 1760 cm– 1. NT-II
65. (D) γ-hydrogen ring opening
+ − H abstraction
NaNH2 −Br
−NH3
Br N Me Br N Me O O H
H H
H
Proceeds through aryne intermediate
O
(aryne)
regeneration of C group
Benzyne intermediate
CSIR-Chemical Sciences (June-2013) | 23

70. (A) H H 74. (D) Mössbauer spectra are shown by nuclei


which have value of I > O and also have
different value of I in excited and ground state
H H of nucleus.
1,5-diene Same product
Among given molecule, only 57Co nuclei
A molecular rearrangement in which the are mössbauer inactive because it decays as
principal product is indistinguishable (in the 1e –
absence of isotopic labelling) from the prin- 57
27
Co ⎯→ 26Fe
57

cipal reactant, is known as ‘degenerate rearr- 129 I 121Sb


which is M.B. active, and is also
angement’. active.
Part–C 75. (C)
71. (B) Initial decay, moisture
76. (A) [CoCl4]– 2 ⎯⎯⎯→ [Co(H 2 O) 6 ]+ 2
NO = 34500 disintegrations/minute dark blue (H2O) pale pink
tetrahedral excess octahedral
After 75 minutes, + 2 O.S. + 2 O.S.
Nt = 21500 disintegrations/minute Tetrahedral complexes are darker in colour.
We know that, disintegration constant, 77. (B) 78. (B)
2·303 ⎛N ⎞ 79. (B) [Bu4 N] 2 Re 2 Cl8 → 2 NBu4+ + Re2 Cl8– 2
λ = log ⎜ O⎟
t ⎝ Nt ⎠ R e : 5d5 6s2
R e+++ : 5d4
2·303
75
= log
34500
21500 ( ) σ2 π4δ2 → σ2 π4δ1 δ*1
= 0·00626 15000 cm– 1
0·693 0·693 (Royal blue)
And, also t1/2 = =
λ 0·00626 σ∗
π∗
= 110·7 minute δ∗
72. (C) Since MeTiCl3 have no e – s for back δ∗ δ → δ * transition
π
donation with π-accepting ligands like PMe3 σ
and CO, therefore, It reacts in order
80. (A)
NMe 3 > PMe 3 > CO
81. (C) (base) CH3 COOH + H2SO4
(more π-accepting tendency) ⊕
Now, in (CO)5 Mo(thf), Mo have sufficient e – s → CH3 COOH2 + HSO 4 –
and vacant site to react with π-acceptor ligand (base) HNO 3 + H2 SO4 → H2NO3⊕ + HSO4 –
Thus, the order is (acid) HClO4 + H2 SO4 → ClO 4 – + H3 ⊕SO4 –
CO > PMe3 > NMe3 behaves as acid
73. (A) In given option, only I is correct, if we (base) H2O + H2 SO4 → H3O⊕ + HSO4 –
consider only attached molecules lone-pairs, 82. (C)
F I
O O
O O O O
F Xe F C
N N N N N N
F
F F F O O O O O O
12 lone-pairs 12 lone-pairs N 2O 3 N 2O 4 N 2O 5
O Cl O N2O3 and N 2 O4 have N–N bond.
O Xe O Cl I Cl F Xe F 83. (D) Br Li
O Cl O
+ n-BuLi + n-BuBr
8 lone-pairs 12 lone-pairs 10 lone-pairs
24 | CSIR-Chemical Sciences (June-2013)

84. (B) highly unstable and readily poly- Thus,


4 × 18 – (4 × 9 + 2 × 12)
merises in its free state when binds with M–M bonds =
2
transition metal complexes, it gains stability
due to formation of C 4 H42–, like 72 – 60
= =6
2
Co 18e – species
4T (P)
88. (B) 1g
4e – donor

Energy
4P 4T (F)
(stable) 1g
85. (D) Among the given ligand CO is most π- 4F 4T
acid ligand. Between C2H4 and C2 F 4 , C2F 4 has 2g
more tendency to behave as π-acid ligand due 4A
2g
to more electronegative F atoms which holds
Strength of ligand
e– density towards F atoms and alkene
behaves as a better π-accepting position from [Cr (NH 3 )6 ]+ 3
metal. NEt3 has very little behaviour towards +2 +1 0 −1 −2
π-back bonding. Thus, the order is Cr+ 3 :
NEt3 < C2H4 < C2F 4 < CO 2S + 1 = 4
86. (A) eg e1g eg L = 3 (F)
4F
+2g a1g +2g
2g ⎯→ T2g
4A 4

VCl6–2
e2g
(n5 – C5H5)2 Cr [Ni (EDTA)]– 2
⎯→ 4 T1g strong bands }
2g → Eg intensity is lowest
Thus, 4A 2
μ = 3·9 BM μ = 2·9 BM ||| μ = 2·9 BM
O 89. (A) Due to lanthanide contraction, the
O O covalent radii of Nb(1·34 Å), Ta(1·34Å) and
Mo(1·29), W(1·30) are almost similar.
N
N
Ni
O
O
Chemical Reactivity = f
Charge
Radius ( )
No increase in covalent radii due to L.C.
eg
O O
+2g O 90. (B) μS = n (n + 2)
[Co (NO2)6]– 3 μ=0 7 11
Thus, VCl 6 – 2 has highest magnetic moment.
87. (B) Metal-Metal bonds
Gd (III) : μS =
7
2 (72 + 2) = ×
2 2

n × 18 –Total no. of valence e– sin 77


complex = = 4·38
= 2
2
n = total no. of metals 3 7 21
Nd (III) : μS = × =
Ir 4 (CO) 12, CO is 2e– donor ligand 2 2 2 = 2·29
Ir have 9 e–s in valence shell 91. (A) As strong field ligand have high crystal
OC CO CO field stabilisation energy, according to spec-
trochemical series of ligand, the given ligand
Ir in order
OC CO Br – < Cl– < NCS–
OC Ir Ir CO Thus, Δ t of given complexes follow the
OC CO order :
Ir [Co(NCS)4 ]– 2 > [CoCl4 ]– 2 > [CoBr 4 ]– 2
92. (C)
OC CO CO
93. (B) (A) [Cr (EDTA)]– is a chiral molecule,
18 e– species having d-l pair (enanntiones) see structure of
(Stable) question 86 (iv)
CSIR-Chemical Sciences (June-2013) | 25
– 3/2
(B)
N
N
N N
N
N
[A2 ]
0
=
3
2
3
([A]–0 3/2 + nK t1/2
2
+ [A 0 ]– 3/2 = n Kt1/2
R4 R4
t1/2 ∝ [A]–0 3/2
N N N N
N N
(iii) According to Lindeman theory, Unimole-
cular reactions (gas phase) are second order at
Enantiomers (d–l pair) low pressure and become first order at high
where N N = bypyridine pressure.
98. (B) 99. (A)
100. (C)
N N Radial function = r2 (α1 – r)(α2 – r) e– βr
Optically active Indicater that r2 = r l
(C) [Pt Cl (diene)]+ has square planar struc- ⇒ l = 2 means d-orbital
tures so it is a chiral. Since it have two nodes, r = α1 and r = α2
94. (C) 95. (B) Thus, orbital (hydrogenic) is identified as 5d
96. (C) 2 AB + B2 → 2 AB2 because radial node
rate is given as : = n –l–1
rate = K [AB] [B2 ] = 5–2–1
According to this rate law, mechanism is = 2
slow
AB + B2 ⎯⎯→ AB3 101. (C) [x, (x, p )] = [x, i–h . 2p]
2
K1
= x . 2p i–h – 2i–h px = 0
fast
AB3 + AB ⎯⎯→ 2 AB2 – [ p [x2, p]] = – [ p, – i–h . 2x]
K2
rate = K1 [AB] [B 2 ], = – [– i–h . 2x . p – (– p . i–h
slowest step is rate determining step. . 2x)]
– –
= + 2ih xp – 2 ih xp = 0
97. (A) (i) As reaction proceed, pressure of system
increases and radical become closer, so rate 102. (C) Wave function,
of propagation become fast. At a certain point
2
radicals value become infinite and this respon-
sible for explosion due to increase in pressure.
Ψ(x) = L sin L
nπx
( )
(ii) nA → P (products) Thus, < x3>


L
rate = K [A] 5/2 = Ψ(x) x3 . Ψ(x)* dx
d [A] 0
= nK [A] 5/2

L
dt =
2
sin2
nπx
( ) . x3 dx)


L 0 L

At d [A] t
+ 5/2 = – nK 0 dt L3
Ao [A] =
4
Integrating it gives,
σh

A A5/2 + 1 103. (B) E Cn n C2 i
+ = – n Kt
Ao – 5/2 + 1 1 1 –1 –1 –1
2 If below principle axis is + ve sign, there is A
– [A– 3/2] A t
= – n Kt notation in irreducible representation.
3 A o

2 If below C2 : – ve, then 2


[[At]– 3/2 – [A0 ]– 3/2] = n Kt If below σh : – ve then double dash ”
3
3 If below i : – ve then u
[A t]– 3/2 = ([A]–0 3/2 + n Kt) Thus, overall irreducible representation = A24 "
2
26 | CSIR-Chemical Sciences (June-2013)

104. (C) JAX = (difference between any two adja-


4 1
105. (B) 41Nb : [Kr] 4d 5s cent peaks) × (instrument frequency)
4d 5s
= (4·72 – 4·60) ppm × 100 MHz
+2 −1 0 −1 −2 0 = 0·12 ppm × 100 MHz
2S + 1 = 6 = 12 Hz
L = +2×1=2D 4·72 + 4·60 + 1·12 + 1·00
δAX =
Thus, ground state term symbol = 6 D 4
106. (A) 11·44
= = 2·86 ppm
107. (C) 1 2 3 4 4
⎪⎪ ⎪⎪ nj
1 2
1 x 1 0 1 112. (A) = g e– (Ej – Ei)/KBT
ni
⎪ ⎪
2 1 x 1 0
4 3
Since g = 2 (doubly-degenerate)
3⎪ 0 1 x 1 ⎪
4⎪ 1 x⎪
and Ej – Ei = 2 units
0 1 KBT = 1 unit
.. . α–E nj
x =
β Thus,
ni
= 2e– 2
⇒ E = α – βx
113. (D)
∴ Huckel determinant 114. (B) Ionic strength is given as
⎪⎪
α–E
β
1 0 ⎪⎪ 1
I =
1 i
2 i∑
mi zi2
⎪ 0 ⎪
α–E =0
1 1
β
⎪ 1 ⎪
= mi is molality of ion and zi is charge
α–E Thus, for NaCl
⎪⎪ α–E⎪
0 1
β

Z+ = 1,
1 0 1 Z– = 1,
β
m+ = 0·01 mol = m–
⎪⎪ α – E β 0 β ⎪⎪ 1
I = (0·01 × 12 + 0·01 × 12 )
= ⎪ β0 α–E β 0
⎪ 2
⎪⎪ β α–E β ⎪ 0·02
α–E⎪
= = 0·010 mol kg– 1
2
β 0 β
For Na2 SO4
108. (C) Z+ = 1, Z – = 2
120 mL of 0·05 M CH3 COOH m+ = 2 × 0·01 = 0·02,
40 mL of 0·05 M NaOH m– = 0·01 molal
moles of H+ = 0·05 × 120 = 6·0 mM 1

moles of OH = 0·05 × 40 = 2·0 mM I = (0·02 × 12 + 0·01 × 22 )
2
remaining H+ = 4 mM 0·02 + 0·04
4 × 10– 3 4 1 =
[H +]net = = = 2
160 × 10– 3 160 40 = 0·030 mol kg– 1
= 0·025 mole l– 1 115. (B)
pH = – log (2·5 × 10– 2) 116. (B) O + ne– R
= 2 + log 2·5 = 2·39 RT [R]
E = E° – ln
109. (B) 110. (B) nF [O]
111. (C) AX system gives 4 lines at 4·72, 4·6, (According to Nernst equation)
1·12 and 1·0 ppm 100 MHz instrument is RT [R]
Thus, E – E° = – ln
given. nF [O]
CSIR-Chemical Sciences (June-2013) | 27

nF [R] +
(E – E°) = – ln OH
RT [O] OH H
– nF
[O] H
⇒ = e RT (E – E°) Ph
[R] Ph
117. (C) We know Bragg equation that Me Major

2d sin θ = n λ Cram product (front side attack)


where n = order of diffraction 124. (D) 125. (D)
Given that 2d sin θ = λ …(1) 126. (B) Among all the options, only option (B)
A plane of spacing 2d shows have aromaticity character after charge
2 . 2d sin φ = λ distribution or keeps in that position and this
From (1), 4d sin φ = 2d sin θ leads to highest dipole moment in its.
sin θ
sin φ =
2
sin θ
⇒ φ = sin – 1
2 ( ) aromatic compound
13 C
118. (D) 119. (A) 120. (D) (C) The broadband decoupled NMR
121. (D) spectrum.
O OH OH
O 4 2 5 7
MeMgBr Me H3PO4 CH2 4 3 OH 4 6
1 8
Et2O 3 5 3 9
H H 2 7
2 5 3 2 10 6
1 3 4 1 3 5
1 HO OH
4 3 CH2 2 4 2 4
8 13C NMR
+5 2
Signal 3
Five signals Ten signals
Major Minor
5 4
127. (C) 128. (A) 129. (C)
5 13C NMR Signal
130. (A) 1H NMR spectral data is given as
122. (A)
δ 7·80 (2H, d, J = 8Hz)
OH O
aromatic m-hydrogens w.r.t.
OH MnO2 OH > C = 0 group
Selectively
oxidises benzylic 6·80 (2H, d, J = 8 Hz)
alcohol into aromatic o–hyrogens w.r.t.
Ketone (CH2OH)2/P-TSA
protecting group for Ketone > C = 0 group
O O O O
4·10 (2H, q, J = 7·2 Hz)
CHO – CH2 environment adjacent
PCC
OH to electronegative group
Oxidises − OH
into − CHO 1·25 (3H, t, J = 7·2 Hz)
123. (A) J value indicates that coupling
H O Back +2 − with – CH 2 group
CHO Me MgBr
Ph H 2·4 (3H, S) adjacent to > C = 0 group or
Me Ph H2O benzylic position
Group ‘L’ must trans/anti Less hindered 7.80 d H O O
to – C = 0 group side-major H 2.4
OH OH CH3
H S
t q
H CH3 CH2 O Hd EtO
Ph 1.25 4.10 H 6.80
Ph Me
Minor 131. (C) 132. (A)
28 | CSIR-Chemical Sciences (June-2013)

133. (C) CN O 139. (B)


CN
140. (D)
Δ
OTs OTs
H
(base)
O K H

CN THF
H O −H2
O
OH O
Δ

H O
134. (B) H H O
O
H
Ph O N OH 141. (A) 142. (B)
N
H O O
O O hυ εH
Ph 143. (D) H
Dipeptide α
Et2O ε γ
NT-II clearage δ β
H2 O
Ph CH2 H + CO2 + H N OH abstraction due to ring cyclisation
10% Pd/c 2
Toluene Carbon N
reduction OH
dioxide H OH
Ph O
Phe-Leu H 1
6 2
135. (B) 136. (A) 5
4
3
137. (A) Let overall phenol is x H
Thus, overall formation of p-hydroxyace- ring cyclisation
tanilide (6-membered) is
70 90 90 important in this case
= x× × × × 100
100 100 100 144. (A)
x × 567
= × 100 = 56·7% hυ disrotation
1000
cyclization
And, overall formation of o–hydroxyaceta-
nilide O3, Me2S
25 90 90 reductive
= x× × × × 100 oxidation
100 100 100 EtOOC
= 20·25% Ph3P = CHCOOEt O
138. (D) F excess witting
H F reaction
EtOOC O

145. (D) H H
H H
H hυ
1, 2–difluoroethane most stable conformation disrotation
due to φ = 60° (staggered) H H
Me Me Δ disrotation cis-product
H OH HO H Stereochemistry is important
H in pericyclic reactions

H OH HO H
Me Me
θ = 60° dl–2, 3–butanediol H
(gauche/staggered) cis-product
CSIR UGC-NET/JRF Exam., December 2013 Solved Paper
Chemical Sciences
PART–A Which of the following statements is correct ?
1. A cylinder of radius 1 cm and height 1 cm is (A) The speed is never zero
broken into three pieces. Which of the (B) The acceleration is zero once on the path
following must be true ? (C) The distance covered initially increases
(A) At least one piece has volume equal to and then decreases
1 cm3
(D) The car comes back to its initial position
(B) At least two pieces have equal volumes
once
(C) At least one piece has volume less than
1 cm3 6. If a + b + c + d + e = 10 (all positive
(D) At least one piece has volume greater numbers), then the maximum value of a × b
than 1 cm3 × c × d × e is—
2. For real number x and y, x2 + (y – 4)2 = 0. (A) 12 (B) 32
Then the value of x + y is— (C) 48 (D) 72
(A) 0 (B) 2
7. How many nine-digit positive integers are
(C) √⎯2 (D) 4 there, the sum of squares of whose digits is
3. Every time a ball falls to ground, it bounces 2?
back to half the height it fell from. A ball is (A) 8 (B) 9
dropped from a height of 1024 cm. The (C) 10 (D) 11
maximum height from the ground to which it
can rise after the tenth bounce is— 8. A circle of radius 7 units lying in the fourth
(A) 102·4 cm (B) 1·24 cm quadrant touches the x-axis at (10, 0). The
(C) 1 cm (D) 2 cm centre of the circle has coordinates—
4. A farmer gives 7 full, 7 half-full and 7 empty (A) (7, 7) (B) (– 10, 7)
bottles of honey to his three sons and asks (C) (10, – 7) (D) (7, – 7)
them to share these among themselves such
that each of them gets the same amount of 9. One of the four—A, B, C and D committed a
honey and the same number of bottles. In how crime. A said, “I did it.” B said, “I didn’t.” C
many ways can this be done ? (bottles cannot said, “B did it.” D said, “A did it.” Who is
be distinguished otherwise, they are sealed lying ?
and cannot be broken)— (A) A (B) B
(A) 0 (B) 1 (C) C (D) D
(C) 2 (D) 3
1 , 1 ,
5. A car is moving along a straight track. Its 10. What is the arithmetic mean of
1×2 2×3
speed is changing with time as shown. 1 , 1 ,…, 1
?
3×4 4×5 100 × 101
1
Speed

(A) 0·01 (B)


101
1 1
+
49 × 50 50 × 51
O Time (C) 0·00111… (D)
2
2 | CSIR-Chemical Sciences (D-13)

11. A circle circumscribes identical, closepacked 17. Consider the sequence of ordered sets of
circles of unit diameter as shown. What is the natural numbers—
total area of the shaded portion ? {1}, {2, 3}, {4, 5, 6}, …
What is the last number in the 10th set ?
(A) 10 (B) 19
(C) 55 (D) 67
18. A student buys a book from an online shop at
20% discount. His friend buys another copy
(A) 2 (B) 2π of the same book in a book fair for 192
(C) 1/2 (D) π/2 paying 20% less than his friend. What is the
12. There are 2 hills, A and B, in a region. If hill full price of the book ?
A is located N30°E of hill B, what will be the (A) 275 (B) 300
direction of hill B when observed from hill (C) 320 (D) 392
A ? (N 30°E means 30° from north towards 19. 366 players participate in a knock-out
east). tournament. In each round all competing
(A) S 30°W (B) S 60°W players pair together and play a match, the
(C) S 30°E (D) S 60°E winner of each match moving to the next
13. What is the next number in the following round. If at the end of a round there is an odd
sequence ? number of winners, the unpaired one moves
39, 42, 46, 50, …… to the next round without playing a match.
(A) 52 (B) 53 What is the total number of matches played ?
(C) 54 (D) 55 (A) 366 (B) 282
(C) 365 (D) 418
14. What is the perimeter of the given figure,
where adjacent sides are at right angles to 20. What does the diagram establish ?
each other ?
5 cm
4 cm

Note—The diagram is a circle inside a


square.
1cm

(A) π > 3
(A) 20 cm (B) π ≥ 2√⎯2
(B) 18 cm (C) π < 4
(C) 21 cm (D) π is closer to 3 than to 4
(D) Cannot be determined
PART–B
15. Three fishermen caught fishes and went to
sleep. One of them woke up, took away one 21. The boiling point of a solution of non-volatile
fish and 1/3rd of the remainder as his share, solid is higher than that of the pure solvent. It
without others’ knowledge. Later, the three of always indicates that—
them divided the remainder equally. How (A) the enthalpy of the solution is higher than
many fishes were caught ? that of the pure solvent
(A) 58 (B) 19 (B) the entropy of the solution is higher than
(C) 76 (D) 88 that of the pure solvent
(C) the Gibbs free energy of the solution is
16. (25 ÷ 5 + 3 – 2 × 4) + (16 × 4 – 3) = higher than that of the pure solvent
(A) 61 (B) 22 (D) the internal energy of the solution is
(C) 41/24 (D) 16 higher than that of the pure solvent
CSIR-Chemical Sciences (D-13) | 3

22. When Frenkel defects are created in an (B) band gap of gold changes with size of the
otherwise perfect ionic crystal, the density of nanoparticle
the ionic crystal— (C) gold in nanocrystalline form undergoes
(A) increases transmutation to other elements
(B) decreases (D) colloidal suspensions diffract light
(C) remains same 29. A reactor contains a mixutre of N2 , H2 and
(D) oscillates with the number of defects NH3 in equilibrium (KP = 3·75 atm – 2). If
23. The correct thermodynamic relation among sufficient he is introduced into the reactor to
the following is— double the total pressure, the value of K P at
∂U ∂H the new equilibrium would be—
(A) ( )
∂V S
=–P (B) ( )
∂V S
=–P (A) 0·94 atm– 2 (B) 3·75 atm– 2
∂G ∂A (C) 7·50 atm– 2 (D) 15·00 atm– 2
(C) ( )
∂V S
= – P (D) ( )
∂V S
=–S
30. Electrolysis of an aqueous solution of 1·0 M
NaOH results in—
24. The molecule in which the bond order
increases upon addition of an electron is— (A) Na at the cathode and O2 at the anode
(A) O2 (B) B 2 (B) H2 at the cathode and O2 at the anode
(C) P 2 (D) N2 (C) Na and H2 at the cathode and O2 at the
anode
25. The volume of a gas adsorbed on a solid
(D) O2 at the cathode and H2 at the anode
surface is 10·0 mL, 11·0 mL, 11·2 mL, 14·5
mL and 22·5 mL at 1·0, 2·0, 3·0, 4·0 and 5·0 31. In a potentiometric titration, the end point is
atm. pressure, respectively. These data are obtained by observing—
best represented by— (A) change in colour
(A) Gibbs’s isotherm (B) jump in potential
(B) Langmuir isotherm (C) increase in current
(C) Freundlich isotherm (D) increase in turbidity
(D) BET isotherm 32. The orbital with two radial and two angular
26. A compound of M and X atoms has a cubic nodes is—
unit cell. M atoms are at the corners and body (A) 3p (B) 5d
centre position and X atoms are at face centre (C) 5f (D) 8d
positions of the cube. The molecular formula
of the compound is— 33. The energy of 2s and 2p orbitals is the same
for—
(A) MX (B) MX2 (A) Li (B) Li2+
(C) M3 X2 (D) M2 X3 (C) Be 2+ (D) H–
27. The angle at which the first order Bragg 34. If a homonuclear diatomic molecule is
reflection is observed from (110) plane in a oriented along the Z-axis, the molecular
simple cubic unit cell of side 3·238 Å, when orbital formed by linear combination of Px
chromium Kα radiation of wavelength 2·29 Å orbitals of the two atoms is—
is used, is— (A) σ (B) σ*
(A) 30° (B) 45° (C) π (D) δ
(C) 60° (D) 90° 35. In the mechanism of reaction, H2 + Br 2
→ 2HBr, the first step is—
28. Michael Faraday observed that the colour of
colloidal suspensions of gold nanoparticles (A) dissociation of H2 into H-radicals
changes with the size of the nanoparticles. (B) dissociation of Br2 into Br-radicals
This is because— (C) reaction of H-radical with Br2
(A) gold forms complex with the solvent (D) reaction of Br-radical with H2
4 | CSIR-Chemical Sciences (D-13)

36. The cell voltage of Daniel cell 42. Amongst the following, the compound which
[Zn]ZnSO 4 (aq)|| CuSO4 (aq)[Cu] is 1·07 V. If has the lowest energy barrier for the cis-trans
reduction potential of Cu2+ [Cu is 0·34 V, the isomerisation is—
reduction potential of Zn2+ |Zn is—
(A) 1·41 V (B) – 1·41 V
(C) 0·73 V (D) – 0·73 V (A) (B)
37. According to Arrhenius equation (k = rate
constant and T = temperature)—
(A) In k decreases linearly with 1/T
(B) In k decreases linearly with T (C) (D)
(C) In k increases linearly with 1/T
(D) In k increases linearly with T
43. For estrone, among the statements A-C, the
38. The IUPAC name of the compound given correct ones are—
below is—
HO 1. it is a steroidal hormone
OH 2. it has two hydroxyl groups
Cl 3. it has one ketone and one hydroxyl
(A) (2E, 4E)-3-chlorohexa-2, 4-diene-1, 6- groups
diol (A) 1, 2 and 3 (B) 1 and 2
(B) (2Z, 4E)-3-chlorohexa-2, 4-diene-1, 6- (C) 1 and 3 (D) 3 and 3
diol
44. An organic compound having the molecular
(C) (2Z, 4Z)-4-chlorohexa-2, 4-diene-1, 6-
formula C10H14 exhibited two singlets in the
diol 1 H NMR spectrum, and three signals in the
(D) (2E, 4Z)-4-chlorohexa-2, 4-diene-1, 6- 13C NMR spectrum. The compound is—
diol
39. A suitable organocatalyst for enantioselective
synthesis of Wieland-Miescher ketone (A) (A) (B)
is—
O O O
(C) (D)
O O
A (optically active) 45. The major product formed in the following
(A) (–)-proline (B) (+)-menthone reaction is—
(C) guanidine (D) (+)-BINOL Cl
40. The major product formed in the reaction of N NaOMe
styrene with an excess of lithium in liquid MeOH
ammonia and t-butylalcohol is— MeO Cl

Cl OMe
(A) (B) MeO
(A) N (B) N
(C) (D) MeO Cl MeO Cl

41. In the IR spectrum of p-nitrophenyl acetate, Cl


the carbonyl absorption band appears at— MeO
N
(C) N (D)
(A) 1660 cm– 1 (B) 1700 cm– 1
MeO Cl
(C) 1730 cm – 1 (D) 1770 cm– 1 MeO OMe
CSIR-Chemical Sciences (D-13) | 5

46. L-DOPA is used for the treatment of— (A) diastereotopic in I and enantiotopic in II
(A) tuberculosis (B) diastereotopic in II and enantiotopic in I
(B) Parkinson’s disease (C) diastereotopic in both I and II
(C) diabetes (D) enantiotopic in both I and II
(D) cancer 51. Deuterium kinetic isotope effect for the
47. The following reaction proceeds through a— following reaction was found to be 4·0. Based
Δ on this information, mechanism of the
reaction is—
O CHO H(D)
(A) 1, 3-sigmatropic rearrangement −
KOH
+
(B) 2, 3-sigmatropic rearrangement MeOH
NMe3
(C) 3, 3-sigmatropic rearrangement (A) E1 (B) E2
(D) 3, 5-sigmatropic rearrangement (C) E1CB (D) Free radical
48. The major product formed in the following 52. The number of nodes present in the highest
reaction is— occupied molecular orbital of 1, 3, 5-
OMe O O hexatriene in its ground state is—
1. heat
2. H2SO4 heat (A) one (B) two
+
(C) three (D) four
NH2
OMe 53. The major product formed in the following
reaction is—
OMe OMe O O
O
MeO Zn(BH4)2
(A) N (B) N
H H OH OH
OMe OMe
MeO
(A) MeO (B)
OMe OMe
O OH
(C) (D)
N HO NH2 (C) (D) HO
OMe OMe

49. The constituent amino acids present in the 54. For acylation with acetic anhydride/
following dipeptide, respectively, are— triethylamine, and oxidation with chromium
COOH
trioxide of the trans- and cis-alcohols A and
H B, the correct statement is—
N NH2 OH OH
H2N
O COOH
(A) (R)-aspartic acid and (S)-lysine A B
(B) (S)-aspartic acid and (R)-lysine (A) A undergoes acylation as well as
(C) (R)-glutamic acid and (S)-arginine oxidation faster than B
(D) (S)-glutamic acid and (S)-arginine (B) B undergoes acylation as well as
50. The two benzylic hydrogens HA and HB in the oxidation faster than A
compounds I and II, are— (C) A undergoes acylation faster than B,
whereas B undergoes oxidation faster
HA HA
HB HB than A
N N
(D) B undergoes acylation faster than A,
Ph Ph whereas A undergoes oxidation faster
(I) (II) than B
6 | CSIR-Chemical Sciences (D-13)

55. Patients suffering from Wilson’s disease 63. In a cluster, H 3 CoRu3(CO) 12, total number of
have— electrons considered to be involved in its
(A) low level of Cu-Zn superoxide dismutase formation is—
(B) high level of Cu-Zn superoxide dismutase (A) 57 (B) 60
(C) low level of copper-storage protein, (C) 63 (D) 72
ceruloplasmin 64. Among the following the correct acid strength
(D) high level of copper-storage protein, trend is represented by—
ceruloplasmin (A) [Al(H2 O) 6 ] 3 + < [Fe(H2 O) 6 ] 3 + <
56. High dose of dietary supplement ZnSO4 for [Fe(H2 O) 6 ]2+
the cure of Zn deficiency— (B) [Fe(H2 O) 6 ] 3 + < [Al(H2 O) 6 ] 3 + <
(A) reduces myoglobin [Fe(H2 O) 6 ] 2+

(B) increases iron level in blood (C) [Fe(H2 O) 6 ] 2 + < [Fe(H2 O) 6 ] 3 + <
(C) increases copper level in brain [Al(H2 O) 6 ]3+
(D) reduces copper, iron and calcium levels (D) [Fe(H2 O) 6 ] 2 + < [Al(H2 O) 6 ] 3 + <
in body [Fe(H2 O) 6 ] 3+

57. The bond order of the metal-metal bond in the 65. An octahedral metal ion M2+ has magnetic
dimeric complex [Re2Cl4(PMe2Ph)4 ]+ is— moment of 4·0 B.M. The correct combination
(A) 4·0 (B) 3·5 of metal ion and d-electron configuration is
(C) 3·0 (D) 2·5 given by—
58. Among the molten alkali metals, the example (A) Co 2+, t2 g5 eg 2
of an immiscible pair (in all proportions) is— (B) Cr2+, t2 g4 eg 2
(A) K and Na (B) K and Cs (C) Mn2+, t2 g3 eg 1
(C) Li and Cs (D) Rb and Cs (D) Fe2+, t2 g4 eg 2
59. Among the following, an example of a
hypervalent species is— 66. The reaction of FeCl3 ·6H2 O with SOCl2
yields—
(A) BF3·OEt 2 (B) SF4
(A) FeCl2(s), SO2(g) and HCl(g)
(C) [PF6]– (D) Sb 2 S 3
(B) FeCl3(s), SO2(g) and HCl(I)
60. Commonly used scintillator for measuring
γ-radiation is— (C) FeCl2(s), SO3(g) and HCl(g)
(A) Nal(Al) (B) NaI(Tl) (D) FeCl3(s), SO2(g) and HCl(g)
(C) CsI(Tl) (D) CsI(Al) 67. Treatment of ClF3 with SbF5 leads to the
61. A sample of Aluminium ore (having no other formation of a/an—
metal) is dissolved in 50 mL of 0·05 (A) polymeric material
MEDTA. For the titration of unreacted (B) covalent cluster
EDTA, 4 mL of 0·05 M. MgSO4 is required.
The percentage of Al in the sample is— (C) ionic compound
(D) Lewis acid-base adduct
(A) 27 (B) 31
(C) 35 (D) 40 68. According to VSEPR theory, the geometry
(with lone pair) around the central iodine in
62. Which of the following is not suitable as I3+ and I3 – ions respectively are—
catalyst for hydroformylation ?
(A) tetrahedral and tetrahedral
(A) HCo(CO)4
(B) trigonal bipyramidal and trigonal
(B) HCo(CO)3PBu3 bipyramidal
(C) HRh(CO)(PPh3 )3 (C) tetrahedral and trigonal bipyramidal
(D) H2Rh(PPh3 )2Cl (D) tetrahedral and octahedral
CSIR-Chemical Sciences (D-13) | 7

69. Two tautomeric forms of phosphorus acid


are— F
F F
OH O OH F Al F
(A) P OH and P F F F F
Al F Al
OH H OH (C) F F
F F F
OH O H F Al F
(B) P OH and P F F
F
H H OH
OH O O F F F F
(C) P OH and H P F F F
(D) Al Al Al Al
OH H H F F F
F F F F
O OH O O
(D) P and H P 74. Oxidised form of enzyme catalase (structure
HO OH HO OH A), prepared by the reaction of [Fe(P)]+ (P =
porphyrin) with H 2 O2, has green color
70. The reason for the chemical inertness of because—
gaseous nitrogen at room temperature is best
given by its—
(A) high bonding energy only N N
(B) electronic configuration Fe
(C) HOMO-LUMO gap only N N
O
(D) high bond energy and HOMO-LUMO
gap
A (substituents on ring are removed
PART–C for clarity)
71. The reactive position of nicotinamide adenine (A) oxidation state of iron changed from Fe III
dinucleotide (NAD) in biological redox to FeIV
reactions is— (B) porphyrin ring is oxidized by one
(A) 2-position of the pyridine ring electron
(B) 6-position of the pyridine ring (C) π - π * transition appears in the visible
region
(C) 4-position of the pyridine ring
(D) FeI V is coordinated with anionic
(D) 5-position of the pyridine ring tyrosinate ligand in axial position
72. The electrophile Ph3 C + reacts with [(η5 75. Substitution of L with other ligands will be
– C5 H5)Fe(CO)2 (CDMe2)] + to give a product easiest for the species—
A. The product A is formed because—
Me
(A) Fe is oxidised Me Me
(B) alkyl is substituted with Ph3C (A) (B)
MLn
(C) Fe-Ph bond is formed Me Me
MLn
(D) alkyl is converted to alkene
73. The solid state structure of aluminum fluoride MLn
(C) (D)
is— MLn
F F F
(A) Al Al 76. The ground state terms of Sm3+ and Eu3+,
F F F respectively, are—
(A) 7F0 and 6H5/2 (B) 6H5/2 and 7F0
(B) F F F
Al Al Al Al
F F F (C) 2F5/2 and 5I4 (D) 7F6 and 2F7/2
8 | CSIR-Chemical Sciences (D-13)

77. The orbital interactions shown below (C) Mn is in + 3 oxidation state and 1 Cr is in
represent + 2 and the other is in + 3 state
sp3 orbitals (D) Cr3+ will have a LFSE in the octahedral
site whereas the Mn2+ ion will not
E 83. Reaction of Ph2 PCH2CH2 PPh2 with
[RHCl(CO)2]2 in a 2 : 1 molar ratio gives a
crystalline solid A. The IR spectrum of
s orbitals complex A shows vCO at 1985 cm– 1. The
31P{1H} NMR spectrum of A consists of two
(A) CH3 —Al interactions in Al2 (CH3)6
(B) B—H interactions in B2H6 doublets of doublets of equal intensities
(103Rh is 100% abundant and I = 1 /2 ). The
(C) CH3 —Li interactions in Li4 (CH3)4 structure of complex A is—
(D) CH3 CH2 —Mg interactions in
Ph2
EtMgBr·(OEt2 )2 P Cl
(A) Rh
78. Compounds K2 Ba[Cu(NO2 )6] (A) and
P CO
Cs 2 Ba[Cu(NO2 )6] (B) exhibit tetragonal Ph2
elongation and tetragonal compression,
respectively. The unpaired electron in A and Ph2 Ph2
B are found respectively, in orbitals— P CO P
(B) Rh2 Cl
(A) dz2 and dx2.y2 (B) dx2 . y2 and dz2
P P
2
(C) dz and dz 2 (D) dx2 .y2 and dx2.y2 Ph2 Ph2
79. Among the following, the correct statement Ph2
is— P CO Cl
(A) CH is isolobal to Co(CO)3 (C) Rh
PPh2
P P
(B) CH2 is isolobal to Ni(CO) 2 Ph2 Ph2
(C) CH is isolobal to Fe(CO) 4
Ph2P O PPh2
(D) CH2 is isolobal to Mn(CO) 4 C Cl
80. In Mossbauer experiment, a source emitting (D) Cl Rh C Rh

at 14·4 Kev (3·48 × 1018 Hz) had to be moved Ph2P O PPh2


towards absorber at 2·2 mm s– 1 for resonance.
The shift in the frequency between the source 84. In a polarographic estimation, the limiting
and the absorber is— currents (μA) were 0·15, 4·65, 9·15 and 27·15
when concentrations (mM) of Pb(II) were 0,
(A) 15·0 MHz (B) 20·0 MHz 0·5, 1·0 and 3·0, respectively. An unknown
(C) 25·5 MHz (D) 30·0 MHz solution of Pb(II) gives a limiting current of
81. In the atomic absorption spectroscopic 13·65 μA. Concentration of Pb(II) in the
estimation of Fe(III) using O2 /H2 flame, the unknown is—
absorbance decreases with the addition of— (A) 1·355 mM (B) 1·408 mM
(A) CO3 2– (B) SO4 2– (C) 1·468 mM (D) 1·500 mM
85. The gases SO 2 and SO3 were reacted separa-
(C) EDTA (D) Cl– tely with ClF gas under ambient conditions.
82. MnCr2 O4 is likely to have a normal spinel The major products expected from the two
structure because— reactions respectively, are—
(A) Mn2+ will have a LFSE in the octahedral (A) SOF2 and ClOSO2F
site whereas the Cr3+ will not (B) SOF2 and SO2F 2
(B) Mn is in + 2 oxidation state and both the (C) SO2 ClF and SO2F 2
Cr are in + 3 oxidation state (D) SO2 ClF and ClOSO2 F
CSIR-Chemical Sciences (D-13) | 9

86. In a specific reaction, 91. Among the following the correct combination
hexachlorocyclotriphosphazene, N3 P 3 Cl6 was of complex and its colour is—
reacted with a metal fluoride to obtain mixed Complex Colour
halo derivatives namely N 3 P 3 Cl5F(1), (A) [Co(CN) 4 ]2– Red
N3P 3 Cl4F 2 (2), N3 P 3 Cl3F 3 (3), N3 P 3 Cl2F 4 (4), (B) [CoCl4] 2– Orange
N3P 3 ClF5 (5). (C) [Co(NCS)4 ]2– Blue
Compositions among these which can give (D) [CoF 4 ]2– Yellow
isomeric products are—
92. On reducting Fe 3 (CO) 12 with an excess of
(A) 1, 2 and 3
sodium, a carbonylate ion is formed. The ion
(B) 2, 3 and 4 is isoelectronic with—
(C) 3, 4 and 5 (A) [Mn(CO)5 ]– (B) [Ni(CO)4 ]
(D) 5, 1 and 2 +
(C) [Mn(CO)5 ] (D) [V(CO)6 ]–
87. Xenon forms several fluorides and
oxofluorides which exhibit acidic behavior. 93. The correct statement for ozone is—
The correct sequence of descending Lewis (A) It absorbs radiations in wavelength
acidity among the given species is represented region 290–320 nm
by— (B) It is mostly destroyed by NO radical in
(A) XeF6 > XeOF4 > XeF4 > XeO 2 F 2 atmosphere
(B) XeOF4 > XeO 2 F 2 > XeOF4 > XeF6 (C) It is non-toxic even at 100 ppm level
(C) XeF4 > XeO 2 F 2 > XeOF4 > XeF6 (D) Its concentration near poles is high due
to its paramagnetic nature
(D) XeF4 > XeF6 > XeOF4 > XeO 2 F 2
94. The most appropriate structure for the
88. Among the following clusters, complex [Pt 2 (NH3 )2(NCS)2 (PPh3)2] is—
A = [(H)Co 6 (CO) 15]–, B = [(H)2Os6 (CO) 18], C
HN NCS PPh3
= [(H)2Os5 (CO) 16] (A) 3 Pt Pt
H 3N SCN PPh3
H is encapsulated in—
Ph3P NCS NH3
(A) A only (B) Pt Pt
H 3N SCN PPh3
(B) B only
(C) B and C only H 3N NCS PPh3
(C) Pt Pt
(D) A and B only Ph3P SCN NH3
89. Number of isomeric derivatives possible for H 3N NCS NH3
the neutral closo-carborane, C2B 10H12 is— (D) Pt Pt
Ph3P NCS PPh3
(A) three (B) two 95. For higher boranes 3c-2e ‘BBB’ bond may be
(C) four (D) six a part of their structures. In B 5 H9, the number
90. The correct statement regarding the of such electron deficient bond(s) present
terminal/bridging CO groups in solid is/are—
CO4 (CO) 12 and Ir4 (CO) 12 is— (A) four (B) two
(C) zero (D) one
(A) both have equal number of bridging CO
groups 96. An organic compound (C8H10O2), which does
(B) number of bridging CO groups in not change the color of ferric chloride
CO4 (CO) 12 is 4 solution, exhibited the following 1 H NMR
spectral data : δ 7·3 (1 H, t, J = 8 Hz), 7·0 (1
(C) the number of terminal CO groups in H, d, J = 8 Hz), 6·95 (1 H, s), 6·9 (1 H, d, J =
Co 4 (CO) 12 is 8 8 Hz), 5·3 (1 H, brs, D 2 O exchangeable), 4·6
(D) the number of bridging CO groups in (2 H, s), 3·9 (3 H, s). Structure of the
Ir 4 (CO) 12 is zero compound is—
10 | CSIR-Chemical Sciences (D-13)

MeO 100. The major product formed in the sulfuric


(A) acid mediated rearrangement of the
HO
sesquiterpene santonin A is—
MeO
(B) OH H2SO4
H
MeO O
(C) H O
OH O
HO H
(D) OMe
(A) H
97. In the following reaction, the intermediate O
and the major product A are— H O
O
CHCl3/aq. NaOH
n-Bu4N+ Br−
A
N (B) H
H
HO
H O
O
(A) CHCl and
N Cl
(C) H
Cl HO
(B) CHCl and H O
N O

(D) H
(C) CCl2 and HO
N Cl H O
O
Cl
101. In the following transformation, the reagent
(D) CCl2 and
A and the major product B, respectively,
N
are—
98. Methyl 4-oxopentanoate exhibited signals at δ Ac2O/Δ
A
208, 172, 51, 37, 32 and 27 ppm in its 13C + B
NMR spectrum. The signals due to the N N
methoxy, Cl, C4 and C5 carbons are— O−
(A) OMe - 32; Cl - 208; C4 - 172; C5 - 51 SO3H
(B) OMe - 51; Cl - 208; C4 - 172; C5 - 32 (A) and OAc
(C) OMe - 32; Cl - 172; C4 - 208; C5 - 51 Cl N
(D) OMe - 51; Cl - 172; C4 - 208; C5 - 32
SO3H
99. The major product formed in the following
reaction sequence is— (B) and OAc
1. BH3, THF; H2O2/NaOH Cl N
2. Ph3P, DEAD; 4-nitrobenzoic acid
3. K2CO3, MeOH (DEAD = diethyl azodicarboxylate) CO3H
(C) and OAc
OH OH N
Cl
(A) (B)
CO3H
OH OH (D) and
(C) (D) OAc
Cl N
CSIR-Chemical Sciences (D-13) | 11

102. The peptide A on reaction with I-fluoro-2, 4- 106. The major product formed in the following
dinitrobenzene followed by exhaustive reaction sequence is—
hydrolysis gave phenylalanine, alanine, OH
1. KH, THF
serine and N-(2,4-dinitrophenyl)glycine. On 2. m-CPBA
the other hand, peptide A after two cycles of BnO SiMe3
Edman degradation gave Phe-Ser as the OH
product. The structure of the peptide A is—
OH OH
(A) Phe-Ser-Ala-Gly
H H
(B) Phe-Ser-Gly-Ala (A) BnO (B) BnO
(C) Gly-Ala-Phe-Ser O O
(D) Ala-Gly-Phe-Ser H H

103. The compound B (labeled) is precursor for H H


biosynthesis of the natural product A. The O O
labeled carbons in the product A are— (C) BnO H
(D) BnO H
COOH OH OH
Me14COOH 107. The major product formed in the following
OH reaction sequence is—
A B O OH 1. PhCHO (1 equiv.), p-TSA
HO
(A) C1, C3, C5 and Me 2. NalO4
HO OH
(B) C2, C4, C6 and Me OH
3. NaBH4

(C) C2, C4, C6 and COOH


OH
(D) C1, C3, C5 and COOH O
(A) OH
104. The major product formed in the following Ph O
reaction sequence is— OH
1. SOCl2 OH
Ph 2. NEt3 HO
COOH O O
Ph 3. CH2 = CH-OEt (B)
Ph
Ph O Ph O
HO OH
Ph Ph
(A) (B)
O O
OEt EtO (C)
Ph
Ph Ph O
Ph O O
(C) (D) HO Ph
(D) O
Ph HO
OEt OEt OH
105. The major product formed in the following 108. The major product formed in the following
reaction sequence is— reaction sequence is—
1. CH 2 I2‚ Zn-Cu‚ CH2 Cl 2
1. TsNHNH2, EtOH 2. PCC‚ CH2 C l2
2. 2 eq. nBuLi, THF cis-but-2-ene-1,4-diol ⎯⎯⎯⎯⎯⎯→
O 3. DMF O
H O H
(A) CHO (B) (A) O (B) O
CHO H H

O H H
COOH CHO
(C) (D) (C) (D)
NMe2 NMe2 COOH CHO
O H H
12 | CSIR-Chemical Sciences (D-13)

109. The conditions A-B, required for the (C) HO OH HO OH


following pericyclic reactions are—
H &
Me Me 90% 10%
A B
Me
(D) HO OH HO OH
Me Me Me
H
(A) A - Δ; B - Δ (B) A - hv; B - Δ &
95% 5%
(C) A - hv; B - hv (D) A - Δ; B - hv
112. The major product formed in the following
110. The major product formed in the following reaction sequence is—
reaction sequence is—
n 1. N-bromosuccinimide, CH2Cl2
H Bu3SnH,AlBN
Br toluene, reflux 2. potassium t-butoxide
OH
COOEt
O H
(A) (B) O
H COOEt
(A)
O H (C) (D)
OH O
H COOEt
113. The number of π electrons participating and
(B) the pericyclic mode in the following reaction
O H are—
H
H
COOEt heat
(C)
O H H
H (A) 4 and conrotatory
COOEt
(D) (B) 4 and disrotatory
O H (C) 6 and conrotatory
(D) 6 and disrotatory
111. Stereoselective reduction of the dione A
with a chiral reducing agent provides the 114. The major product formed in the following
corresponding diol B in 100% reaction sequence is—
diastereoselectivity and 90% ce favouring
O
R,R-configuration. The composition of the
product is— 1. hv
+
2. KOH, MeOH
O O HO OH OAc

O O
A B H

(A) HO OH HO OH (A) (B)


H
& O O
90% 10%
OH H OH H
(B) HO OH HO OH
(C) (D)
&
95% 5% H OMeH
CSIR-Chemical Sciences (D-13) | 13

115. The major product formed in the following 118. The major product formed in the following
photochemical reaction is— reaction sequence is—
O O 1. H2O2, NaOH
hv
2. TsNHNH4
Ph
3. potassium t-butoxide
4. H2, Pd/CaCO3, quinoline
(A) Ph (B) Ph
O O (A) (B)
O O
(C) Ph (D) Ph
O O H
COOH
116. The most suitable reagent combination of A- (C) (D)
C, required in the following conversions
are— H
OH O O 119. The major product formed in the following
B A
reaction sequence is—
SePh

C O 1. H2O2
SO2
2. Heat
O
(A) O (B)
S
O2 O
(A) A = Li/liq. NH 3 ; B = NaBH4 ,
H H
CeCl 3 .7H2 O; C = H2 , (PH3 P)3 RhCl
(B) A = Li/liq. NH 3 ; B = NaBH4 , (C) (D)
CeCl 3 .7H2 O; C = H2 , 10% Pd/C
(C) A = NaBH4 , CeCl 3 .7H2 O; B = Li/liq. H O H O
NH3; C = H2 , (Ph3 P)3 RhCl 120. The major product B formed in the
(D) A = NaBH4 , CeCl 3 .7H2 O; B = Li/liq. following reaction sequence, and overall
yield of its formation are—
NH3; C = H2 , 10% Pd/C aniline LiAIH 4
(S)-glutamic acid ⎯⎯⎯→ A ⎯⎯⎯→ B
117. The major product formed in the following 180°C 60% 80%
reaction sequence is— (A)
1. Hg(OAc)2, H2SO4 NHPh and 48%
Ph N
2. BnNH2, heat H
O
H (B) NHPh and 70%
(A) Ph N Ph N
H
O
(C) NHPh and 48%
(B) Ph N
N Ph H
O H

Ph (D) NHPh and 70%


(C) N N
H
Ph
121. Metallic gold crystallizes in fcc structure
with unit cell dimension of 4·00 Å. The
Ph
(D) N atomic radius of gold is—
(A) 0·866 Å (B) 1·414 Å
Ph (C) 1·732 Å (D) 2·000 Å
14 | CSIR-Chemical Sciences (D-13)

122. A first order gaseous reaction is 25% 127. The molecule that has the smallest diffusion
complete in 30 minutes at 227°C and in 10 coefficient in water is—
minutes at 237°C. The activation energy of (A) Glucose (B) Fructose
the reaction is closest to (R = 2 cal K– 1 (C) Ribose (D) Sucrose
mol –1)—
(A) 27 kcal mol– 1 (B) 110 kcal mol– 1 128. In the reaction between NO and H2 the
– 1 following data are obtained—
(C) 55 kcal mol (D) 5·5 kcal mol– 1
Experiment I : PH2 = Constant
123. The most probable value of r for an electron
in 1s orbital of hydrogen atom is— P NO (mm of Hg) 359 300 152
(A) a0 /2 (B) a0 – dPNO 1·50 1·03 0·25
dt
(C) ⎯ 2 a0
√ (D) 3a0 /2
Experiment II : PNO = Constant
124. The quantum mechanical virial theorem for
a general potential V(x, y, z) is given by P H2 (mm of Hg) 289 205 147

x ∂V + y ∂V + z ∂V = 2(T) where T is the


– dPH2 1·60 1·10 0·79
∂x ∂y ∂z dt
kinetic energy operator and < > indicates The orders with respect to H2 and NO are—
expectation value. This leads to the following (A) 1 with respect to NO and 2 with respect
relation between the expectation value of to H2
kinetic energy and potential energy for a
quantum mechanical harmonic oscillator (B) 2 with respect to NO and 1 with respect
1 1 to H2
problem with potential V = kxX2 + kyY2 +
2 2 (C) 1 with respect to NO and 3 with respect
1 2
to H2
kZ —
2 z (D) 2 with respect to NO and 2 with respect
1 to H2
(A) 〈T〉 = 〈V〉 (B) 〈T〉 = – 〈V〉
2 ^ is—
1 129. The angular momentum operator Ly
(C) 〈T〉 = 〈V〉 (D) 〈T〉 = – 〈V〉
2 h
(A) – (y∂/∂z – z∂/∂y)
i
125. Consider a particle in a one dimensional box
of length ‘a’ with the following potential— h
(B) (z∂/∂x – x∂/∂z)
V(x) = ∞ x<0 i
V(x) = ∞ x>a – ih ∂
(C)
V(x) = 0 0 ≤ x ≤ a/2 2m ∂x
V(x) = V1 a/2 ≤ x ≤ a h
(D) (z∂/∂x – y∂/∂y)
Starting with the standard particle in a box i
hamiltonian as the zeroth order Hamiltonian 130. Both NaCl and KCl crystallize with the fcc
and the potential of V1 from ‘a/2’ to ‘a’ as a structure. However, the X-ray powder
perturbation, the first-order energy correction diffraction pattern of NaCl corresponds to
to the ground state is— the fcc structure whereas, that of KCl
(A) V1 (B) V1/4 corresponds to simple cubic structure. This
(C) – V1 (D) V1/2 is because—
(A) K+ and Cl– are isoelctronic
126. The energy for a single electron excitation in
cyclopropenium cation in Huckel theory (B) Na + and Cl– are isoelctronic
is— (C) K+ and Cl– are disordered in the crystal
(A) β (B) 2β lattice
(C) 3β (D) 4β (D) KCl has anti-site defects
CSIR-Chemical Sciences (D-13) | 15

131. Consider a two-dimensional harmonic 136. The chemical potential (μi) of the ith
1 component is defined as—
oscillator with potential energy V(x, y) =
∂U
2 1
kxX + kyY . If Ψax (x) and Ψny (y) are the
2
2
(A) μi = ( )
∂ni T,P
(B) μi = (∂H
∂n )
i T,P
2 ∂A
eigensolutions and E nx and Eny are the
eigenvalues of harmonic oscillator problem
(C) μi = ( )
∂ni T,P
(D) μi = (∂G
∂n )
i T,P

1 137. The transition that is allowed by x-polarized


in x and y direction with potential kxX2 and
2 light in trans-butadiene is—
1 (The character table for C2h is given below)
k Y2, respectively, the wave functions and
2 y C 2h E C2 i σh
eigenvalues of the above two-dimensional
harmonic oscillator problem are— Ag 1 1 1 1 Rx, x2 , y2,
(A) Ψnx,ny = Ψnx (x) + Ψny (y) z2 , xy
Bg 1 –1 1 – 1 R x, Ry, xz, yz
Enx,ny = Enx + Eny
Au 1 1 –1 –1 z
(B) Ψnx,ny = Ψnx (x) Ψny (y)
Bu 1 –1 –1 1 x, y
Enx,ny = Enx E ny
(A) 1 Au → 1 Au (B) 1 Au → 1 B g
(C) Ψnx,ny = Ψnx (x) Ψny (y)
(C) 1 B u → 1 B g (D) 3 B g → 1 Ag
Enx,ny = Enx + Eny
(D) Ψnx,ny = Ψnx (x) + Ψny (y) 138. The masses recorded when a substance is
Enx,ny = Enx E ny weighted 4 times are 15·8, 15·4, 15·6 and
16·0 mg. The variance (square of the
132. An element exists in two crystallographic standard deviation) is closest to—
modifications with FCC and BCC structures. (A) 0·02 (B) 0·05
The ratio of the densities of the FCC and
BCC modifications in terms of the volumes (C) 0·10 (D) 0·20
of their unit cells (VFCC and VBCC) is— 139. The heat capacity of 10 mol of an ideal gas
(A) VBCC : VFCC (B) 2VBCC : VFCC at a certain temperature is 300 JK– 1 at
constant pressure. The heat capacity of the
(C) VBCC : 2VFCC (D) VBCC : √
⎯ 2 VFCC same gas at the same temperature and at
133. For an electronic configuration of two non- constant volume would be—
equivalent π electrons [π1, π1], which of the (A) 383 JK – 1 (B) 217 JK – 1
following terms is not possible ? (C) 134 JK – 1 (D) 466 JK – 1
(A) 1 Σ (B) 3 Σ
140. Work (w) involved in isothermal reversible
(C) 3 Δ (D) 3 Φ expansion from Vi to V f of n moles of an
134. Given γ (1 H) ~ – 2·7 × 108 T– 1S – 1. The ideal gas is—
resonance frequency of a proton in magnetic (A) w = – nRT In (Vf /Vi)
field of 12·6 T is close to (π = 3·14)—
(A) 60 MHz (B) 110 MHz (B) w = nRT (Vf /Vi)
(C) 540 MHz (D) 780 MHz (C) w = – nRT (Vf/Vi)
135. The atomic masses of fluorine and hydrogen (D) w = nRT log(Vf/Vi)
are 19·0 and 1·0 amu, respectively (1 amu =
141. The Maxwell’s relationship derived from the
1·67 × 10– 27 kg). The bond length of HF is
equation dG = VdP – SdT is—
2·0 Å. The moment of inertia of HF is—
∂V ∂S
(A) 3·2 × 10– 47 kg m2
(B) 6·4 × 10– 47 kg m2
(A) ( ) ( )
∂T P
=
∂P T
(C) 9·6 × 10– 47 kg m2 ∂P ∂T
(D) 4·8 × 10– 47 kg m2
(B) ( ) ( )
∂V T
=
∂S P
16 | CSIR-Chemical Sciences (D-13)

(C) (∂V
∂T ) P
∂S
=–( )
∂P T
5. (B) We know that
acceleration =
Change in velocity
∂P ∂T Change in time
(D) ( ) = – ( )
∂V T∂S P ∴ the graph represents, acceleration is 0 once
on path.
142. The number of ways in which four
molecules can be distributed in two different 6. (B) 7. (A)
energy levels is— 8. (C) It is clear from figure, the centre of circle
is (10, –7)
(A) 6 (B) 3
Y
(C) 16 (D) 8
(−, +) (+, +)
143. Consider the cell—
(10, 0)
Zn | Zn2+ (a = 0·01) || Fe2+ (a = 0·001), Fe3+ X′ X
7
(a = 0·01) | Pt
(−, −) (10,−7)
Ecell = 1·71 V at 25°C for the above cell. (+, −)
The equilibrium constant for the reaction : Y′
Zn + 2Fe3+ Zn2+ + 2Fe2+ at 25°C would 9. (C) A said, “I did it.”
be close to— B said, “I didn’t.”
(A) 1027 (B) 1054 C said, “B did it.”
(C) 1081 (D) 1040 D said, “A did it.”
144. The molecule with the smallest rotation Thus, it is clear from statements that C is
partition function at any temperature among lying.
the following is— 10. (B) 11. (D)
(A) CH3 -C ≡ C-H (B) H-C ≡ C-H 12. (A) Hill A is located N30° E of hill B.
W A
(C) H-C ≡ C-D (D) D-C ≡ C-D
30°
145. The limiting molar conductivities of NaCl,
NaI and RbI are 12·7, 10·8 and 9·1 mS m2 S N
mol – 1, respectively. The limiting molar
conductivity of RbCl would be— 30°

(A) 32·6 mS m 2 mol– 1 B E


(B) 7·2 mS m 2 mol– 1 Thus, according to question, direction of hill
(C) 14·4 mS m 2 mol– 1 B from A = S 30° W.
(D) 11·0 mS m 2 mol– 1 13. (D) 14. (A)
15. (B) Of all the options given in question only
Answers with Hints 1
19 is correct because (19 – 1) × = 6, 7 fish is
3
1. (D) 2. (D) taken by one fisherman.
3. (C) A ball is dropped from a height of 1024 ∴ 19 – 7 = 12 is divided equally among all
cm. the three with 4 fishes.
16. (A) (25 ÷ 5 + 3 – 2 × 4) + (16 × 4 – 3)
= (5 + 3 – 2 × 4) + (16 × 4 – 3)
h
= (5 + 3 – 8) + (64 – 3)
h/2
= (8 – 8) + (64 – 3)
∴ The maximum height from the ground to = 0 + 61 = 61
which it can rise after the tenth bounce 17. (C) {1}, {2, 3}, {4, 5, 6}
1024 in every set 1 number increases proceeded to
= 1 cm
2 ×2×2×2×2×2×2×2×2×2 words higher natural number.
4. (C) Thus, last number in 10th set is 55.
CSIR-Chemical Sciences (D-13) | 17

18. (B) 19. (C) 20. (C) 21. (B) 27. (B) According to Bragg’s Law,
22. (C) Since displacement of cation takes place, nλ = 2d sin θ
so no effect on density. nλ
∴ sin θ =
Cation Vacancy
Cation in 2d
interstitial site
(first order diffraction, n = 1)
A+ B− A+ A+
2·29 Å × √⎯2
A+ sin θ =
2 × 3·238 Å
B− B− A+ B−
1
= = sin 45°
2
A+ B− A+ B− A+
⎛·.· d = a ⎞
⎜ ⎟
⎝ ⎯⎯⎯⎯⎯⎯⎯⎯
√ h 2 + k 2 + l 2⎠
B− A+ B− A+ B−
θ = 45°
23. (A) From this rule, given in figure we know
dU = TdS – PdV
ΔM 28. (B)
S P 29. (B) Mixture of N 2 , H2 & NH3,
ΔV ΔG Kp = 3·75 atm– 2
He is introduced into the reactor to double the
V ΔA (−)
T
(−) total pressure. The addition of inert gas
Differentiating w.r.t. V at constant S N2 + 3H2 → 2NH3
∂U Kp ∝ (Ptotal)
∴ ( )
∂V S
= –P ∴ new Kp ,
24. (B) We know that for B2. M.O. diagram is ∴ Kp′ = (Ptotal)
= P total
3πg* 3πg* ∴ K′ p = 3·75 = 3·75 atm– 2
3σ4* 30. (B) Consider ionization as
(1σg2 ) (1σ4 *2 ) (2σg 2 ) (2σ4 *2 ) –
3σg NaOH Na +(aq) + OH(aq)
1 1 water also dissociates into ions

3π4 3π4 H2O(l) H+ (aq) + OH (aq)
+
Since reduction potential of H is higher than
Addition of an e– increases the value of bond Na +
order due to extra e – will go into bonding-
at cathode
orbital. 1
H+ (aq) + e– → 2 H2 (g)
25. (D)
at anode, 2H2O(l) → O2(g) + 4H+ (aq) + 4e–
26. (D) M atoms are at the corners Net reaction,
1
= ×8 NaOH (aq) + H2 O(l)
8 – 1 1
= 1 → Na+ (aq) + OH (aq) + H2 (g) + O2 (g)
2 2
and at body centre position = 1 31. (B) In potentiometric titration, EMF measure-
∴ M = 2 ment is done. The end point is observed after
X atoms are at face centre positions sharp jump in potential.
1
= ×6 Sharp
2 Jump
E
= 3
∴ X = 3
Thus, formula of compound = M2 X3 V
18 | CSIR-Chemical Sciences (D-13)

32. (B) We know that 39. (A)


angular node = value of l
N CO2H
and radial node O O H (−)-proline N CO2H O
n–l–1 = 2 DMF solvent
n–l = 3 −H2O
O Condensation to O *
∴ 5d orbital have 2 radial & 2 angular nodes. form enamine Carbonyls are diastereotopic
33. (B) 34. (C) enamine elects to attack *
carbonyl
35. (B) H2 + Br 2 → 2HBr
First step – initiation
K ° O O
Br2 ⎯→ 1
2Br CO2H
°
dissociation of Br2 into Br radicals
• K ° O N
°
H2 + Br ⎯→
2
HBr + H
H + Br 2 ⎯→ HBr + Br
K3 ° } propagation Optically active OH

° K ° 40. (C) excess Li excess Li


HBr + Br ⎯→ 4
H + Br 2 – inhibition in liq.NH3 in liq.NH3
t-butyl alcohol t-butyl alcohol
(not important)
° K5
2Br ⎯→ Br – termination
2 alkyl gp acts e – releasing group, facilitate the
36. (D) Daniel cell formation of ortho & meta hydrogenated
Zn|ZnSO4 (aq)||CuSO 4 (aq)|Cu7 product.
E = 1·07 V O
half cell reactions are
41. (D) O2N O C CH3
Zn(s) → Zn+2 + 2e– at anode
Cu + 2 + 2e– → Cu(s) at cathode P-nitro phenyl acetate
·.· E = Ec – Ea esters show band at 1760 – 1780 cm– 1
1·07 = 0·34 – Ezn+ 2/Zn
42. (C) 43. (C)
⇒ Ezn + 2/Zn = – 0·73 V
20 + 12 – 14
37. (A) According to Arrhenius equation, 44. (A) C10H14, DBE = =4
2
K = Ae – Ea/RT
1
1 CH3
CH3 2 H
−Ea/R
lnK 2 3 two singlets in 1H NMR
2 1
lnK
3 2 three signals in 13C NMR
H CH3
1/T 1CH3

Ea 45. (C) 46. (B)


lnK = ln A –
RT
[3,3] sigmatropic
hint – y = mx + c 3 Δ
47. (C) 2
O Formation of 3,3-bond
1 . 2
lnK decreases linearly with 1 & breaking of 1,1-bond
T 3
38. (B)
H H 2 H 2 2 H 1 H
HO 2 4 6 C CHO
2 4
1 3 5 OH O
1 2Z Cl 1 1 4E H2
Cl H
(2Z, 4E)-3-chlorohexa-2, 4-diene-1, 6-dial 48. (C)
CSIR-Chemical Sciences (D-13) | 19

49. (A) 59. (C) F C2H5


COOH B O
hydrolysis SF4
H NH2 C2H5
N F F sp3d
H 2N Coordination bond
O COOH
F
4
4 3 COOH H 3 F F
H H N NH2
OH + 1 2
1 P
H 2N 2 2 COOH
O F F
(R) - aspartic acid (S) - lysine
F
50. (B) P : 1s2 2s2 2p6 3s2 3p3
N HA N HA
5 valence e–s
Ph HB Ph HB This compound have hypervalent covalent
(I) (II) bond.
enantiotopic diastereotopic 60. (B) 61. (B)
CO 2 (CO)8
62. (D) 2RCH = CH2 + 2CO + H2 ⎯⎯⎯⎯→
51. (B) ‘C—H’ or ‘C—D’ bond cleavage in
T.S.—Primary Kinetic isotope effect RCH2 CH2 CHO + RCH2 CH3

H(D) + − H (D) HC = O
K OH
I− − fast
+
MeOH
+ I In hydroformylation reaction, we need one or
NMe3 NH3 two mole of CO from catalyst to give product
rds-E2
by migratory insertion. Thus, the given option
2 4 6 H2RH(PPh3)2Cl is not suitable as catalyst for
52. (B) 1 1,3,5-hexatriene
3 5 hydroformylation.
2 node 63. (B) H3 CORu 3 (CO) 12
Total No. of e–s :— 3 + 9 + (8 × 3) + (12 × 2)
1 node = 60 (valence e– s)
64. (D) For aqua ion complex, acidity is more
0 node when (1) small size & (ii) greater charge
65. (A) Co+2, Octahedral, eg
53. (A) 54. (C) 55. (C) 56. (D) t2g
d7, system
57. (B) dimeric complex [Re2Cl4(PMe2Ph)4 ]+ M.M. –~ 3·9 B.M.
Oxidation state 66. (D)
67. (C) ClF3 + SbF5 → SbF6 – + ClF 2 +
2x – 4 = + 1 base acid ionic product
x = 2·5 68. (C)
I
1
∴ Bond order = I
2
(B.M.O’s e–s – ABMO’s e – s)
I I
1
= (8 – 1)
2
= 3·5 I
I
Tetrahedral (I3+) Trigonal bipyromidal (I3−)
58. (C)
20 | CSIR-Chemical Sciences (D-13)

69. (A) Phosphorus Acid (ortho) For Sm+ 3, we have ground state term symbol,
2S + 1L = 6 H +3 6
O OH J 5 /2 Eu , f system

P P
reducing H OH OH OH
agent OH 6
two tautomeric forms
S =
=3
2
70. (D) 2S + 1 = 7, L = 3 (F), J = 0
H O ∴ 7F
0
71. (C) 43 C NH2
O 5 77. (C)
6 1 78. (B) K2Ba[Cu(NO2 )6]
2
O P O O N
(A)
O N N – tetragonal elongation (Zout)
O P O O N N
unpaired e– will go in dx2 – y2 orbital.
Thus, orbital order –
O
dx2 – y2 > dz2 > dxy > dxz = dyz
Nicotinamide adenine dinucleotide (NAD)
Cs 2 Ba[Cu(NO2 )6]
4-position of pyridine ring is most favourable (B)
position for nucleophilic attack in this case
– tetragonal compression (Zin)
72. (D) dz2 > dx2 – y2 > dxz = dyz > dxy
CPh3
Ph3C unpaired e– will go in dz2 orbital.
Fe Fe
79. (A) In C—H, the three valency of C is remain
OC OC
CO CDMe2 CO CDMe2 and in Co(CO)3 , 15e – system also have lack
of 3e–s for 18 electronic system.
73. (C) In solid state, Alluminium flouride have
three-dimensional network structure unlike So, CH & Co(CO)3 are isolobal.
AlCl3 (dimeric) 80. (C) We know that, Doppler shift in frequency,
υ
F Δυ = υ
F F c
F Al F 3·48 × 1018 Hz × 2·2 mms– 1
F F F F ∴ Δυ =
Al F Al 3 × 1011 mms– 1
F
F F F F = 2·55 × 107 Hz
F Al F
= 25·5 MHz
F F
F 81. (B)
82. (D) MnCr 2 O4 – normal spinel structure
74. (B) 75. (C)
(Mn+2)Td (Cr2 +3)0 O4 because Cr + 3 will have a
76. (B) Sm+3, f 5 system
LFSE in octahedral site whereas Mn+ 2 ion will
+3 +2 +1 0 −1 −2 −3
not.
83. (A)
S = 5/2
Ph2PCH2CH2PPh2 + [RhCl(CO)2]2 O
2S + 1 = 6
2 : 1 molar ratio
L = +3+2+1+0–1 −2CO C
CO Cl
= 5(H) Ph2 Rh Rh
P Cl Cl Cl
∴ J = L–S H2C C
2 Rh
(less than half-filled orbital) H2C P CO O
Ph2 υCO = 1985 (terminal Co)
= 5/2
CSIR-Chemical Sciences (D-13) | 21

CH2 couple with P to form doublet and this structure is NCS is coordinated with both side
doublet with another CH 2 gives doublets of N & S with 2e– donation
doublets. H3N NCS PPh3
84. (D) 85. (D) 86. (B) 87. (A) 88. (A) Pt Pt
PPh3P SCN NH3
89. (A) Closo-carborane – C2B 10H12
95. (D) For higher borones 3C–2e ‘BBB’ bond
(i) 1, 2–ortho isomer 1
may be a part of their structures. For B5 H9
(ii) 1, 7–meta
Styx code : 4120
(iii) 1, 12–para 5 6
4 2 H H H
3 B B
8 H
9 7 H H
B
10 11
B B
H H H
12

90. (D) Co4(CO)12 96. (B) C8H10O2,


CO 16 + 2 – 10
DBE = = 4 (one ring)
OC CO 2
Co δ7·3 (1H, + J = 8Hz)
7·0 (1H, d J = 8Hz)
O 6·95 (1H, S)
C 6·9 (1H, d, J = 8Hz)
OC CO 5·3 (1H, brs, D2 O exchangeable)
Co Co
OC CO 4·6 (2H, S)
C
O Co C 3·9 (3H, S)
OC O 3 double bond
CO
6.95,s
72 – (24 + 36) 3.9, s H 4.6, s
M–M bond = =6 CH3O H2 5.3, brs
2
OH
Terminal CO = 9
bridging CO = 3 H H 6.9, d
7.0, d H
Ir4(CO)12 7.3, t
CO MeO
OC CO
OH correct structure
Ir
aq.NaOH _ Cl
97. (D) CHCl3 _H O CCl3 : CCl2
OC CO 2
OC Ir Ir CO Cl
OC CO Cl
CCl2
Ir
N N −Cl −
OC CO CO H H

72 – (24 + 36) Cl
M–M bond = =6
2 +
N
Terminal CO = 12 H
bridging CO = 0 −H
91. (C) 92. (B) 93. (A) Cl
94. (C) Pt complexes are 16e– species generally,
therefore there is not Pt-Pt bond formation & N
22 | CSIR-Chemical Sciences (D-13)

98. (D) 51 O OH
OMe 2
37 208 OH
1 H
172 27 4 32 BnO m-CPBA
3 5 BnO
O O
Methy-4-Oxopentanoate H
13C NMR spectrum data suggest OH
OMe – 51, C1 = 172, C4 = 208, C5 = 32 BnO SiMe3
99. (A)
OH O
1. BH3.THF 2. Ph3P, DEAD
H2O2/NaOH 4-nitrobenzoic acid
107. (A)
3. K2CO3, MeOH 108. (A)
OH OH OH OH OH OH
OH
CH2I2, Zn-Cu
CH2Cl2
H H H H
100. (B) CH2 CH2
CH2Cl2 PCC
H
H H
H2SO4 OH
O HO
H O O O CHO
O
(A) O O
H H H H
CH2
H
Me
H H hυ Δ
HO HO 109. (B) Me disrotation disrotation
O H O Me
Me
O O
H
101. (D) Me
CO3H Cl
AC2O/Δ Me
OAC H
N N N
Br H
nBu SnH
O COOEt 3
110. (D) AIBN
102. (C) O H toluene
reflux
103. (C)
O H
Ph 1. SOCl2 Ph COOEt
104. (B) C COOH C Cl
Ph Ph O H
Ph
O
1. NEt3 Ph H
H
3. CH2=CH OET
OEt COOEt
105. (A) O H
H
106. (B)
OH
OH H
BnO SiMe3 BnO SiMe3
COOEt
OH OH − O
KH, THF H H
CSIR-Chemical Sciences (D-13) | 23

111. (D) 124. (A) Its first option,


O O
OH OH 〈T〉 = (V〉, all explanations are true.
Chiral
125. (D) 126. (C) 127. (D)
reducing 128. (B) From given data, it is clear that
agent
R, R-configuration 100%
diastereoselectivity rate ∝ [NO]2
90% ee means 95% R,R–configured product. and rate ∝ [H 2 ]
112. (B) Thus, orders are 2 w.r.t. NO and 1 w.r.t. H2 .
H H
129. (B) The angular momentum operator ^L is y
113. (D) heat

H
dissotation
_
6 π e s participating
^Ly = –i –h
(z ∂x∂ – x∂z∂ )
∂ ∂
= – –h (z – x )
i·i
114. (A) 115. (B)
i ∂x ∂z
116. (A)
–h ∂ ∂
OH > c=o bond O = (z – x ) (i2 = – 1)
reduction B A
i ∂x ∂z
NaBH4 Li/Liq.NH3
CeCl3.7H2O double bond 130. (A) NaCl & KCl crystallize with fcc structure
reduction (1,4)
NaCl — fcc
H2,RhCl (PPh3)3 C
KCl — simple cubic
less hindered double bond reduction
because K+ and Cl– are iso electronic and
O O form simple cubic structure simply.
1 1
131. (C) V(x,y) = kx X2 + ky Y2
2 2
117. (D) 118. (B) 119. (D) Thus, ψnx,ny = ψnx (x).ψny (y)
CH2CH2COOH and Enx, Eny = Enx + Eny
NH2 aniline LiAlH4
120. (A) 132. (B) We know
180°C 80%
COOH H 60% nM
ρ =
NV
N NHPh 4M
H 48% ∴ ρFCC =
NVFCC
121. (B) Unit cell dimension, a = 4·00Å 2M
atomic radius in fcc structure is given by and ρBCC =
NVBCC
a 4
r = = ρFCC 4M NVBCC
2√⎯ 2 2√ ⎯2 ∴ = ·
ρBCC NVFCC 2M
= 1·414 Å
2VBCC
122. (C) =
123. (B) The most probable value of r for e is 1s – VFCC
orbital of Hydrogen 133. (D) π 1 π1
σ4*
rmp = P(r) = R(r) ·r 2 2 For this, we have πg* πg*
d – 2r 2 Λ = 0(Σ)
⇒ 4 (e ·r ) = 0 1 1
dr 2Σ + 1 = 1 or 3
π 4 π4
(·. · R1s = 2·e )
– r
= 1π σg
rmp = 1 = a0 = 2Δ
3a0 There is no term symbol like 3φ because it
expectation value, <r> =
2 has Λ = 3 which is not possible in this case.
24 | CSIR-Chemical Sciences (D-13)

134. (C) We know that angular frequency, 140. (A) Isothermal reversible expansion work,
ω = γ B0
⎛ Vf⎞
where γ is gynomagnetic ratio & B0 = w = – nRT ln ⎜ ⎟
magnetic field.
⎝ Vi⎠
Thus, ω = 2·7 × 108 T – 1S – 1 × 12·6T – sign for expansion,
= 34·02 × 108 Hz If compression then + (positive) sign.
∴ resonance frequency,
141. (C) Maxwell’s relationships are
ω 34·02 × 108 Hz
υ= =
2π 2 × 3·14
= 5·14 × 108 Hz
(i) (∂S∂V) = (∂P∂T)
T V

= 514 MHz
135. (B) Moment of Inertia, (ii) (∂S∂P) = – (∂V
T ∂T ) P
J = μr2

I =
m1 m2
m1 + m2
× r2
(iii) (∂V
∂S )P
∂T
= ( )
∂P S

=
19
20
× 2 × 2 × 10– 20 m2 (iv) (∂P∂S) = – (∂T∂V)
V
S

× 1·67 × 10– 27 kg 142. (C)


= 6·6 × 10– 47 kgm2 143. (B) Zn/Zn+ 2 (a= 0·01) || Fe+ 2 (a = 0·001),
136. (D) The chemical potential is given as Fe+ 3 (a = 0·01)/Pt Ecell = 1·71V
ΔH
S P Zn + 2Fe+ 3 Zn+ 2 + 2Fe+ 2
ΔV ΔG Zn → Zn+ 2 + 2e–
V T 2e–
(−) ΔA (−) 2Fe+ 3 ⎯→ 2Fe+ 2

μi = (∂G
∂n ) i T,P
∴ log keg =
n × Ecell
0·0591
at constant temp. & pressure. 2 × 1·7100 ~
137. (B) The transition that is x-polarised light in = ~ 54
0·0591
trans-butadiene is for
1A → 1B keg = 1054
u u

⎛x u⎞B 1 1 1 1 144. (B) All explanation are correct, but option is


Au ⎜⎜ y B u ⎟⎟ Bu = 1 1 1 1 (B) H—C ≡ C—H – smallest partition
⎝ z Au ⎠ 1 –1 1 –1 function, less molecular mass.
→ x-polarised (all + ve) 145. (D) Limiting molar conductivities of reactant
138. (B)
= λ∞ of product
139. (B) n = 10 mol, CP = 300 JK– 1, C V = ?
∴ C P – C V = nR ∴ NaI + RbCl → NaCl + RbI mSm 2 mol– 1
10·8 12·7 9·1
C V = C P – nR
∴ λRbCl = 12·7 + 9·1 – 10·8
= 300 – 10 × 8·314 JK– 1
= 217 JK – 1 = 11·0 mSm2 mol– 1
Chemical Sciences
CSIR-UGC NET/JRF Exam.
Solved Paper
June 2014
Chemical Sciences
PART A dispersion (D) = (maximum marks – minimum
marks), and
1. The following diagram shows 2 perpendi- dispersion
relative dispersion (RD) = · Then,
cularly inter-grown prismatic crystals (twins) mean
of identical shape and size. What is the (A) RD of group A = RD of group B
volume of the object shown (units are (B) RD of group A > RD of group B
arbitrary) ?
(C) RD of group A < RD of group B
(D) D of group A < D of group B
5. In 450 g of pure coffee powder 50 g of chicory
10

is added. A person buys 100 g of this mixture


and adds 5 g of chicory to that. What would
be the rounded-off percentage of chicory in
2 this final mixture ?
2
(A) 10 (B) 5
(A) 60 (B) 65
(C) 72 (D) 80 (C) 14 (D) 15

2. Suppose in a box there are 20 red, 30 black, 40 6. The time gap between the two instants, one
blue and 50 white balls. What is the minimum before and one after 12·00 noon, when the
number of balls to be drawn, without angle between the hour hand and the minute
replacement, so that you are certain about hand is 66°, is—
getting 4 red, 5 black, 6 blue and 7 white (A) 12 min (B) 16 min
balls ? (C) 18 min (D) 24 min
(A) 140 (B) 97
7. Suppose
(C) 104 (D) 124
x Δ y = (x – y)2
3. In the growing years of a child, the height
x o y = (x + y)2
increases as the square root of the age while
the weight increases in direct proportion to x * y = (x × y)– 1
the age. The ratio of the weight to the square x·y = x×y
of the height in this phase of growth— +, – and × have their usual meanings. What is
(A) is constant the value of
(B) reduces with age [(197 o 315) – (197 Δ 315)} · (197 * 315) ?
(C) increases with age (A) 118 (B) 512
(D) is constant only if the weight and height (C) 2 (D) 4
at birth are both zero
8. If A × B = 24, B × C = 32, C × D = 48 then
4. Students in group A obtained the following A × D—
marks : 40, 80, 70, 50, 60, 90, 30. Students in
(A) cannot be found (B) is a perfect square
group B obtained 40, 80, 35, 70, 85, 45, 50,
75, 60 marks. Define (C) is a perfect cube (D) is odd
4 | CSIR-UGC Chemical Sciences (J-14)

9. If all horses are donkeys, some donkeys are 15. Consider a right-angled triangle ABC where
monkeys and some monkeys are men, then AB = AC = 3. A rectangle APOQ is drawn
which statement must be true ? inside it, as shown, such that the height of the
(A) All donkeys are men rectangle is twice its width. The rectangle is
(B) Some donkeys may be men moved horizontally by a distance 0·2 as shown
schematically in the diagram (not to scale).
(C) Some horses are men
C
(D) All horses are also monkeys
10. A rectangular area of sides 9 and 6 units is to O S
Q
be covered by square tiles of sides 1, 2 and 5
T
units. The minimum number of tiles needed
for this is—
(A) 3 (B) 11
(C) 12 (D) 15 A P B
11. Suppose n is a positive integer. Then (n2 + n) Area of ΔABC
What is the value of the ratio ?
(2n + 1)— Area of ΔOST
(A) may not be divisible by 2 (A) 625 (B) 400
(B) is always divisible by 2 but may not be (C) 225 (D) 125
divisible by 3
16. 80 gsm paper is cut into sheets of 200 mm ×
(C) is always divisible by 3 but may not be 300 mm size and assembled in packets of 500
divisible by 6 sheets. What will be the weight of a packet ?
(D) is always divisible by 6 (gsm = g/m2)
12. There is a train of length 500 m, in which a (A) 1·2 kg (B) 2·4 kg
man is standing at the rear end. At the instant (C) 3·6 kg (D) 4·8 kg
the rear end crosses a stationary observer on a
platform, the man starts walking from the rear 17. Find the missing letter
to the front and the front to the rear of the train A B C D
at a constant speed of 3 km/hr. The speed of F I L O
the train is 80 km/hr. The distance of the man K P U Z
from the observer at the end of 30 minutes P W D ?
is— (A) P (B) K
(A) 41·5 km (B) 40·5 km (C) J (D) L
(C) 40·0 km (D) 41·0 km 18. A merchant buys equal numbers of shirts and
13. Three identical flat equilateral-triangular trousers and pays Rs. 38,000. If the cost of 3
plates of side 5 cm each are placed together shirts is Rs. 800 and that of a trouser is Rs.
such that they form a trapezium. The length 1,000, then how many shirts were bought ?
of the longer of the two parallel sides of this (A) 60 (B) 30
trapezium is— (C) 15 (D) 10
(A) 5
⎯√ 34 cm (B) 5√
⎯ 2 cm 19. Consider the set of numbers {17 1 , 172 , …,
17300 }. How many of these numbers end with
(C) 10 cm (D) 10⎯
√ 3 cm the digit 3 ?
(A) 60 (B) 75
14. An archer climbs to the top of a 10 m high
building and aims at a bird atop a tree 17 m (C) 100 (D) 150
away. The line of sight from the archer to the 20. Find the missing number in the triangle.
bird makes an angle of 45° to the horizontal. 7 1 8
What is the height of the tree ?
(A) 17 m (B) 27 m 90 13 ?
(C) 37 m (D) 47 m 3 5 6 4 2 6
CSIR-UGC Chemical Sciences (J-14) | 5

(A) 16 (B) 96
(C) 50 (D) 80
Br
PART B

21. The correct order of basicity for the following


anions is— Br
− (A) molecule is chiral and possesses a chiral
O
plane
− −
O O (B) Molecule is chiral and possesses a chiral
NO2 axis
(C) Molecule is achiral as it possesses a
NO2 NO2 plane of symmetry
I II III (D) Molecule is achiral as it possesses a
(A) II > III > I (B) I > II > III centre of symmetry
(C) II > I > III (D) III > II > I 25. Consider the following statements about cis-
22. The major product formed in the reaction of and trans-decalins—
2, 5-hexanedione with P2O5 is— 1. cis-isomer is more stable than trans-
isomer
(A) 2. trans-isomer is more stable than cis-isomer
O 3. trans-isomer undergoes ring-flip
O 4. cis-isomer undergoes ring-flip
The correct statements among the above are—
(B) (A) 2 and 4 (B) 1 and 3
(C) 1 and 4 (D) 2 and 3
O 26. In bis (dimethylglyoximato) nickel(II), the
number of Ni—N, Ni—O and intramolecular
O hydrogen bond(s), respectively, are—
(C) (A) 4, 0 and 2 (B) 2, 2 and 2
O (C) 2, 2 and 0 (D) 4, 0 and 1
27. Among the following species, (1) Ni(II) as
O dimethylglyoximate, (2) Al(III) as oxinate,
(3) Ag(I) as chloride, those that precipitate
(D) with the urea hydrolysis method are—
(A) 1, 2 and 3 (B) 1 and 2
O (C) 1 and 3 (D) 2 and 3
23. The absolute configuration of the two 28. If an enzyme fixes N2 in plants by evolving
stereogenic (chiral) centres in the following H2, the number of electrons and protons
molecule is— associated with that, respectively are—
O (A) 6 and 6 (B) 8 and 8
(C) 6 and 8 (D) 8 and 6
29. The particles postulated to always accompany
the positron emission among—
(A) 5R, 6R (B) 5R, 6S (1) neutrino, (2) anti-neutrino, (3) electron,
(C) 5S, 6R (D) 5S, 6S are—
24. The correct statement about the following (A) 1, 2 and 3 (B) 1 and 2
molecule is— (C) 1 and 3 (D) 2 and 3
6 | CSIR-UGC Chemical Sciences (J-14)

30. Toxicity of cadmium and mercury in the body (A) η5 – C 5 H5


is being reversed by proteins, mainly using (B) η1 – C 5 H5
the amino acid residue—
(C) η5 – C 5 H5 and CO
(A) Glycine (B) Leucine
(C) Lysine (D) Cysteine (D) η1 – C 5 H5 and CO
31. NiBr2 reacts with (Et)(PH)2P at – 78°C in CS 2 37. [CoL6 ]3+ is red in colour whereas [CoL′6]3+ is
to give red compound, ‘A’, which upon green. L and L′ respectively corresponds to—
standing at room temperature turns green to (A) NH3 and H 2 O
give compound, ‘B’, of the same formula. (B) NH3 and 1, 10-phenanthroline
The measured magnetic moments of ‘A’ and
‘B’ are 0·0 and 3·2 BM, respectively. The (C) H2O and 1, 10-phenanthroline
geometries of ‘A’ and ‘B’ are— (D) H2O and NH3
(A) Square planar and Tetrahedral 38. The oxidation state of Ni and the number of
(B) Tetrahedral and Square planar metal–metal bonds in [Ni2 (CO) 6 ] 2+ that are
(C) Square planar and Octahedral consistent with the 18 electrons rule are—
(D) Tetrahedral and Octahedral (A) Ni(– II), 1 bond (B) Ni(IV), 2 bonds
32. The correct non-linear and iso-structural pair (C) Ni(–I), 1 bond (D) Ni(IV), 3 bonds
is— 39. Structures of SbPh5 and PPh5 respectively
(A) SCl2 and I3 (B) SCl2 and I3 + are—
(C) SCl2 and ClF2– (D) I3+ and ClF2– (A) trigonal bipyramidal, square pyramidal
33. Ozone present in upper atmosphere protects (B) square pyramidal, trigonal bipyramidal
people on the earth— (C) trigonal bipyramidal, trigonal bipyramidal
(A) due to its diamagnetic nature (D) square pyramidal, square pyramidal
(B) due to its blue colour 40. The typical electronic configurations of the
(C) due to absorption of radiation of transition metal centre for oxidative addition
wavelength at 255 nm are—
(D) by destroying chlorofluoro carbons (A) d 0 and d 8 (B) d 5 and d 8
34. If L is a neutral monodentate ligand, the (C) d 8 and d 10 (D) d 5 and d 10
species, [AgL4 }2+ , [AgL6 }2+ and [AgL4 ]3+ 41. Gelatin added during the polarographic
respectively are— measurement carried out using dropping
(A) paramagnetic, paramagnetic and mercury electrode—
diamagnetic (A) reduces streaming motion of Hg drop
(B) paramagnetic, diamagnetic and
(B) decreases viscosity of the solution
paramagnetic
(C) diamagnetic, paramagnetic and (C) eliminates migrating current
diamagnetic (D) prevents oxidation of Hg
(D) paramagnetic, diamagnetic and 42. The pK a values of the following salt of
diamagnetic aspartic acid are indicated below. The
35. Chromite ore on fusion with sodium predominant species that would exist at pH =
carbonate gives— 5 is—
(A) Na 2 CrO4 and Fe2O3 (pKa = 9.9) H3N COOH (pKa = 2.0)
(B) Na 2 Cr2 O7 and Fe2O3
(C) Cr2 (CO3)3 and Fe(OH) 3 COOH (pKa = 3.9)
(D) Na 2 CrO4 and Fe2(CO3)3
H 3N COO
36. The ligand(s) that is (are) fluxional in [(η5 (A)
– C5 H5) (η 1 –C 5 H5)Fe(CO)2 ] in the tempera-
ture range 221 – 298 K, is (are)— COOH
CSIR-UGC Chemical Sciences (J-14) | 7

45. The major product formed in the following


H 3N COO
reaction is—
(B)
H Me
COO
Ph AcOH
OTs
H 2N COO
(C) Me H

COO H Me
Ph
H 3N COOH OAc
(D) H
(A) Me
COO H Me
43. The major product formed in the following Ph
photochemical reaction is— OAc
(B) H Me
t-Bu
t-Bu Me Me
hv

(C) Ph H
t-Bu Me H
t-Bu
t-Bu
(D) Ph Me
(A) 46. The correct order for the rates of electrophilic
t-Bu aromatic substitution of the following com-
H
pounds is—
t-Bu Cl

(B) N N
t-Bu Me
t-Bu I II III
(A) I > II > III (B) II > I > III
t-Bu
t-Bu (C) III > II > I (D) I > III > II
47. The commutator of the kinetic energy
(C) ^
t-Bu operator, T x and the momentum operator, ^px
H for the one-dimensional case is—
d
t-Bu (A) i –h (B) i –h
dx
(C) 0 –
(D) i hx
(D) 48. The major product formed in the reaction of
t-Bu trans-1-bromo-3-methylcyclobutane with
t-Bu
sodium iodide in DMF is—
44. The pair of solvents in which PCl5 does NOT Me Me
ionize is—
(A) (B)
(A) CH3 CN, CH3 NO2
Me Me
(B) CH3 CN, CCl4
(C) C 6 H6, CCl4 (C) (D)
(D) CH3 CN, C 6 H6 l l
8 | CSIR-UGC Chemical Sciences (J-14)

49. When Si is doped with a Group V element— 55. Examine the following first order consecutive
(A) donor levels are created close to the reactions. The rate constant (in s– 1 units) for
valence band each step is given above the arrow mark—
(B) donor levels are created close to the 105 108
1. P ⎯→ Q ⎯→ R
conduction band
105 103
(C) acceptor levels are created close to the 2. P ⎯→ Q ⎯→ R
valence band 107 107
(D) acceptor levels are created close to the 3. P ⎯→ Q ⎯→ R
conduction band 102 106
4. P ⎯→ Q ⎯→ R
50. The symmetry point group of propyne is—
Steady-state approximation can be applied
(A) C 3 (B) C 3v to—
(C) D3 (D) D3d (A) 1 only (B) 3 only
51. For a first order reaction A → products, the (C) 2 and 3 only (D) 1 and 4 only
⎛ [A] ⎞
plot of ln ⎜ t ⎟ vs. time, where [A]0 and [A] t 56. The figure below represents the path followed
⎝ [A]0⎠ by a gas during expansion from A → B. The
refer to concentrations at time t = 0 and t work done is (L atm.)
respectively, is— 5 A
P(atm)
(A) a straight line with a positive slope 4
passing through origin 3
(B) a straight line with a negative slope 2
passing through origin 1 B
(C) an exponential curve asymptotic to the
time axis 1 2 3 4 5
⎛ [A] ⎞ V(L)
(D) a curve asymptotic to the ln ⎜ t ⎟ axis
⎝ [A]0⎠ (A) 0 (B) 9
(C) 5 (D) 4
52. In radical chain polymerization the quantity
given by ‘the rate of monomer depletion, 57. An aqueous solution of an optically pure
divided by the rate of propagating radical compound of concentration 100 mg in 1 mL of
formation’ is called— water and measured in a quartz tube of 5 cm
(A) kinetic chain length length was found to be – 3°. The specific
rotation is—
(B) propagation efficiency
(A) – 30° (B) – 60°
(C) propagation rate constant
(C) – 6° (D) + 6°
(D) polymerization time
58. Two phases (α and β ) of a species are in
53. Number of rotational symmetry axes for
equilibrium. The correct relations observed
triclinic crystal system is—
among the variables, T, p and μ are—
(A) 4 (B) 3
(A) Tα = Tβ, pα ≠ pβ, μα = μβ
(C) 1 (D) 0
(B) Tα ≠ Tβ, pα = pβ, μα = μβ
54. Generally, hydrophobic colloids are (C) Tα = Tβ, pα = pβ, μα = μβ
flocculated efficiently by ions of opposite
type and high charge number. This is (D) Tα = Tβ, pα = pβ, μα ≠ μβ
consistent with the— 59. The number of configurations in the most
(A) peptization principle probable state, according to Boltzmann
(B) Krafft theory formula is—
(C) Hardy-Schulze rule (A) e S/kB (B) e – S/kB
(D) Langmuir adsorption mechanism (C) e – E/kB T (D) e – ΔG/kBT
CSIR-UGC Chemical Sciences (J-14) | 9

60. The correct match of the ′HNMR chemical (A) 1 and 3 (B) Only 3
shifts (δ) of the following species/compounds (C) 2 and 4 (D) 2 and 3
is—
64. Pericyclic reaction involved in one of the
− + steps of the following reaction sequence is—
i. Heat OH
I II III + O
ii. (EtO)3P
(A) I : 5·4; II : 7·2; III : 9·2 S
(B) I : 9·2; II : 7·2; III : 5·4 H
(C) I : 9·2; II : 5·4; III : 7·2 (A) [1, 3] sigmatropic shift
(D) I : 7·2; II : 9·2; III : 5·4 (B) [3, 3] sigmatropic shift
61. The major products formed in the following (C) [1, 5] sigmatropic shift
reaction are— (D) [2, 3] sigmatropic shift
O
65. Atorvastatin (structure given below) is a—
i. Ph3C Na+
OEt OH
ii. H3O+ HOOC
OH
O F
Me
(A) + Ph3C–CH 2 CH3 N
OH
Me
CPh3 PhHN
HO Ph
(B) + EtOH O
CPh3
(A) cholesterol lowering drug
O O (B) blood sugar lowering drug
(C) + EtOH (C) anti-plasmodial drug
OEt
(D) anti-HIV drug
O
66. The maximum bond order obtained from the
(D) + CH 2 =CH 2 molecular orbitals of a transition metal dimer,
OH
formed as linear combinations of d-orbitals
62. In a Diels-Alder reaction, the most reactive alone, is—
diene amongst the following is— (A) 3 (B) 4
(A) (4E)-1, 4-hexadiene (C) 5 (D) 6
(B) (4Z)-1, 4-hexadiene 67. The term symbol that is NOT allowed for the
(C) (2E, 4E)-2, 4-hexadiene np2 configuration is—
(D) (2Z, 4Z)-2, 4-hexadiene (A) 1 D (B) 3 P
(C) 1 S (D) 3 D
63. Consider the statements about the following
structures X and Y. 68. If the ionization energy of H atom is x, the
ionization energy of Li2+ is—
Ph CN Ph (A) 2x (B) 3x
NH
X Y (C) 9x (D) 27x
1. X and Y are resonance structures 69. If temperature is doubled and the mass of the
2. X and Y are tautomers gaseous molecule is halved, the rms speed of
3. Y is more basic than X the molecule will change by a factor of—
4. X is more basic than Y (A) 1 (B) 2
The correct statement(s) among the above is 1 1
(C) (D)
(are)— 2 4
10 | CSIR-UGC Chemical Sciences (J-14)

70. In the graph below, the correct option, (B) The ‘phen’ is a π-donor ligand that
according to Kohlrausch law is— enhances the rate of electron transfer
A (C) The ‘phen’ forms charge transfer
complex with iron and facilitates the
Λ B electron transfer
C (D) The ‘phen’ forms kinetically labile
complex with iron and facilitates the
D
electron transfer
73. The compound [Re2 (Me 2 PPh)4Cl4] (M)
C
having a configuration of σ 2 π4,δ2δ*2 can be
(A) A is a weak electrolyte and B is a strong oxidized to M+ and M2+ . The formal metal
electrolyte – metal bond order in M, M+ and M2+
respectively are—
(B) A is a strong electrolyte and B is a weak
electrolyte (A) 3·0, 3·5 and 4·0 (B) 3·5, 4·0 and 3·0
(C) C is a strong electrolyte and D is a weak (C) 4·0, 3·5 and 3·0 (D) 3·0, 4·0 and 3·5
electrolyte 74. In low chloride ion concentration, the
(D) C is a weak electrolyte and D is a strong anticancer drug cis-platin hydrolyses to give a
electrolyte diaquo complex and this binds to DNA via
adjacent guanine
PART C O
N NH
71. Reduction of [Ru(NH3)5 (isonicotinamide)]3+
with [Cr(H2 O) 6 ] 2+ occurs by inner sphere N N NH2
mechanism and rate of the reaction is H
determined by dissociation of the successor (guanine)
complex. It is due to the— The co-ordinating atom of guanine to Pt(II)
(A) inert ruthenium bridged to inert is—
chromium centre (A) N1 (B) N3
(B) inert ruthenium bridged to labile (C) N7 (D) N9
chromium centre 75. The 19F-NMR spectrum of CIF3 shows—
(C) labile ruthenium bridged to inert (A) doublet and triplet for a T-shaped
chromium centre structure
(D) labile ruthenium bridged to labile (B) singlet for a trigonal planar structure
chromium centre (C) singlet for a trigonal pyramidal structure
(D) doublet and singlet for a T-shaped
72. Consider the second order rate constants for structure
the following outer-sphere electron transfer
reactions 76. The low temperature (–98°C) 19 F NMR
spectrum of SF4 shows doublet of triplets. It
[Fe(H2 O) 6 ]3+/[Fe(H2 O) 6 ] 2+ 4·0 M– 1 sec– 1
is consistent with the point group symmetry—
[Fe(phen)3]3+/[Fe(Phen)3 ]2+ 3·0 × 107 M– 1
(A) C 3v (B) C 4v
sec– 1
(C) Td (D) C 2v
(phen = 1, 10-phenanthroline)
The enhanced rate constant for the second 77. Amongst organolithium (A), Grignard (B)
reaction is due to the fact that— and organoaluminium (C) compounds, those
react with SiCl 4 to give compound containing
(A) The ‘phen’ is a π-acceptor ligand that Si — C bond are—
allows mixing of electron donor and
acceptor orbitals that enhances the rate of (A) A and B (B) B and C
electron transfer (C) A and C (D) A, B and C
CSIR-UGC Chemical Sciences (J-14) | 11

78. In its electronic spectrum, [V(H2 O) 6 ]3+ 83. The use of dynamic inert atmosphere in
exhibits two absorption bands, one at 17,800 thermogravimetric analysis (TGA)—
(υ1 ) and the second at 25,700 (υ2 ) cm – 1. The (A) decreases decomposition temperature
correct assignment of these bands, respec- (B) decreases weight loss
tively is— (C) reduces rate of decomposition
(A) υ 1 = 3 T1g(F) → 3T2g(F) (D) increases weight loss
υ 2 = 3 T1g(F) → 3T1g(P)
84. The correct statements for hollow cathode
(B) υ 1 = 3 T1g(F) → 3T1g(P) lamp (HCL) from the following are—
υ 2 = 3 T1g(F) → 3T2g(P) 1. HCL is suitable for atomic absorption
(C) υ 1 = 3 A2g → 3 T1g(F) spectroscopy (AAS)
υ 2 = 3 A2g → 3 T2g(F) 2. lines emitted from HCL are very narrow
(D) υ 1 = 3 A2g → 3 T2g(F) 3. the hardening of lamp makes it unsuitable
for AAS
υ 2 = 3 A2g → 3 T1g(F) 4. transition elements used in the lamps
79. Reactions of elemental As with hot and conc. have short life
HNO 3 and H2 SO4 , respectively, give— (A) 1, 2 and 3 (B) 2, 3 and 4
(A) As4 O6 and As 2 (SO4 )3 (C) 3, 4 and 1 (D) 4, 1 and 2
(B) As(NO3 )5 and As 2 (SO4 )3 85. Identify the correct statement about
(C) As4 O6 and H 3 AsO4 [Ni(H2 O) 6 ]2+ and [Cu(H2 O) 6 ]2+
(D) H3AsO4 and As 4 O6 (A) all Ni–O and Cu–O bond lengths of
individual species are equal
80. The total valence electron count and the
structure type adopted by the complex (B) Ni–O (equatorial) and Cu–O (equatorial)
[Fe5(CO) 15C] respectively, are— bond lengths are shorter than Ni–O
(axial) and Cu–O (axial) ones respectively
(A) 74 and nido (B) 60 and closo
(C) all Ni–O bond lengths are equal whereas
(C) 84 and arachno (D) 62 and nido Cu–O(equatorial) bonds are shorter than
81. 1H NMR spectrum of [η5 –C 5 H5)Rh(C2 H4)2] Cu–O(axial) bonds
at – 20°C shows a typical AA ‘XX’ pattern in (D) all Cu–O bond lengths are equal whereas
the olefinic region. On increasing the Ni–O(equatorial) bonds are shorter than
temperature to ~70°C, the separate lines Ni–O(axial) bonds
collapse into a single line which is due to—
86. Reaction of nitrosyl tetrafluoroborate to
(A) free rotation of the ethylene ligand about Vaska’s complex gives complex A with
the metal-olefin bond ∠M–N–O = 124°. The complex A and its
(B) intramolecular exchange between the N–O stretching frequency are, respectively—
ethylene ligands (A) [IrCl(CO)(NO)(PPh 3 )2]BF 4 , 1620 cm– 1
(C) intermolecular exchange between the (B) [IrCl(CO)(NO)2 (PPh3)](BF4)2, 1720 cm– 1
ethylene ligands
(C) [IrCl(CO)(NO)2 (PPh3)](BF4)2, 1520 cm– 1
(D) change in hapticity of the cyclopenta-
dienyl ligand (D) [IrCl(CO)(NO)(PPh 3 )2], 1820 cm– 1
82. The nuclides among the following, capable of 87. The correct order of decreasing electro-
undergoing fission by thermal neutrons, are— negativity of the following atoms is—
1. 233 U 2. 235 U (A) As > Al > Ca > S
3. 239 Pu 4. 232 Th (B) S > As > Al > Ca
(A) 1, 2 and 4 (B) 1, 3 and 4 (C) Al > Ca > S > As
(C) 2, 3 and 4 (D) 1, 2 and 3 (D) S > Ca > As > Al
12 | CSIR-UGC Chemical Sciences (J-14)

88. A 1 : 2 mixture of Me2 NCH2 CH2 CH2 PPh2 (A) Sm is a better one electron reductant than
and KSCN with K 2 [PdCl4] gives a square Eu
planar complex A. Identify the correct pairs (B) Sm is a better one electron oxidant than
of donor atoms trans to each other in complex Eu
A from the following combinations— (C) facile oxidation state is +2 for both the
1. P, N 2. N, S elements
3. P, S 4. N, N (D) both of these display similar redox
(A) 1 and 2 (B) 1 and 4 behaviour
(C) 2 and 3 (D) 3 and 4 93. The cooperative binding of O2 in hemoglobin
89. For a low energy nuclear reaction, 24Mg(d, is due to—
α) 22 Na, the correct statements from the (A) a decrease in size of iron followed by
following are— changes in the protein conformation
1. kinetic energy of d particle is not fully (B) an increase in size of iron followed by
available for exciting 24Mg changes in the protein conformation
2. total number of protons and neutrons is (C) a decrease in size of iron that is NOT
conserved accompanied by the protein conforma-
3. Q value of nuclear reaction is much tional changes
higher in magnitude relative to heat of (D) an increase in size of iron that is NOT
chemical reaction accompanied by the protein conforma-
4. threshold energy is ≤ Q value tional changes
(A) 1, 2 and 3 (B) 1, 2 and 4
94. Amongst the following which is not isolobal
(C) 2, 3 and 4 (D) 1, 3 and 4 pairs—
90. At pH 7, the zinc(II) ion in carbonic (A) Mn(CO)5, CH3 (B) Fe(CO) 4, O
anhydrase reacts with CO2 to give— (C) Co(CO)3, R 2 Si (D) Mn(CO)5, RS
H OH O 95. The correct order of the size of S, S 2–, S 2+ and
(A) Zn O
O (B) Zn C S 4+ species is—
C O (A) S > S2+ > S4+ > S2–
O (B) S 2+ > S4+ > S2– > S
H (C) S 2– > S > S2+ > S4+
OH O (D) S 4+ > S2– > S > S2+
(C) Zn O (D) Zn C O 96. The major product formed in the following
O
C O reaction is—
H
91. Molybdoenzymes can both oxidize as well as Br n-Bu3SnH
reduce the substrates, because— AlBN
(A) Mo(VI) is more stable than Mo(IV) benzene
reflux
(B) Mo(IV) can transfer oxygen atom to the H
substrate and Mo(VI) can abstract oxygen
atom from the substrate H H
(C) conversion of Mo(VI) to Mo(IV) is not (A) (B)
favoured
(D) Mo(VI) can transfer oxygen atom to the H H H H
substrate and Mo(IV) can abstract oxygen SnBu3
Br
atom from the substrate
92. A comparison of the valence electron con- (C) (D) H
figuration of the elements, Sm and Eu sug-
gests that— H H H
CSIR-UGC Chemical Sciences (J-14) | 13

97. The correct combination of reagents to effect 100. The correct 13 C NMR chemical (δ) shift
the following conversion is— values of carbons labeled a-e in the following
O COOH ester are—
O a
d
H H e Me
Me c
b O
H H (A) a : 19; b : 143; c : 167; d : 125; e : 52
(B) a : 52; b : 143; c : 167; d : 125; e : 19
(A) (i) Ph3 P +CH2 OMeCl– , BuLi; (ii) H3 O+; (C) a : 52, b : 167; c : 143; d : 125; e : 19
(iii) Jones’ reagent (D) a : 52, b : 167; c : 125; d : 143; e : 19
(B) (i) H2 N–NHTs; (ii) BuLi (2 equiv.);
(iii) DMF 101. The products A and B in the following
(C) (i) H2 N–NHTs; (ii) BuLi (2 equiv.); reaction sequence are—
NH2
(iii) CO2 O O

(D) (i) ClCH3 CO2 Et, LDA; (ii) BF3 ·OEt 2 ; OH MeO Cl A B
Et3N
(iii) DMSO, (COCl)2, Et3 N – 78°C to rt
98. The major product formed in the following O O O
reaction is— (A) A : O OMe B : N
H
Bu3N+Br−
O O O O
anhyd. CF3COOH
CH2Cl2 (B) A : O Cl B : O N
H
Br
H O O O
(A) (C) A : O OMe B : MeO N
H
H O O
(B) O
(D) A : O Cl B: O
Br
H O
(C) 102. The biosynthesis of isopentenyl pyrophos-
CF3COO phate from acetyl CoA involves—
1. Four molecules of acetyl CoA
(D) 2. Three molecules of ATP
O 3. Two molecules of NADPH
99. Consider the following reaction. 4. Two molecules of lipoic acid
O O Ph The correct options among these are—
N (A) 1, 2 and 4 (B) 1 and 2
CF3COOH
+ Ph-N3 (C) 2 and 3 (D) 1, 3 and 4
The appropriate intermediate involved in this 103. Amongst the following, the major products
reaction is— formed in the following photochemical reac-
Ph tion are—
O H HO
N O hυ
N
(A) Ph (B) H

Ph Ph O
O N HO N O
N N HO
(C) (D) O
(1) (2) (3) (4)
14 | CSIR-UGC Chemical Sciences (J-14)

(A) 1 and 3 (B) 2 and 3


3B
(C) 1 and 4 (D) 1 and 2
104. The products A and B in the following H
reaction sequence are—
X
O

BnO OMe H2N-NH2 Heat


O A B and
N (A)
H Zn
O H

BnO BnO O
Zn
(A) A : B: and
O NH (B)
H 2N H
MeO H
HO
O O O Zn
O (C) H
(B) A : N N and
MeO B: O
O O H

BnO
O O O H
O (D) and
(C) A : N B: N O
HO H Zn
O O

OMe
107. Using Boltzmann distribution, the
BnO O probability of an oscillator occupying the
(D) A : B: first three levels (n = 0, 1 and 2) is found to
H 2N O HN be p0 = 0·633, p1 = 0·233 and p2 = 0·086.
MeO
The probability of finding an oscillator in
105. Anthranilic acid, on treatment with iso-amyl energy levels n ≥ 3 is—
nitrite furnishes a product which displays a (A) 0·032 (B) 0·048
strong peak at 76 (m/e) in its mass spectrum.
(C) 0·952 (D) 1·000
The structure of the product is—
NO 108. The major products A and B in the
(A) (B) following reaction sequence are—
COOH i. PhNCO
Et3N H2, Raney Ni
OH NO2 A B
ii. MeOH, H2O
(C) (D) AcOH
COOH COOH
N O OH
106. The organoborane X, when reacted with (A) A : O B:
Et2 Zn followed by p-iodotoluene in the
presence of catalytic amount of Pd(PPh3 )4
furnishes a trisubstituted alkene. The inter- N O OH
mediate and the product of the reaction, (B) A : O B:
respectively, are—
CSIR-UGC Chemical Sciences (J-14) | 15

N O O Ph
(C) A : O B: H3COOC N COOCH3
(A)
N O OH Ph Ph
(D) A : O B:
Ph
H3COOC N COOCH3
109. The correct combination of reagents required (B)
to effect the following conversion is—
Ph Ph
COOCH3 O
Ph
H3COOC N
COOCH3 Ph
OH
(C)
(A) (i) Na, xylene, Me 3 SiCl, heat; (ii) H 3 O+ H3COOC Ph
(B) (i) Na, xylene, heat; (ii) H2 O2, NaOH Ph
(C) (i) NaOEt, EtOH; (ii) Na, xylene, heat H3COOC N Ph
(D) (i) TiCl 3 , Zn-Cu, Me3 SiCl, heat; (ii)
(D)
H3 O+ Ph
H3COOC
110. An organic compound gives following
spectral data : 112. The correct combination of reagents for
effecting the following sequence of reactions
IR : 2210, 1724 cm– 1; 1H NMR : δ 1·4 (t, J = is—
7·1 Hz, 3H), 4·4 (q, J = 7·1 Hz, 2H), 7·7 (d, J
A B
= 8·0 Hz, 2H), 8·2 (d, J = 7·0 Hz, 2H); 13C O O
NMR : δ 16, 62, 118, 119, 125, 126, 127, O O
168. The compound is— (A) A = O3 /O2 ; B = K+ – OOC–N=N–COO–
O K+, AcOH
(A) O (B) A = O2 , Rose Bengal, hυ; B = K+
NC – OOC–N=N–COO–K+, AcOH

O (C) A = O2 , Rose Bengal, hυ; B = H2, Pd/C


(B) NH2 (D) A = O2 , Rose Bengal, Δ; B = H2 , Pd/C
O
113. The correct combination of reagents required
O to effect the following conversion is—
O I
(C)
NC N N
O O O
CN (A) I2, HNO 3
(D)
O (B) s-BuLi, – 78°C followed by KI
(C) NaOEt followed by ICH2 CH2 I
111. The major product formed in the following (D) s-BuLi, – 78°C followed by ICH2 CH2 I
reaction is—
Ph 114. Consider a particle confined in a cubic box.
Δ The degeneracy of the level, that has an
:N + Ph Ph energy twice that of the lowest level, is—
(A) 3 (B) 1
H3COOC COOCH3 (C) 2 (D) 4
16 | CSIR-UGC Chemical Sciences (J-14)

115. Only two products are obtained in the CHO


following reaction sequence. The structures O N
(A) A : B:
of the products from the list I-IV are—
i. NaNH2 CHO
O O
ii. BrCH2CH2Br (B) A : B:
O
O
O A: B:
(C) CHO
O O

N
O O (D) A : CHO
B:
I II III IV O

(A) I and II 118. The spatial part of the wave function of the
(B) II and IV atom in its ground state is 1s (1)1s (2). The
(C) I and III spin part would be—
(D) III and IV (A) α(1)α(2)
116. The major product A formed in the following (B) β(1)β(2)
reaction is— 1
(C) [α(1)β(2) + β(1)α(2)]
O
MeOOC COOMe
P ⎯2

Heat
1
(D) [α(1)β(2) – β(1)α(2)]
COOMe ⎯2

MeOOC
H 119. The number of phases, components and
(A) degrees of freedom, when Ar is added to an
H equilibrium mixture of NO, O2 and NO2 in
gas phase are, respectively,
COOMe (A) 1, 3, 5 (B) 1, 4, 5
MeOOC
(C) 1, 3, 4 (D) 1, 4, 4
(B)
O 120. The major product formed in the following
reaction is—
COOMe
MeOOC
O TsCl
(C) pyridine
H
H
OH
COOMe
(D)
COOMe
(A) TsO (B)
OH
OTs
117. The products A and B in the following
reaction sequence are—
SeO2 aq. NaCN, MnO2
A B (C) (D)
dioxane i-PrOH, Me2NH
reflux
OTs OTs
CSIR-UGC Chemical Sciences (J-14) | 17

121. A particle in a one dimensional harmonic Cl


oscillator in x-direction is perturbed by a
potential λx (λ is number). The first-order (B)
correction to the energy of the ground state— HX = HCl and P = Br
(A) is zero Br
(B) is negative
(C) is positive (C)
(D) may be negative or positive but NOT HX = HBr and P = Br
zero
122. The products A and B in the following (D) Br
HX = HBr and P = Br
sequence of reactions are—
OH OH 124. Match the following natural products in
CHO MeOH/H A PhCH(OMe)2 (1 equiv) B
+
HO column A with their structural features in
reflux, 24h + H
column B.
OH OH
Column A Column B
OH
Ph (a) Colchicine 1. Tetrahydrooxepine
O OMe O O (b) Strychnine 2. Phenanthrene
HO O OMe
(A) A = HO B = HO
(c) Quinine 3. Tropolone
OH OH
(d) Ephedrine 4. Phenylethylamine
OH 5. Quionoline
O Ph O O
HO O 6. Benzofuran
(B) A = HO B= HO
HO OMe HO OMe Identify the correct match from the
following—
OH (a) (b) (c) (d)
HO O OMe Ph HO O
O
(C) A = HO B= O (A) 3 1 5 4
OH H HO OMe (B) 6 1 2 5
OH (C) 1 4 6 4
HO O OMe (D) 3 1 5 6
HO O OMe
(D) A = HO B = HOO
OH O 125. A particle in a one-dimensional box
Ph H
(potential zero between 0 to a and infinite
outside) has the ground state energy E0 =
123. The mass spectrum of the product A, formed 0·125h 2
in the following reaction, exhibits M, M + 2, · The expectation value of the above
ma2
M + 4 peaks in the ratio of about 1 : 2 : 1. Hamiltonian with ψ(x) = x (x – a) yields an
The reagent HX and the product P are— energy E1. Using a linear combination of
OH two even functions x (x – a) and x 2 (x – a)2,
HX we obtain variational minimum to the
A ground state energy as E2 . Which of the
Br following relations holds for E0 , E1 and E 2 ?
F (A) E0 < E1 < E2
(B) E0 < E2 < E1
(A) (C) E1 < E0 < E2
HX = HF and P = Br (D) E2 < E0 < E1
18 | CSIR-UGC Chemical Sciences (J-14)

126. The dissociation constant of a weak acid HX Using the approximate data given above, the
at a given temperature is 2·5 × 10– 5. The pH frequency factor (A) for a reaction of the
of 0·01 M NaX at this temperature is— type : atom + diatomic molecule →
(A) 7·3 (B) 7·7 nonlinear transition state → product,
(C) 8·3 (D) 8·7 according to the conventional transition state
theory is—
127. The ground state energy of hydrogen atom is
(A) 2 × 103 (B) 6 × 107
– 13·598 eV. The expectation values of
kinetic energy, 〈T〉 and potential energy, 〈V〉, (C) 2 × 1012 (D) 6 × 1013
in units of eV, are— 132. The interplanar spacing of (110) planes in a
(A) 〈T〉 = 13·598, 〈V〉 = – 27·196 cubic unit cell with lattice parameter a =
(B) 〈T〉 = – 27·196, 〈V〉 = 13·598 4,242 Å is—
(C) 〈T〉 = – 6·799, 〈V〉 = – 6·799 (A) 3 Å (B) 6 Å
(D) 〈T〉 = 6·799, 〈V〉 = – 20·397 (C) 7·35 Å (D) 2·45 Å
128. If ψ = 0·8 ϕ A + 0·4 ϕB is a normalized
133. A compound AxBy has a cubic structure with
molecular orbital of a diatomic molecule AB,
A atoms occupying all the corners of the
constructed from ϕ A and ϕB which are also
cube as well as all the face center positions.
normalized, the overlap between ϕA and ϕB
The B atoms occupy four tetrahedral voids.
is—
The values of x and y respectively, are—
(A) 0·11 (B) 0·31
(A) 4, 4 (B) 4, 8
(C) 0·51 (D) 0·71
(C) 8, 4 (D) 4, 2
129. At a given temperature consider
Fe2 O3(s) + 3CO(g) 2Fe(s) + 3CO2 (g); 134. The number of lines in the ESR spectrum of
K1 = 0·05 CD3 is (the spin of D is 1)—
2CO2 (g) 2CO(g) + O2 (g); K 2 = 2 × 10– 12 (A) 1 (B) 3
The equilibrium constant for the reaction (C) 4 (D) 7
2Fe2 O3(s) 4Fe(s) + 3O2 is— 135. The C = O bond length is 120 pm in CO2.
(A) 1 × 10– 13 (B) 2 × 10– 38 The moment of inertia of CO2 would be
(C) 4 × 10– 15 (D) 2 × 10– 24 close to (masses of C and O are 1·9 × 10– 27
kg and 2·5 × 10– 27 kg, respectively)—
130. In a bomb calorimeter, the combustion of
0·5 of compound A (molar mass = 50 g (A) 1·8 × 10– 45 kg m2
mol – 1) increased the temperature by 4 K. If (B) 3·6 × 10– 45 kg m2
the heat capacity of the calorimeter along
with that of the material is 2·5 kJ K– 1, the (C) 5·4 × 10– 45 kg m2
molar internal energy of combustion, in kJ, (D) 7·2 × 10– 45 kg m2
is—
136. The fluorescence lifetime of a molecule in a
(A) 1000 (B) – 1000 solution is 5 × 10– 9 s. The sum of all the
(C) 20 (D) – 20 noradiative rate constants (Σknr)for the decay
131. The translational, rotational and vibrational of excited state is 1·2 × 108 s– 1. The
partition functions for a molecule are— fluorescence quantum yield of the molecule
is—
f translation ~
– 1010 m– 1, f rotation ~
– 10, f vibration ~

1, (kBT/h) ~ 13
– 10 at room temperature, (A) 0·1 (B) 0·2
NA ~ – 6 × 1023 (C) 0·4 (D) 0·6
CSIR-UGC Chemical Sciences (J-14) | 19

137. Solutions of three electrolytes have the same 142. The E ⊗ E direct product in D3 point group
ionic strength and different dielectric contains the irreducible representations
constants as 4, 25 and 81. The corresponding D3 E 2C 3 3C 2
relative magnitude of Debye-Huckel A1 1 1 1
screening lengths of the three solutions A2 1 1 –1
are—
E2 2 –1 0
(A) 4, 25 and 81
(A) A1 + A2 + E (B) 2A1 + E
(B) 2, 5 and 9 (C) 2A2 + E (D) 2A1 + 2A2
(C) 1/2, 1/5 and 1/9
143. The result of the product C2 (x) C2(y) is—
(D) 1, 1 and 1 (A) E (B) σxy
138. Simple Hu··ckel molecular orbital theory— (C) C 2 (z) (D) i
(A) considers electron-electron repulsion 144. Given
explicitly 1. Fe(OH)2 (s) + 2e – → Fe(s) + 2OH– (aq);
(B) distinguishes cis-butadiene and trans- E° = – 0·877 V
butadiene 2. Al3+ (aq) + 3e– → Al(s); E° = – 1·66V
(C) distinguishes cis-butadiene and 3. AgBr(aq) + e– → Ag(s) + Br– (aq);
cyclobutadiene E° = 0·071 V
(D) has different coulomb integrals for non- The overall reaction for the cells in the
equivalent carbons direction of spontaneous change would be—
(A) Cell with A & B : Fe reduced
139. For the nondissociative Langmuir type Cell with A & C : Fe reduced
adsorption of a gas on a solid surface at a (B) Cell with A & B : Fe reduced
particular temperature, the fraction of
Cell with A & C : Fe oxidized
surface coverage is 0·6 at 30 bar. The
(C) Cell with A & B : Fe oxidized
Langmuir isotherm constant (in bar– 1 units)
at this temperature is— Cell with A & C : Fe oxidized
(D) Cell with A & B : Fe oxidized
(A) 0·05 (B) 0·20
Cell with A & C : Fe reduced
(C) 2·0 (D) 5·0
145. The reagent X used and the major product Y
140. For a set of 10 observed data points, the formed in the following reaction sequence
mean is 8 and the variance is 0·04. The are—
‘standard deviation’ and the ‘coefficient of O
variation’ of the data set are, respectively— A BrCN
B
N N
(A) 0·005, 0·1% (B) 0·02, 0·2% Me Me
(C) 0·20, 2·5% (D) 0·32, 1·0% CN
(A) A : LiAH4 B : Br N
141. In the Lineweaver-Burk plot of (initial
rate)– 1 vs. (initial substrate concentration)– 1 Br
for an enzyme catalyzed reaction following (B) A : LiAH4 B:
NC N
Michaelis-Menten mechanism, the y-
CN
intercept is 5000 M– 1 s. If the initial enzyme (C) A : NaBH4 B : Br N
concentration is 1 × 10– 9 M, the turnover
number is— O
(A) 2·5 × 103 (B) 1·0 × 104 (D) A : H2/Pd-C B :
N
(C) 2·5 × 10 4 (D) 2·0 × 105 Me
20 | CSIR-UGC Chemical Sciences (J-14)

Answers with Hints 10. (C) area of rectangle = 54 units


perimeter of square tiles = 4, 8, 20
1. (C) 2. (D)
Thus, minimum no. of tiles needed = 12.
3. (D) According to question, 11. (D) (n 2 + n) (2n + 1), n is a + ve integer
h ∝ √⎯⎯⎯
age n = 1, (1 + 1) (2 + 1) = 6
∴ h ∝ age
2 n = 2, 6 × 5 = 30
and w ∝ age n = 3, 12 × 7 = 84
w A number is divisible by 6, when it is also
Thus, = 1 (constant) divisible by 2 and 3.
h2
Thus, it is always divisible by 6.
if weight and height at birth are both zero.
12. (B)
4. (B) Mean of A
13. (C) Length of longer of two parallel sides of
40 + 80 + 70 + 50 + 60 + 90 + 30 trapezium = 5 × 2 = 10 cm
= = 60
7 D C
Dispersion of A = 90 – 30 = 60
relative dispersion (RD) of A 5
60
= =1 A B
60
Now, Mean of B Trapezium
5
40 + 80 + 35 + 70 + 85 + 45 + 50 + 75 + 60
=
9 5 5 5 5
= 60
dispersion of B = 85 – 35 = 50 5 5
50 14. (B) Figure clearly indicates that we have to
RD of B = = 0·83
60 find out BC + BD = h + BD
Thus, RD of A > RD of B. C bird
5. (C) 6. (D) tree
h
7. (D) {(197 + 315)2 – (197 – 315) 2 }
45°
· (197 × 315)– 1 Archer A 17 m B
1
10 m

= + 4 × 197 × 315 ×
10 m

=4 building
197 × 315
A×B 24
8. (B) ×C×D = × 48 E D
B×C 32
BC
3 Thus, tan 45° =
⇒ A × B = × 48 AB
4
h
= 3 × 12 1 = ⇒ h = 17
17
= 36 = (6)2 Thus, height of tree
⇒ Perfect square. h + BD = 17 + 10
9. (B) horses = donkeys …(1) = 27 m
15. (C) Area of ΔABC
some donkeys = monkeys …(2)
1
some monkeys = men …(3) = × AB × AC
2
go through options, eq. (1), (2) and (3) gives 1 9
Some donkeys may be men. = ×3×3=
2 2
CSIR-UGC Chemical Sciences (J-14) | 21

In rectangle, h = 2b In case of o and p – substitution – NO2 group


b acts as electron withdrawing group with
Thus, OS = mesomeric effect and it is more dominant
2
0·2 + 0·2 with p-substitution. Hence, the order of
= = 0·2 basicity is—
2
II > III > I
and ST = 0·2
22. (A)
1
Area of ΔOST = × OS × ST O
2 P2O5
1
= × 0·2 × 0·2 as acid anhydride
2 O & dehydrating agent O
Thus, value of ratio 2,5-hexanedione furan
intramolecular aldol
1
×3×3 OH
2
= O
1
× 0·2 × 0·2
2
= 225 Cyclopentanone
16. (B) 23. (C)
17. (B) A +→1 B +→1 C +→1 D 4
F +→3 I +→3 L +→3 O 21 O For 5-position 2
view
3 3 2
3
K +→5 P +→5 U +→5 Z 4 6 1 1
5
For 6-position
P +→7 W +→7 D +→7 K H
4
1 2
800
18. (B) Price of one shirt = 3
3
Price of one trousers = 1‚000 CH2
24. (A) CH2
Thus, go through options—
Thus, price of 30 shirts Br
800 × 30 CH2 CH2
= = 8,000
3
and that of 30 trousers Br
= 1‚000 × 30 = 30,000 Due to chiral plane molecule is chiral.
Total = 30,000 + 8,000 25. (A) (CSIR gave option (D), but it is wrong)
= 38,000 H H
a ring-flipping a
Thus, right option is 30. e e
(I) H H
19. (B) 20. (D)
O cis-decalin
21. (A)
H
O flipping not possible -
N (II) , ,
called as Locking group
H
O
trans-decalin
Mesomerism is not possible in case of m-
substitution, hence basicity increases due to Two factors are responsible for less stability
availability of electrons. of (I)
22 | CSIR-UGC Chemical Science (J-14)

(i) non-bonding interaction in concave area. 32. (B)


(ii) 1,3-diaxial interaction with atoms or S 201 Pm I 201 Pm

groups. Cl 103° Cl I I
26. (A) Structure of bis (dimethylglyoximato) non-linear and iso-structural pair
nickel (II)
33. (C) Ozone absorbs UV-B. UV-B (255 nm)
O H O Ni − N bond = 4 would otherwise damage the DNA of all
Ni − O bond = 0 DNA-based life on/near Earth’s Surface. It
N N
C C H − bonds = 2 acts as a greenhouse sunlight can come in but
Ni it can’t go out. Ozone protects earth by
(intramolecular)
C N N C
keeping it warm. Ozone is a protectant in
upper atmosphere but if found in lower
O H O atmosphere where we live, it is poisonous.
27. (B) 28. (B) 34. (A) [AgL4]2+ [AgL6]2+ [AgL4]3+
Ag(II) Ag(II) Ag(III)
29. (D) Positron (positive electron) result from 4d 4s 4p
Same diamagnetic
the transformation of a proton to a neutron. octahedral (no unpaired e−)

1 1 0 – unpaired e−
1P → 0n + 1e + υ paramagnetic
proton neutron positron anti-neutrino fusion
35. (A) FeCr2 O4 ⎯⎯→ Na 2 [CrO 4 ] + Fe 2 O3
anti-neutrino balance the spiner. When the
Chromite ore Na2CO 3 sodium
positron is ejected from the nucleur it very chromate
quickly collides with an electron in the
Spinel— FeII(CrIII)2O4 ↓
surroundings. Sometimes positron emission
also gives positron and neutrino with 2Al 2C
2Cr ←⎯⎯ Cr2O3 ←⎯⎯ Na2 Cr2 O7
characteristic energy spectrum. pure – Al2 O3 –Na2 CO 3
chromium –CO
30. (D) The thiol group has a high affinity for
heavy metals, so that proteins containing 36. (B) (Fluxional)
η5
cysteine, such as metallo thionein, will bind η1 (221-298 K)
metals such as Hg & Cd, Pb tightly and Fe
removes toxicity of such metals. OC CO
O
37. (A) [Co(NH3)6]+ 3 – d 6 system, Red colour
HS OH L = NH3
(thiol gp) NH2 cysteine
[Co(H 2 O) 6 ]+ 3 – d 6 system, green colour
31. (A) L′ = H2O
[NiBr2.P(Ph)2(Et)] 38. (C) [Ni2 (CO) 6 ] 2

2(Et)(Ph)2P
NiBr2 M.M. = 0.0 A (Red) 36 – (12 + 20 + 2)
−78°C M–M bond = =1
in CS2 Square planar (dsp2) 2
−2
room temperature OC CO
tetrahedral [NiBr2.P(Ph)2(Et)] Ni (– I) state OC Ni Ni CO
(sp3) green, B
OC CO
18 e– system
sp3 39. (B) SbPh5
M.M. = 3.2 BM (two unpaired e− + orbital Shows square pyramidal structure due to prior
contribution) availability of d-orbital
CSIR-UGC Chemical Sciences (J-14) | 23

Ph 43. (A) 1, 2, 4-tri-t-butyl benzene gives the 1,2,5-


Ph Ph tri-t-butyl Dewar structure. The driving force
for the reaction is probably the relief of steric
Sb compression of the t-butyl groups. While the
Ph Ph ring closure could be regarded as an allowed
disrotatory closure of a 4N-system, the orbital
{s p } d symmetry rules for aliphatic systems cannot
PPh5 = be arbitrarily extended to aromatic systems.
sp3d t-Bu
t-Bu t-Bu
Ph t-Bu H t-Bu t-Bu
Ph hυ

Ph t-Bu t-Bu
P t-Bu H
1,2,5-tri-t-butyl
T.B.P. Dewar structure
Ph
Ph 44. (C) Eliminate options, all given options have
40. (C) For an oxidative addition reaction to polar solvents except option (C). So the right
occur : answer is C. Benzene and carbon tetra
chloride are non-polar solvents and do not
(i) non-bonding e – density on the metal ionize PCl5 .
(ii) Two vacant coordination site H Me H Me
45. (A)
(iii) A metal with stable O.S. separated by 2 Ph Slow Ph
units. OTs
RDS
Thus, d 8 and d10 is correct option. Me H Me H

41. (A) Gelatin is added during the polaroghaphic ACOH fast


measurement carried out using droping
mercury electrode to— H Me
Ph
(i) reduce streaming motion of falling Hg + OTS OAc
drop (Dissociation)
Me H
(ii) increase I d (association)

(iii) increase E1/2 Due to planar structure of carbocation, attack


at both side is possible.
(iv) eliminate residual current
46. (D)
H3N COOH (pKa = 2.0) H E H E H E
(pKa = 9.9) E
42. (B) E
COOH (pKa = 3.9)
N N N N
aspartic acid
unstable electron unstable e– deficient
At pH = 5 (acidic character), deficient cation cation
But we know that H E
E E
lower pKa = strong acid
higher pKa = weak acid N
at 3-position
N N
Therefore, the species exist at pH = 5 is— Me CH3
Me
benzene ring
H 3N COO intact
Thus, I > III > II ⇒ rate of electrophilic
COO aromatic substitution.
24 | CSIR-UGC Chemical Sciences (J-14)

Cl 51. (B) A → products, First order reaction


We know that
is deactivating towards further EASR. [A]f = [A]0 e– nKt
^P ^ ^ ^ ^ ^ ⎛ [A]f ⎞
47. (C) [T x x] = TxP x – P xTx = 0 ∴ ln ⎜ ⎟ = – nKt
(same axis operator in this case do not ⎝ [A]0⎠
commute)
–h2 d 2
∴ ^
Tx = –
2μ dx 2 ln ( (
[A]f
[A]0
−nK
t

^P = – ih– d
x
dx
t
48. (C)
Straight line with –ve slope passing through
Me I Me origin.
in DMF SN2
+ 52. (A) The kinetic chain length can be defined as
Na−I cis
Br the average number of monomer moleculers
trans-1-bromo-3-methyl small
loosely
large
Me
consumed by each effective free radical
cyclobutane
held generated by initiator.
more nucleophilic
rate of propagation
I υ =
rate of initiation
49. (B) Group V element is added to the Si. The rate of propagation
fifth electron on Group V element is not =
rate of termination
bonded and excited into conduction band rate of initiation = rate of monomer depletion.
where they act as charge carriers (extrinsic
53. (D) Triclinic crystal system-most unsymme-
conduction)
trical exists in primitive form (a ≠ b ≠ c, α
Conduction ≠ β ≠ γ ≠ 90°)
band
γ
Energy c
donor band α
β
b
a
Valence band
Examples : K2 Cr2 O7, CuSO4·5H2 O, H3BO3
50. (B) One C3 axis (passing through the C* and
So, no rotational symmetry axis is found.
— C ≡ C — H group and also through the
centre of the triangle formed by joining the 54. (C) Hardy-Schulze Rule—
three hydrogen atoms), σn absent, 3σv present (1) Coagulation is brought about by ions
(each passing through C* , —C ≡ C—H group having opposite charge to that of the sol.
and one of the H atoms, and bisecting the line (2) The efficacy of an ion to cause
joining the other two H atoms). coagulation depends upon its valency (high
H charge).
(3) The minimum concentration of an
C
electrolyte required to cause coagulation or
C flocculation of a sol is called its flocculation
C3V value (millimoles per litre) efficacy varies
C directly as the square of the valency of the
H H H ion.
1 55. (D) In the state of steady approximation,
Complete list of symmetry elements — E, C 3
2 reactions are investigated under such condi-
C 3 , 3σv point group— C3v tions that the slowest rate-determining step
CSIR-UGC Chemical Sciences (J-14) | 25

does not exist, one assumes the steady state 61. (C)
approximation (s.s.a.) for the transient, i.e., O O
short-lived intermediate. Ph3C Na
In case of consecutive rn , K 1 ≠ K2 , so (3) is H
OEt
−Ph3CH
OEt O
wrong and in (2), K1 > K2 which is not OEt
possible due to long lifetime of Q. So (1) and
(4) shows s.s.a. H3O
O O
56. (D) Expansion against a particular constant EtOH EtO +
OEt
external pressure, P ext ∝ P
Then, Work done 62. (C) (2E, 4E) – 2, 4-hexadiene

V2
W = – Pext dv [expansion (–)] H
V1 H most reactive diene
W = – Pext (V2 – V1) H S-cis conformation
H
= – 1 (5 – 1) = – 4
–ve sign only indicates expansion. (2z, 4z) – 2, 4-hexadiene
57. (B) Specific rotation is given by H
T α H it also give Diels-Alder
[α]λ =
Cl but least reactive as
α = – 3°, C = 100 mg in 1 mL = 0·1 g in 1 mL H compared to previous
l = 5 cm = 0·5 dm (decimeter) H
T –3
∴ [α]λ = Other two options are not working well for
0·1 × 0·5
D–A reaction.
= – 60°
58. (C) Two phases (α and β) of a species are in 63. (D) CH2 C N CH C NH
equilibrium. Then following conditions—
(i) Thermal Equilibrium
⇒ Tα = Tβ
X Y
(ii) Mechanical equilibrium
⇒ Pα = Pβ Clearly, X and Y are toutomers because
sharing of H takes place with hetero atom.
(iii) Chemical Equilibrium
Y is more basic due to availability of electrons
⇒ μα = μβ on N-atom and forming ability of NH2 with
59. (A) The Boltzmann formula for the entropy, external source.
S = KB ln W 64. (D)
where W is number configurations in the S
most probable state of the system Δ O Ph
O [2,3]
S
∴ ln W = S
KB
Ph (EtO)3P
⇒ W = eS/KB
OH Et
O−H
O P (EtO)3
60. (A) 1HNMR − + +
S
δ 5.4 7.2 9.2 Ph
I II III
In Benzene, Hydrogens are deshielded by the Side Reaction : P (OEt)3 PhSEt
large anisotropic field generated by the e–s in OEt
+
the ring’s II system. I, III also show ring S EtOH (EtO) P = O
3
current. Ph
26 | CSIR-UGC Chemical Sciences (J-14)

65. (A) 66. (C) 71. (B) [CSIR gave wrong answer option (A)]
67. (D) np2 configuration [Ru(NH3 )5 (isonicotinamide)]3+
np Inert
3P inner
2S + 1 = 3 + [Cr(H2 O) 6 ]2+ ⎯⎯⎯⎯⎯⎯→
+1 0 –1 L = 1(P) Labile sphere mechanism
2S + 1 = 1 1D isonicotinamide can act as a bridge for
electron transfer.
L = 2
1S
72. (A) [Fe(H2O) 6 ]3+
2S + 1 = 1
For outer electron transfer reaction :
L = 0
3 D is not possible for np2 configuration.
(i) electron transfer from t 2g takes place.
(ii) spin state should be same.
68. (C) According to Bohr, ionization energy of
(iii) not much difference in bond length.
H and H like atom is given by
(iv) unsaturated π-acceptor ligand (mixing of
2π2 Z 2 me4 e– donar and acceptor orbitals).
En = –
(4πε0 )2 n2 h2 The presence of all the above factors in
Thus, En ∝ Z2 (Z = atomic number) [Fe(Phen)3 ]+ 3 makes it’s reaction faster than
For Li +2, Z = 3 [Fe(H2 O) 6 ]3+.
And for H, En = x 73. (A) [Re2(Me 2 PPh)4Cl4]
Configuration : σ2 π4 δ2 δ*2
Therefore, E n for Li + 2 = 9x
δ*
69. (B) Root Mean Square Speed of the molecule
δ
is
π
3RT 1/2
C rms = ( ) M σ
Re2[Me2PPh)4Cl4] (M+) (M+2)
T 1/2
C rms ∝ () M
(M) 3.0 3.5

BMO – ABMO
4.0

1 M—M bond order =


T= 2T1 , M = M1 2
2 8–2
= = 3 for M
C rms = ⎛⎜ ⎞⎟ = (4)1/2 = 2
2 1/2 2

⎜⎝ 1⎟⎠ 74. (C) Cis-isomer is active at low concentration
2 cis–PT (NH3 )2Cl2 + H2 O
70. (C) Strong electrolyte— cis-[Pt(NH3 )2Cl (H2O)]+
+ Cl
Strong
Electrolyte Pt binds at 7-position of guanine.
C When a self-complementary aligomer (a
Weak
D portion of a DNA chain) reacts with the cis
isomer, two adjacent guaniners are bound and
C
Watson-Crick base pairing is disrupted [See
linear increase with dilution, curve can be also James E. Huheey–Page No.-700].

extrapolated and the value of Λ m can be F
75. (A)
determined.
Weak electrolyte— F
Cl
Variation is non-linear, initially very slow, as
C approaches zero increase become very fast T-Shape
but never approaches limiting value (Y-axis). F
CSIR-UGC Chemical Sciences (J-14) | 27

19FNMR spectrum Observed spectrum


Two axial F
υ1
1 A υ2
2 × 2 × + 1 = 3 (triplet) υ3
2
equatorial F.
1
2 × 1 × + 1 = 2 (doublet) υ
2
76. (D) low temp. 19F NMR spectrum of SF4 — υ1 = 3T
1g (F) → 3 T2g (F)
doublet of triplets υ2 = 3T
1g (F) → 3 T1g (P)
F 79. (D)
80. (A) [Fe5 (CO) 15C]
C2V F Total valence electron count
S
F = 8 × 5 + 15 × 2 + 4
= 74
F structure type = 40 + 30 + 4
= 74 – 60 = 14
1
equatorial F = 2×2× +1=3 = 7e– pair
2
1 = (m + 2) = Nido
axial F = 2×2× +1=3 81. (A) [(η5 – C 5 H5) Rh (C2 H4)2]
2
One C 2 axis is present (passes through S and at – 20°C shows 1 H NMR
bisects the line joining the two F atoms each. typical AA1 XX1 pattern
No. σh .2σV present (molecular plane and the at ~ 70°C – single line
plane bisecting the line joining the two F
H H
atoms passing through S).
Complete list of symmetry elements = E, C2, H C Rh C H
2σV C H C
+ +2 − SiCl4 Cl H
Cl Si R MgBrCl H
77. (D) RMgBr H
Cl
due to free rotation of the ethylene ligand
like this, all compounds given in question about the metal-olefin bond.
form Si—C bond due to R species which 82. (D) Very heavy nuclei have a lower binding
attacks at Si centre. energy per nucleon than nuclei with an
78. (A) [V(H2O) 6 ]3+, d2 system intermediate mass. U233 and U235 are used as
3A2g(F) fission by thermal neutrons.
3p 3T1g(P) β
υ2 υ3
238 1 239 239
92 U + 0n → 92 U ⎯⎯⎯⎯→ 93 Np
3F 3T2g(F) ty2 = 23·5 min
υ1
3T1g(F) β 239
⎯⎯⎯⎯→ 94 Pu
+2 +1 0 −1 −2 ty2 = 2·3 days
All isotopes of Pu are fissile and used as a
2S + 1 = 3 3 F, 3 P nuclear fuel.
L = 3(F) 83. (A) 84. (A)
[3T 1g (F) → 3A 2g (F) – not observed low 85. (C) [Ni(H2 O) 6 ] 2+ is d 8 system and have
intensity and high energy portion of regular octahedral geometry (all Ni–O bond
spectrum] lengths are equal). Whereas [Cu(H2O) 6 ]2+ is
28 | CSIR-UGC Chemical Sciences (J-14)

d 9 system, shows large John-Teller 91. (D)


distortion with Zout (tetragonally elongated). 92. (B) Sm+2 : [Xe] 4f 6
Therefore, Cu–O (equatorial) bonds are
shorter than Cu–O (axial) bonds. Eu+2 : [Xe] 4f 7
+ Thus, Sm is better oxidant (oxidising agent)
86. (A) Ir(Ph3P)2 (CO)Cl + NOBF–4 →
statement 1, 3 and 4 are wrong.
[Ir(Ph3 P)2 (CO) (NO)Cl]+BF4–
Square pyramidal Sm+ 2 + e – → Sm+ 1 : [Xe] 4f 7
bent nitrosyl ligand, ∠M–N–O = 124° at 93. (A) (i) Binding of dioxygen molecule
apical position 1 e– donor, less e – density on PFe II + O2 PFe II O2 ↔ PFeIIIO2–
metal, high Bond Order, so N—O stretching
(ii) Forming μ-peroxo complex with second
frequency, υ NO ~ – 1620 cm– 1.
heme
87. (B) Across the period from left to right,
PFe II O2 + Fe II → PFe III —O—O—FeIII P
electro-negativity increases.
Down the group, electronegativity decreases (iii) PFeIII—O—O—FeIII P → 2PFeIII—O· ↔
combining both factors, we have the order 2PFeIV = O
S > As > Al > Ca (iv) 2FeIV = O + PFeII → PFeIII—O—FeIII P
−2
88. (A) Cl Cl (decrease in size, Fe+ 2 → Fe+ 3)
Me2NCH2CH2CH2PPh2
1 : 2 ratio Followed by changes in protein conformation
K2 Pd
− takes place.
KSCN
Cl Cl 94. (C) Isolobal pairs – electronically equivalent
Me groups
2
SCN N Mn(CO)5 = 7 + 10 = 17
CH2 less than 1 for
CH2 CH3 = 6 + 3 = 9 inertness
Pd
CH2
NCS
Fe(CO)4 = 8 + 8 = 16 less than 2 for
PPh2
O=8 inert gas configuration
On warming,
S–bonded isomer is converted to the N- Mn(CO)5 = 17 Co(CO)3 = 9 + 6 = 15
different
bonding isomer, which is presumably slightly RS = 17 R2Si = 4 + 2 = 6
more stable.
So, P opposite to N and S opposite to N. 95. (C) anion > atom > cation – order of size
24 (d‚ α) 22 Therefore, S2– > S > S2+ > S4+
89. (A) 12Mg + 1 H2 ⎯⎯→ 11Na + 2He 4
Due to increased ionic radii in anions.
Denterium
96. (A)
(i) Q >> heat of chemical reaction. Br H H
(ii) total no. of P and n is conserved.
(iii) no enough energy in d-particle. n-Bu3 SnH
90. (A) At PH = 7 AIBN
N H Benzene reflux H
O C O
N Zn O H CO2 Zn O H
H O H O H
N H H H
interconversion of CO2 and
hydrogen carbonate ion. O cis at
C O ring joint
Zn O H H H H
H O Final
H product
CSIR-UGC Chemical Sciences (J-14) | 29

Side Rh H O a
100. (D) CH3
C N N C C + N2 d b
H 3C c O
e H
CN CN CN
AIBN initiator
> C = O group shows maximum value
C CN around 170 alkene carbon – 120 – 150
Bu3Sn H Bu3Sn + C CN
H (almost)
97. (C) Me(a) adjacent to electronegative –O atom
O (more value).
N−NHTs
H H2 N−NHTs H In case of c and d there is α, β-unsaturated
−H2O carbonyl case and (d) carbon have more
H H value due to more deshielding effect.
O 2 BuLi Thus, a : 52; b : 167; c : 125; d : 143; e : 19
C O N=N Ts N−N Ts 101. (A)
Li Li Li
H −N2 H H
H −Ts H H
H 2O COOH

98. (B)
NBu4
Bu4N+Br−
anti-addition 102. (C)
Br
retention of anhyd. CF3COOH O OH
configuration CH2Cl2 2ATP HO2C
3X SCoA 2NADPH OH
H acetyl CoA mevalonic acid
ATP

O OH
Br H elimination

OPP HO OPP
99. (D) H H
H
Ph isopentenyl
O H pyrophosphate
N=N=N O N
N N
Ph
N N N Ph there is no role of lipoic acid.
CF3COOH
Intermediate ATP – 3 molecules
−N2
NADPH – 2 molecules
H Ph
O Ph O
acetyl CoA – 3 molecules.
N N
{PP = pyrophosphate group transferred from
−H
ATP}
30 | CSIR-UGC Chemical Sciences (J-14)

103. (D) 108. (A)


O O O O
hv H γ OH O C N
Ph
α β N N
O O

H H H
OH O N O
OH HO Ph
touto-
merisation O
(B) (A)
N N
Et3N
O N O O
104. (A) −CO2 H
−NH2−PH HN O
O
(A) H2 Raney Ni Ph
BnO OMe
NH O OH
O O OH MeOH, H2O
N
AcOH
O H2N NH2 BnO OMe
109. (A)
NH2 NH2 O ONa ONa
BnO O
BnO OCH3
OCH3 Na OCH3 radical
NH heat
OCH3 Xylene OCH3 dimeriza-
−HOMe H 2N O tion
OCH3
OMe O ONa ONa
(B) (A) ketyl radical unstable initial
anions product

O
O O O O
105. (A) C 1st e−
OH iso amyl O transfer
nitrite
NH2 N N O O O
2nd e− transfer
1,2-diketone
anthronilic acid
−N2
O OSiMe3 O
−CO2 Me3SiCl H3O

O OSiMe3 OH
dimerization
enediolate
m/e = 76
110. (C) IR : 2210 cm– 1
106. (D) (presence of ≡ bond C ≡ C, C ≡ N)
13CNMR : 8 types of C–atom.
organozinc
Et2Zn
+ δ 1·4 (t‚ J = 7·1 Hz‚ 3H)
—CH2—CH 3 with O
H 3B H Zn H 2B δ 4·4 (q‚ J = 7·1 Hz‚ 2H)
(X) organoborane intermediate I δ 7·7 (d‚ J = 7·0 Hz‚ 2H)
Pd(PPh3)4 aromatic protons
δ 8·2 (d‚ J = 7·0 Hz‚ 2H)
CH3 Thus, we have structure :
O
H
product CH3 O

107. (B) NC
CSIR-UGC Chemical Sciences (J-14) | 31

111. (B)
Br CH2CH2Br
Ph H NaNH2
115. (A)
N Ph
Δ O O
H3COOC N COOCH3
Br
H3COOC COOCH3 H
NaNH2
Ph Ph CH2
Ph CH2
O
N
H3COOC COOCH3 Br −HBr
O Br
Ph Ph (I)
O CH2
Br CH2CH2Br
A CH2
O2, hv O
112. (B) O O O
rose bengal (II)
O
116. (A)
Rose Bengal is used in synthetic chemistry
H COOMe COOMe
to generate singlet oxygen from triplet
heat
oxygen, which reacts with an alkene. O O
(4 + 2)CA
A : O2, Rose Bengal, hυ H H
COOMe COOMe
O O (4 + 2)CA

O KO C N N C OK O COOMe
O B, AcOH O MeOOC
H
(H 2 , Pd/C – not useful here) endo
product
H Li O
113. (D) S-BuLi
N −78°C N Stable due to secondary interaction between
−S-BuH double bond and lone pair of electrons on
O O oxygen atom.
ICH2CH2I 117. (D)
I O Se O H
dioxane C
N reflux (A)
α−oxidation
O O
i−PrOH aq. NaCN
Strong base like LDA and Buhi can abstract N Me2 NH MnO2
H attached with benzene ring. (Oxidizing
C agent)
114. (A) Energy of a particle confined in cubic
box is O
h2 118. (D) The four normalized two electrons spin
E3D = (nx2 + ny2 + nz2 )
8ma2 eigen functions with correct exchange
h2 properties are—
Let K =
8ma 2 ⎪⎧α(1) α(2)
⎪β(1) β(2)
Thus, the degeneracy of the level, E = σK, Symmetric : ⎨⎪
twice that of the lowest level = 3 ⎪⎩[α(1) β(2) + β(1) α(2)] /√
⎯2
(2, 1, 1) (1, 1, 2) (1, 2, 1) ⎯⎯⎯ E = 6K
(1, 1, 1) ⎯⎯⎯⎯ E = 3K Antisymmetric : [α(1) β(2) – β(1) α(2)/⎯
√2
32 | CSIR-UGC Chemical Sciences (J-14)

1 122. (B) The reaction is simply known as acetal


Thus, ψ(0) = 1s(1) 1s(2) · formation of glucose which is driven by
⎯2
√ anomeric effect.
[α(1) β(2) – β(1) α(2)]
OH OH OH
1
So, spin part = [α(1) β(2) – β(1) α(2)] HO CHO HO O MeOH/H+
⎯2
√ HO reflux, 24h
Spatial part = 1s(1) 1s(2) OH OH
OH OH
119. (C) All species are in gaseous phase, Hence OH bonding
P=1 Ph O PhCH(OMe)2 O interaction
O O HO
C−O σ*
number of components, C = N — E HO
H HO
OH OMe
where N = 4 OH
OMe axial anomer
E = equations = 1, viz, anomeric effect
2NO + O2 → 2NO2 123. (C) If M, M + 2 and M + 4 in the ratio of 1 :
2 : 1, this confirms two Br group in A as
So, C = 4–1=3
(a + b)2 = a2 + b2 + 2ab
degrees of freedom, = 1:2:1
F = C–P+2 Thus,
= 3–1+2=4 OH Br
Thus, P = 1‚ C = 3‚ F = 4 HBr
(HX)
120. (B) 1
Br −H2O Br
2 3
NGP
TsCl
7
4 124. (A) Quinine belongs to the quinoline group
−HCl 6 5 of alkaloids and is known as a cinchona
OH OTs alkaloid.
Natural product Group
Ephedrine — Phenyl ethly amine
Colchicine — Tropolone
OTs
OTs Strychnine — Tetrahydrooxepine
0·125h 2
121. (A) Wave function for ground state (one-D 125. (B) E0 =
ma2
harmonic oscillator)
ψ(x) = x (x – a), E1
α 1/4 – αx2 /2
ψ(0) = ()
π
e even function linear combination of x (x – a) and x2 (x – a)2,
E2
∴ First order correction to the energy
Variational principle tells that ground state
ΔE = ∫ ^
ψ(0)* H(1) ψ(0) dt energy is always less than other higher level
∞ energies.
∫ (απ)
1/4
2
= e– αx /2.λx . So, E0 < (E1, E2 )
–∞
Now, E2 is obtained from even functions
α 1/4
() π
e– αx
2 /2
dx (two), it have high value than E1
∞ Thus, E 0 < E2 < E1
(απ) ∫
1/2
2
= λ x . e– αx dx
–∞ 1 1
126. (C) pH = 14 – (p Kw) + p Ka
· . · the integrand here is overall an odd 2 2
function, so the integral vanishes. 1
+ (log C) ×
ΔE = 0 2
CSIR-UGC Chemical Sciences (J-14) | 33

p Ka =
– log Ka [Fe] 4 [CO2 ]6
K1 2 = ,
– log (2·5 × 10– 5)
= [Fe2O3]2 [CO]6
=
4·60 [CO] 6 [O2 ]3
p Kw =
14 K2 3 =
[CO2]6
log C =log (0·01) = – 2 [Fe] 4 [O2 ]3
1 1 Now, K12 ·K2 3 =
∴ pH = 14 – (14) + (4·6) [Fe2O3]2
2 4
Thus, Keq = K12 ·K2 3
1
+ (– 2) × = (0·05)2 ·(2 × 10– 12)3
2
1
= 14 – 7 + 2·3 – 1 = × 8 × 10– 36
400
= 8·3
= 2 × 10– 38
127. (A) We know that for H-atom, energy, K.E. 130. (B) In bomb calorimeter, the heat of
and P.E. holds following relation combustion, q V at constant volume is
1 internal energy and given by
〈T〉 = – 〈V〉 = – 〈E〉
2 M
qV = C × θ ×
Given that 〈E〉 = – 13·598 m
·.· 〈T〉 = – 〈E〉 50 g mol– 1
= 2·5 kJ K – 1 × 4K ×
0·5
∴ 〈T〉 = 13·598 – 1
= 1000 kJ mol
and 〈V〉 = 2 〈E〉 Since qV always has –ve sign.
= 2 × (– 13·598) Thus, molar internal energy = – 1000 k J
〈V〉 = – 27·196 131. (B) atom + diatomic molecule →
non-linear transition state
128. (B) ψ = 0·8 ϕA + 0·4 ϕB

– normalized M.O.
product
ϕA and ϕB are also normalized
f
An overlap integral is a direct measure of the
extent of the overlap of the orbitals centered
Thus, A = ( )
RT rot
h ftrans ·frot
on two different nuclei.
We know that,
= ( )
kBT
h
·NA
f
ftrans ·frot
132. (A) (h K 1) = (1 1 0)
ψ = aϕA + bϕB
a
a = 0·8, b = 0·4 ∴ dh kl =
Thus, overlap integral, ⎯⎯⎯⎯⎯⎯⎯⎯
√ h + k2 + l2
2

∫ aϕ ·bϕ
4·242 Å
S AB = dt = = 3Å
A B 1·414
= ab = 0·8 × 0·4 133. (A) AxBy
= 0·32 A atoms – all corners and face centre
[Fe] 2 [CO2 ]3 1 1
129. (B) K1 = , = ×8+ ×6
[Fe2O3] [CO]3 8 2
[CO] 2 [O2 ] = 1+3=4
K2 =
[CO2]2 B atoms – 4 tetrahedral voids
[Fe] 4 [O2 ]3 Thus, x = 4, y = 4
Keq = , A4 B 4
[Fe2O3]2
34 | CSIR-UGC Chemical Sciences (J-14)

• (iii) for same and different orbitals – overlap


134. (D) CD3, For D, I = 1
integral.
Thus, no. of lines in ESR spectrum (iv) distinguishes cis-butadiene and
= 2nI + 1 cyclobutadiene.
= 2×3×1+1=7 139. (A) Nondissociative Langmuir adsorption
135. (B) We know, 1 1
I = μr2 θ
=
Keq P
+1
2r = 2 × 120 × 10– 12 m
1 1
2r Thus, –1 =
←⎯⎯⎯→ θ Keq P
O=C=O

μ =
m1 m2 ⇒
1
Keq
= P ( )1
θ
–1
m1 + m2

=
2·5 × 10– 27 × 2·5 × 10– 27
1
Keq ( )
= 30 bar
10
0·6
–1
(2·5 + 2·5) × 10– 27
Thus, I = 1·25 × 10– 27 × (2·4 × 10– 10)2 ( )
= 30 ×
2
3
bar
= 7·2 × 10– 45 kg m2 3 1
Thus, Keq = =
Since, C atom is on axis of rotation = do not 30 × 2 20
contribute to I. = 0·05 bar– 1
136. (C) φf =
Kf 140. (C) Variance = σ2
Kf + KIC + KISC σ is standard deviation
fluorescence lifetime,
σ = ⎯√⎯⎯⎯⎯⎯
variance
φ
τ0 = f = √⎯⎯⎯
0·04 = 0·20
Kf
coefficient of variation
1
τ = σ
Kf + KIC + KISC = — × 100
10 x
=
12 × 108 s– 1 –
(x is mean of data)
= 8·3 × 10– 9 s 0·20
τ 0·83 × 10– 8 s = × 100
Thus, φf = = 8
τ0 5 × 10– 9 s
= 2·5%
137. (B) We know that Debye-Huckel Screening
141. (D) [E]0 = 1 × 10– 9 M
length
⎛ ε kBT 1 ⎞ 1/2 1
=
Km 1
· +
1
K– 1 = ⎜ 4π Σ n 0 z 2 e 2⎟ r rmax [s] 0 rmax
⎜⎝ 0⎟

j i
i

same ionic strength 1


r
ε = dielectric constants
k – 1 ∝ (ε)1/2 5000 M−1s
Thus, it should be 2, 5 and 9. 1/[s]0
138. (C) Simple Huckel molecular orbital theory
is— 1
= 5000 M– 1s
(i) Energy of similar orbital coulomb rmax
integral. 1
rmax = × 10– 3 Ms– 1
(ii) adjacent atoms – Resonance integral 5
CSIR-UGC Chemical Sciences (J-14) | 35

1 143. (C) There are two fold axes at right angles to


× 10– 3 Ms– 1 one another, there must necessarily be a
5
K2 =
1 × 10– 9 M third at right angles to both.
C2(x) C (y)
1
= × 106 S– 1 [x1, y1, z1] [x1, − y1, − z1] 2
5 C2(x) [− x1, − y1, z1]
= 2 × 105 S– 1 [x1, − y1, z1]
rmax Thus, C 2 (x).C2(y) = C 2 (z)
TON = K2 =
[E]0 144. (B) A B C
ΔG° = 1·654F 4·98F – 0·071F
142. (A) D3 E 2C 3 3C 2 Cell with A and B ⇒ since less +ve value
A1 1 1 1 of A,
A2 1 1 –1 Thus, Fe will reduce
Cell with A and C ⇒ Fe oxidized
E2 2 –1 0
145. (A) O
E⊗E 4 1 0 LiAlH4 Br C N

Let us check with options reducer Br


N C = O bond N
(i) A1 + A2 + E 4 1 0 Me Me

(ii) 2A1 + E 4 1 1 CN Br
(iii) 2A2 + E 4 1 –2 Br N
N
C N
(iv) 2A 1 + 2A2 4 4 0 Me
Chemical Sciences
CSIR-UGC NET/JRF Exam.
(December 2014)
Solved Paper
December 2014
Chemical Sciences
PART–A 4. If n is a positive integer, then
n(n + 1) (n + 2) (n + 3) (n + 4) (n + 5) (n + 6)
1. Average yield of a product in different years is divisible by—
is shown in the histogram. If the vertical bars (A) 3 but not 7 (B) 3 and 7
indicate variability during the year, then (C) 7 but not 3 (D) neither 3 nor 7
during which year was the per cent variability
over the average of that year the least ? 5. The area (in m2 ) of a triangular park of
dimensions 50 m, 120m and 130 m is—
(A) 3000 (B) 3250
(C) 5550 (D) 7800
6. Lunch-dinner pattern of a person for m days is
given below. He has a choice of a VEG or a
NON-VEG meal for his lunch/dinner—
(1) If he takes a NON-VEG lunch he will
have only VEG for dinner.
(A) 2000 (B) 2001 (2) He takes NON-VEG dinner for exactly 9
days.
(C) 2002 (D) 2003
(3) He takes VEG lunch for exactly 15 days.
2. A rectangle of length d and breadth d/2 is (4) He takes a total of 14 NON-VEG meals.
revolved once completely around its length
and once around its breadth. The ratio of What is m ?
volumes swept in the two cases is— (A) 18 (B) 24
(A) 1 : 1 (B) 1 : 2
(C) 20 (D) 38
(C) 1 : 3 (D) 1 : 4
7. A bank offers a scheme wherein deposits
3. A long ribbon is wound around a spool up to made for 1600 days are doubled in value. The
a radius R. Holding the tip of the ribbon, a interest being compounded daily. The interest
boy runs away from the spool with a constant accrued on a deposit of 1000 over the first
speed maintaining the unwound portion of the 400 days would be —
ribbon horizontal. In 4 minutes, the radius of (A) 250 (B) 183
R (C) 148 (D) 190
the wound portion becomes . In what
⎯2
√ 8. What is the next number of the following
R sequence ?
further time, it will become ?
2 2, 3, 4, 7, 6, 11, 8, 15, 10 …
(A) 12 (B) 13
(C) 17 (D) 19
9. Two locomotives are running towards each
other with speeds of 60 and 40 km/h. An
(A) √ 2 min
⎯ (B) 2 min object keeps on flying to and fro from the
front tip of one locomotive to the front tip of
(C) 2√
⎯ 2 min (D) 4 min the other with a speed of 70 km/h. After 30
CSIR-UGC Chemical Sciences (D-14) (II) | 3

minutes, the two locomotives collide and the 15. A person sells two objects at 1035 each. On
object is crushed. What distance did the the first object he suffers a loss of 10% while
object cover before being crushed ? on the second he gains 15%. What is his net
(A) 50 km (B) 45 km loss/gain percentage ?
(C) 35 km (D) 10 km (A) 5% gain (B) < 1% gain
10. Weights (in kg) of 13 persons are given (C) < 1% loss (D) no loss, no gain
below— 16. Continue the sequence
70, 72, 74, 76, 78, 80, 82, 84, 86, 88, 90, 92, 2, 5, 10, 17, 28, 41, –, –, –
94. (A) 58, 77, 100 (B) 64, 81, 100
Two new persons having weights 100 kg and (C) 43, 47, 53 (D) 55, 89, 113
79 kg join the group. The average weight of
the group increases by— 17. A ladder rests against a wall as shown. The
top and the bottom ends of the ladder are
(A) 0 kg (B) 1 kg marked A and B. The base B slips. The
(C) 1·6 kg (D) 1·8 kg central point C of the ladder falls along—
11. A code consists of at most two identical
letters followed by at most four identical A
digits. The code must have at least one letter
and one digit. How many distinct codes can C
be generated using letters A to Z and digits 1
to 9 ?
(A) 936 (B) 1148
(C) 1872 (D) 2574 B
12. Two solid iron spheres are heated to 100°C (A) a parabola (B) the arc of a circle
and then allowed to cool. One has the size of (C) a straight line (D) a hyperbola
a football; the other has the size of a pea.
Which sphere will attain the room tempera- 18. 20% of students of a particular course get jobs
ture (constant) first ? within one year of passing. 20% of the
remaining students get jobs by the end of
(A) The bigger sphere second year of passing. If 16 students are still
(B) The smaller sphere jobless, how many students had passed the
(C) Both spheres will take the same time course ?
(D) It will depend on the room temperature (A) 32 (B) 64
13. Find the missing letter— (C) 25 (D) 100
A ? Q E 19. Binomial theorem in algebra gives (1 + x)n =
a0 + a1 x + a2 x2 + …… + a n xn, where a0 , a 1 ,
C M S C
……, an are constants depending on n. What
E K U A
is the sum a0 + a1 + a2 + …… + an ?
G I W Y
(A) 2n (B) n
(A) L (B) Q
(C) n2 (D) n2 + n
(C) N (D) O
14. The least significant bit of an 8-bit binary 20. A sphere is made up of very thin concentric
number is zero. A binary number whose value shells of increasing radii (leaving no gaps).
is 8 times the previous number has— The mass of an arbitrarily chosen shell is—
(A) 12 bits ending with three zeros (A) equal to the mass of the preceding shell
(B) 11 bits ending with four zeros (B) proportional to its volume
(C) 11 bits ending with three zeros (C) proportional to its radius
(D) 12 bits ending with four zeroes (D) proportional to its surface area
4 | CSIR-UGC Chemical Sciences (D-14) (II)

PART–B 29. The rate of the reaction


hv
Ni(CO) 4 + PPh3 ⎯→ [Ni(CO)3 (PPh3 )] + CO
21. The correct order of the retention of cations depends on—
on a sulfonated cation exchange resin column
is— (A) concentration of both the reactants
(A) Ag+ > K+ > Na+ > Li + (B) concentration of Ni(CO)4 only
(B) K+ > Na+ > Ag+ > Li + (C) concentration of PPh3 only
(C) Li+ > Na+ > K+ > Ag+ (D) the steric bulk of PPh3
(D) Li+ > Na+ > Ag+ > K+
22. Among F–, Na+ , O 2– and Mg2+ ions, those 30. The hapticities ‘x ’ and ‘y ’ of the arene
having the highest and the lowest ionic radii moieties in the diamagnetic complex [(η x -
respectively are— C 6 H6) Ru(ηy - C6 H6)] respectively are—
(A) O2 – and Na+ (B) F – and Mg2 + (A) 6 and 6 (B) 4 and 4
2 –
(C) O and Mg 2 + (D) Mg2 + and O2 – (C) 4 and 6 (D) 6 and 2
23. In a polarographic measurement, (aqueous 31. The correct order of the isomeric shift in
KCl solution used as supporting electrolyte) Mossbauer spectra (57 Fe source) of iron
an applied potential more than + 0·4 V, compounds is—
results mainly in the formation of— (A) Fe(II) > Fe(III) > Fe(IV)
(A) HgI (B) HgII (B) Fe(III) > Fe(II) > Fe(IV)
(C) Cl2 (D) O2
(C) Fe(IV) > Fe(III) > Fe(II)
24. The reaction between SbF5 and two equiva- (D) Fe(IV) > Fe(II) > Fe(III)
lents of HF leads to the formation of— 32. The δ-bond is formed via the overlap of—
(A) H2SbF3 + 2F2 (B) HSbF2 + 3F2 (A) dx2 – y2 and dx2 – y2 orbitals
(C) SbF3 + H2 + 2F2 (D) [SbF6]– [H2 F]+ (B) dxz and dxz orbitals
25. The extent of π-electron conjugation in mac- (C) dxy and dxy orbitals
rocyclic rings of (a) heme, (b) coenzyme B12 (D) dyz and dyz orbitals
and (c) chlorophyll follows the order—
(A) (a) > (c) > (b) (B) (a) > (b) > (c) 33. Reductive elimination step in hydrogenation
(C) (c) > (a) > (b) (D) (b) ≈ (a) > (c) of alkenes by Wilkinson catalyst results in
(neglecting solvent in coordination sphere of
26. The point group symmetries for trans- Rh)
[Cr(en)2F 2 ]+ and [TiCl6 ]3–, respectively, are— (A) T-shaped [Rh(PPh3 )2Cl]
(A) D4d and D3d (B) D3d and D4d
(B) Trigonal-planar [Rh(PPh3 )2Cl]2 +
(C) D4h and D3h (D) D3h and D4h
(C) T-shaped [Rh(H)(PPh3 )Cl]+
27. The S and L values for 15N atom respectively,
are— (D) Trigonal-planar [Rh(H)(PPh 3 )2]
1 1 34. In the following reaction
(A) and 1 (B) and 0
2 2 NH3
3 [PtCl4 ]2 – + NO2 ⎯→ A ⎯→ B
(C) 1 and 0 (D) and 0
2 compound B is—
28. The product of the reaction of propene, CO (A) trans–[PtCl2 (NO2 )(NH3 )] –
and H2 in the presence of CO2 (CO) 8 as a (B) cis–[PtCl2(NO2 )(NH3 )] –
catalyst is—
(C) trans–[PtCl2 (NH3 )2]
(A) butanoic acid (B) butanal
(C) 2-butanone (D) methylpropanoate (D) cis–[PtCl2(NO2 )2]2–
CSIR-UGC Chemical Sciences (D-14) (II) | 5

35. Co 4 (CO) 12 adopts the— (A) 2, 3 and 3, 3 (B) 3, 3 and 2, 3


(A) closo—structure (C) 3, 3 and 2, 2 (D) 2, 4 and 3, 2
(B) nido—structure 41. In the most stable conformation of neomen-
(C) arachno—structure thol, stereochemical orientation of the three
(D) hypho—structure substituents on the cyclohexane ring are—
36. The major product formed in the following (A) OH : equatorial; i-Pr : equatorial and Me
reaction is— : equatorial
(B) OH : axial; i-Pr : equatorial and Me :
equatorial
(C) OH : equatorial; i-Pr : equatorial and Me
: axial
(D) OH : equatorial; i-Pr : axial and Me :
equatorial
42. The reaction of 1-bromo-2-fluorobenzene
(A) (B) with furan in the presence of one equivalent
of Mg gives—

OCH2OCH3 (A) (B)


HOOC
(C) (D)
H3C

37. The compound that is antiaromatic is— (C) (D)

43. Identify correct statements for mercury as an


I II III IV
environment pollutant :
(A) I (B) II 1. Carbanionic biomethylation converts it
(C) III (D) IV to MeHg+.
38. The decreasing order of basicity of the 2. Thiol group of cysteine has strong
following compounds is— affinity for mercury.
3. Mercury containing industrial catalyst
H
N N N release caused Minamata disaster.
The correct answer is—
N H N N N
N (A) 1 and 2 (B) 1 and 3
(C) 2 and 3 (D) 1, 2 and 3
I II III IV 44. The increasing order of pK a value of the
(A) III > II > III > IV (B) IV > I > II > III circled hydrogens in the following compounds
(C) III > II > I > IV (D) IV > III > II > I is—
39. The configurations of carbon atoms C 3 and C4
in D-ribose, respectively, are—
(A) R and S (B) S and R
(C) R and R (D) S and S
40. The number of histidine amino acid nitrogen
atoms coordinated to bimetallic active site of I II III
oxyhemocyanin and oxyhemerythrin, respec- (A) I < II < III (B) I < III < II
tively, are— (C) II < I < III (D) II < III < I
6 | CSIR-UGC Chemical Sciences (D-14) (II)

45. The absolute configurations of the chiral 49. The major product of the following reaction
centres of starting ketone in the following is—
reaction is—

(A) (B)
Major product
(A) 3R, 6S (B) 3S, 6S Br
(C) 3R, 6R (D) 3S, 6R (C) (D)
S
46. The product for the following sequence of
reactions is— Br
50. The major product of the following reaction
is—

(A) (B)

(A) (B)

(C) (D)
(C) (D)
47. A compound with molecular formula C 4 H6O2
shows band at 1770 cm– 1 in IR spectrum and 51. For the cell reaction,
peaks at 178, 68, 28, and 22 ppm in 13C NMR Sn(s) + Sn4 + (aq) 2Sn 2 + (aq),
spectrum. The correct structure of the com- separate electrode reactions could be written
pound is— with the respective standard electrode poten-
tial data at 25°C as—
(A) (B) Sn 4 + (aq) + 2e → Sn2 + (aq), E0 = + 0·15 V
Sn 2 + (aq) + 2e → Sn(s), E0 = + 0·14 V
When RT/F is given as 25·7 mV, logarithm of
(C) (D) the equilibrium constant (ln K) is obtained
as—
(A) 22·6 (B) 226
48. The cyclic product(s) of the following photo-
chemical reaction is(are)— (C) 2·26 (D) 2·26 × 10– 1
52. For a particle of mass m confined in a box of

⎯⎯⎯⎯⎯⎯→ length L, assume Δx = L. Assume further that
1
Vapour phase Δ p (min) = (p 2 ) /2 . Use the uncertainty
principle to obtain an estimate of the energy
(A) only cis-1, 2-dimethylcyclopentane of the particle. The value will be—
(B) only trans-1, 2-dimethylcyclopentane h2 h2
(A) (B)
(C) a mixture of cis- and trans-1, 2-dimethy- (8mL2 ) (8mL2 )
lcyclopentanes h2 h2
(C) 2 (D)
(D) only 2, 6-dimethylcyclohexanol (32mL ) (2mL2 )
CSIR-UGC Chemical Sciences (D-14) (II) | 7

53. The mass of metastable ion produced due to 59.


decomposition of F1 + in the following mass
fragmentation sequence is—
Diethyl phthalate ⎯→ F 1 + ⎯→ F2 + + CO
M+‚ 222 (177)
(A) 141·2 (B) 125·4
(C) 45·0 (D) 210·2
54. If the component of the orbital angular mom-
entum along the molecular axis of a heteronu- Identify the speed distribution functions of
clear diatomic molecule is nonzero, the rota- Ne, Ar, and Kr with the curves in the figure
tional-vibrational spectrum will show— above.
(A) P and R branches only (A) Ne-A, Ar-B, Kr-C
(B) P and Q branches only (B) Ne-B, Ar-C, Kr-A
(C) Q and R branches only (C) Ne-C, Ar-B, Kr-A
(D) all the P, Q and R branches
(D) Ne-C, Ar-A, Kr-B
55. The ratio of the relative intensities of the
carbon signals in the first order 13C NMR 60. For a process in a closed system, temperature
spectrum of CD3 CI is— is equal to—
∂H ∂A
(A) 1 : 4 : 6 : 4 : 1
(B) 1 : 3 : 3 : 1
(A) ( )
∂P S
(B) – ( )
∂V T
∂G ∂H
(C) 1 : 6 : 15 : 20 : 15 : 6 : 1 (C) ( ) (D) ( )
(D) 1 : 3 : 6 : 7 : 6 : 3 : 1 ∂P T ∂S P

56. Bond lengths of homonuclear diatomic 61. A reaction A + B + C → D follows the


molecules can be determined with the help of mechanism—
both— A+B AB
(A) rotational and vibrational spectroscopy AB + C → D
(B) rotational and rotational Raman spectro- in which first step remains essentially in
scopy equilibrium. If ΔH is the enthalpy change for
(C) rotational Raman and electronic spectro- the first reaction and E0 is the activation
scopy
energy for the second reaction, the activation
(D) vibrational and electronic spectroscopy energy of the overall reaction will be given
57. The biosynthetic precursor of abietic acid by—
is— (A) E0 (B) E0 – ΔH
(A) shikimic acid (B) mevalonic acid
(C) chorismic acid (D) cinnamic acid (C) E0 + ΔH (D) E0 + 2ΔH
58. The amino acid constituents of artificial 62. Hydrogen is adsorbed on many metal surfaces
sweetener given below are— by dissociation (S represents a surface site)—
H H
| | | |
H2 + —S—S — —S —S —
If the pressure of H2 (p) is small, the fraction
(A) D-Glutamic acid and L-phenylglycine of the surface covered by hydrogen is propor-
tional to—
(B) L-Glutamic acid and L-phenylalanine
(C) L-Aspartic acid and L-phenylalanine (A) p (B) p2
(C) p 1/2 (D) p3/2
(D) L-Aspartic acid and L-tyrosine
8 | CSIR-UGC Chemical Sciences (D-14) (II)

63. The exact differential df of a state function average molar mass and (Mw ) for the weight
f (x, y), among the following, is— average molar mass. The variance of Mn will
x then be—
(A) xdy (B) dx – dy
y (A) 39 (B) 3
1 x (C) 1 (D) 87
(C) ydx – xdy (D) dx – 2 dy
y y
70. Wavelength (λ in nm) of the Lyman series for
∂ an one-electron ion is in the range 24 ≤ λ ≤
64. The angular momentum operator Lz = – ih
∂φ 30. The ionization energy of the ion will be
has eigenfunctions of the form exp [iAφ]. The 1019
condition that a full rotation leaves such an (
closest to 1J =
1·6 )
eV —
eigenfuction unchanged is satisfied for all the
(A) 32 eV (B) 42 eV
values of A—
1 2 4 (C) 52 eV (D) 62 eV
(A) 0, ± , ± , ± 1, ± , …
3 3 3
(B) 0, ± 1, ± 2, ± 3, … PART–C
1 3
(C) 0, ± , ± 1, ± , …
2 2 71. 1 H NMR spectrum of free benzene shows a
1 3 5 peak at ~ 7·2 ppm. The expected chemical
(D) 0, , , , … shift (in ppm) of C6 H6 ligand in 1 H NMR
2 2 2
spectrum of [(η6 –C 6 H6)Cr(CO)3 ] and the
65. X-ray diffraction does not give any structural
information for— reason for it, if any, is/are—
(A) metallic solids (B) ionic solids (A) 4·5; disruption of ring current
(C) molecular solids (D) amorphous solids (B) 9·0; inductive effect
(C) 7·2
66. For an enzyme-substrate reaction, a plot
1 1 (D) 2·5; combination of inductive effect and
between and yields a slope of 40s. If the disruption of ring current
v [s] 0
enzyme concentration is 2·5 μM, then the 72. Match the metalloproteins in column A with
catalytic efficiency of the enzyme is— their function in column B—
(A) 40 Lmol– 1 s– 1 (B) 10– 4 Lmol– 1 s– 1 Column A Column B
(C) 107 Lmol– 1 s– 1 (D) 104 Lmol– 1 s– 1 (a) Oxyhemocyanin 1. hydrolysis of C-
terminal peptide
67. 10 ml of 0·02 M NaOH is added to 10 ml of
bond
0·02 M acetic acid (pKa = 4·75). The pH of
(b) Carbonic anhydrase 2. methylation
the solution will be closest to—
(c) Cytochrome P450 3. conversion of
(A) 7·0 (B) 8·4
CO2 to H2 CO3
(C) 5·6 (D) 9·6
(d) Carboxypeptidase A 4. oxidation of
68. For a polydispersed macromolecular colloid, alkene
osmometry gives— 5. oxygen storage
(A) weight–average molecular weight 6. oxygen transport
(B) number–average molecular weight
Codes :
(C) both weight–average and number aver-
age molecular weight (a) (b) (c) (d)
(D) viscosity–average molecular weight (A) 6 3 4 1
69. A sample experiment revealed that PVC (B) 5 3 1 6
formed in the medium has (Mn ) = 13, and (C) 6 2 3 1
(Mw) = 16, where (Mn ) stands for the number (D) 5 4 3 1
CSIR-UGC Chemical Sciences (D-14) (II) | 9

73. The geometric cross-section (in barn) of a (C) Sigmoidal and pH independent; hyperbo-
nucleus A = 125, r0 = 1·4 × 10–15 m approxi- lic and pH dependent.
mately is— (D) Hyperbolic and pH dependent; sigmoidal
(A) 1·05 (B) 1·54 and pH independent.
(C) 2·05 (D) 2·54 79. The compound that undergoes oxidative addi-
74. Na[(η5–C tion reaction in presence of H 2 is—
5 H5 ) Fe(CO2 )] reacts with Br2 to
give A. Reaction of A with LiAlH4 results in (A) [Mn(CO)5 ]–
B. The proton NMR spectrum of B consists of (B) [(η5 –C 5 H5) Mo(CO)3]–
two singlets of relative intensity 5 : 1. (C) [IrCl(CO) (PPh3 )2]
Compounds A and B, respectively, are—
(D) [η5–C 5 H5)2 ReH]
(A) (η5–C 5 H5) Fe(CO)2Br and (η5 –C 5 H5)
Fe(CO) 2 H 80. The spin-only magnetic moment and the
spectroscopic ground state term symbol of
(B) (η4–C 5 H5) Fe(CO)2Br2 and (η4 –C 5 H5) manganese centre in [MnF6 ]3 – ion respecti-
Fe(CO) 2 HBr vely, are—
(C) (η5–C 5 H5) Fe(CO)2 Br and (η 4 –C 5 H5) (A) 4·9 BM and 5 D (B) 4·9 BM and 4 F
Fe(CO) 2 (H)2 (C) 3·9 BM and 3 D (D) 4·9 BM and 3 F
5 5
(D) (η –C 5 H5)Fe(CO)2 Br and (η –C 5 H5) 81. The main products of the reaction of equimo-
Fe(CO) 2 HBr lar quantities of XeF6 with NaNO3 – are—
(A) XeOF4, NaF and NO2 F
75. Which of the following will result in devia-
tion from Beer’s law ? (B) XeO2 F 2 , NaF, NOF and F2
1. Change in refractive index or medium. (C) XeOF4, NaNO2 and F2
2. Dissociation of analyte on dilution. (D) XeF4, NaNO2 and F2 O
3. Polychromatic light. 82. A borane (X) is reacted with ammonia to give
4. Path length of cuvette. a salt of borohydride (Y). The 11 B NMR
(A) 1, 2 and 3 (B) 2, 3 and 4 spectrum of Y consists of a triplet and a
quintet. The borane X is—
(C) 1, 3 and 4 (D) 1, 2 and 4
(A) B 2 H6 (B) B 3 H9
76. The number of stereoisomers of trans— (C) B 4 H8 (D) B 5 H9
[CoCl2(triethylenetetramine)]Br is—
83. Base hydrolysis of [CoCl(NH3 )5] 2 + is an
(A) One (B) Two
overall second order reaction, whereas that of
(C) Three (D) Four [Co(CN) 6 ]3 – is of first order. The rates depend
77. The gas commonly used in generating plasma in both cases solely on the concentrations of
in Inductively Coupled Plasma-Atomic Emis- the cobalt complex. This may be due to :
sion Spectroscopy (ICP—AES) is— 1. Presence of ionizable proton in
(A) argon (B) carbon dioxide [CoCl(NH3)5]2 + but not in [Co(CN)6 ]3 –
(C) nitrous oxide (D) hydrogen 2. S N1 CB mechanism in the case of
78. Under physiological condition, oxygen is bin- [CoCl(NH3)5]2 + only
ding to deoxyhemoglobin and deoxymyoglo- 3. S N1 CB mechanism in the case of
bin, the binding curve and its pH dependence, [Co(CN) 6 ]3 – only
respectively, are—
4. S N1 CB mechanism in both the complexes
(A) Sigmoidal and pH dependent; hyperbolic
and pH independent. Correct explanation(s) is/are—
(B) Hyperbolic and pH independent; sigmoi- (A) 1 and 2 (B) 1 and 3
dal and pH dependent. (C) 2 only (D) 1 and 4
10 | CSIR-UGC Chemical Sciences (D-14) (II)

84. Hindered β—diketonates like dpmH (dpmH = (B) PPh3 , [Ph3 PI]Me, Ph2 P(n Bu)3
dipivaloylmethane) are used for the separation
of lanthanides because complexes formed (C) PPh3 , [Ph3 PMe]I, Ph3 P == CH2
with dpmH can be separated by— (D) [PPh4]Cl, [Ph 3 P == CH2 ]I, Ph3P(n Bu)]Li
(A) Gel permeation chromatography
89. The reaction between diphenyldichlorosilane
(B) Gas chromatography
and water in 1 : 2 molar ratio gives product A
(C) Gel filtration chromatography which on heating above 100°C yields a cyclic
(D) Ion exchange chromatography or polymeric product B. The products A and
85. The final product in the reaction of B respectively, are—
[Cp* 2 ThH] with CO in an equimolar ratio
is— (A)
(A)

(B)
(B)
(C) and (Ph2 SiO)n (n = 3, 4, or ∞)

(C)
(D)
and (Ph2 SiO)n (n = 3, 4, or ∞)
86. An aqueous solution of [Mn(H2 O) 6 ]2+ comp-
lex is pale pink in color. The probable reasons
for it are— (D)
1. presence of 6A1g ground state.
2. disallowed transition by spin selection 90. The spin-only (μS) and spin plus orbital
rule. (μS + L) magnetic moments of [CrCl6 ]3 – are—
3. presence of 2T2g ground state. (A) 3·87 BM and 5·20 BM
4. charge transfer transition. (B) 2·84 BM and 5·20 BM
The correct answer is— (C) 3·87 BM and 6·34 BM
(A) 1 and 2 (B) 1 and 3 (D) 2·84 BM and 6·34 BM
(C) 2 and 3 (D) 3 and 4 91. The three dimensional structure of compound
87. According to Wade’s rule, anion C 2 B 9 H12 [Co(Co(NH3 )4 (OH) 2 )3]Br6 has—
adopts— (A) twelve Co—O and twelve Co—N bonds
(A) closo—structure (B) ten Co—O and ten Co—N bonds
(B) nido—structure (C) fourteen Co—O and ten Co—N bonds
(C) arachno—structure (D) twelve Co—O and ten Co—N bonds
(D) hypho—structure
92. 12-Crown-4 binds with the alkali metal ions
88. The reaction of phosphorus trichloride with in the following order :
phenyllithium in 1 : 3 molar ratio yields Li+ >> Na+ > K+ > Cs+. It is due to the—
product ‘A’, which on further treatment with
methyl iodide produces ‘B’. The reaction of B (A) right size of cation
with n BuLi gives product ‘C’. The products (B) change in entropy being positive
A, B and C, respectively, are— (C) conformational flexibility of crown ether
(A) [PPh4]Cl, [Ph 2 P == CH2 ]I, Ph2P(n Bu) (D) hydrophobicity of crown ether
CSIR-UGC Chemical Sciences (D-14) (II) | 11

93. Complexes HM(CO)5 and [(η5–C 5 H5) (C)


M′(CO)3 ]2 obey the 18-electron rule. Identify
M and M′ and their 1 H NMR chemical shifts
relative to TMS—
(A) M = Mn, – 7·5; M′ = Cr, 4·10
(B) M = Cr, – 4·10; M′ = Mn, – 7·5
(C) M = V, – 7·5; M′ = Cr, 4·10
(D) M = Mn, – 10·22; M′ = Fe, 2·80
94. Gel permeation chromatography can be used
to separate which of the following ?
1. Lanthanides
2. Alkaline earths
3. Fatty acids
4. Low molecular weight peptides
The correct answer is—
(A) 1 and 2 (B) 2 and 3 (D)
(C) 3 and 4 (D) 1 and 4
95. The correct schematic molecular energy
diagram for SF 6 molecule is—
(A)

96. The major product formed in the following


reaction is—

(B)
(A) (B)

(C) (D)

97. The product B in the following reaction sequ-


ence is—
Br2 (1 equiv) excess Me2 NH
⎯⎯⎯⎯⎯→ A ⎯⎯⎯⎯⎯→ B
hexane‚ rt.
12 | CSIR-UGC Chemical Sciences (D-14) (II)

(A) 99. The major product of the following reaction


is—

(B) 1. H3 O+‚ hν
⎯⎯⎯⎯⎯→
2. NaOH

(C)
(A) (B)

(D)

(C) (D)
98. The major product formed in the following
transformation is—

100. The major product of the following reaction


is—

(A) CF3CO2 H
⎯⎯⎯⎯→

(A)

(B)

(B)

(C)
(C)

(D) (D) OCOCF3

MeO
CSIR-UGC Chemical Sciences (D-14) (II) | 13

101. The products A and B in the following reac- 103. The products A and B in the following reac-
tion sequence are— tion sequence are—
O
LDA PdCl2 ‚ O2 1. NaOEt
⎯⎯→ A ⎯⎯⎯⎯⎯⎯→ B ⎯⎯⎯→
Br CuCl DMF.H2 O 2. H3O+

(A) 1. 2.
(A) 1. 2.

(B) 1. 2.
(B) 1. 2.

(C) 1. 2.
(C) 1. 2.

(D) 1. 2.
(D) 1. 2.
102. The major product for the following reaction
is—
104. The products A and B in the following reac-
tion sequence are—

(A) 1. 2.
(A)

(B)

(B) 1. 2.

(C)

(D) (C) 1. 2.
14 | CSIR-UGC Chemical Sciences (D-14) (II)

107. The correct combinations of the reactions and


the reagents are—
(D) 1. 2. Reactions
(A)

(B)

105. The major product of the following reac-


tion—
(C)
Reagents
(P) PPh3 and EtO 2 CN== NCO 2 Et
(Q) POCl3.EtyN
(R) H2SO4

(A) A—P, B—Q, C—R


(A) (B) (B) A—Q, B—R, C—P
(C) A—P, B—R, C—Q
(D) A—Q, B—P, C—R
108. The products A and B in the following
(C) (D) reaction sequence are—

106. The major product formed in the following


reaction sequence is—
(A) 1. 2.

(B) 1. 2.

(A)

(C) 1. 2.

(B)

(D) 1. 2.

(C)
109. The major product of the following reaction
is—

(D)
CSIR-UGC Chemical Sciences (D-14) (II) | 15

(A) (A) X = PhSO2H, BF3OEt2 and Y =


CH2 == CHCOOEt, BF3 OEt 2
(B) (B) X = 1. PhSH, PTSA; 2. m-CPBA and Y =
CH2 == CHCOOEt, BF 3 OEt 2
(C) X = PhSO3 H, BF3 OEt2 and Y = LDA,
(C) CH2 == CHCOOEt
(D) X = 1. PhSH, PTSA; 2. m-CPBA and Y
= LDA, CH 2 == CHCOOEt
113. The reactive intermediate and the product
(D) formed in the following reaction are—
(n-Bu3Sn)2
110. The major product of the following reaction ⎯⎯⎯⎯⎯→
is— BF3OEt2 ‚ hν
(A) Free radical and 4-iodomethyloxepan-2
one
(B) Free radical and 5-iodooxacan-2-one
(A) (B) (C) Carbene and 3-oxabicyclo [5.1.0] octane
2-one
(D) Carbene and (E)-5-iodopent-3-en-1-yl
acetate
(C) (D)
114. An organic compound having molecular for-
mula C10H12O2 exhibits the following spectral
data—
111. The following reaction gives a product
(racemic) which exhibits the following NMR IR : 3400 (br), 1600 cm– 1.
data— 1 H NMR : δ 1·85 (3H, d, J = 6Hz), 3·8 (3H, s),

1 H NMR : δ 2·67 (2H, s), 5·60 (2H, s) ppm; 5·0 (1H, s, D 2 O exchangeable), 6·0 (1H, dq, J
13C NMR : δ 170·3, 129·0, 105·0, 25·4 ppm. = 18, 6 Hz), 6·28 (1H, d, J = 18 Hz), 6·75
The structure of the product (racemic) is— (1H, d, J = 8 Hz), 6·8 (1H, s), 6·90 (1H, d, J =
8 Hz) ppm; 13C NMR : δ 146·5, 144·0, 131·0,
130·5, 123·0, 119·0, 114·0, 108·0, 55·0, 18·0
ppm.
The structure of the compound is—

(A) (B) (A) (B)

(C) (D) (C) (D)

112. In the following reaction sequence, the rea-


gents X and Y are, respectively— 115. The major product formed in the following
reaction sequence is—
16 | CSIR-UGC Chemical Sciences (D-14) (II)

(A) (B)
(A)

(C) (D)
(B)

(C)

116. The correct combination of the following reac- (D)


tions and their ρ values is—
Entry Reaction Entry ρ value
A ArNH2 + PhCOCl in P + 2·01 119. The major products A and B formed in the
benzene following reaction sequence are—
B ArO + EtI in EtOH Q – 0·99
C ArCO2Et + aq NaOH R – 2·69
in EtOH
S + 0·78
(A) A—P; B—R; C—P
(A) 1. 2.
(B) A—R; B—Q; C—P
(C) A—R; B—P; C—Q
(D) A—Q; B—R; C—S
117. In the following reaction sequence, the struc- (B) 1. 2.
ture of the product is—

(C) 1. 2.

(A) (B)
(D) 1. 2.

(C) (D) 120. The major product of the following reaction


is—

118. In the following reaction sequence, the struc-


tures of A and B are, respectively—
(A) (B)
CSIR-UGC Chemical Sciences (D-14) (II) | 17

(C) (D) (A) 0 (B) 1


L 2
(C) (D)
2 L
–∞
[Hint : ∫ + ∞ f (x) δ(x – a)dx = f (a)]
124. For a gaseous reaction, 2NO(g) + Cl2(g) →
121. Species A undergoes a unimolecular reaction Non-linear T.S. → 2NOCl, the pre-
as follows— exponential factor in the rate constant is
k1 proportional to—
A+A A* + A (A) T1/2 (B) T– 1/2
k– 1 – 5/2
(C) T (D) T– 7/2
k2 125. The probability of finding the harmonic
A* ⎯→ P oscillator in the energy level n = 1 is (neglect
For this reaction, the first order rate constant zero point energy and assume hν = kB T).
at high pressure is k∞. The first order rate (A) e (B) e2
k∞ (C) 1 – e – 2 (D) e– 2 (e – 1)
constant becomes when pressure of A is
2
[A]1/2. 126. At high pressure, the fugacity coefficient of a
real gas is greater than one, because—
(A) attractive term overweighs the repulsive
term
(B) repulsive term overweighs the attractive
term
(C) repulsive term is equal to the attractive
term
(D) the system is independent of both the
attractive and repulsive terms

The value of k1 will be— 127. If the bond length of a heteronuclear diatomic
molecule is greater in the upper vibrational
k∞ state, the gap between the successive
(A) (B) k∞[A]1/2 absorption lines of P-branch—
[A]1/2
(A) increases non-linearly
[A]1/2
(C) k∞ – [A]1/2 (D) (B) decreases non-linearly
k∞
(C) increases linearly
122. The low and high temperature limits of vibra- (D) decreases linearly
tional partition function are (θ = hν/k). 128. A quantum particle with fixed initial energy
T T E0 < V is allowed to strike the following four
(A) e– θ/T and e– θ/T (B) e– θ/2T and e– θ/2T
θ θ barriers separately. The transmission probabi-
lity is maximum in—
T θ
(C) e– θ/2T and e– θ/T (D) e– θ/2T and e– θ/2T
θ T (A) (B)

123. A particle in a 1-dimentional box of length L


is perturbed by a delta function potential,

( ) L
δ x – , in the middle of the box. The first
2
(C) (D)

order energy correction to the ground state


will be—
18 | CSIR-UGC Chemical Sciences (D-14) (II)

129. EPR spectrum of a free radical containing (A) D0(H 2 ) = D0 (H 2 +) + I(H) – I(H2 )
nuclei with nonzero nuclear spin is obtained if (B) D0(H 2 ) = D0 (H 2 +) – I(H) + I(H2 )
the following selection rules are observed—
(A) Δms = 0, Δm1 = 0 (C) D0(H 2 +) = D0 (H 2 ) + I(H) + I(H 2 )
(B) Δms = ± 1, Δm1 = 0 (D) D0(H 2 +) = D0 (H 2 ) – I(H) – I(H2)
(C) Δms = ± 1, Δm1 = ± 1 135. Fuel cells provide clean electrical energy to a
(D) Δms = 0, Δm1 = ± 1 variety of applications including automobiles
and stationary power sources. Normally
130. Given the following two relations, hydrogen combines with oxygen to give
x1dμ1 + x2dμ2 = 0 …(1) electrical energy and water. If we use butane
– – instead of hydrogen at 1·0 bar and 298 K, the
and x1dV1 + x2dV2 = 0, …(2) following reaction occurs—
for a binary liquid mixture at constant 13
temperature and pressure, the true statement C 4 H10(g) + O2 (g) → 4CO2(g) + 5H 2 O(l)
2
is that,
If the change in the Gibbs free energy of this
(A) both the relations are correct. reaction is—
(B) relation A is correct, but B is not.
2746·06 kJ mol – 1, involving 26 electrons, its
(C) relation B is correct, but A is not. open circuit voltage is—
(D) both the relations are incorrect except for
very dilute solution. (A) 1·55 V (B) 1·09 V
(C) 3·15 V (D) 2·06 V
131. The operators S are defined by—
±
S ± = Sx ± iS y, 136. A solid consisting of only X atoms has a
close-packed structure with X-X distance of
where Sx and Sy are components of the spin 160 pm. Assuming it to be a closed-packed
angular momentum operator. The commutator structure of hard spheres with radius equal to
[Sz, S +] is— half of the X-X bond length, the number of
(A) hS + (B) hS – atoms in 1 cm3 would be—
(C) – hS + (D) – hS – (A) 6·023 × 1027 (B) 3·45 × 1023
(C) 6·02 × 10 21 (D) 3·8 × 1021
132. The configuration [Ne] 2p1 3p1 has a 3D term.
Its levels are— 137. The character table of C2v point group is
3 3
(A) D3/2, D1/2 given below. In cis-butadiene molecule the
vibrational modes belonging to A2 irreducible
(B) 3 D5/2, 3 D3/2, 3 D1/2
representation are IR inactive. The remaining
(C) 3 D3, 3 D2, 3 D1 IR active modes are—
(D) 3 D3, 3 D2, 3 D1, 3 D0 C E C σ σ′ y
2v 2 v
133. The fraction of groups condensed at time t in A1 1 1 1 1 z, x2, y2, z
any stepwise condensation polymerization
(overall second order) reaction is— A2 1 1 –1 –1 R z, xy
1
(A) 1 + kt[A]0 (B) B1 1 –1 1 –1 x, Ry, xz
1 + kt[A]0
kt[A]0 1 + kt[A]0 B2 1 –1 –1 1 y, Rx, yz
(C) (D)
1 + kt[A]0 kt[A]0 (A) 7A1 + 5B1 + 8B2
134. If D0 (A) and I(A) refer respectively to the (B) 9A1 + 4B1 + 7B2
dissociation energy and ionization potential of
(C) 7A1 + 3B1 + 7B2
A (where A is either H, H2 , or H2 + species),
the correct relation among the following is— (D) 9A1 + 3B1 + 8B2
CSIR-UGC Chemical Sciences (D-14) (II) | 19

138. The product σxy.S z4 (S4z is the four fold impro- (C) 1sa α (1) 1sb α (1)
per axis of rotation around the z axis, and σxy 1sa α (2) 1sb α (2)
is the reflection in the xy plane) is—
(D) 1sa α (1) 1sb β (1)
(A) C z4 (B) C z4 .i
1sa α (1) 1sb β (2)
(C) C y4 (D) C z2
143. The number of microstates that are possible,
139. Among the following figures, when two particles are distributed in four
states such that the resulting wave functions
are antisymmetric with respect to exchange of
the particles, is—
(A) 16 (B) 12
(C) 8 (D) 6
the variations of mass adsorbed with pressure 144. When T → ∞, value of the single-particle
for a monolayer and a multilayer are repre- partition function will be (given : degeneracy
sented by— of level j = gj).
(A) A and C respectively (A) 1 (B) g0
(B) A and B respectively 1
(C) C and A respectively (C) ∑j gj (D)
∑j gj
(D) B and A respectively
145. The rate constant for a reaction
140. According to Huckel theory, the π electron A1 + + Bn + ⎯→ P
charge on the central carbon atom in propenyl
is measured in two different aqueous solu-
cation (CH2CHCH2 )+ is (in units of electronic
tions of ionic strengths 0·01 M and 0·04 M. If
charge). k
1 1 log 0·04 = 0·3, the charge n on B is closest
(A) (B) k0·01
2 ⎯2
√ to—
(C) 1 (D) 2 (A) 1 (B) 2
141. For some one-electron system with l = 0 and (C) 3 (D) 6
m = 0, the functions N0 e – σ and N1(2 – σ)e– σ/2
refer respectively to the ground (E0 ) and first Answers with Hints
excited (E1 ) energy levels. If a variational
–σ 1. (B) In 2001,
wave function– N2 (3 – σ)e yields and ave- (250 + 75) + (250 – 75)
rage energy E, it will satisfy— = 250
– – 2
(A) E ≥ 0 (B) 0 ≤ E ≤ E 0 75
and × 100 = 30%
– – 250
(C) E ≥ E 1 (D) E0 ≤ E ≤ E 1
(least per cent variability)
142. A Slater determinant corresponding to the Other years it is 33%, 37·5% and 50%
ionic part of the ground state valence bond 2. (B) When a rectangle is revolved around any
wave function of H2 molecule is (1sa α, 1sa β, axis, its create cylinder with height according
1sb α, 1sb β are atomic spin orbitals of hydro- to that axis.
gen atoms a and b of the hydrogen molecule) Thus, Volume V1 = lwh
(A) 1sa α (1) 1sa β (1) = length × width × height
1sa α (2) 1sa β (2) d d d3
= d× × =
2 2 4
(B) 1sa α (1) 1sb β (1) d d3
1sa α (2) 1sb β (2) and V2 = d × × d =
2 2
20 | CSIR-UGC Chemical Sciences (D-14) (II)

V1 d3 2 1 10. (B) Average


Thus, ratio = · =
V2 4 d3 2 70 + 72 + 74 + 76 + 78 + 80 + 82
+ 84 + 86 + 88 + 90 + 92 + 94
⇒ V1 : V2 : : 1 : 2 =
13
3. (B) Its very common that in 4 minutes the = 82 kg
R R
wound portion becomes = . Then in Two new persons average weight
⎯2
√ 1·41
100 + 79
R = = 89·5 kg
further app. 2 minutes, it will become = . 2
2 1066 + 179
Thus, total average = = 83 kg
2(
1·41
)
= 0·70 15
Thus, increase in average = 1 kg
4. (B) A number is divisible by 3 if the sum of 11. (C) 1872 codes can be generated using letters
its digits is divisible by 3. A to Z and digits 1 to 9.
A number is divisible by 7 if after doubling 12. (B)
last digit and subtracting it from remaining
leading truncated number is divisible by 7.
Thus, given combination is divisible by 3 and
7.
5. (A) B

Cooling is much faster in smaller sphere, so


130 attain room temperature first.
50
13. (D) From below, we find the gap of 1 letter in
English alphabet, so, I J K L M N O
A 14. (B)
120 C
10
1 15. (B) 1035 × = 103·5 ⇒ 1138·50
Area of Δ ABC = × base × height 100
2
1 15 + 879·75
= × 120 × 50 1035 × = 155·25 ⇒
2 100 2018·25
= 3000 m2 2018·25
Thus, % gain = × 100 ≈ < 1%
6. (C) According to given statements, it is clear 2070
that the value of m is 20 then he takes 14 non- 16. (A) 2 + 3 = 5 + 5 = 10 + 7 = 17 + 11 = 28 +
veg meals. 13 = 41 + 17 = 58 + 19 = 77 + 23 = 100
7. (D) Don’t use formula, just think, how ?
addition of prime numbers in increasing order.
Amount doubled in 1600 days (4·4 years)
17. (B)
Then in 1·1 years (400 days) interest of ` 1000
should be around 200. Thus right option is
190. A
8. (D) Series is increasing and decreasing and
option is 19. C arc of a circle
9. (C) Since locomotives are running towards
each other and collide and object has crushed.
Then only speed of object which matters.
Thus, distance = speed × time B
1 20
= 70 km/h × hour 18. (C) 25 × = 5 students get job within 1
2 100
= 35 km year of passing
CSIR-UGC Chemical Sciences (D-14) (II) | 21

20 26. (*) F TiCl6 3–


20 × = 4 students get job by end of 2nd
100
D4h : C4 + 4C2 + 4σv
year of passing remaining students = 16
So, correct option is 25, (we have to check en Cr en + σn
options)
19. (A) (1 + x)n = a0 + a1 x + a2 x2 + …… an xn F
Then, a0 + a1 + a2 + a3 + …… an = 2 n D2h : C2 + 2C2 + 2σv + σn
20. (D)
None of the given alternative are correct.
27. (D) 14N or 15N

arbitrarily
chosen shell
no. of protons same
n 3
S = =
2 2
L = +1×1+0×1+–1×1=0
Surface area of sphere = 4πr2
r is radius, 3
Thus, S = ‚L=0
2
mass depends upon surface area
Thus, the mass is proportional to its surface 28. (B) Hydroformylation (Oxo synthesis)
area. CO + H2
CH3 —CH== CH2 ⎯⎯⎯⎯→ CH3 CH2 CH2 CHO
21. (A) Cation exchange resin–exchanges cations CO2 (CO) 8 (normal) butanol
and depends on bigger size which is attached
till last time. Thus, the correct order of + (CH3)2CHCHO
retention of cations is (distribution constant, isobutyraldehyde
k) Ag+ > k+ > Na+ > Li + hν
29. (B) Ni(CO)4 + PPh3 ⎯→ [Ni(CO)3 PPh3 ]
22. (C) All have 10 e–s but ionic radius is greater
in case of highest – ve charge which gives + CO
increased radius (ionic). Thus, O2 – has highest 18 e– s complex.
and Mg+ 2 has lowest ionic radii. For substitution, it must follow dissociative
23. (A) In a polarographic measurement, (aq. KCl mechanism (SN1 ). Hence, it depends only on
sol. used as supporting electrolyte) an applied Ni(CO) 4 concentration.
potential more than + 4V, results the 30. (C) [(η x – C6 H6)Ru (ηy – C6 H6)]
formation of Hg+. – diamagnetic
DME worked at potential + 0·4V to – 0·2V Ru – 8 e– in valence shell, want 10 more e–
24. (D) SbF5 + 2HF ⎯→ [SbF 6 ]– [H2 F]+ for 18e – system, so hepticity will be 4 and 6.
x = 4, y = 6.
strongest acid in the system, is 1016 times
stronger than 100% H2 SO4 . 31. (A) Isomeric shift is given by—
δ = k(Re2 – R g 2 ) {[ψs2 (0)]b – [ψs2 (0)]a }
25. (A) Heme – porphyrin ring (11 π)
oxidised ferric ions (Fe ) e– density difference
+3
Coenzyme B12 — Corrin ring have lower isomeric on nucleus (a =
Chlorophyll — Chlorin ring (10 π) effect than Fe+ 2 source, b = sample)
Thus, π – e– extent of conjugation is because s – e – density at nucleus of Fe+3 is
greater due to a weaker screening effect by
Heme > Chlorophyll > Coenzyme B12 delectrons.
(a) > (c) > (b) So, δ : Fe(II) > Fe(III) > Fe(IV)
22 | CSIR-UGC Chemical Sciences (D-14) (II)

32. (C) dxy and dxy orbitals (Re2Cl8)2– 39. (C) R and R
Two nodal planes which
contain the internuclear axis
and go through both axis.
= delta bond formation 2d x y or 2dx2–y2
orbitals interacting.
33. (A) after reductive climination step D-ribose
Cl (I)
PPh3 40. (B)
Rh 16 VE
PPh3 S S = Solvent
⇒ neglecting solvent, [PPh3—Rh —PPh3 ] Oxyhemocyanin
|
Cl
34. (A) [P + Cl4]– 2 + NO2 – →

Cl NO2 Cl NO2
NH3
Pt Pt
Cl Cl NH3 Cl
Trans
Oxyhemerythrin
trans effect of NO2– > Cl
41. (B)
35. (B) CO4 (CO) 12
Total Valence electron = 4 × 9 + 12 × 2 = 60
Putting n = 4 in 14n + 4 Wade’s Rule ≡
14n + 2 ⇒ Closo
We have 14n + 4 = 60
⇒ Nido structure 14n + 6 ⇒ arachno
36. (C) OH : axial, i-Pr : equato-
rial, Me : equatorial

42. (C) Br MgBr


Mg

F F
arene intermediate

Diels-Alder reaction
O

O
37. (A)
43. (D) Biomethylation is a process whereby
living organisms produce a direct linkage of a
methyl group to a metal or metalloid, thus
(I) (II) (IV) (III) forming metal-carbon bonds found in soil
4nπ system, aromatic non-aromatic extensively.
Huckel rule
antiaromatic Thus, all statements are common and correct.
38. (C) Availability of lone pairs on N-atom deci- 44. (C) pka = – log ka, greater ka leads to lower
des basicity in these cases. Clearly, pyridine value of pka .
is least basic due to N- is part of aromatic ring
(sp2 system). Electron withdrawing group ⇒ greater ka ⇒
Greater e– density leads to greater basic lower pka.
character. Thus, Electron releasing group ⇒ lowers acidity
III > II > I > IV (ka) ⇒ greater pka .
CSIR-UGC Chemical Sciences (D-14) (II) | 23

Thus, Birch reduction e– withdrawing gp, reduction


at ipso and 4-position and alkynes into
2-cyano guanidine alkenes with trans-geometry
51. (A) S n(s) → S n + 2 (aq) + 2e–,
E1 ° = + 0·14V
II < I < III E2 ° = + 0·15V
45. (A) Crams Rule, smaller nucleophile, trans S n (aq) + 2e– → S n + 2 (aq)
+ 4
aldol product
S n(s) + Sn+ 4 (aq) → 2S n + 2 (aq)
– nFE3° = – nFE1° – nFE 2 °
4E3 ° = 2E1 ° + 2E2 °
0·29
46. (C) E3 ° = = 0·145
2
nFE 3 ° 4 × 0·145
Thus ln k = =
RT 25·7 × 10– 3V
Ketone Cyclohexane = 22·56 ≈ 22·6
carboxaldehyde 52. (B) We know that
P(OEt)3 h
Br—CH2—O—Me ⎯⎯⎯→ Δp Δx ≥
DME 2
α-haloether
h
wittig reagents ⇒ Δp =
2L
47. (C) C4H6O2
Energy of the particle
IR : 1770 cm– 1 implier γ-lactone
(Δp)2 ⎛ h ⎞2 1 h2
= =⎜ ⎟ =
hence, 2m ⎝ 2L⎠ 2m 8mL2
13C 53. (B)
NMR : 178, 68, 28, 22
48. (C) hν
⎯⎯→
vapour
phase
2,6-dimethyl cyclohexanone α-cleavage
free rotation metastable ion peak,
⎯⎯⎯⎯⎯⎯→ Mixture of cis & trans- m2
of C—C bond 1‚ 2-dimethyl cyclopentanes m* = 2
m1
49. (C) Br2 (excess)
⎯⎯⎯⎯→ (149) 2
= ~ 125·429
CHCl3‚ r+ 177 –
Benzothiophene
54. (D) The selection rule have two consequen-
ces—
(1) Both vibrational and rotational quantum
Bromination at 2,3-position numbers must change. The transition—
50. (A) ΔV = ± 1, ΔJ = 0 (Q-branch) is forbidden.
(2) The energy change of rotation can be
either subtracted from or added to the energy
change of vibration, giving the p- & R-bran-
ches of the spectrum respectively.
24 | CSIR-UGC Chemical Sciences (D-14) (II)

55. (D) CD3Cl; n = 3, I = 1


2nI + 1 = 7 non-pascal triangle
⇒ (∂M∂x ) = – y1
y
2

so, intensity ratio : : 1 : 3 : 6 : 7 : 6 : 3 : 1


56. (C) Since homonuclear diatomic molecules
Q (∂N∂y ) = (∂M∂x )
x y
are both rotational & IR inactive (absence of ∴ equation is exact.
dipole moment). So rotational Raman and ∂
electronic spectroscopy can be used for 64. (B) LZ = – ih
∂φ
determination of bond-lengths.
The eigen function equation is
57. (B) ∂
LZ Ylm = – ih Ylm
∂φ
Ylm = f(0).eiAφ
f is arbitrary function φ must be periodic for Y
is to be unchanged with period 2π.
Thus, value of A should be A = 0, ± 1, ± 2, …
58. (C) 65. (D) X-Ray Diffraction method is used for
crystal structure identification and study.
Amorphous solids don’t show XRD.
66. (D) The Lineweaver burk equation is given
by—
L-aspartic acid L-phenylalanine
1 1 k 1
= + m ·
2kT V Vmax Vmax [s] 0
59. (C) ν mp = M , thus, greater the mass, km
lesser will be the νmp. Given = 40s, [E] = 2·5 × 10– 6 M
Vmax
Thus, Ne – C, Ar – B, Kr – A. We know that at maximum velocity,
60. (D) We know that [ES] = [E]
dH = TdS + VdP
∴ Vmax = k2[E]
∂H
Thus, ( )∂S P
= T (dP = 0)
Thus, Efficiency = 2 =
k Vmax
61. (C) The difference between the activation km [E]x Vmax × 40s
energy for forward & backward reaction is = 104 Lmol– 1 s– 1
ΔH. Since given that E0 is activation energy 67. (B) CH3 COOH CH3 COO– + H+
for second reaction. The activation energy of
I 0·01 0 0
overall reaction will be E0 + ΔH.
C –x +x +x
62. (C) Fractional coverage (θ) for diatomic
molecule on metal surface, E + 0·01 – x +x +x
(KP)1/2 0·2
θ = [CH3COO– ] = 10 × = 0·01
1 + (KP)1/2 10 + 10
since P is small, 1 > (KP)1/2. pka = 4·75
θ = K1/2P 1/2 ⇒ ka = 1·8 × 10– 8
⇒ θ ∝ P 1/2 Thus, kb = 5·6 × 10– 10
1
63. (D) N = x2
Y =
0·01
∂N
⇒ ( ) ∂y x
= – 2
1
y

x = [OH] = 2·36 × 10– 6
x pOH = 5·62
M = – 2
y Hence, pH = 14 – pOH = 8·38 –~ 8·4.
CSIR-UGC Chemical Sciences (D-14) (II) | 25

68. (B) f00 polydispersed macromolecular colloid; Br2


— — 74. (A) Na[(η5-C5 H5) Fe(CO)2] ⎯⎯→
M w > M n osmometry gives number average 17e– system

molecular weight, M n . LiAlH4
[(η5 -C5 H5) Fe(CO)2 Br] ⎯⎯⎯⎯→ system
69. (A) Mn " = 13, 18e–
Mw " = 16, substitution of Br with H.
The variance, σ2 for molecular weight deter- [(η5 -C5 H5) Fe(CO)2H]
mination 1 H NMR appears for 5H of Cp and 1H in

⎛—
—2 ⎜ M w
⎞ ration 5 : 1.
σ = M n ⎜ — – 1⎟
2 ⎟ 75. (A) Path length of curvetle is not responsible
⎜⎝ M ⎟⎠ for deviation from ideal behaviour. All the
n
remaining statement, A, B and C will result in
= (13) 2 (1613 – 1) deviation form.
76. (C) Three isomers shown as—
3
= 169 × = 39
13
hc 6·626 × 10– 34 × 3 × 108 × 1019
70. (C) I.E. = =
λ 24 × 10– 9 × 1·6
= 52 eV
Ionization energy is the energy required to (a) (b)
remove an electron from outermost orbital of
gaseous system.
71. (A) All the given options are incorrect except
(1). There is ring current disruption after
formation of complex in [η6 – (C6 H6)
Cr(CO) 3 ]. The shift in δ is also attributed to (c) mirror plane
this reason from ~ 7·2 ppm to 4·4 (almost). 77. (A) Also known as inductively coupled
plasma optical emission spectroscopy (ICP-
OES) is an analytical technique used for the
detection of trace metals. Argon gas is
typically used to create the plasma.
78. (A) Myoglobin is pH independent
72. (A) Oxyhemocyanin—oxygen transport speci-
fically in invertebrates.
Carbonic anhydrase—In Red Blood Cells,
CO2 to HCO3 .
Cytochrome P450 —Oxydation of alkene,
monooxygenose, green catalyst.
Carboxypeptidase A—Hydrolysis of C- Lungs (gills) Hb + 4O2 → Hb(O2 )4
terminal peptide bond.
73. (B) Geometric cross-section = πr2 Tissues Hb(O2 )4 + 4 Mb → 4 Mb(O2 ) + Hb
and r = r0 (A)1/3 pH dependence shown by hemoglobin is
known as Bohr effect (binds one H+ for every
Thus, r = 1·4 × 10– 15 m × (125)1/3 dioxygen molecules).
= 0·7 × 10– 14 m 79. (C) Iridium complex is 16 e– system and
Hence, GC-S = 3·14 × (0·7 × 10– 14)2 m2 favourable for oxidative addition reaction in
– 1·54 × 10– 28 m2
~ presence of H2, commonly known as Vaska’s
(1 barn = 10– 28 m2) complex. Rest are 18 e– stable complex
~
– 1·54 barn compound.
26 | CSIR-UGC Chemical Sciences (D-14) (II)

80. (A) Mn + 3 : 87. (B) C2B 9 H12, According to Wade’s rule


= (CH) × 2 + (BH) × 9 + 1 + 1
25 + 1 = 5, L = 2(D), μ = ⎯ √⎯⎯⎯⎯⎯
n(n + 2) = 3 × 2 + 2 × 9 + 1 + 1 = 26 or 13 e– pair
no. of unpaired electrons = 4 = (m + 2) e– pair ⇒ Nido structure.
So, μS.O. = 4·9 BM.
6Na
81. (A) XeF6 + NaNO3 → XeOF4 + NaF + NO2 F 88. (C) PCl3 + 3PhCl ⎯⎯→ PPh 3 + 6NaCl
Nitrodes react with XeF6 to form XeOF4 . This (A) ↓ MeI
safe & convenient route involves reaction of BuLi
XeF6 with a stoichiometric deficiency of Ph 3 P == CH2 ⎯⎯⎯⎯⎯→ [Ph3 PMe]+ I–

NaNO3 . ylides (C) – BuH‚ – Li (B)
2H2O
82. (A) H H H 2NH3 89. (C) Ph2SiCl2 ⎯⎯⎯→ Ph 2 Si(OH)2
B B Unsymmetric – 2HCl
H H H Cleavage (A) Δ 100°C – H2 O
Ph
[(BH 2 ) (NH 3 )2] [BH4 ]
2H → triplet Polymeric |
4H → quinlet structure Si—O
| h
Other product is Borohydride salt [BH 2 (NH3 )2] (B)
Ph
[BH4] which gives triplet and quinlet in B. 11
90. (A) Cr+ 3 :
NMR spectrum due to 2H and <| H respecti-
vely.
– μS.O. = n(n + 2)
OH
83. (A) [CoCl(NH3)5] ⎯⎯⎯→
2 +
– H2 O = 3(3 + 2) = 3·87 B.M.
S N1 CB
[CoCl(NH2) (NH 3 )4]+ and μS + L = n(n + 2) + L(L + 1)
– Cl ↓ Slow = 15 + 12 = 27 = 5·20 BM
H2 O
[CO(OH) (NH 3 )5]2 + ←⎯⎯ 91. (A)
fast
[Co(NH2 ) (NH 3 )4]2 +
3 –
In [Co(CN)6] , no ionizable proton.
Thus, statements A & B are correct.
84. (B) By gas chromatography.
85. (D) O

Th—C—H
(η5 CP2 ) TH—H + CO → (η5 (CP2 ) Inorganic optically active complex—
CO insertion occurs in case of Thorium 12Co—O and 12 Co—N bonds
hydride A driving force for this reaction is the 92. (A) The size of the cavity is responsible for
strong interaction of oxygen of inserted CO binding with alkali metals. So, crown ether
with thorium atom. binds different metals. Thus, right size of
86. (A) Mn + 2 : cation is important factor.
2s + 1 = 6, L = 0 (s)
Spin selection rule, Δs = 0
6s ⎯→ 6A1g ground state
Violated and disallowed transition charge
12-crown-4
transfer impossible and 2 T2g is not ground
state. 93. (A) HMn(CO)5 and [(η5 -C5 H5) Cr(CO)3]2
Hence, A & B are correct. δ : – 7·5 4·10
CSIR-UGC Chemical Sciences (D-14) (II) | 27

94. (C) Type of size exclusion chromatography 99. (A)


(SEC), that separates analytes on the basis of
size. Analysis of polymers is done by this
technique. Thus, from given options, fatty
acids low molecular weight peptides can be
separated by this method.
95. (A) SF6 : 6 S-F bonds, 12 e– s involved
hypervalent
4-orbitals (1a1 , 1t1 ) : bonding
4-orbitals (2t1 , 2a1 ) : anti-bonding
2-orbitals (1e) : non-bonding
96. (C)

Hoffmann—Loffler—Fretyag reaction
⇒ Free radical mechanism

100. (B)
⎯→

Demethylation
⎯⎯⎯⎯⎯→
97. (C)
Br
Br Br Br 101. (A)
Br anti addition
Br
Me2NH
(SN2)
inversion
NMe2 Br
NMe2
NMe2 Me2NH NMe2

98. (A) Diastereoselective proplem-formation of


cis enolode thus cis-aldol product.

102. (A)

small ligand (Bu), good


leaving gp (OTf) and
hindered amine (i-Pr2 NEt)
facilitates the reaction.
28 | CSIR-UGC Chemical Sciences (D-14) (II)

103. (A)
2. TMSCl
⎯⎯⎯⎯⎯⎯⎯⎯→
LDA‚ THF – 78°C

CH CH3

C OTMS 3. Δ
O ⎯⎯⎯⎯⎯⎯⎯⎯⎯→
Claisen rearrangement
OMOM
O
104. (A) O 4. H3O+
⎯⎯⎯⎯⎯→
TMSO C 5. CH2 N2
O OMOM methylation
Me
MeOOC
O OMOM
Me
107. (D) A is dehydration reaction using POCl3,
Et3 N. B is esterification using …… PPh3 and
EtO2 CN== NCO2 Et. C is butylation using
H2SO4 .
108. (A)
CO 2(CO)8‚ CH2Cl 2
105. (A) ⎯⎯⎯⎯⎯⎯⎯⎯→
Me3N+—O–

Pauson-Khand Reaction (PKR) [2 + 2 + 1] CAR

THF EtOH
⎯⎯⎯⎯⎯⎯⎯⎯→
SmI 2 reducing agent
(A)

3. H2O
←⎯⎯⎯
Θ (B)
OH 109. (C)

4. H⊕
⎯⎯→

Variation of Stork enamine reaction.


106. (D)
1. EtCOCl
⎯⎯⎯⎯→
Et3 N
CSIR-UGC Chemical Sciences (D-14) (II) | 29

110. (A) ⇒ δ 6·28 & J = 18 Hz indicates trans-geometry


of double bond.
115. (A)

111. (C) δ 2·67 (2H, S) indicates13C


NMR shows 4 signals. Thus "== entity
structure may be (2) or (3).
112. (D)

116. (B) δ = reaction constant


O
·· || benzene
(A) ArN H2 + Cl—C —Ph ⎯⎯⎯⎯→
rds
moderate negativeδ value (– 2·69) R
O
C Ph
113. (B) Ar H2N
positive charge near ring
EtOH
(B) ArO – + EtI ⎯⎯⎯⎯→ ArOEt
rds
no electron change small δ value (– 0·99) Q.
O
|| EtOH
(C) Ar—C—OEt + aq. NaOH ⎯⎯⎯⎯→
114. (A) C 10H12O2 rds
O
||
Ar—C —OH
Moderate positive δ value (+ 2·01) P/e–s flow
into transition state.
30 | CSIR-UGC Chemical Sciences (D-14) (II)

117. (B) N
N
heat
O O ⎯⎯⎯→
– CO2
N N
Ts Ts
oxazolidone azomethine ylide
Me
cis-product H N
⎯⎯⎯⎯⎯⎯⎯⎯⎯⎯⎯⎯→
intramolecular dipolar CAR H
N
Ts
118. (A) 121. (A) We know that for unimolecular reaction
first order rate constant
k1k2 [A]
kuni =
k– 1 [A] + k2
When k– 1[A] = k2,
k
then kuni = ∞ and [A] = [A]1/2
2
k1[A]
kuni =
2
k00 k1[A]1/2
⇒ =
2 2
119. (C) k∞
⇒ k1 =
[A]1/2
122. (B) Vibrational partition function is given
by—
e– θ/2T
qvib = ,
1 – e– θ/T

where θ =
k
(1) At low T, θ/T >> 1, so e– θ/T is negligible
Thus, qvib = e– θ/2T
(2) At high T, θ/T << 1
120. (D) CH3 —NH Then, e– θ/T = 1 – θ/T + ……
|
e– θ/2T
CH2 COOH Hence, qvib =
⎯⎯⎯⎯⎯⎯⎯⎯→ 1 – (1 – θ/T)
T – θ/2T
⇒ qvib = e
θ
123. (D) First order energy correction is—
n
E(1)
n = ψ(0)
n | H | ψn "
(0)*

2
iminium ion
ψ(0)
n = L sin (nπxL )
CSIR-UGC Chemical Sciences (D-14) (II) | 31

L
∫ – L sin2 ( L ) 127. (A) Rotational-vibrational
2 nπx spectrum
Thus, E(1)
n = L ~ ~ ~
ΔE = 2mB + ν (1 – 2x ) e e
δ (x – ) dx
L In upper vibrational state
2 B < B 1 0
for ground state n = 1 h
L πx B = cm– 1
E(1)
n =
2
L ∫
–L
sin2 ()
L
8π2 (μr2) C
r = bond length
δ x– ( )
L
2
dx Thus, the gap between the successive
absorption lines of p-branch increases non-
∞ linearly.

Q – ∞ δ(x – a)dx = 1
128. (B) Transmission probability is inversely
2 proportional to barrier width and height.
Thus, E(1)
n = L 129. (B) Selection rule of EPR spectrum for a free
radical containing nuclei with non-zero
kBT qt.s. – ΔE0 nuclear spin
124. (D) Rate constant k(T) = e
n qNO qCl RT Δms = ± 1,
2
ΔmI = 0
⎛ 2mπ kBT⎞3/2
Since qtrans = ⎜ ⎟ 130. (A) Relation (A) is true and it is Gibbs-
⎝ h2 ⎠ Duhem equation.
3/2
T.T .T. 1 We know that
k(T) ∝ 3/2 3/2 ∝ 7/2 i
T .T.T.T .T.T. T
∑ Nidui = – SdT + Vdp
k(T) = T– 7/2 i=1
At constant T and P, relation (B) is also true.
125. (D) En = n + ( ) 1
2
hν 131. (A) [Sz, S +] = [Sz, S x] + i[Sz, S y]
= ihSy + i(– ihS x)
e– E/kB T
Probability, P (E) = = h(Sx + iSy) = hS +
∑ e– E/kB T
i 132. (C) [Ne]
e– 3/2 e– 1
= – 1/2 – 3/2 = 2P 3P
e +e (1 + e– 1)
Term = 3D
e– 1.e (e – 1) e – 1
= × = 2 n
e + 1 (e– 1) e – 1 S = = 1,
2
e2 >> 1 L = 2(D)
e–1 J = (L – S) to (L + S)
=
e2 = 1, 2, 3
⇒ – 2
p(E) = e (e – 1) Thus, 3 , 3
D3 D2 D1 , 3

126. (B) Fugacity coefficient is given by— 133. (C) Stepwise condensation polymerization,
P (Z – 1) overall rate law is given as
ln r = ∫0 P
dP d[A]
= – k[A]2
dt
PV
Q Z = [A]0
RT ∴ [A] =
1 + kt [A]0
At high pressure, z >> 1
Fraction of groups condensed at time t.
Thus r > 1
[A]0 – [A] k + [A]0
Thus, repulsive term overweighs the attractive f = =
term. [A] 0 1 + k + [A]0
32 | CSIR-UGC Chemical Sciences (D-14) (II)

134. (B) B represents BET curve for multilayer


adsorption C also represents multilayer
adsorption for benzene on Fe 2 O3 at 50°C.
140. (C) Wave function for central carbon
in propenyl cation
1 1
ψ2 = φ1 – φ3
⎯2
√ ⎯2

Thus, D0 (H 2 ) = D0 (H 2 +) – I(H) + I(H2 ) π-electron charge
r (internuclear distance). qr = ∑ ηi cir2
ΔG = 2746·06 kJ mol– 1
q2 = 2 × ⎛⎜
135. (B) 1⎞2 + 0 + 0
We know, ΔG = nFE
So, ⎟
⎝√
⎯ 2⎠
ΔG 2746·06 × 103 J mol– 1
So, E = = = 1
nF 26 × 96500
141. (C)
= 1·0944 V –~ 1·09V
⎪1S a α (1) 1S a β (1) ⎪⎪
136. (B) ccp means fcc structure 142. (A) ⎪
⎪⎪1S α (2) 1S β (2) ⎪⎪
a a
r = 160 pm
= 1Sa α(1) 1Sa β(2) – 1Sa β(1) 1Sa α(2)
Q a√
⎯ 2 = 4r
α and β are spin up and down
4r According to Pauli’s principle, 2 electrons in
a =
⎯2
√ ‘a’ atomic orbital with spin up and down.
no. of atoms in fcc = 4 143. (D) No. of microstates
So, no. of atoms in 1 cm3 (one atom) 4
=
=
4 4 √
=
( )
⎯2 2
2 4–2
a3 (4r) 3 4×3× 2
= =6
4 × (⎯√ 2)3 2 ×2×1
=
4 × 4 × 4 × (160)3 × (10– 10)3
144. (C) Partition function
= 3·45 × 1023
f = ∑ gj e– εi /kT
137. (D) 1 E C 2 σV σV 1 j

no. of 10 0 0 10 When T → ∞, e– εi /kT → 1


unshifted Thus, f = ∑ gj
atoms j
Character 3 – 1 1 1 order of 145. (C) We know that
per atom 30 0 0 10 group, h = 4 log k = log k0 + 1·018 | ZAZB| ⎯ √I
A1 = 10, A2 = 5, B1 = 5, B2 = 10
log k0·04
ltrans = B1 + B2 + A1 , lrot = B2 + B1 + A2 = 1·018 (1·x) [(0·04)1/2
log k0·01
Thus, IR active model = 9A1 + 3B1 + 8B2 – (0·01) 1/2]
138. (A) σ .S4 = C 4 (rotation axis)
xy z z
0·3 = 1·018 × x[0·2 – 0·1]
139. (B) A represents Langmuir adsorption 0·30
x = = 2·77 ≈ 3
isotherm for monolayer adsorption. 0·1080
Chemical Sciences
CSIR-UGC-NET/JRF Exam.
(June 2015)
Solved Paper
June 2015
Chemical Sciences
PART A 06. Find the height of a box of base area 24 cm
× 48 cm, in which the longest stick that can
01. Each of the following pairs of words hides a be kept is 56 cm long—
number, based on which you can arrange them (A) 8 cm (B) 32 cm
in ascending order. Pick the correct answer : (C) 37·5 cm (D) 16 cm
I. Cloth reel 07. The product of the perimeter of a triangle, the
J. Silent wonder radius of its in-circle, and a number gives the
K. Good tone area of the triangle. The number is—
L. Bronze rod (A) 1/4 (B) 1/3
(A) L, K, J, I (B) I, J, K, L (C) 1/2 (D) 1
(C) K, L, J, I (D) K, J, I, L 08. An infinite row of boxes is arranged. Each
02. Which of the following values is same as box has half the volume of the previous box.
2
22 2 ? If the largest box has a volume of 20 cc, what
(A) 26 (B) 28 is the total volume of all the boxes ?
(C) 216 (D) 2222 (A) Infinite (B) 400 cc
(C) 40 cc (D) 80 cc
03. A 12 m × 4 m rectangular roof is resting on
four 4 m tall thin poles. Sunlight falls on the 09. Find the missing element based on the given
roof at an angle of 45° from the east, creating pattern—
a shadow on the ground. What will be the area 1. 2. 3.
of the shadow ? 1. 2. 3. ?
(A) 24 m2 (B) 36 m2 (A) (B)
(C) 48 m 2 (D) 60 m2
(C) (D)
04. If 2a
10. By reading the accompanying graph, deter-
×b2
mine the INCORRECT statement out of the
c6 following—
84
liquid
8d6
Pressure

gas
Here a, b, c and d are digits. solid
Then a + b =
(A) 4 (B) 9
(C) 11 (D) 16 Temperature
05. The maximum number of points formed by (A) Melting point increases with pressure
intersection of all pairs of diagonals of convex (B) Melting point decreases with pressure
octagon is— (C) Boiling point increases with pressure
(A) 70 (B) 400 (D) Solid, liquid and gas can co-exist at the
(C) 120 (D) 190 same pressure and temperature
Chemical Sciences (CSIR) J-15 | 3

11. If you change only one observation from a set 16. An ant can lift another ant of its size whereas
of 10 observations, which of the following an elephant cannot lift another elephant of its
will definitely change ? size, because—
(A) Mean (B) Median (A) ant muscle fibres are stronger than
(C) Mode (D) Standard deviation elephant muscle fibres
(B) ant has proportionately thicker legs than
12. A man starts his journey at 0100 Hrs. local elephant
time to reach another country at 0900 Hrs (C) strength scales as the square of the size
local time on the same date. He starts a return while weight scales as cube of the size
journey on the same night at 2100 Hrs. local (D) ants work cooperatively, whereas
time to his original place, taking the same elephants work as individuals
time to travel back. If the time zone of his 17. Consider a series of letters placed in the
country of visit lags by 10 hours, the duration following way :
for which the man was away from his place U…G…C…C…S…I…R
is— Each letter moves one step to its right and the
(A) 48 hours (B) 20 hours extreme right letter takes the first position,
(C) 25 hours (D) 36 hours completing one operation. After which of the
following numbers of operations do the Cs
13. Let r be a positive number satisfying not sit side by side ?
r (1/1234) + r (–1/1234) = 2 (A) 3 (B) 10
Then r 4321 + r – 4321 = ? (C) 19 (D) 25
(A) 2 (B) 2 (4321/1234) 18. An inclined plane rests against a horizontal
(C) 2 3087 (D) 2 1234 cylinder of radius R. If the plane makes an
angle of 30° with the ground, the point of
14. A float is drifting in a river, 10 m downstream contact of the plane with the cylinder is at a
of a boat that can be rowed at a speed of 10 height of—
m/minute in still water. If the boat is rowed (A) 1·500 R (B) 1·866 R
downstream, the time taken to catch up with (C) 1·414 R (D) 1·000 R
the float—
19. What is the maximum number of parallel,
(A) will be 1 minute non-overlapping cricket pitches (length 24 m,
(B) will be more than 1 min width 3 m) that can be laid in a field of
(C) will be less than 1 min diameter 140 m, if the boundary is required to
be at least 60 m from the centre of any pitch ?
(D) can be determined only if the speed of
the river is known (A) 6 (B) 7
(C) 12 (D) 4
15. ABC is a right angled triangle inscribed in a
semicircle. Smaller semicircles are drawn on 20. In a fast moving car with open windows, the
sides BC and AC. If the area of the triangle is driver feels a continuous incoming breeze.
a, what is the total area of the shaded lumes ? The pressure inside the car, however, does not
keep increasing because—
B (A) air coming in from the front window
goes out from the rear
(B) air comes in as well as goes out through
every window but the driver only feels
A the incoming one
C
(C) no air actually comes in and the feeling
of breeze is an illusion
(A) a (B) πa (D) cool air reduces the temperature therefore
(C) a/π (D) a/2π the pressure does not increase
4 | Chemical Sciences (CSIR) J-15

PART B 26. The species having he strongest gas phase


proton affinity among the following—
21. The biological functions of carbonic anhy- (A) N3– (B) NF3
drase and carboxypeptidase A, respectively, (C) NH3 (D) N(CH3)3
are— 27. Consider the following statements regarding
(A) interconversion of CO 2 and carbonates the diffusion current at dropping mercury
and hydrolysis of peptide bond electrode.
(B) gene regulation and interconversion of 1. It does not depend on mercury flow rate.
CO2 and carbonates 2. It depends on drop time.
(C) gene regulation and hydrolysis of peptide 3. It depends on temperature.
bond Correct statement(s) is/are—
(D) interconversion of CO 2 and carbonates (A) 1 only (B) 2 only
and gene regulation (C) 1 and 2 (D) 2 and 3
22. Fe–Nporphyrin bond distances in the deoxy- and 28. Q value for the reaction 13N(n, p)13C is 3·236
oxy-haemoglobin, respectively are— MeV. The threshold energy (in MeV) for the
(A) ~ 2·1 and 2·0 Å (B) ~2·0 and 2·0 Å reaction 13C(p, n)13N is—
(C) ~2·2 and 2·3 Å (D) ~2·3 and 2·5 Å (A) – 3·236 (B) – 3·485
(C) 3·485 (D) 3·845
23. The binding modes of NO in 18
electron compounds [Co(CO)3 (NO)] and 29. The 119 Sn NMR chemical shift (approximately
in ppm) corresponding to (η5 -Cp)2Sn (relative
[Ni(η5 – Cp)(NO)], respectively are—
to Me 4 Sn) is—
(A) linear and bent
(A) – 4 (B) + 137
(B) bent and linear
(C) + 346 (D) – 2200
(C) linear and linear
30. All forms of phosphorus upon melting, exist
(D) bent and bent
as—
24. The role of copper salt as co-catalyst in P
Wacker process is— (A)
(A) oxidation of Pd(0) by Cu(II) n P P
(B) oxidation of Pd(0) by Cu(I) P
(C) oxidation of Pd(II) by Cu(I) P P
(B)
(D) oxidation of Pd(II) by Cu(II)
P P P P
25. For typical Fischer and Schrock carbenes, n
P P
consider the following statements—
1. Oxidation state of metal is low in Fischer (C) n (P ≡ P)
carbene and high in Schrock carbene. P P P P
(D) P P P
2. Auxiliary ligands are π -acceptor in P P P P P P P
Fischer carbene and non-π-acceptor in P P P P
P P
Schrock carbene. P P P
P P
3. Substituents on carbene carbon are non-
π-donor in Fischer carbene and π-donor 31. For the oxidation state(s) of sulphur atoms in
in Schrock carbene. S 2 O, consider the following—
4. Carbene carbon is electrophilic in 1. – 2 and + 4
Fischer carbene and nucleophilic in 2. 0 and + 2
Schrock carbene. 3. + 4 and 0
The correct statements are— The correct answer(s) is/are—
(A) 1, 2 and 3 (B) 1, 2 and 4 (A) 1 and 2 (B) 1 and 3
(C) 2, 3 and 4 (D) 1, 3 and 4 (C) 2 and 3 (D) 3 only
Chemical Sciences (CSIR) J-15 | 5

32. The correct set of pseudohalide anions is— (B) Equilibrium will shift towards left in
(A) CN– , ClO 4 – , BF4– , PF6 – case of both A and B
(B) N3– , NO3– , HSO4 – , AsF 6 – (C) Equilibrium will shift towards right in A
and left in case of B
(C) SCN–, PO4 3–, H2PO4 – , N3–
(D) Equilibrium will shift towards right in B
(D) CN– , N3– , SCN–, NCN2–
and left in case of A
33. In transition metal phoshine (M—PR 3 ) com-
38. The compound that exhibits sharp bands at
plexes, the back-bonding involves donation 3300 and 2150 cm– 1 in the IR spectrum is—
of electrons from—
(A) 1-butyne (B) 2-butyne
(A) M(t2g) → PR3(σ*)
(C) butyronitrile (D) butylamine
(B) M(t2g) → PR3(π*)
(C) M(e g ) → P(d) 39. The 1H NMR spectrum of a dilute solution of
(D) PR3(π) → M (t2g) a mixture of acetone and dichloromethane in
CDCl3 exhibits two singlets of 1 : 1 intensity.
34. The refluxing of RhCl3 .3H2 O with an excess Molar ratio of acetone to dichloromethane in
of PPh3 in ethanol gives a complex A. the solution is—
Complex A and the valence electron count on (A) 3 : 1 (B) 1 : 3
rhodium are, respectively—
(C) 1 : 1 (D) 1 : 2
(A) [RhCl(PPh3 )3], 16
(B) [RhCl(PPh3 )5], 16 40. Intense band generally observed for a carbonyl
group in the IR specturm is due to—
(C) [RhCl(PPh3 )3], 18
(A) The force constant of CO bond is large
(D) [RhCl(PPh3 )5], 18
(B) The force constant of CO bond is small
35. The β-hydrogen elimination will be facile
in— (C) There is no change in dipole moment for
CO bond stretching
(A) M (B) M
H H
(D) The dipole moment change due to CO
bond stretching is large
(C) M (D) M H
41. The compound that gives precipitate on
H warming with aqueous AgNO3 is—
36. The reaction [Co(CN)5H2O] 2– + X– → Br Br
[Co(CN) 5 X] 2– + H2 O follows a/an— (A) (B)
(A) Interchange dissociative (Id ) mechanism
(B) Dissociative (D) mechanism
(C) Associative (A) mechanism Br
(D) Interchange Associative (Ia) mechanism Br
(C) (D)
37. Correct statement on the effect of addition of
aq. HCl on the equilibrium is— N
OH 42. Following reaction goes through—
O
HO … Eq. A
O COOAg Br
O Br2

O−
O+ CN− … Eq. B (A) Free radical intermediate
CN (B) carbanion intermediate
(A) Equilibrium will shift towards right in (C) carbocation intermediate
case of both A and B (D) carbene intermediate
6 | Chemical Sciences (CSIR) J-15

43. The most stable conformation for the Me Me Me Me Me


following compound is— Me
Me Me
Me Me
I II III IV
Me (A) I > II > III > IV (B) I > III > II > IV
(C) IV > I > III > II (D) IV > II > I > III
Me Me 47. Among the following, the correct statement (s)
about ribose is (are)—
Me Me
1. On reduction with NaBH 4 it gives
(A) H (B) Me optically inactive product.
Me H 2. On reaction with methanolic HCl it gives
a furanoside.
Me Me
Me 3. On reaction with Br2 –CaCO3 –water it
Me gives optically inactive product.
(C) H
(D) 4. It gives positive Tollen’s test.
Me
Me H (A) 1, 2 and 4 (B) 1, 2 and 3
(C) 2 and 3 (D) 4 only
44. The major product formed in the following 48. Biogenetic precursors for the natural product
reaction is— umbelliferone among the following are—
O
Me NaBH4, CeCl3
CHO MeOH, H2O HO O O
umbelliferone
OH 1. L-tryptophan 2. cinnamic acid
(A)
OH
Me 3. L-methionine 4. L-phenylalanine
(A) 1 and 2 (B) 2 and 4
Me (C) 2 and 3 (D) 3 and 4
(B) 49. Number of signals in the 13C{H} NMR
OH spectrum of (R)–4-methylpentan-2-ol are—
O (A) 3 (B) 4
OH (C) 5 (D) 6
(C) OH
Me 50. The major product formed in the following
reaction is—
OH Me Me
(D)
EtO2C
Me NaBH4
CHO 0°C
H MeOH/THF
45. The correct relation between the following O
compounds is—
(A) Me Me (B) Me Me
H H H Cl
Cl Me Me H HO HO
OH H OH
HO H H
(A) enantiomers OH H
(B) diastereomers (C) Me Me (D) Me Me
(C) homomers (identical) EtO2C EtO2C
(D) constitutional isomers
H OH
46. The correct order of heat of hydrogenation for H H
the following compounds is— OH H
Chemical Sciences (CSIR) J-15 | 7

51. The major product formed in the following 55. A particle is in a one-dimensional box with a
reaction is— potential V 0 inside the box and infinite
heat outside. An energy state corresponding to
Me n = 0 (n : quantum number) is not allowed
H because—
Me
(A) the total energy becomes zero
(A) (B) (B) the average momentum becomes zero
Me (C) the wave function becomes zero every
H
where
H H
(D) the potential V0 ≠ 0
(C) (D) 56. An eigenstate of energy satisfies HΨn = En Ψn .
H Me H Me In the presence of an extra constant potential
V0 —
52. The major product formed in the following
reaction is— (A) both En and Ψn will change
Me (B) both En and the average kinetic energy
O will change
O
H2N-NH2.HCl (C) only En will change, but not Ψn
Et3N, CH3CN, rt
(D) only Ψn will change, but not E n
57. The intensity of a light beam decreases by
Me
50% when it passes through a sample of 1·0
Me Me cm path length. The percentage of trans-
O O mission of the light passing through the same
(A) (B) sample, but of 3·0 cm path length, would be—
(A) 50·0 (B) 25·0
(C) 16·67 (D) 12·5
Me Me Me
58. The electric-dipole allowed transition among
Me Me the following is—
HO HO
(A) 3 S → 3D (B) 3 S → 3P
(C) (D) (C) S → D
3 1 (D) 3 S → 1F
x x
59. The product C 2 σxy (C 2 is the two-fold rotation
Me Me axis around the x-axis and σxy is the xy mirror
53. The magnitude of the stability constants for plane) is—
K+ ion complexes of the following supra- (A) σx z (B) σy z
molecular hosts follows the order— y z
H H
(C) C 2 (D) C 2
N N S
O O O O O O 60. The simplest ground-state VB wave function
of a diatomic molecule like HCl is written as
O O O O O O
N O S Ψ = ΨH (1s, 1) ΨCl (3pz, 2) + B where B
H
stands for—
A B C (A) ΨH (3pz, 2) ΨCl (1s, 1)
(A) B > A > C (B) C > A > B (B) ΨH (1s, 2) ΨCl (3pz, 1)
(C) A > B > C (D) C > B > A
(C) ΨCl (1s, 2) ΨCl (3pz, 1)
54. Antitubercular drug(s) among the following is (D) ΨCl (1s, 2) ΨH (3pz, 1)
(are)—
1. Salbutamol 2. Ethambutanol 61. Heat capacity of a species is independent of
3. Isoniazid 4. Diazepam temperature if it is—
(A) 1 and 2 (B) 2 and 3 (A) tetratomic (B) triatomic
(C) 3 and 4 (D) 4 alone (C) diatomic (D) monatomic
8 | Chemical Sciences (CSIR) J-15

62. In a chemical reaction : PCl5 (g) PCl3 (g) + aZn(s)


(C) ΔG° – RT ln
Cl2 (g ), xenon gas is added at constant aCu2+
volume. The equilibrium— a 2+
(A) will shift towards the reactant (D) ΔG° + RT ln Zn
aCu2+
(B) will shift towards the products
68. The lowest energy-state of an atom with
(C) will not change the amount of reactant
electronic configuration ns1 np1 has the term
and products
symbol—
(D) will increase both reactant and products
(A) 3 p1 (B) 1 p1
63. The temperature-dependence of a reaction is
give by (C) 3 p2 (D) 3 p0
k = AT2 exp (– E0/RT). 69. Energy of interaction of colloidal particles as
The activation energy (Ea ) of the reaction is a function of distance of separation can be
give by— identified as (1) van der Waals, (2) double
1 layer, (3) van der Waals and double layer.
(A) E0 + RT (B) E0 The correct order of interactions in the figure
2
corresponding to curves (a), (b) and (c)
(C) E0 + 2RT (D) 2E0 + RT
respectively, is—
64. For a reaction, 2A + B → 3Z, if the rate of
consumption of A is 2 × 10– 4 mol dm– 3 s– 1, E (b)
(a)
the rate of formation of Z (in mol dm – 3 s– 1)
will be—
r
(A) 3 × 10– 4 (B) 2 × 10– 4
4 (c)
(C) × 10– 4 (D) 4 × 10– 4
3
65. Dominant contribution to the escaping (A) 1, 2, 3 (B) 2, 3, 1
tendency of a charged particle with uniform (C) 3, 1, 2 (D) 1, 3, 2
concentration in a phase, depends on— 70. The packing factor (PF) and number of
(A) chemical potential of that phase atomic sites per unit cell (N) of an FCC
(B) electric potential of the phase crystal system are—
(C) thermal energy of that phase (A) PF = 0·52 and N = 3
(D) gravitational potential of that phase (B) PF = 0·74 and N = 3
66. The intrinsic viscosity depends on the molar (C) PF = 0·52 and N = 4
mass as [η] = KMa . (D) PF = 0·74 and N = 4
The empirical constants K and a are dependent
on—
PART C
(A) solvent only
(B) polymer only 71. Differential pulse polarography (DPP) is
(C) polymer-solvent pair more sensitive than D.C. Polarography (DCP).
(D) polymer-polymer interaction Consider following reasons for it—
1. Non-faradic current is less in DPP in
67. The correct ΔG for the cell reaction involving comparison to DCP.
steps
2. Non-faradic current is more in DPP in
Zn(s) → Zn 2+ (aq) + 2e– comparison to DCP.
Cu 2+ (aq) + 2e– → Cu(s) is— 3. Polarogram of DPP is of different shape
a 2+ than that of DCP.
(A) ΔG° – RT ln Zn
aCu2+ Correct reasons(s) is/are—
a 2+ (A) 1 and 3 (B) 2 and 3
(B) ΔG° + RT ln Zn
aCu(s) (C) 2 only (D) 1 only
Chemical Sciences (CSIR) J-15 | 9

72. Considering the following parameters with 78. Considering the intert pair effect on lead, the
reference to the fluorescence of a solution : most probable structure of PbR2 [R = 2, 6–
1. Molar absorptivity of fluorescent mole- C 6 H3 (2, 6—Pr2 C 6 H3)2] is—
cule. RR
2. Intensity of light source used for R
excitation. R
(A) Pb Pb (B) Pb Pb R
R
3. Dissolved oxygen
R
The correct answer for the enhancement of R
fluorescence with the increase in these
R R R
parameters is/are— Pb Pb R
(C) R (D) Pb Pb R
(A) 1 and 2 (B) 2 and 3 R R
(C) 1 and 3 (D) 3 only
79. The reaction of SbCl3 with 3 equivalents of
73. The geometric cross section of 125 Sn (in barn) EtMgBr yields compound X. Two equivalents
is nearly— of SbI3 react with one equivalent of X to give
(A) 1·33 (B) 1·53 Y. In the solid state, Y has a 1D-polymeric
(C) 1·73 (D) 1·93 structure in which each Sb is in a square
pyramidal environment. Compounds X and Y
74. Match column A (coupling reactions) with respectively, are—
column B (reagents)— (A) SbEt3 and [Sb(Et)I2]n
Column A Column B (B) Sb(Et2 )Cl and [Sb(Et2)Cl]n
a. Suzuki coupling i. CH 2 =CHCO2 CH 3 (C) SbEt3 and [SbEt2Br2 ]n
b. Heck coupling ii. RB(OH)2 (D) Sb(Et)Br2 and [SbEt(I) (Br)] n
c. Sonogashira iii. PhCO(CH2)3 ZnI
coupling
80. Match the complexes given in column I with
the electronic transitions (mainly responsible
d. Negeshi coupling iv. HC≡CR
for their colours) listed in column II
v. SnR 4 I II
The correct match is— a. Fe(II)-protopor- 1. π → π*
(a) (b) (c) (d) phyrin IX
(A) ii i iv iii b. [Mn(H2 O)6]Cl2 2. spin allowed d → d
(B) i v iii iv c. [Co(H2 O)6]Cl2 3. spin forbidden d → d
(C) iv iii ii i 4. M → L charge transfer
(D) ii iii iv v The correct answer is—
75. The oxoacid of phosphorus having P atoms in (a) (b) (c)
+ 4, + 3 and + 4 oxidation states respectively, (A) 1 3 2
is— (B) 4 2 3
(A) H5P 3 O10 (B) H5P 3 O7 (C) 1 3 4
(C) H5P 3 O8 (D) H5P 3 O9 (D) 1 2 3
81. The following statements are given regarding
76. The geometries of [Br 3 ]+ and [I5 ]+ respec-
the agostic interaction C—H…Ir observed in
tively, are—
[Ir(Ph3 P)3 Cl].
(A) trigonal and tetrahedral
1. Upfield shift of C—H proton in 1H NMR
(B) tetrahedral and trigonal bipyramidal
spectrum.
(C) tetrahedral and tetrahedral
2. Increased acid character of C—H.
(D) linear and trigonal pyramidal
3. v C – H in IR spectrum shifts to higher
77. According to Wade’s theory the anion wavenumber.
[B12H12]2– adopts— The correct answer is/are—
(A) closo - structure (B) arachno - structure (A) 1 and 3 (B) 2 and 3
(C) hypo - structure (D) nido - structure (C) 1 and 2 (D) 3 only
10 | Chemical Sciences (CSIR) J-15

82. Amongst the following : 87. Match the action of H2 O2 in aqueous medium
1. [Mn(η5 – Cp) (CO)3 ], given in column A with the oxidation/ reduc-
tion listed in column B :
2. [Os(η5 –Cp)2 ],
A : action of H2O2 B : type of reaction
3. [Ru(η5–Cp)2 ] and
a. Oxidation in acid 1. [Fe(CN)6]3– →
4. [Fe(η5 –Cp)2 ],
[Fe(CN)6]4 –
the compounds with most shielded and
deshielded Cp protons respectively, are—
b. Oxidation in base 2. [Fe(CN)6]4 – →
[Fe(CN)6]3–
(A) 4 and 1 (B) 4 and 2
(C) 3 and 1 (D) 3 and 2
c. Reduction in acid 3. MnO4 – → Mn2+
d. Reduction in base 4. Mn 2+ → Mn4+
83. Total number of vertices in metal clusters
[Ru6 (C)(CO)1 7 ], [Os 5 (C)(CO)1 5 ] and The correct answer is—
[Ru5 (C)(CO)16 ] are 6, 5 and 5 respectively. (a) (b) (c) (d)
The predicted structures of these complexes, (A) 1 2 3 4
respectively, are— (B) 2 4 3 1
(A) closo, nido and nido (C) 3 4 2 1
(B) closo, nido and arachno (D) 4 1 3 2
(C) arachno, closo and nido 88. The reduced form of a metal ion M in a
(D) arachno, nido and closo complex is NMR active. On oxidation, the
complex gives an EPR signal with g || ≈ 2·2
84. Among the complexes,
and g⊥ ≈ 2·0. M··ossbauer spectroscopy cannot
1. K4[Cr(CN)6 ]
characterise the metal complex. The M is—
2. K4[Fe(CN)6 ] (A) Zn (B) Sn
3. K3[Co(CN) 6 ] (C) Cu (D) Fe
4. K4[Mn(CN)6 ] 89. The least probable product from A on
Jahn-Teller distortion is expected in— reductive elimination is—
(A) 1, 2 and 3 (B) 2, 3 and 4 Ph Ph
(C) 1 and 4 (D) 2 and 3 P CH3
M
85. The reductive elimination of Ar—R (coupled P CH3
product) from A is facile when— Ph Ph
Ph Ph A
P Ar CH3
Pd (A) (B) CH4
A R H 3C
P
Ph Ph CH3
CH3
(C) (D)
(A) R = CH3 (B) R = CH2Ph H 3C
H 3C CH3
(C) R = CH2COPh (D) R = CH2CF3
90. Water plays different roles in the following
86. The total number of metal ions and the reactions—
number of coordinated imidazole units of
1. 2H2O + Ca → Ca 2+ + 2OH– + H2
histidine in the active site of oxy-hemocyanin,
respectively, are— 2. nH2O + Cl → [Cl(H2O) n ]–
(A) 2Cu 2+ and 6 (B) 2Fe2+ and 5 3. 6H2O + Mg2+ → [Mg(H2 O) 6 ]2+
(C) 2C u+ and 6 (D) Fe2+ and 3 4. 2H2O + 2F2 → 4HF + O2
Chemical Sciences (CSIR) J-15 | 11

The correct role of water in each reaction is— 1. CO2 combines with Ni(PR3 )2 (1, 5-cyclo-
(A) (1) oxidant, (2) acid, (3) base and octadiene)
(4) reductant 2. Insertion of CO2 occurs
(B) (1) oxidant, (2) base, (3) acid and
(4) reductant 3. Insertion of Et Et takes place
(C) (1) acid, (2) oxidant, (3) reductant and The correct answer is—
(4) base
(A) 1 and 2 (B) 2 and 3
(D) (1) base, (2) reductant, (3) oxidant and
(4) base (C) 3 and 1 (D) 1, 2 and 3
91. With respect to σ and π bonding in Pt—||| in 95. Consider the following statements for
the structure given below, which of the (NH4 )2 [Ce(NO3 )6] (Z)
following represent the correct bonding ?
1. Coordination number of Ce is 12
Ph
2. Z is paramagnetic
Ph3P C 3. Z is an oxidising agent
Pt 1.32 A
Ph3P 4. Reaction of Ph3 PO with Z gives a
C
complex having coordination number 10
Ph for Ce.
(A) M(σ) → L(σ) and M(π) → L(π * ) The correct statements are—
(B) L(σ) → M(π) and L(π) → M(π) (A) 1, 2 and 3 (B) 2, 1 and 4
(C) L(π) → M(π) and L(σ) → M(π) (C) 2, 3 and 4 (D) 1, 3 and 4
(D) L(π) → M(σ) and M(π) → L(π* )
96. The major product formed in the following
92. The complex [Fe(phen) 2 (NCS)2 ] (phen = 1, reaction sequence is—
10-phenanthroline) shows spin cross-over
behaviour. CFSE and μeff at 250 and 150 K HO2C 1. (i) SOCl2,
(ii) NaN3, MeOH
respectively, are— O 2. t-BuOK
(A) 0·4 Δ •, 4·90 BM and 2·4 Δ •, 0·00 BM
3. H3O+
(B) 2·4 Δ •, 2·90 BM and 0·4 Δ •, 1·77 BM
(C) 2·4 Δ •, 0·00 BM and 0·4 Δ •, 4·90 BM O
(D) 1·2 Δ •, 4·90 BM and 2·4 Δ •, 0·00 BM (A)
93. Consider the following statements with respect NH
HO2C
to uranium H
1. UO22+ disproportionates more easily than
UO22+ O
2. U3O8 is its most stable oxide of U (B)
NH
3. Coordination number of U in
[UO2 (NO3)2 (H2 O) 2 ].4H2 O is six H
HO2C
4. UO22+ is linear
The correct set of statements is— HO2C
(A) 1, 2 and 4 (B) 1, 3 and 4 H
(C) N
(C) 2, 3 and 4 (D) 1, 2 and 3 O
94.
Et H
Et Et
(R3P)2Ni(1,5-cyclooctadiene) H
2 Et Et + CO2 (D) HO2C N
Et O O O
For the above conversion, which of the
following statements are correct ? H
12 | Chemical Sciences (CSIR) J-15

97. The major products A and B in the following CO2H CO2H


reaction sequence are—
R (B) A = B=
N CO2Et N
R = OH R = Me H H
A B
NaNH2 NaNH2
NH3 (I) NH3 (I) CO2Et CO2H
Br
(C) A = B=
OH Me Me N CO2H N
H H
(A) A = B= +
NH2 NH2 CO2Et
NH2 (1:1)
OH Me (D) A = B=
N CO2H N
H H
(B) A = B=
NH2 100. The major products formed in the following
NH2 reaction are—
OH OH Me Me 0.5 equiv. PhC(Me)2OOH
1.0 equiv. Ti(OiPr)4
(C) A = + B= + 1.2 equiv. (−)-DIPT
NH2 NH2 OH OMe CH2Cl2, −20°C
NH2 (1:1) NH2 (1:1)
O
OH Me
(D) (A) A = B=
OH OMe OH OMe
A= B=
NH2 NH2 O
98. The major product formed in the following A= B=
(B)
reaction is— OH OMe OH OMe
AcO p-TsNH-NH2
AcOH O
O NaBH3CN A= B=
(C)
OH OMe OH OMe
AcO O
(A) (B) (D) A= B=
OH OMe OH OMe

AcO AcO 101. The correct statement about the following


reaction is—
(C) (D)
O

NH2 Br2
99. The major products A and B in the following NaOH
reaction sequence are— N F
O O
H2N + CO2Et aq KOH aq KOH (A) The product is 2-fluoropyridin-3-amine
A B
EtO2C rt reflux and reaction involves nitrene inter-
CO2H CO2H mediate
(B) The product is 2-fluoropyridin-3-amine
(A) A = B=
CO2Et and reaction involves radical inter-
N N CO2H
H H mediate
Chemical Sciences (CSIR) J-15 | 13

(C) The product is 2-hydroxynicotinamide 104. The major products A and B formed in the
and reaction involves benzyne-like following reactions are—
intermediate O
1. PdCl2, CuCl
(D) The product is 2-hydroxynicotinamide O2, DMF-H2O
i. Li, NH3 (l)
and reaction involves addition-elimina- A B
ii. allyl bromide 2. ethanolic KOH
tion mechanism
102. The major product formed in the following O
reaction is— B= O
OAc Pd(OAc)2 (A) A =
PPh3, Et3N H
CH3CN
N Ph O
O
H OH
Ph (B) A = B=
(A) N Me
O
O
H Me
Ph (C) OH
N B=
A=
(B)
O
H
B= O
H Ph (D) A =
N H
(C)
105. An organic compound shows following
H spectral data :
IR (cm– 1) : 1680
HO 1 H NMR (CDCl ) : δ 7·66 (m, 1H), 7·60 (m,
3
1H), 7·10 (m, 1H), 2·25 (s, 3H)
(D) N 13C NMR (CDCl ) : δ 190, 144, 134, 132,
Ph 3
Ac 128, 28
m/z (EI) : 126 (M+, 100%), 128 (M+ + 2,
103. The major products A and B formed in the 4·9%)
following reactions are— The structure of the compound is—
Me Me OAc
KH Br Ph3P
A B
THF n-BuLi (A) (B)
HO Me 18-crown-6 rt 0 °C O
O S
Me Me
Me CHO B = Me
(A) A = (C) (D)
Me Me O CO2Me S
O
Me Me
Me CHO B = Me 106. The correct set of reagents to effect the
(B) A = following transformation is—
Me Me
O O
Me Me CO2Me
Me CHO B = Me
(C) A =
Me Me
(A) (i) (a) NaOMe, Mel; (b) NaCl, wet
Me Me
Me Me DMSO, 160°C;
CHO B =
(D) A = (ii) (a) LDA, – 78°C, TMSCl; (b) t-
Me Me BuCl, TiCl4, 50°C
14 | Chemical Sciences (CSIR) J-15

(B) (i) (a) NaOMe, MeI; (b) aq. NaOH then OH


HCl, heat OH
(ii) (a) Et3 N, TMSCl, rt; (b) t-BuCl, (C) A : LiHMDS, AcOEt B = N
TiCl4, 50°C N
(C) (i) LDA, t-BuCl; (ii) LDA, MeI; (iii) OH
aq. NaOH then HCl, heat OH
(D) (i) (a) NaCl, wet DMSO, 160°C; (b) (D) A : n-BuLi, AcOEt B = N
N
NaH, t-BuCl
109. The major product of the following reaction
(ii) (a) morpholine, H +; (b) MeI then sequence is—
H3 O+
Br NHAc OH
107. The correct structures of the intermediates
CO2Me
[A] and [B] in the following reaction are— Pd(OAc)2 Pd(OAc)2
N PPh3, Et3N PPh3, Et3N
POCl3 PH NH2
[A] [B] Ts
N O N N Ph CO2Me
H H (A) OH

(A) A = AcHN
Cl B= Cl Cl
N OP(O)Cl2 N P
Cl N
H H O
Ts
CO2Me
(B) A = B= Cl (B)
N O N Cl
P(O)Cl2 P(O)Cl2 AcHN

N
(C) A = Cl B=
N Ts
N OP(O)Cl2 Cl
H (C) CO2Me

NHAc
(D) A = B= Cl Cl
N O N P
Cl N
P(O)Cl2 H O
Ts
108. The correct reagent combination A and the (D) OH CO2Me
major product B in the following reaction
sequence are— NHAc
O O O
A H2N-NH2 N
B
EtO2C EtO2C Ts
110. The major product formed in the following
reaction is—
(A) A : LiHMDS, AcCl B = N O
EtO2C N Cp Me
H Ti AI
O Cp Cl Me
H Pyridine, toluene, −40°C
(B) A : n-BuLi, AcCl B = O
N
EtO2C N COOEt
H PhMe2Si
Chemical Sciences (CSIR) J-15 | 15

O 112. The major product formed in the following


(A) reaction is—
PhMe2Si H O

EtOOC hv, acetone


O

O H
(B)
Me
PhMe2Si H
Me
EtOOC (A) (B)
O
O O
O O
(C) Me
PhMe2Si H (C) (D)
O
EtOOC O 113. The hydrocarbon among the following
having conformationally locked chair-boat-
O
chair form is—
(D) H H H H
PhMe2Si H
(A) (B)
EtOOC O H H H H
H H H H
111. The major products A and B in the following
synthetic sequence are— (C) (D)
O H H H H
i. PhMgBr
Me CuI NaOEt 114. The major product formed in the following
A B reaction sequence is—
ii. H3O+ Br2
O
1. (Boc)2O, pyridine
O O
HO 2. TBSCl, Imidazole
NH2 3. LiAlH(Ot-Bu)3
Me Ch2Br EtOH, −78°C
(A) A = B=
Ph Ph OH

O O (A) TBSO
NHBoc
Me Ch2Br
(B) A = B= OH
Ph Ph
(B) BocO
O Br O NHTBS
OH
Me Me
(C) A = B=
Ph Ph (C) TBSO
NHBoc
O O
Br OH
Me Me
(D) A = B= (D) BocO
Ph Ph NHTBS
16 | Chemical Sciences (CSIR) J-15

115. The major product in the following reaction 117. IN the following reaction, the ratio of A:B:C
sequence is— is (* indicates labelled carbon)
O N2 Me Br Br Br
hv, OTIPS NBS
Me AIBN
vycor filter
+ +
CCl4
ClCH2CH2Cl, 80°C heat
A B C
(A) Me Me (A) 1 : 1 : 1 (B) 1 : 2 : 1
H
HO OTIPS (C) 2 : 1 : 1 (D) 3 : 2 : 1
H
118. Structure of the major product in the
following synthetic sequence is—
CO2Me
(B) Me Me 1. CuI
2. SeO2
O OTIPS
N2

H
Me
(A) HO
(C) Me Me
H CO2Me
HO OTIPS
H
HO Me
(B)
H CO2Me
(D) Me Me
HO OTIPS H
HO Me
(C)
H CO2Me

HO H
116. Structures of A and B in the following Me
synthetic sequence are— (D)
O CO2Me
1. Ph3P=CHCO2Me i. LiAlH4 H
N A B
AcO H CHO 2. heat ii. H3O+
119. Major product formed in the following
CO2Me B = synthetic sequence on the monoterpene
(A) A = N N pulegone is—
OAc 1. Br2
O O
2. NaOEt, EtOH
H H
3. KOH, EtOH
O
(B) A= N CO2Me B = N CO2OH

O COOH
O H O H (A) (B)
CO2H
(C) A = AcO N B = Me N
H H

MeO2C HOH2C
H H
(C) (D)
(D) A = N CO2Me B = N CH2OH O
HO
O O OEt
Chemical Sciences (CSIR) J-15 | 17

120. Optically pure isomers A and B were heated 125. The spectroscopic technique, by which the
with NaN3 in DMF. The correct statement ground state dissociation energies of
from the following is— diatomic molecules can be estimated is—
NMe2 NMe2 (A) microwave spectroscopy
(B) infrared spectroscopy
Br Br
A B (C) UV-visible absorption spectroscopy
NMe2 NMe2 (D) X-ray spectroscopy
126. The term symbol for the first excited state of
N3 N3 Be with the electronic configuration 1s2 2s1
C D
3s1 is—
(A) A gives optically pure D and B gives
optically pure C (A) 3 S 1 (B) 3 S 0
(B) A gives racemic mixture of C and B (C) 1 S 0 (D) 2 S 1/2
gives optically pure C
127. Which of the following statements is
(C) A gives optically pure C and B gives INCORRECT ?
racemic C
(A) A Slater determinant is an antisym-
(D) A gives optically pure D and B gives metrized wavefunction
racemic D
(B) Electronic wavefunction should be
121. A molecular orbital of a diatomic molecule represented by Slater determinants
changes sign when it is rotated by 180°
(C) A Slater determinant always corres-
around the molecular axis. This orbital is—
ponds to a particular spin state
(A) σ (B) π
(D) A Slater determinant obeys the Pauli
(C) δ (D) ϕ exclusion principle
122. IR active normal modes of methane belong
128. Compare the difference of energies of the
to the irreducible representation—
first excited and ground states of a particle
Td E 8C 3 3C 2 6S 4 6σd
confined in (i) a 1-d box (Δ1 ), (ii) a 2-d
A1 1 1 1 1 1 x 2 + y 2 + z2 square box (Δ2 ) and (iii) a 3-d cubic box
A2 1 1 1 –1 –1 (Δ3). Assume the length of each of the boxes
E 2 –1 2 0 0 2z2 – x2 – y2 , is the same. The correct relation between the
x2 – y2 energy differences Δ1 , Δ2 and Δ 3 for the
T1 3 0 –1 1 –1 Rx, R y, R z three cases is—
T2 3 0 –1 –1 1 x, y, z, xy, yz, (A) Δ1 > Δ2 > Δ3 (B) Δ1 = Δ2 = Δ3
zx (C) Δ3 > Δ2 > Δ1 (D) Δ3 > Δ1 > Δ2
(A) E + A1 (B) E + A2
129. The correct statement about both the average
(C) T1 (D) T2 value of position (〈x〉) and momentum (〈p〉)
123. The symmetric rotor among the following of a 1-d harmonic oscillator wavefunction
is— is—
(A) CH4 (B) CH3 Cl (A) 〈x〉 ≠ 0 and 〈p〉 ≠ 0
(C) CH2 Cl2 (D) CCl4 (B) 〈x〉 = 0 but 〈p〉 ≠ 0
124. The nuclear g-factors of 1H and 14N are 5·6 (C) 〈x〉 = 0 and 〈p〉 = 0
and 0·40 respectively. If the magnetic field (D) 〈x〉 ≠ 0 but 〈p〉 = 0
in an NMR spectrometer is set such that the
proton resonates at 700 MHz, the 14 N 130. The value of the commutator [x, [x, px]] is—
nucleus would resonate at— (A) i hx (B) – i h
(A) 1750 MHz (B) 700 MHz
(C) 125 MHz (D) 50 MHz (C) i h (D) 0
18 | Chemical Sciences (CSIR) J-15

131. The equilibrium constants for the reactions 136. For a reaction on a surface
CH4 (g) + 2H 2 O(g) CO2(g) + 4H 2 (g) and H H
CO(g) + H2 O(g) CO2 (g) + H2 (g) are K1
H2 + S S S S
and K 2 , respectively. The equilibrium
constant for the reaction H H H
CH4 (g) + H 2 O(g) CO(g) + 3H2 (g) is— slow
S S S S +H
(A) K1·K2 (B) K1 – K2
(C) K1/K2 (D) K2 – K1 At low pressure of H2 , the rate is propor-
132. Consider the progress of a system along the tional to—
path shown in the figure. ΔS (B → C) for one (A) [H 2 ] (B) 1/[H2]
mole of an ideal gas is then given by— 1/2
(C) [H 2 ] (D) 1/[H2]1/2
A(T1, V1)
137. The temperature-dependence of an electro-
chemical cell potential is—
p Adiabatic ΔG ΔH
process (A) (B)
nFT nF
C(T3, V1) B(T2, V2) ΔS ΔS
(C) (D)
V nF nFT
T1 T3 138. The single-particle partition function (f) for a
(A) R ln (B) R ln
T3 T1 certain system has the form f = AVeBT. The
V V average energy per particle will then be (k is
(C) R ln 2 (D) R ln 1
V1 V2 the Boltzman constant)—
133. A thermodynamic equation that relates the (A) BkT (B) Bk T2
chemical potential to the composition of a (C) k T/B (D) k T/B2
mixture is known as—
(A) Gibbs-Helmholtz equation 139. The indistinguishability correction in the
(B) Gibbs-Duhem equation Boltzmann formulation is incorporated in
(C) Joule-Thomson equation the following way : (N = total number of
(D) Debye-H ··uckel equation particles; f = single-particle partition func-
134. According to transition state theory, the tion)
temperature-dependence of pre-exponential (A) Replace f by f / N!
factor (A) for a reaction between a linear and
(B) Replace f N by f N/N!
a non-linear molecule, that forms products
through a non-linear transition state, is given (C) Replace f by f / ln (N!)
by— (D) Replace f N by f N/ln(N!)
(A) T (B) T2
(C) T– 2 (D) T– 1·5 140. In a photochemical reaction, radicals are
formed according to the equation—
135. For a given ionic strength, (I) rate of reac-
tion is given by k1
k C 4 H10 + hv 2C2 H5
log = – 4 × 0·51 (I)1/2. Which of the k–1
k0
following reactions follows the above k2
C 2 H5 + C2H5 ⎯→ C2 H6 + C2H4
equation ?
(A) S 2 O82– + I– If I is the intensity of light absorbed, the rate
(B) Co(NH3)5Br2+ + OH– of the overall reaction is proportional to—
(C) CH3 COOC2 H5 + OH– (A) I (B) I1/2
(D) H+ + Br + H2O2 (C) I [C4 H10] (D) I1/2 [C4H10]1/2
Chemical Sciences (CSIR) J-15 | 19

141. Conductometric titration of a strong acid Answers with Hints


with a strong alkali (MOH) shows linear fall
of conductance up to neutralization point 1. (A) I. Cloth reel → three
because of— J. Silent wonder → two
(A) formation of water K. Good tone → one
(B) increase in alkali concentration L. Bronze rod → zero
(C) faster moving H+ being replaced by
slower moving M+ Ascending order L, K, J, I.
2
(D) neutralization of acid 2. (C) 22 2 = 24 2 = 2 16
12m × 4m
142. Find the probability of the link in polymers 3. (C)
where average values of links are (1) 10, 4m
(2) 50 and (3) 100— 4m 45°
4m
(A) (1) 0·99, (2) 0·98, (3) 0·90
Area = 12 × 4 = 48 m2
(B) (1) 0·98, (2) 0·90, (3) 0·99 20 a
4. (C)
(C) (1) 0·90, (2) 0·98, (3) 0·99 10b 200b 10ab = 840
(D) (1) 0·90, (2) 0·99, (3) 0·98 2 40 2a = c6

143. The stability of a lyophobic colloid is the and 2 × a gives 6 at unit place
consequence of— 2×3 = 6
(A) van der Waals attraction among the or 2 × 8 = 16
solute-solvent adducts So, a can be 3 or 8.
According to question,
(B) Brownian motion of the colloidal 840 + c 6 = 8d6
particles which gives c+4 = d
(C) insolubility of colloidal particles in as digit at hundred place is 8, so c will be
solvent between 0 and 5.
(D) electrostatic repulsion among double- So, from table we have
layered colloidal particles 40 + 2a = c 6
c should be 4 or 5
144. In a conductometric experiment for estima- 40 + 2 × 3 = 46 …(I)
tion of acid dissociation constant of acetic and 40 + 2 × 5 = 50 → (not possible)
acid, the following values were obtained in So, a = 3, c = 4
four sets of measurements : Now check from option
1·71 × 10– 5, 1·77 × 10– 5, 1·79 × 10– 5 and We obtain a + b = 11.
1·73 × 10– 5. 5. (A) We use combination n C 4 point of inter-
The standard deviation of the data would be section. For octagon n = 8.
in the range of— 8!
8C =
(A) 0·010 × 10– 5 – 0·019 × 10– 5 4
4! × 4!
(B) 0·020 × 10– 5 – 0·029 × 10– 5 8 ×7×6×5×4
= = 70
4! × 4!
(C) 0·030 × 10– 5 – 0·039 × 10– 5
(D) 0·040 × 10– 5 – 0·049 × 10– 5 6. (D)
145. Silver crystallizes in face-centered cubic
structure. The 2nd order diffraction angle of
a beam of X-ray (λ = 1Å) of (111) plane of base area = 24cm × 48cm
the crystal is 30°. Therefore, the unit cell Longest stick can be kept inside the body
length of the crystal would be— diagonally.
(A) a = 3·151 Å (B) a = 3·273 Å So, (24) 2 + (48)2 + h2 = 562
(C) a = 3·034 Å (D) a = 3·464 Å after solving we get, h = 16 cm.
20 | Chemical Sciences (CSIR) J-15

So, duration of Man


7. (C)
= Elapse time including time lag
+ Return time + staying time
= 18 hrs + 18 hrs + 12 hrs
Area of triangle = N. = 48 hours.
Radius of incircle × P of triangle. 1
13. (A) Let = x
N = ? 1234
x
r +r = 2 – x
We know incircle touches the triangle at mid
point. 1
rx + x = 2
r
1
Area of triangle = base × height r2x + 1
2 = 2
rx
1 x
= × radius × perimeter 2x r
r –2 +1 = 0
2
(rx – 1)2 = 0
1
N = or rx = 1
2 and r x = 1

8. (C) So, r 4321 +r – 4321 = 2


Distance
Let volume be x 14. (A) Time taken =
Speed in still water
x x x
Series : x, , , … , n – 10 m
2 4 2 =
10m/min.
x = 20 cc = 1 min.
1
Common ratio = 15. (A) 16. (C)
2
17. (D) U … G … C … C … S … I … R
Sum of the series (as series is in G.P.)
No. of letters = 7
x 20cc
= = Position of first C = 3
1–r 1
1– Second C = 4
2
Permutation 3 × 7 × 4 = 25
= 40 cc
9. (B) 1. 2. 3. 90°
18. (B) A
1. 2. 3. C R
Ratation of 180° clockwise. B
R
10. (A) Melting point decreases with pressure, 30°
boiling point increases and also solid, liquid D
and gas can co-exist at the same pressure and Height = AB + CD
temp. AB = R cos 30°
Sum of observation Height = R + R cos 30°
11. (A) Mean =
No. of observation ⎯√ 3 R
If sum of observation change then mean will = R+
2
also change. 19. (B) Length = 24 m
12. (A) Journey start Width = 3 m
0100 Hrs local time ⎯ 0900 Hrs. Boundary is required to be at least 60 m
So, elapsed time = (9 – 1) hrs + lag time first pitch is at 60 m from boundary = 0
= 8 hrs + 10 = 18 hrs. Diameter of field = 140 m
Elapse time for staying Next pitch at 63, 66, 69, 72, 75, 78, 81
= (21 – 9) 60 + 81 = 141
= 12 hrs So, leave 81
Chemical Sciences (CSIR) J-15 | 21

So, total possible pitches at 60, 63, 66, 69, 70, 25. (B) [reference → Inorganic chemistry +
75, 78 m position Atkins]
Total = 7 In Fischer Carbene Oxidation State of metal
20. (B) is low and ligand is π acceptor and nature of
carbonic Fischer carbene is electrophilic
21. (A) CO2 + H2 O H2 CO3
Anhydrase OH
Hydration occur in blood at high pH value. Ex. : (CO)5 Cr C (Fischer Carbene)
CH3
Dehydration occur in lung at low pH value.
Carboxypeptidase A is pancreatic exopep- While in Schrock carbene metal are in high
tidase that hydrolise peptide bond of C termi- positive oxidation state, ligand are non-π
nal. acceptor and nature of Schrock carbene is
22. (A) Deoxy. Hb. Oxy. Hb. nucleophilic.
Fe+2 HS Fe+2 LS Ex. : Grubb Catalyst
t2 g 4 eg2 t2 g 6 eg0 Pcy3
Fe is above the Fe is fitted well Cl H
plane which tends inside the plane Ru C
to increase the thus reducing Cl H
Fe—N distance Fe—N bond dis- Pcy3
tance.
26. (A) [Inorganic Chemistry – By Greenwood]
23. (C) When NO is linear it donate 3e –
Since both complex follow 18e – rule. Proton affinity of an anion or a neutral atom
or molecule is a measure of its gas phase
So [Co(CO) 3 NO] ⇒ Total valence electron
basicity. Higher the proton affinity stronger
Co = 9 (d7 s2 ) the base and weaker the conjugate acid in gas
CO = 3 × 2 = 6 phase.
NO = 3 So N3– is strongest base.
[Ni(η5 – Cp)NO] = 18 e– 27. (D) [Ref. → Analytical Chemistry – S.M.
Total Valence Electron Kopkar]
Ni = 10 (d8 s2 ) ilkovic equation id = 607 nD1/2 m2/3 t1/6 c
η Cp = 5
5
From this eq. id ∝ t1/6
NO = 3 i.e., depends upon drop time (t = drop time)
TV E = 18 e – and its value also depend upon temperature
24. (A) Wacker process— the value of id (diffusion Current) increases at
a rate of 1 – 2% per °C.
O
[PdCl4]2− 28. (C) The threshold energy is the minimum
CH2 CH2 + O2 CuCl2 CH3 H projectile energy necessary to satisfy mass
energy and momentum conservation in a
i.e., formation of carbonyl compound by
Nuclear reaction to form product in their
Reaction of Alkene and O2 in presence of
ground state.
Pd +2.
Mechanism— ⎛ mp + mc⎞
T.E. = – Q ⎜ ⎟
CH2 CH2 + [PdCl4]2− + H2O ⎝ mc ⎠
O
+ Pd° + 2HCl
= – (– 3·23) (1 +1313)
CH3 H = 3·485
13N(n,p)13C Q = 3·236
Regeneration of Wacker catalyst‚
II
Pd° + 2Cu Cl2 + 2Cl– → [PdCl4 ]2– + 2CuCl So 13C(p, n)13N = – 3·236
22 | Chemical Sciences (CSIR) J-15

29. (D) ⇒ – 2200 ppm Reference—Principles of Structure and


Reference → Tin Chemistry—Fundamental Reactivity. INORGANIC CHEMISTRY by
Frontiers and Application By – Marcel James E. Huheey IV edition Ch-13, page 552.
Gielen. 37. (A) Explanation—After addition of HCl in
30. (A) At high temperature or at melting form equation (A) Primary OH group protonated
phosphorous exist as symmetrical P4 type and H2 O removed early. i.e.,
structure
i.e., n P HO OH H H2O OH
P P O O
P O
− −H2O
−2 +4 O O
31. (A) S2O exist as S S
32. (D) Pseudohalogen are polyatomic analogous In equation (B), Carbonyl group protonated
of Halogen whose chemistry resemble with and Electrophilic charactor of C = O group
true Halogen common example. These anions increases so, CN ion attacks easily,
contain nitrogen atoms.
CN– , N3– , SCN–, NCN2–, OCN, Co(CO)4 etc. δ(−) H
O
33. (A) M (t2g) → PR3 (σ* ) O + CN
H CN
Phosphines (PR3 ) primarily function as Lewis
base, interacting with metal as σ donar Therefore, equilibrium will shift towards right
Ligand. PR3 can accept electron density from in case of both A and B.
metal into P—C (σ * ) Antibonding orbital 38. (A) Explanation—
having π symmetry. 3300 cm–1 peak is for ≡ C—H stretch.
34. (A)
Ethanol 2150 cm–1 is for C ≡ C—H stretch.
RhCl3 ·3H2 O + PPh3 ⎯⎯→ [RhCl(PPh3)3] Thus, CH3 —C ≡ C—H
RhCl(PPh 3 )3 is Wilkinson catalyst and
common method for preparation of 39. (B) Explanation—Acetone have 6H and
Dichloromethane have 2H, when concentra-
RhCl(PPh 3 )3 is Refluxing of RhCl 3 ·3H2 O
tion is same then intensity ratio is 3 : 1. in
with PPh 3 . Moreover, on counting the total 1 H–NM R spectra.
valence electrons,
[Rh(PPh3)2Cl] = 9 + 3 × 2 + 1 = 16e– But intensity ratio obtained is 1 : 1. So,
concentration of Acetone and Dichloro-
{Cl atom cannot be bridging ∴ its contri-
– methane must be in ratio of 1 : 3.
bution is 1}. Wilkinson catalyst is 16e
species. 40. (D) Explanation—The intensity of IR-spec-
35. (A) In M⎯⎯ — trum bands is dependent mainly on the mag-
— H, the unit is anti-periplanar and nitude of dipole.
α
thus β-H elimination not possible. β More the polar charactor of a bond, the
M
greater the intensity of IR-band. So, In case of
H CO the intense band observed in IR-spectrum
For β hydride elimination reaction β hydrogen due to large dipole moment change of CO due
should be closer to metal in other 3 option all to bond stretching.
hydrogen atoms are far so elimination not 41. (C) Explanation—Bromo-Heptatriene give
possible (sp 2 carbon) Bromide ion easily and becomes aromatic, so
36. (A) The reaction follows SN, CB mechanism AgNO 3 give precipitate of AgBr.
in which M—Y bond is fully broken before
M—X bond begins to form. +AgNO3
Br + AgBr
Thus, Id mechanism is the most evidential −Br Tropyllium cation
mechanism. Aromatic (Most stable)
Chemical Sciences (CSIR) J-15 | 23

42. (A) It is Hunsdiecker reaction. This reaction Me Me Me


follows a free radical mechanism. Me Me Me
Me
COOAg COOBr COO Me
Br2 (I) (II) (III) (IV)
−AgBr −Br order, I > III > II > IV
because heat of hydrogenation of alkene
−CO2
1
COOBr ∝
COO Br Stability of Alkene
+ i.e., more the stable Alkene less will be heat
of hydrogenation.
(Product) (Free Radical) Hence order of stability of Alkene →
43. (A) Explanation—In general, 1, 2 Hetero I < III < II < IV
substitution, ee is the most stable form but 47. (A) Explanation—The correct statement(s)
due to repulsion between the Methyl groups, for ribose is—
ae form here is most stable. (A) On reduction with NaBH 4 it give
H Me optically inactive product
Equitorial
Me CHO CH2−OH
H NaBH4
(CHOH)3 (CHOH)3
Me axial
(ae) CH2OH CH2OH
(Optical inactive)
44. (D)
O (B) On reaction with methanolic HCl it gives
OH
NaBH4, CeCl3 a furanoside.
Me CHO Me CHO
MeOH, H2O H
C O
NaBH 4 give chemoselective reduction of
H OH MeO O CH2OH
ketone in presence of CHO group, when HCl, MeOH
H OH H H
CeCl 3 is mixed with NaBH4 . H H
H OH
45. (C) Explanation—Both structures are CH2OH OH OH
identical; because both are super-imposable to (furanoside)
each other, when one structure is rotated out (C) On reaction with Br2 -CaCO3–water its
of plane as shown below :— gives following compounds—
H H H Cl CHO CHO
Me , Me H OH Br2−CaCO3 H OH
Cl H H OH H OH
HO OH water
H OH CH2OH
(I) (II)
CH2OH
180 Rotn
H H
which is optical active.
out of plane Me
Cl (D) It gives positive follen’s test.
OH
CHO COOH
(III)
H OH Ag2O H OH
I and III are Homomers. So, both structures H OH , H OH
Tollen s
are Homomers. H OH Reagent H OH
CH2OH CH2OH
46. (B) The correct order of heat of hydrogenation
of the following compounds are— So, statement A, B and D is correct.
24 | Chemical Sciences (CSIR) J-15

48. (B) Cinnamic acid is biogenatic precursors for 53. (A) Explanation—Magnitude of stability
the natural product umbelliferone constant for K+ ion complexes of the supra
molecular hosts is directly proportional to
ionic interaction and size of cavity.
HO O O So, more the ionic interaction of hetero atom
13C-
with K+ ion more will be stability. So, stability
49. (C) (R)-4-methylpentane-2-ol give 5; order is B > A > C.
NMR signal.
54. (B) Explanation—Antitubercular drug(s)
5
CH3 3 among the given compounds are—
4 2 CH3 1
(A) Salbutamol—It is used for relief of
OH CH3 bronchospasm in conditions such as ‘asthma’
and ‘chronic abstructive’.
1 signal → 2 Methyl group OH
1 signal → CH
1 signal → CH2 HO
HN
1 signal → CH (attached to OH) HO
1 signal → CH3 (B) Ethambutanol
50. (D) NaBH4 is chemoselective reducing agent OH
for carbonyl group in presence of ester. In H
N
presence of steric hindrence it give hydride N
ion from less hindered side. So, product (D) is H
HO
major product
Me Me Me Me (C) Isoniazid O NH
NH2
EtOOC EtOOC
NaBH4
H 0°C H OH
MeOH/THF
N
Less O more hindered H (D) Diazepam—It is used treatment of
hindered due bridging
‘anxiety’, alcohol withdrawal syndrome,
H Me
muscle spams, trouble sleeping and restless
51. (A) Me legs syndrome.
Heat
CH3
[ene reaction] O
N
CH2
Cl N
CH2 H

52. (C) O Me
O 55. (C) We know that
H2N−NH2.HCl
2 sin n πx if n = 0
[Walf-Kishner reduction]
Ψ =
a a ( )
then Ψ = 0 (i.e., wavefunction become zero)
Me Me Me and it is not possible. In I-D box for ground
O O HO
H state n = 1. n = 0 exists only for Simple
Et3N
Harmonic Oscillator.
56. (C) When we add potential in the Hamaltonian
the energy eigen value will shift but the eigen
[Product] function will not change.
Chemical Sciences (CSIR) J-15 | 25

A1 E.C. L1 because upon addition of inert gas at constant


57. (D) = volume, the total pressure will increase. But
A2 E.C. L2
the concentration of the product and reactant


log (IIt ) = L
0
1
(i.e., ratio of their moles to the volume of
container) will not change.
A2 L2 – E0
100 63. (C) k = ATn exp.
log RT
50 1 is the modified form of Arrhenius Eqn.
⇒ =
A2 3 – E0
k = AT2 exp. (given)
log 2 1 RT
=
A2 3 Comparing eqn. (i) and (ii) we get
A2 = 3 log 2 = 3 × 0·3010 Ea = 2RT + E 0
A2 = 0·9030 64. (A) 2A + B → 3Z
1 D[A] D[B] 1 D[Z]
1 – = = …(i)
We know that A = log 2 Dt Dt 3 Dt
T
D[A]
1 – = 2 × 10– 4 (given)
= Antilog of A2 Dt
T from eqn. (i)
1
⇒ = Antilog 0·903 –
1 D[A]
=
1 D[Z]
T 2 Dt 3 Dt
1 D[Z] 1
= 8 = 3 × × 2 × 10– 4
T Dt 2
1 1
T = % = × 100 = 12·5 = 3 × 10– 4 mol dm– 3 s– 1
8 8 65. (B) The escaping tendency of a charged
58. (B) For allowed transition particle from a phase can be affected by the
ΔS = 0 charge state of the phase and the variable used
ΔL = ± 1 to describe the differences is escaping
tendency of electrostatic potential φ and the
3S → 3P electrostatic potential at a point (a) is defined
ΔS = 3 – 3 = 0 as the work per unit charge required to bring a
ΔL = (1 – 0) = ± 1 positive test change reversible from infinity to
(For P, L = 1 for S, L = 0) the point
⎪⎪x⎪⎪ σxy ⎪⎪ x⎪⎪ φ(a) =
δω∞ → a
59. (A) ⎪y⎪ ⎯⎯→ ⎪–y⎪ …(1) dθ
⎪⎪z⎪⎪ ⎪⎪ z⎪⎪
Reference—‘Principle of thermodynamics’
⎪⎪x⎪⎪ C2 (x) ⎪⎪ x⎪⎪ σxy ⎪⎪ x⎪⎪ by ‘Myron Kaufam’.
C 2 x.σxy = ⎪y⎪ ⎯→ ⎪–y⎪ ⎯→ ⎪–y⎪ 66. (C) η = KMa (given)
⎪⎪z⎪⎪ ⎪⎪–z⎪⎪ ⎪⎪ z⎪⎪ It is Mark-Hauwink equation where K, a is
constant for polymer solvent pair.
From eqn. (i) and (ii) 67. (D) We know that,
C 2 x.σxy = σx z. ΔG = ΔG° + RT ln Q
60. (B) HCl is a covalent molecule. Thus, both the Q = Reaction quotient,
electrons cannot reside only in Cl atom. Thus, [Product]
option (C) is wrong. In option (A) and (D) for Q =
[Reactant]
H-atom, 3p orbital is not present (H → 1s; Cl [Zn+2] [Cu]
= 3s2 3p5 ). ΔG = ΔG° + RT ln
[Zn] [Cu+2]
61. (D) For monoatomic gas it only posses Since activity of Cu and Zn is unity
translational degree of freedom so variation of
heat capacity with temperature is ignored. a +2
So, ΔG = ΔG° + RT ln Zn +2
62. (C) At constant volume addition of inert gas aCu
does not cause any effect on equilibrium where a = activity.
26 | Chemical Sciences (CSIR) J-15

68. (D) Term symbol = 2S + 1LJ 125 Sn ⇒ A = 125


S = 1, L = 0 + 1 = 1 = P r = 1·2 × 10– 15 m
(A) 1/3 = (125) 1/3 = 5
2S + 1 = 3
J = |L + S| ……… |L – S| r = 1·2 × 10– 15 × 5
= 6 × 10– 15 m
= |1 + 1| ……… |1 – 1|
Geometrical cross section = π r2
= 2, 1, 0
1 barn = 10– 28 m2
As this is half filled orbital Cross section Area
∴ Lower value of J will be considered. 22
∴ 3P 0 . = × (6 × 10– 15 m)2 ≈ 1·53
7
69. (B) Double layer interaction is determined by 74. (A)
Gany–Chapmann potential (Ψ0 e– kr) Suzuki coupling—Palladium catalyzed cross
a = represents double layer coupling between organoboranic acid and
b = represents vander Wall’s and double layer. halides.
c = Van der Wall’s. Heck coupling—Palladium catalyzed C—C
70. (D) In FCC, Packing fraction is 74% and No. coupling between aryl halides or vinyl halides
of atoms per unit cell is ‘4’. and activated alkenes in the presence of a
base.
Sonogashira coupling—This coupling
terminal alkynes with aryl or vinyl halides is
performed with a palladium catalyst, Cu(I)
cocatalyst and amine base.
Negeshi coupling—This reaction is the
No. of atom per unit cell organic reaction of an organohalide with an
1 1 organic-zinc compound Pd and Ni catalyst.
= 8× +6× =4
8 2 75. (C) H5 P 3 O8
Packing fraction (φ)
−2 O−2 −2
Volume occupied by the particle O O
= × 100
Volume of unit cell +4
+3
P +4
4 −1 P P −1
4 × π r3 HO −1 −1 −1 OH
3 OH
= × 100 = 74% OH OH
a3
71. (D) Differential pulse polarography (DPP) is 76. (B)
more sensitive then (DCP) from analytical [Br3]+ Hybridisation = sp3
point of view because Non-foradic current is Geometry = Td
less in DPP then DCP. Shape = Bent shap
72. (A) Fluorescence is directly proportional to
the amount of absorbed radiation where F = [Br3]+ ≈ Br2 Br+
S

KP∈bc. S ⇒ SBr2 Br Br
The fluorescence signal can be increased if
the radiat power of the incident beam is [I 5 ]+
Hybridisation = sp3 d
increased, therefore always use more intense Geometry = TBP
sources. 77. (A) Wade’s theory,
Dissolved oxygen largely limits fluorescence F = 3b + 4c + h + x – 2n
since it promoters intersystem crossing b = No. of boron atoms
because it is paramagnetic. c = No. of carbon atoms
73. (B) We know that, According to Fermi model h = No. of hydrogen atoms
r = r0 (A)1/3 x = Amount of negative ions
A = mass number. n = No. of vertices = b + c
Chemical Sciences (CSIR) J-15 | 27

F = 3 × 12 + 4 × 0 + 12 + 2 – 2(12 + 0) [Os5(C)(CO)15]
= 26 1
F = 2n + 2 for Closo. =[74 – 12 × 5]
2
26 = 2 × 12 + 2 1
= 24 + 2 = 26. = [14] = 7 (M + 2) ⇒ nido
2
Hence, closo. [Ru5 (C)(CO)16]
R 1
78. (A) R = [76 – 60]
2
Pb Pb 1
= [16] = 8 (M + 3) Aracho.
R 2
R 84. (C) J. T. distortion occurs where there is
Pb show the inert pair effect. unsymmetrical filling of electrons in t 2g or eg
Due to inert pair effect Pb(II) is more stable orbital.
forming banana bond in which structure is K4[Cr(CN)6 ]
only correct one. Cr = + 2 O.S. low spin complex
79. (A) SbCl3 + 3EtMgBr → SbEt3 + 3MgBrCl Cr(II) = d4 t 2g4 eg0 (unsym)
SbEt3 + 2SbI 3 → [Sb(Et)I2 ]n K4[Fe(CN)6 ] low spin complex
Polymeric structure Fe(II) = d 6 t 2g6 eg0 (symmetrical)
80. (A) Fe–porphyrin complex – colour show is K3[Co(CN) 6 ] low spin complex
due to π – π* transition. Co(III) ⇒ d 6 t 2g6 eg0 (symmetrical)
[Mn(H2 O) 6 ]Cl2 :
K4[Mn(CN)6 ] low spin complex
Mn+ 2 ⇒ d 5 configuration which is both spin
Mn(II) ⇒ d5 t2g5 eg0 (unsym)
and Laporte forbidden
[Co(H 2 O) 6 ]Cl2 : Hence only A and D is having unsymmetrical
filling, hence undergo distortion.
Co(II) ⇒ d7 configuration which is spin
85. (A) The reductive elimination of (A) gives
allowed but laporte forbidden.
toluene
81. (C) Agostic interaction can be detected by the
presence of a 1H NMR peak. A Ar R CH3
Agostic interaction most commonly refers to
a C—H bond on a ligand that undergoes an So, R = CH3 option A is correct.
interaction with the metal complex. 86. (A) Structure of oxy-haemocyanin.
82. (A) Cp proton of complex (A) is highly His
deshielded because of CO ligand. CO is very N N
good π-acceptor ligand. His
N N
Complex (D) will be highly shielded because N ++ O ++ His
Fe is small in size. It is more closed to N Cu Cu N N
nucleus. His O
N N
83. (B) (M + 1) = closo, (M + 2) = nido, (M + 3)
= Aracho, (M + 4) = Hypo = (M = no. of His N
metal atom) N
His
1
Weadge Rule = [TVE – 12 × No. of metal] Blue colour O.S. Cu = II
2
Ru 6 (C) (CO)17 B.O. of O2 = I
1 87. (B) In strong acidic condition
= [8 × 6 + 4 + 17 × 2 – 12 × 6] H+
2 KMnO4 ⎯⎯→ Mn+2
1 Base
= [86 – 72]
2 Mn+2 ⎯⎯→ Mn+ 4
1 So, option (B) is correct., Acidified Mn+2 is
= [14] = 7
2 not oxidised by H2O2 but Alkaline
(M ± 1) = closo Mn(OH)2 + H2 O2 → Mn2 O3
28 | Chemical Sciences (CSIR) J-15

88. (C) Oxidation of Zn to Zn+2 will be diamag- 5 H.S.


netic hence EPR inactive. 4
3
Fe and Sn – Inactive. Sharp change 175
μ 2
EPR spectroscopy generally used for the 1
species have one or more unpaired electron. 0 L.S.
175 T
Cu(II) are EPR active because non-paired
electron. CFSE Δ = (– 0·4p + 0·6q)
89. (C) CH3
where p = No. of e–s in t2g orbital
CH3
q = No. of e–s in eg orbital
Alkene does not undergo reductive elemina-
tion reaction easily as compared to Alkane. ⇒ d6H.S. Δ = (– 0·4 × 4 + 0·6 × 2)
90. (A) Reaction (i) = – 1·6 + 1·2 = – 0·4 Δ
Water acting as an oxidant oxidizing Ca to ⇒ d6 L.S. Δ = – 0·4 × 6 = – 2·4 Δ.
Ca2+ getting itself reduced to H2 .
Reaction (ii) Water act as Lewis acid accept- (–) ve sign is used for CFSE.
ing elecron from Cl – which act as a ligand. 93. (A) UO2+ disproportionates more easily than
Reaction (iii) Water act as Lewis base donat- UO2+2 and U3 O8 is most stable oxide of U.
ing electron to Mg2+ which act as a.
But coordination No. of U in
Reaction (iv) Water acting as an reductant
reducing F to F –. [UO2 (NO3)2·(H2 O) 2 ]4H2O is eight.
91. (D) Alkyne can act as 2 or 4 electron doner. Structure of UO2 2(+) is linear (Reffrac. = J.D.
L(ππ) → M(σ σ) : Lee)
C 94. (B) CO2 does not combines with [Ni(PR3 )2
M (1,5 – cyclooctadiene]. Only insertion of CO2
C occurs and Et − − Et is inserted to complex

(Empty metal (Filled acetylene


and formed compound.
d-orbital) π-orbitals) 95. (D) (NH 4 )2[Ce(NO3)6] complex NO 3 is
σ-Type donation bidented ligand and structure is Icosahedran
π) → M(σ
and L(π σ) : i.e., coordination number of Ce is 12.
C 2−
O
M N
O
C N OO O
O N O
(Metal filled (Empty acetylene O
Ce O
d-orbital) π* orbitals) O O
π-type donation N OO OO N O
O N
92. (A) Fe(phen)2 (NCS) 2
O.S. of Fe = + 2 O
Fe(II) = d 6
High spin – low spin equilibria or spin cross In Ce+ 4 more stable
over Ce = 4f 1 5d 1 6s 2
d6 H.S. d6 L.S. Ce (IV) = 4f 0 5d 1 6s 0
i.e., Z is diamagnetic
No. of unpaird electron No. of unpaired e– = 0 Ce + 4 is a strong one electron oxidizing agent.
μ = 4·9 BM μ = 0·0 BM Ce + 4 used in redox process.
Chemical Sciences (CSIR) J-15 | 29

96. (C)
OH Cl N3 N
O O O O O O
C C C C C N C N
O SOCl2 O NaN3 O −N2 O O t-BuOK O

[Nitrene] [Isocynate]
O
O O O
C NH C N C N C N
O
O H H O O O
H
H2O

O
C NH O O
C NH C NH
O O O COOH
NH
O
H
C C
H H O H
HO O

H
97. (D) 99. (C)
– O
OH O OH O
O CO2Et
NaNH2 NH2 H 2N + EtO2C CO2Et
NH3(l)
H N
H NH2 H O CO2Et
NH2 Benzyne O O
Br
(OH at para position shows (e– releasing)
Me Me Me CO2Et CO2Et
H N H N H OH Aq. KOH
NaNH2 NH2
NH3(l) CO2Et HO 2Et
CO
H NH2 Aq. KOH
Benzyne
Br NH2 H O O
HO CO2Et
98. (C) OH
CO2Et CO2Et
AcO AcO H N H H N N CO2Et
p−TsNH−NH2/AcOH
CO2Et CO2Et H
O N N Ts
H
CO2OH CO2Et CO2Et
aq.KOH aq.KOH
NaBH3CN
reflux
N N CO2OH N CO2Et
AcO AcO
H [B] H [A] H
N N N NH Ts 0.5 equiv PhC[Me]2OOH
100. (A) 1.0 equiv. Ti[OiPr]4
H H 1.2 equiv. (−) DIPT
OH
OMe CH2Cl2, −20°C
AcO
O

OH OH
OMe and OMe
(Product) [A] [B]
30 | Chemical Sciences (CSIR) J-15

The asymetric synthesis of chiral secondary allylic alcohol or their corresponding epoxide.
101. (A)
O O O O
N C O
NH2 OH N Br−Br N Br OH N Br
N F N F H N F H N F N F
Nitrene Isocynate
OH

NH2 NH C O N C O
H OH
O OH
N F N F N F
[Product]

Ph Ph
OAc H H H
Pd[OAc]2/PPh3; N N
102. (B) Et3N
Et3N, CH3CN
N Ph
H H H
103. (B)
Me Me Me Me
Me
KH/THF Me Me Me CHO
+O
18−Crown−6 rt Me OHC
HO Me K
Me Me [A]
Me
Me
BrPh3P

Me
[B]
104. (D) O
O O IR 1680 C (Carbonyl group)
Li, NH3(l) Allyl bromide 1H
–NMR → δ 7·66 (M, 1H), 7·60 (M, 1H)
CH2 CH CH2
Br
7·10 (M, 1H), 2·50 (S, 3H)
–NMR → δ·190, 144, 134, 132, 128, 28
13C
O O
H3C
Ethanolic KOH
M/Z → 126 [M+, 100%], 128 [M+ + 2, 4·9%]
O PdCl2, CuCl
H3C O2, DMF.H2O H3C 106. (A)
[A]
O
O H OH CO2Me
H2C
NaOMe
H
O O
O OMe–
H3C H H
CH3 CH3 O–
CO2Me O
[B]
Me CO2Me
105. (D) {7.10; (m1 1H)} H H{7.66 (m, 1H)} CH3–I

CH3
NaCl, wet
H S (2.50; S1 3H)
DMSO, 160°C
{7.60, (m1 1H)}
O
Chemical Sciences (CSIR) J-15 | 31

OSiMe3 Cl O H
110. (C)
Me LDA
t−BuCl Me LDA
– 78°C O Cp Me
TMSCl O Ti Al
Cp Cl Me
tBu-Cl TiCl4 (LDA removes e–s from least ,
hindered side) H Tabbe s Reagent
O
O
Me PhMe2Si
COOEt
[Product]
O
107. (C) (4+2) Cyclo Addition
POCl3 H2C Reaction
Cl Cl
H
N O N OP(O)Cl2 N OP(O)Cl2 PhMe2Si
H Cl H H O
[A] COOEt
Ph NH2 −H+ O
N N Ph N N Cl
Cl
[B] H PhMe2Si
H
108. (A) EtOOC O
O
O O
LiHMDS, AcCl CH3 Cl 111. (A)
O
EtO2C CH3 EtO2C CH2 O O
CH3 Cl

O N NH2 O O Me PhMgBr/CuI Me
H2N−NH2
Ph H3O
EtO2C CH3 EtO2C CH3
O HO
Br Br NaOEt Me
O CH3 O CH3
Ph Ph
HO N O
CO2Et N CO2Et N
N
N N EtO2C CH2−Br
H H H H
Ph
(Product)
N
EtO2C N
112. (A)
NHAc
Br I NHAc
Br
CO2Me Me O Me O O
109. (D) CO2Me
hv, Acetone
Pd |OAc|2, Me
N PPh3, Et3N N
Ts Ts H H H
OH NHAc
Me O O
OH CO2Me
Me Me
Pd (OAc)2
PPh3, Et3N N (Product) O
Ts H H
32 | Chemical Sciences (CSIR) J-15

113. (D) 116. (B)


H H H 2
H O
3 1 Ph3P CHCO2Me
6 1 2 3
6 4 AcO N CHO Stabilised
H 5
5 4 H
H H H
O
(Chair-Boat-Chair
CH
Conformation)
AcO N CH CO2Me
114. (C) E-form (trans)
H
O O
(Boc)2O, Pyridine H
CH CO2Me CO2Me
HO HO N
NH2 NHBoc CH2 N
O
O [A]
OAc O
TBSO TBSCl, Imidazole
H LiAlH4/H3O+
NHBoc CH2OH
OH N
LiAlH, (Ot−Bu)3
[B]
EtOH, −78°C TBSO
step - (2)
N 117. (C) NIBS gives bromination at allylic posi-
Step 2 : Chelation-controlled reduction of tion. In this reaction the ratio of A : B : C =
the ketone produces the anti-alcohol 2 : 1 : 1.
diastereo selectivity. Br
115. (D) NIBS, AIBN
+ +
O CCl4, heat

O O Br
C [A] [C]
N2

Br
+
Me Me Carbene Ketene Br
O [A] [B]
O Ratio of A : B : C = 2 : 1 : 1.
Me
C
OTIPS OTIPS 118. (A)
Me CO2Me
[2 + 2] Cycloaddition
Reaction
Ketene N2
CuI
CO2Me
Me Me Me Me
O O OTIPS
H OTIPS H [Carbene]

Me
(Unstable)
+H −H
H CO2Me
Me Me SeO2

Me
HO OTIPS HO
CO2Me
H
Chemical Sciences (CSIR) J-15 | 33

119. (B) 1
nA1 = [15 × 1 × 1 + 0 × 1 × 8 +
24
Br2 NaOEt, EtOH (–1) × 1 × 3 + (– 1) × (+1)
O + 6 + 3 × 1 × 6] = 1
O
Br 1
nA2 = [15 × 1 × 1 + 0 × 1 × 8 +
24
COOH (– 1) × 1 × 3 + (– 1) × 1 × 6
KOH/EtOH
OH + 3 × (– 1) × 6] = 0
O O 1
Br
Br nE = [15 × 1 × 1 + 0 × (– 1) × 8
(Product) 24
120. (B) A gives Racemic mixture of ‘C’ and ‘B’ + ( 1) × 2 × 3 + (– 1) × (0) × 6
gives optically pure ‘C’. + 3 × 0 × 6] = 1
1
nT1 = [15 × 1 × 1 + 0 × 0 × 8 +
NMe2 NMe2 24
NMe
( – 1) × (– 1) × 3 + (– 1) × 1 × 6
+ 3 × (– 1) × 6] = 1
Br 1
(A) Br nT2 = [15 × 1 × 1 + 0 × 0 × 8 +
24
Me2 Me2 (– 1) × (– 1) × 3 + (– 1) × (– 1) × 6
N N
+ 3 × 1 × 6] = 3
So reducible representation.
A1 + E + T1 + 3T2
N3
Since IR activity is count by x , y , z co-
NMe2 NMe2 ordinate so T2 mode is IR active by using
character table.
123. (B) Explanation CH4 , CCl4 → Spherical
N3 N3 mols. (All Td mols are spherical).
Racemic Mixture
H
|
NMe2 NMe2 NMe2 NMe2 CH2 Cl2 ⇒ C has C2 V point
N3 Cl | Cl
Br H
Br N3
N3 (C) group and thus is assymmetric top rotor.
121. (B) In diatomic molecule only σ and π-bond Remaining CHCl3 is symmetric top rotor.
is possible Source–Molecular spectroscopy by Banwell.
180° 124. (D) We know that
σ(gerade) spherical ⎯⎯→ No change occur. γ β0
180° υ =
π(ungerade) ⎯⎯→ change in sign occur. 2π
and g ∝ υ
122. (D) gH υ
or = H
C gN υN
5·6 700
= × = 50 MHz
0·4 x
order = 24 E 8C 3 3C 2 6S 4 6σd 125. (B) The dissociation energy g a diatomic
molecule is calculated from vibration
No. of unshifted atom 5 2 1 1 3 spectroscopy (IR)
Contribution per atom 3 0 –1 –1 1 — ωe (1 – x e2 )
Reducible representation 15 0 –1 –1 3 De =
4 xe
34 | Chemical Sciences (CSIR) J-15

126. (A) Term Symbol = 2S + 1LJ Substracting equation (1) and (2), we get
For orbital l = 0 0 Eqn. (3) and According to equilibrium
K1
2s 3s K3 =
K2
1 1
L = 0 → S, S = + = 1 T2 V
2 2 132. (D) ΔS = C v ln + R ln 2
2S + 1 = 2 × 1 × 1 = 3 T1 V1
In this case, for process B → C,
J = L+S
T2 T
= 0+1=1 = 3
T1 T2
Term symbol = 3S 1 V2 V
127. (C) A slater determinent always corresponds and = 1
V1 V2
to a particular spin state. T V
* Slater determinate changes by changing ∴ ΔS = C v ln 3 + R ln 1
T2 V2
sign upon exchange of 2 electron.
∫ ∫
T3 T3
dV PdV
η 2 h2 ΔS = +
T2 T T
128. (B) Energy (Δ1 ) = x 2 for 1 - D Box T2
8m l x ⎛T ⎞
C v ln ⎜ 3 ⎟ = 0
h 2 ⎡⎢ ηx2 ηy2 ⎤⎥
If
Δ2 = ⎝ T2 ⎠
8m ⎢⎣ Lx2 Ly2⎥⎦
Similarly, +
V1
then, (ΔS)B → C = R ln
⎡⎢ ηx2 ηy2 ηz2⎤⎥
h2 V2
Δ3 =
8m ⎢⎣ Lx2 Ly2 Lz2⎥⎦
+ + 133. (B) Gibbs-Duhem equation gives relationship
between chemical potential and component
So, 1D BOX 2D BOX 3D BOX of a thermodynamic system
nx2 h2 h 2 ⎡⎢ nx2 ny2 ⎤⎥ h 2 ⎡⎢ nx2 ny2 nz2 ⎤⎥ I
Δ=
8m lx2 8m ⎢⎣ Lx2 Ly2⎥⎦ 8m ⎢⎣ Lx2 Ly2 Lz2⎥⎦
+ + + Σ ni dμi = – SdT + VdP
i=1

Ground n=1 n x = 1, ny = 1 n x = 1, ny = 1, 134. (D) According to Transition state theory


state nz = 1 qTranslational ∝ T3/2
Ist excited n = 2 n x = 2, ny = 1 n x = 2, ny = 1, qV/b ∝ T°
state nz = 1 qrotational ∝ T (linear)
Energy difference qrotational ∝ T3/2 (Non-linear)
Linear + Non-linear Non-linear T.S.
3h2 3h 2 3h2
= Pre exponential factor
8m L 8mL 8mL2
2 2
k T Q#
So, Δ1 = Δ2 = Δ3 A = b ×
h Qa.Qb
129. (C) 〈x〉 = 0 T1 ·T 3/2 × T3/2·T 0
or A = 1 3/2 3/2 3/2
and 〈p〉 = 0 T ·T ·T T
Reference ⇒ Physical chemistry by ‘Alkins’. = T – 1·5
130. (D) x.(x, Px) = x.i h = 0 135. (B) Explanation : Primary salt effect
k
Since = [x . Px] = i h log = 2A | Z+ Z – |⎯√I
k0
When position vector commulate with where A = 0·51
constant term, it is always zero. k
log = 2 × 0·51 Z+Z– ⎯ √I
131. (C) k0
CH4 + 2H2 O CO2 + 4H2 ; K1 …(1)
= – 4 × 0·51⎯
√I
CO + H2O CO2 + H2 ; K 2 …(2) Z+Z– = – 2
CH4 + H2 O CO + 3H2; K3 = ? …(3) For option (B); + 2 × – 1 = – 2
Chemical Sciences (CSIR) J-15 | 35

136. (C) From Longmuir Chemisorption isotherm ∂P


140. (A) = k [C2 H5]2 …(1)
dissociation isotherm ∂t
k·[P]1/n Applying steady state Approximation on
θ =
1 + k [P]1/n C 2 H5
For Diatomic molecule n = 2 ∂[C2 H5]
k · [H2]1/2 = 2k1Ia – 2k–1[C2 H5]2 – 2k2 [C2 H5]2
θ = ∂t
1 + k [H2]1/2
2k1Ia = 2k–1[C2 H5]2 + 2k2[C2 H5]2
So, at low pressure
2k1Ia
θ = k [H2 ]1/2 [C2 H5]2 =
2(k–1 + k2)
or θ ∝ [H2 ]1/2 Substituting above value in eq. (1)
137. (C) We know that, ∂P kk 1 Ia ∂P
∂∈ = or ∝I
ΔS = nf ( ) ∂T P
∂t k–1 + k 2 ∂t
141. (C) Basic principle of conductometric
∂∈ ΔS
or ( )
∂T P
=
nf
titration; when faster moving ion is replace
by slower moving ion the conductance
Temperature dependence of Electrochemical decreases linearly.
cell. — 1
142. (C) Since N =
138. (B) F = Partition function 1–P
= AVe BT P = Probability of polymer link
where, A.V. = Constant. —
— N = Average size
Average Energy ( E ) 1
∂ (a) 10 = ⇒ P = 0·9
= – mf …(1) 1–P
∂β 1
1 (b) 50 = ⇒ P = 0·98
But β = 1–P
k BT 1
∂β 1 (c) 100 = ⇒ P = 0·99
= – 1–P
∂T kB T 2
143. (D) Electrostatic repulsion among double-
∂T
∂β = – layered colloidal particle
kB T 2
Reference : Physical chemistry by Puri,
— ∂
then E = kBT2 · mf Sharma and Pathania.
∂T
∂ 144. (C) The mean
= kBT2 · · m (Av eBT) [1·71 × 10– 5 + 1·77 × 10– 5 + 1·79 × 10– 5
∂T
∂ – + 1·73 × 10– 5
= kBT2 [mA + mU + BT] x=
∂T 4
— ∂ [1·71 × 1·77 + 1·79 + 1·73] × 10– 5
E = k BT2 [BT] =
∂T 4
— 7·0 × 10 – 5
E = k BT2 B = = 1·75 × 10– 5
4
139. (B) For indistinguisable particles, the Standard Deviation (s)
canonical partition function over-counts the
number of microstates by the total number (1·71 – 1·75) 2 × 10– 10
of possible swaps among the particle + (1·77 – 1·75) × 10– 10
identities ‘N’ correcting this = + (1·79 – 1·75)2 × 10– 10
qN + (1·73 – 1·75)2 × 10– 10
Q =
N! (4 – 1)
36 | Chemical Sciences (CSIR) J-15

(0·04)2 + (0·03)2 + (0·04)2 a2


d2 =
= + (0·02)2 × 10– 10 12 + 12 + 12
3 a2
d2 =
⎡⎢ ·0016 + 0·0009 + 0·0016⎤⎥ 3
= ⎢⎢ + 0·0004⎥
⎣ ⎥⎦ × 10– 5 d =
a
3
⎯3

0·0045
= × 10– 5 Putting the value of ‘d’ in equation (1)
3
a
(s) = √
⎯⎯⎯⎯⎯
0·0015 × 10– 5 2×1 = 2× sin 30°
On solving, we get standard deviation data ⎯3

of Range = 0·030 × 10– 5 – 0·039 × 10– 5 a 1
145. (D) From Bragg’s Equation 2 = 2× ·
n λ = 2d sin θ …(1) ⎯3 2

1 h 2 k2 l 2 a = 2√
⎯3
and = 2 + 2+ 2
d2 a a a = 2 × 1·732
a 2
or d2 = 2 a = 3·464 Å
h + k2 + l2

⎯⎯⎯⎯⎯⎯⎯⎯⎯⎯

You might also like